Female Reproductive System/Obstetric Gynaecology Flashcards

1
Q

x

On examination of a 28-year-old nulliparous woman at 37 weeks gestation, her fundal height is found to be 35cm and her fetus in transverse lie. An ultrasound exam at 18th week, revealed a low-lying placenta. Which one of the following is the most likely cause of this presentation?

A. Placenta previa.
B. Ruptured membrane.
C. Small for gestational age.
D. Nulliparity.
E. A normal variation.

A

A. Placenta previa

Shoulder presentations, unstable lie, transverse lie and oblique lie may be detected in late pregnancy. These conditions occur in 1 in 200 pregnancies, usually in multiparous women. The etiology is varied. They may occur in a lax multiparous uterus with no other complications of pregnancy but may be associated with other contributing factors.

Contributing factors to fetal transverse lie include:
-High parity
-Pendulous abdomen
-Placenta previa
-Polyhydramnios
-Pelvic inlet contracture and / or fetal macrosomia
-Uterine abnormalities (e.g. bicornuate uterus or uterine fibroids)
-Fetal anomaly (e.g. tumors of the neck or sacrum, hydrocephaly, abdominal distension)
-Distended maternal urinary bladder
-Poorly formed lower segment
-Wrong dates i.e. more premature than appears
-Undiagnosed twins
-Preterm delivery (fetus)

When transverse lie is found, an ultrasound exam is required to exclude placenta previa as a likely cause and a contraindication to vaginal delivery.

85% of low-lying placentas found on ultrasound at 18 weeks will be normally situated at term. In 15%, placenta remains in the lower segment of the uterus. This woman still has a 15% chance of having low placentation as the main contributing factor to the transverse lie.

After 20weeks gestation, the fundal height in centimeters correlates to the gestational age in weeks. A discrepancy of >2 cm between the gestational age and the fundal height (more than 2 cm) is considered significant and can be caused by:

  1. Fundal height < gestational age:
    -Dating errors
    -Oligohydramnios
    -Transverse/oblique lie Small for gestational age
  2. Fundal height > gestational age:
    -Dating errors
    -Large for gestational age
    -Multiple pregnancy
    -Polyhydramnios
    -Molar pregnancy

In this case, a discrepancy of only 2 cm is not that significant but can be caused by the transverse lie.

Over all, some reports show that extreme prematurity is the most common finding in pregnancies complicated by transverse lie, but in this case and with low-lying placenta in history, placenta previa remains the most likely cause to consider and exclude.

Transverse lie near term is not a normal finding. In early pregnancy, it is very common to see in fetus in transverse lie, but not a normal finding near term.

  • South Australian Perinatal Practice Guideline - Unstable lie of the fetus
  • Williams Obstetrics – McGraw Hill - 24th Edition – page 468
How well did you know this?
1
Not at all
2
3
4
5
Perfectly
2
Q

You are working at a busy GP clinic. Your next patient is 24 -year-old woman with irregular menstrual and a positive pregnancy test. She is keen to know the age of her baby. Which one of the following methods would be the most accurate one for estimation of gestational age?

A. Bimanual examinations at 10 weeks.
B. Ultrasound at 16 weeks.
C. Transvaginal ultrasound at 8 weeks.
D. Transvaginal ultrasound at 20 weeks.
E. Transabdominal ultrasound at 20 weeks.

A

C. Transvaginal ultrasound at 8 weeks

By eight weeks gestation, the fetus and its heart beat can be detected relatively easy with transvaginal ultrasound. Dating scans are usually recommended if there is doubt about the validity of the last menstrual period, such as in the following conditions:

-Patient does not know when the first day of her last period or the likely day of conception was
-Patient with irregular periods
-Patients who has become pregnant while on hormonal contraceptives

Transvaginal ultrasonography performed between weeks 8 and 12 (within the first trimester) can predict the gestational with three to five days difference and is the most accurate method of determining the gestational age.

As the pregnancy advances, sonography becomes less accurate in estimation of gestational age.

  • http://sogc.org/wp-content/uploads/2014/02/gui303C
How well did you know this?
1
Not at all
2
3
4
5
Perfectly
3
Q

A 37-year-old woman presents to you with history of irregular periods and decreased libido for the past 6 months. Which one of the following would be the investigation of choice?

A. Serum prolactin level.
B. Ultrasound scan of the ovaries.
C. Serum FSH and LH.
D. Serum FSH and estradiol.
E. Thyroid function tests.

A

D. Serum FSH and estradiol

In women younger than 40 years, who present with secondary amenorrhea or menstruation irregularities and signs of estrogen deficiency such as decreased libido, atrophic vaginitis, etc, the most common cause is found to be primary ovarian failure (POF) (premature menopause) or premature ovarian insufficiency (POI), for which an elevated FSH associated with decreased estradiol level is diagnostic.

Measuring serum FSH level is the core study to establish the diagnosis of POI/POF after pregnancy has been excluded. By convention, 2 FSH levels in the menopausal range for the specific assay (>40 μIU/mL by radioimmunoassay), measured at least 1 month apart, are diagnostic of POI/POF. A parallel test of serum estradiol is necessary. As a rule, serum estradiol is low in women with POI/POF and is similar to or less than the early follicular phase estradiol of women who cycle normally. The combination of low estradiol and high gonadotropins defines POI/POF.

Measurement of serum LH is also important. In most cases of spontaneous POI/POF, FSH is higher than LH. If autoimmune oophoritis is present, FSH may be only mildly elevated, sometimes below the cutoff of 40 μIU/mL, while LH is markedly elevated.

Occasionally, women with POI/POF may have spontaneous follicular activity. In such women, if hormonal tests are performed during such episodes, FSH, LH, and estradiol levels could be in the normal range or only minimally elevated. This may lead to an erroneous exclusion of POI/POF. In these cases, persistent amenorrhea or oligomenorrhea accompanied by menopausal symptoms necessitates a repeat of the above tests in 1-2 months.

*http://emedicine.medscape.com/article/271046-worku

How well did you know this?
1
Not at all
2
3
4
5
Perfectly
4
Q

Which of the following is not a risk factor for isolated spontaneous abortion?

A. Age more than 35 years.
B. Cigarette smoking.
C. High doses of caffeine.
D. Uterine adhesions.
E. Retroverted uterus.

A

E. Retroverted uterus

The following are known risk factors for spontaneous abortion or miscarriage:

-Advanced maternal age (≥35)
-Previous miscarriage(s)
-Antiphospholipid syndrome
-Parentral chromosomal derangements
-Embryonic chromosomal abnormalities
-Congenital uterine malformations
-Cervical weakness
-Diabetes mellitus (subclinical disease excluded) and thyroid disease (subclinical disease excluded)
-Immune factors
-Infections
-Inherited thrombophilic defects
-Caffeine, smoking and alcohol use (dose dependent)

A retroverted uterus has not shown association with increased risk of spontaneous abortions. A retroverted uterus is a uterus that is tilted backwards instead of forwards.

  • The royal college of obstetricians and Gynaecologists – Green-top Guideline No.17
How well did you know this?
1
Not at all
2
3
4
5
Perfectly
5
Q

A 24-year-old woman underwent dilation and curettage for septic abortion. Now, she has developed amenorrhea of six months duration. An office pregnancy test excludes pregnancy. She smokes 10 cigarettes and drinks two standard units of alcohol every day. Which one of the following is the most appropriate next investigation to reach a cause for this presentation?

A. Liver function tests.
B. Urine analysis and microscopy.
C. Transvaginal ultrasound.
D. Full hormone assay.
E. MRI of the brain.

A

C. Transvaginal ultrasound

Intrauterine adhesions are a common complication of curettage. Approximately 90% cases of severe intrauterine adhesions are related to curettage for pregnancy complications such as missed or incomplete abortion, postpartum hemorrhage, or retained placental tissue.

Intrauterine adhesions can be asymptomatic and of no clinical significance.
If there are symptoms they can include:
-Infertility
-Menstrual irregularities (amenorrhea)
-Cyclic pelvic pain
-Recurrent miscarriages

When intrauterine adhesions are suspected, transvaginal ultrasonography is the next best investigation to confirm the diagnosis. Although not always necessary, diagnostic hysteroscopy remains the gold standard diagnostic investigation.

  • UpToDate - Intrauterine adhesions
How well did you know this?
1
Not at all
2
3
4
5
Perfectly
6
Q

A 20-year-old female presents to your practice at 18 weeks pregnancy with right iliac fossa pain that is particularly brought on by getting up from a chair, sneezing and coughing. On examination she has normal vital signs and is otherwise healthy. Abdominal examination reveals no tenderness, rebound tenderness or guarding. Which one of the following is the most likely diagnosis?

A. Round ligament pain.
B. Acute appendicitis.
C. Ovarian torsion.
D. Ruptured ectopic pregnancy.
E. Uncomplicated ectopic pregnancy.

A

A. Round ligament pain

This patient has signs and symptom suggestive of round ligament pain. It is considered a normal finding during pregnancy and does not require any intervention.

Round ligament pain most frequently occurs during the second trimester of pregnancy when women report sharp unilateral or bilateral pain in the iliac fossa that may radiate to the groin. The pain is often sudden-onset, sharp and spastic ,lasting few seconds. The pain is aggravated by standing, getting off chairs, sneezing, laughing or rolling in bed. Sudden change of position is a well-known trigger.

Rest and avoiding sudden changes in body position is the cornerstone of management.

OPTION B : Although appendicitis presents with pain in the right iliac fossa, absence of tenderness makes this diagnosis less likely.

OPTION C : Ovarian torsion presents with aute onset severe pain followed by signs and symptoms of peritoneal irritation such as tenderness, rebound tenderness, guarding, etc.

OPTION D and E : Presentation of ectopic pregnancy and its complications (e.g. rupture) almost always occurs in the first trimester; furthermore, absence of tenderness, rebound tenderness and other localized findings make this diagnosis even less
likely.

NOTE - It should be noted that the question asks about the most likely diagnosis, not the next best step in management. Although, round ligament strain comes top on the list, but more serious conditions should be excluded first through appropriate examination and investigations.

  • Danforth’s Obstetrics and Gynaecology – 10th Edition – page 18
How well did you know this?
1
Not at all
2
3
4
5
Perfectly
7
Q

A 30-year-old woman presents with a 12-month history of secondary infertility. Her first child, fathered by the same partner, was born 4 years ago after she was conceived spontaneously, and through an uneventful vaginal delivery. She has always had irregular periods occurring every 2 to 4 months. On examination, she is obese (BMI>32) and otherwise normal. Ultrasonography of the pelvis reveals 12 small cyst of 3-6 mm in diameter in the left and 20 cysts of about the same size in the right ovary. A sperm analysis of the partner is normal. Which one of the following is the most appropriate next step in management?

A. Metformin.
B. Clomiphene citrate.
C. Ovulation induction with gonadotropins.
D. Laparoscopic ovarian drilling.
E. In-vitro fertilization.

A

B. Clomiphene citrate

The findings on ultrasonography are suggestive of polycystic ovarian syndrome (PCOS). PCOS is the most common cause of infertility due to anovulation. Infertility in women with PCOS, however, is not absolute and many women can conceive even without treatment. In women with infertility due to PCOS, different options are available:

Non-pharmacological treatment:

If a woman is younger than 35 years of age and has a BMI>25, and no other cause of infertility is suspected an intensive lifestyle program addressing weight loss, without any pharmacological treatment for the first 6 months, is recommended. Small amounts of weight loss (~5%) may restore menstrual cycle regularity and ovulation, providing benefit even if pharmacological intervention is subsequently required

`Pharmacological treatment:

1-If pharmacological treatment is required, the best first-line treatment is clomiphene citrate, which has a pregnancy rate of 30–50% after six ovulatory cycles.

2-In women with a BMI <30–32 kg/m2, metformin may have a similar efficacy to clomiphene citrate, and is the first-line treatment (with or without clomiphene citrate) if there is concomitant impaired glucose tolerance).`

3-If clomiphene citrate, metformin or a combination of the two is unsuccessful in achieving pregnancy, gonadotropins are the next pharmacological options.

4-Laparoscopy with ovarian surgery/drilling (LOS) is an appropriate second-line treatment if clomiphene citrate with metformin has failed. The pregnancy rate with LOS is as effective as 3-6 cycles of gonadotropin ovulation induction.

5-If all of the above are unsuccessful or if there are other factors contributing to infertility such as endometriosis or male factors, in vitro fertilization or intra-cytoplasmic sperm injection is recommended.

*http://www.racgp.org.au/afp/2012/october/polycysti
* Therapeutic Guidelines – Endocrinology; available from http://tg.org.au

How well did you know this?
1
Not at all
2
3
4
5
Perfectly
8
Q

A 30-year-old pregnant woman presents to the Emergency Department with severe right-sided throbbing head ache, nausea, and vomiting. She is 24 weeks pregnant. Her medical history is remarkable for migraine. Which one of the following is the most appropriate management of this patient?

A. Paracetamol.
B. Paracetamol and codeine.
C. Codeine and metoclopramide.
D. Codeine and promethazine.
E. Sumatriptan.

A

C. Codeine and metoclopramide

This woman is suffering from a migraine attack associate with nauseas and vomiting. In pregnant women with migraine paracetamol is the treatment of choice for mild attacks. For more severe attacks, codeine alone or in combination with paracetamol can be used.

Codeine is not shown to lead to miscarriage or have teratogenic effects on fetus; however, long-term use of opiates can cause withdrawal (abstinence) syndrome in the neonate.

Because paracetamol alone or with codeine would not stop vomiting, an antiemetic should be added as well to control the nausea and vomiting. Metoclopramide is the antiemetic of choice during pregnancy (category A). Promethazine is category C and should be avoided.

TOPIC REVIEW

A step-wise approach to management of an acute migraine attack in a pregnant woman is as follows:

  1. Paracetamol (first-line) - For acute treatment, paracetamol is safe but often inadequate to control the symptoms. Migraine that does not respond to paracetamol alone may be relieved with combination therapy such as paracetamol (650 to 1000 mg) and metoclopramide (10 mg); paracetamol-codeine.

For migraine with nausea and vomiting, metoclopramide (category A) can be safely added. Prochlorperazine can be used as an alternative to metoclopramide.

Women with migraine that has not responded to these drugs after several days should be evaluated for provoking factors and treated more aggressively with the following medications in a step-wise approach.

  1. NSAIDs and aspirin (second-line) - NSAIDs are second-line options, and safest in the second trimester. In the first trimester, an association with miscarriage and some birth defects (ventricular septal defect, gastroschisis) has been suggested. In the third trimester, their use should be limited to fewer than 48 hours due to concerns about premature ductus arteriosus closure, platelet inhibition, and oligohydramnios.
  2. Opioids (third-line) - Opiates (e.g., oxycodone, meperidine, morphine, etc.) can be given by rectal, intravenous, or intramuscular administration.

Opioids can be useful for treatment in women with nausea and vomiting.

  1. Triptans (fourth-line) - For moderate to severe symptoms in patients who do not respond to other drugs, triptans can be considered.

NOTE - Ergotamine is absolutely contraindicated throughout pregnancy.

Of the options, codeine plus metoclopramide is the most appropriate one for a severe headache and nausea of this woman.

  • AAFP - Treatment of Acute Migraine Headache
  • UpToDate - Headache in pregnant and postpartum women * Australian Prescriber
  • Therapeutic Guidelines – Neurology
How well did you know this?
1
Not at all
2
3
4
5
Perfectly
9
Q

A 35-year-old woman pregnant woman presents to the Emergency Department with left-sided retro-orbital and occipital severe headache associated with nausea and vomiting. She has been suffering from migraine for the past 10 years, and has been on treatment with sumatriptan. Which one of the following drugs if used for treatment of migraine would lead to premature closure of fetal ductus arteriosus?

A. Codeine.
B. Paracetamol.
C. Metoclopramide.
D. Non-steroidal anti-inflammatory drugs (NSAIDs).
E. Sumatriptan.

A

D. Non-steroidal anti-inflammatory drugs (NSAIDs)

For treatment of migraine in a pregnant woman, NSAIDs should be used with caution and only if paracetamol with or without codeine/metoclopramide fails to control the pain. If NSAIDs are indicated, they should be used not more than 48 hours, and not in late pregnancy, because they are associated with premature closure of fetal ductus arteriosus. Aspirin has the same effect and should be avoided as well.

Other effects of NSAIDs on fetus include:
-Delayed labor and birth
-Oligohydramnios via decreasing the glomerular filtration rate in the fetus

Other mentioned drugs are not associated with premature closure of ductus arteriosus.

  • UpToDate- Headache in pregnant and postpartum women
  • Australian Prescriber - Treatment of nausea and vomiting in pregnancy
How well did you know this?
1
Not at all
2
3
4
5
Perfectly
10
Q

A pregnant woman has come to you because two days after babysitting her friend’s son, he was diagnosed with infectious mononucleosis. She is worried about herself and her baby’s well-being. Currently, she is asymptomatic. Which one of the following will be the most appropriate action for now?

A. Order an ultrasound for assessment of fetal hydrops.
B. Check serology for antibody titres.
C. Reassure her.
D. Tell her to come back if any symptoms develop.
E. Refer her to specialist.

A

D. Tell her to come back if any symptoms develop

Epstein-Barr virus (EBV) is a human herpes virus with variable incubation period that may cause infectious mononucleosis. This virus can remain latent in the body and become reactivated at a later time.

In Australia, EBV is more common among women aged 15 and 19 years, but in developing countries it is more common among children. The route of transmission is sharing oral secretions (saliva). EBV has an incubation period of 2-7 weeks after exposure.

Clinical features of EBV include:
-Fever
-Sore throat
-Lymphadenopathy
-Characteristic increase in the percentages of monocytes and lymphocytes (mononucleosis and lymphocytosis)
-Hepatosplenomegaly
-Rise in hepatic transaminases

Of all pregnant women, only 3.0% to 3.4% are susceptible and of those infected, only 50% develop clinical disease.

In several recent studies, EBV infection was not transmitted to the fetus and there were no adverse effects. The risk of intrauterine transmission of EBV infection is considered to be low, even when the mother is symptomatic; however, reactivation of EBV in pregnancy may carry a small risk of a shortened pregnancy duration and lower birth weight.

Recent primary EBV infection is diagnosed by the presence of viral capsid antigens (VCA) IgG and IgM antibodies in the absence of antibodies to EBV-associated nuclear antigen which develop 3 to 4 weeks after primary infection.

It is recommended that serology for IgG and IgM antibodies against viral capsid antigens (VCA) be obtained soon after symptoms of infection. About 80% of those infected form antibodies to early antigens, which usually fall to undetectable levels by six months afterwards. The presence of antibodies against early antigens at later times after acute infection indicates possible viral reactivation.

Management is supportive with rest, fluids and analgesia if required. Most pregnant women will have a gradual, uneventful recovery after an acute phase lasting several days to 3 or 4 weeks. Fever usually resolves within two weeks. Abnormal liver transaminases occur in about 10 % of cases. Nausea, anorexia and possibly vomiting can be expected. Significant organomegaly usually resolves within 1 to 3 months. Recovery from severe fatigue may occur quickly, however a full recovery to a feeling of wellbeing may take several months.

This woman has a 3.0 – 3.4% risk of infection with EBV. Even if she shows clinical infection, the potential risks to the fetus are negligible, and she can be safely reassured that no harm from EBV threatens her pregnancy. She, however, may have become infected and show clinical infection, in that case further management (including serology) would be required. For this reason she should be warned that the risk of developing the disease is small yet possible and she should come back for further management if any symptom develops.

How well did you know this?
1
Not at all
2
3
4
5
Perfectly
11
Q

Yasmin®, containing drospirenone 3mg and ethinylestradiol 30 mcg, has been released for use in Australia. Which one of the following has made it the preferred oral contraceptive pill among Australian women?

A. It has not weight gain as an adverse effect and maybe associated with weight loss.
B. It has a failure rate of less than other OCPs.
C. It has a protective effect against cervical cancer.
D. It causes less spotting even at the very beginning of use.
E. It has a good effect on acne.

A

A. It has not weight gain as an adverse effect and maybe associated with weight loss

Yasmin® has been shown to be associated with less fluid retention and weight gain as one of the complication of COCs, and this the main reason Yasmin® is preferred by most women suffering from this adverse effect. Unlike older progestogens, drospirenone is associated with no increase in weight or eve slight weight loss due to its anti-mineralocorticoid effects.

OPTION B : The failure rate Yasmin® is about the same as other COCs. There is no study showing that use of Yasmin® is associated with less incidence of cervical cancer as a long-term adverse effect of COCs (option C) . As with all COCs, Yasmin can cause irregular bleeding and spotting within the first few months of use (option D).

OPTION E : The progesterone component – drospirenone has antiadorgenic effects and is slightly more effective in treatment of acne, but compared to other COCs, the difference is not significant enough to make it preferable in terms of acne treatment of prevention.

  • AMC Handbook of Multiple Choice Questions – page 533
    *http://www.ncbi.nlm.nih.gov/pubmed/16203653
    *http://www.uptodate.com/contents/hormonal-therapy-
How well did you know this?
1
Not at all
2
3
4
5
Perfectly
12
Q

A 29-year-old obese woman comes to you for prescription of oral contraceptive pill (OCP). She weighs 115kg and has a BMI of 35. She also has hirsutism and acne. She mentions that she has migraine headaches at occasions, associated with pins and needles in her left arm. Which one of the following would be the most appropriate contraception method for her?

A. Condoms.
B. Implanon®.
C. An OCP containing ethinyl estradiol and norgestrel.
D. An OCP containing ethinyl estradiol and cyproterone acetate.
E. An OCP containing ethinylestradiol and drospirenone.

A

A. Condoms

This woman has classic migraine associated with focal neurological findings. In such patients use of OCP preparation of any kind containing estrogen is absolutely contraindicated. Progesterone has androgenic effects such as hirsutism, acne and weight gain. For a woman of her weight, progesterone of any kind (norgestrel, drospirenone, cyproterone, etc) is better avoided; therefore, a barrier method such as male condoms will be the most appropriate advice.

When choosing a combined oral contraceptive pill (COCP), it is recommended that preparations containing 20-30 mcg ethinylestradiol be considered first. The progesterone component can be norgestrel, drospirenone, cyproterone, etc; however, the preparations containing norgestrel are cheaper and more affordable for patients.

Considerations should be given to the progesterone type for particular patients:

i) Patients with bothersome fluid retention and weight gain as a side effect of COCPs can be prescribed preparations containing drospirenone (Yaz®, Yasmin®). Drospirenone has anti-mineralocorticoid activity and does not lead to fluid retention. It may even be associated with slight weight loss.
ii) If the patient has probable polycystic ovarian syndrome (PCOS), a preparation containing cyproterone acetate is preferred.

TOPIC REVIEW

Combination oral contraceptives (COCs) with antiandrogenic progestins are a subclass of COCs. These include agents that contain cyproterone acetate, drospirenone, or dienogest plus an estrogen (ethinylestradiol).

Yasmin ® (ethinylestradiol 30mcg/drospirenone 3 mg) and Yaz ® (ethinylestradiol 20mcg/drospirenone 3mg) are the two drospirenone-containing oral contraceptives. These two COCs are shown in studies to be effective in treatment of acne and other androgenic effects such as hirsutism, PCOS, etc; however, they are only slightly superior to other COCs. The effective component of COCs against acne is estradiol and all low-dose COCs are estrogen dominant, which effectively makes all of these agents antiandrogenic and effective.

Some studies, however, suggest that their use is associated with a 2- to 3-time increase in venous thromboembolic events. Although their use is only contraindicated in the presence of active venous thromboembolism (e.g. DVT, PE, etc)

  • AMC Handbook of Multiple Choice Questions – page 533
    *http://www.uptodate.com/contents/hormonal-therapy-
    *http://www.fpnotebook.com/mobile/gyn/pharm/Ysmn.ht
How well did you know this?
1
Not at all
2
3
4
5
Perfectly
13
Q

A 36-year-old woman presents to your clinic for advice regarding diabetes mellitus. She has 3 children, and was diagnosed with gestational diabetes mellitus in her second pregnancy at the age of 32 years. Which one of the following tests would be the most appropriate screening test for her?

A. Fasting blood sugar (FBS), 3yearly.
B. Oral glucose tolerance test (OGTT), 2 yearly.
C. OGTT, yearly.
D. HbA1C, now.
E. FBS, 2 yearly.

A

D. HbA1C, now

All women diagnosed with GDM should have a 75 g OGTT at 6-12 weeks postpartum. Additionally and due to the fact that women with GDM have a 50% risk of developing type 2 DM within 20 years, they need to be tested for DM. Based on current guidelines by the Royal Australian College of General Practitioners (RACGP) all women with GDM should undergo a fasting OGTT with 75 g glucose at weeks 6-12 postpartum and fasting blood sugar (FBS) or glycated hemoglobin (HbA1C) every 3 years thereafter. Since this woman has not been screened for diabetes until now, the most important step would be ordering an FBS or HbA1c now.

NOTE - Different guidelines mention different intervals for DM screening in women with history of GDM. For example Therapeutic Guidelines and Australian Diabetes in Pregnancy Society, recommend 2-yearly 75 g OGTT as the screening test of choice. We have chosen the correct answer based on RACGP guidelines which are important AMC MCQ references for the AMC MCQ exam.

  • RACGP - Gestational diabetes mellitus
  • Therapeutic Guidelines
How well did you know this?
1
Not at all
2
3
4
5
Perfectly
14
Q

A 28-year-old primigravida woman presents at 24 weeks gestation after she noticed vaginal bleeding of approximately 50ml. Last week, she also had a 10ml vaginal bleeding, for which ultrasonography was performed revealing placenta previa grade IV. Which one of the following would be the most appropriate next step in management?

A. Transvaginal ultrasonography.
B. Blood group and cross match.
C. Anti-D (RhoGAM).
D. Abdominal ultrasonography.
E. Induction of labor.

A

B. Blood group and cross match

The scenario describes vaginal bleeding in the second half of the pregnancy (antepartum hemorrhage), most likely caused by the previously diagnosed grade IV placenta previa.

The usual history includes a first episode of vaginal bleeding occurring at home. Bleeding is not often heavy. This can be followed by more episodes of bleeding.

The first priority in cases presenting with antepartum hemorrhage is assessment of vital signs, estimation of the blood loss (both clinically and by using laboratory tests), determination of blood group and cross matching.

Even an asymptomatic currently-stable placenta previa can lead to massive bleeding at anytime. This is even more likely with higher grades of placenta previa, or if the first episode of bleeding occurred prior to 30 weeks’ gestation.

The abdomen should be palpated for any uterine tenderness, contraction and fetal presentation. An abdominal ultrasonography is needed to confirm the diagnosis as well as gathering additional information regarding the situation.

Fetal well-being should be assessed by cardiotocography, and corticosteroids administered (if indicated) to promote fetal lung maturity because the feared risk of premature labor.

No vaginal examination should be made before placenta previa is excluded by ultrasound examination, because this can lead to torrential bleeding. Transvaginal ultrasonography is more accurate than transabdominal and first choice of investigation if indicated. The probe is entered into the vaginal to the extent and in an angle that prevents the probe from reaching the cervical os and is safe to perform in experienced hands.

Anti-D (RhoGAM) should be administered, but not as the first priority.

TOPIC REVIEW

Placenta previa is implantation of placenta, either partially or wholly, in the lower uterine segment below (previa) the fetal presenting part. Placenta previa is classified as:

*Grade I – placenta is in lower segment, but the lower edge does not reach the internal os.
*Grade II – Lower edge of placenta reaches the internal os, but does not cover it.
*Grade III – Placenta covers the internal os partially.
*Grade IV – placenta covers the internal os completely

Placenta previa occurs is 0.5-1.0% of pregnancies and accounts for 20% of all cases of antepartum hemorrhage. It is three times more common in multiparous women.

Risk factors for placenta previa include:
-Large placental area e.g. multiple pregnancy
-Advanced age
-High parity
-Deficient endometrium due to pre-existent uterine scar (e.g. previous cesarean section) -Endometritis
-Manual removal of placenta
-Curettage (especially for miscarriage or termination of pregnancy)
-Submucous fibroids

NOTE - if a routine ultrasound examination is made at 18 weeks, the report may show that there is a low-lying placenta, but in over 85% of cases, the placenta will be normally situated by the time of delivery, as the lower uterine segment does not develop fully until late in the third trimester. Another ultrasound examination should be performed at about the 34th week, or earlier if vaginal bleeding occurs.

Presentation

Most cases of placenta previa present with painless bright red vaginal bleeding of different amount without uterine contractions. Rarely, uterine contraction and tenderness may be present.

In approximately one-third of affected pregnancies, the initial bleeding episode occurs prior to 30 weeks of gestation; this group is more likely to require blood transfusions and is at greater risk of preterm delivery and perinatal mortality than women whose bleeding begins later in the course of pregnancy. An additional one-third of patients become symptomatic between 30 and 36 weeks, while most of the remaining patients have their first bleed after 36 weeks. About 10% of women reach term without bleeding.

For an individual patient, it is not possible to predict whether a bleed will occur, nor the gestational age, volume, or frequency of bleeding. However, higher grades of placenta previa bleed earlier and more compared with lower grades. There is often fetal malpresentation or usually high and mobile fetal presenting part.

Management :
1. Check vital signs, establish IV access and start fluids (if indicated)
2. Cross-matched blood and blood products should be readily available in anticipation of massive hemorrhage.
3. Gentle abdominal palpation to see is uterine tenderness/contractions are present, and to estimate the gestational age as well as the fetal presenting part.
4. Ultrasonography to confirm the diagnosis
5. Assessment of fetal well-being
6. Anti-D (RhoGAM) if indicated
7. Corticosteroids (if indicated)
8. Decide for outpatient versus inpatient management.

Outpatient management maybe considered:
If the patient is stable and there is no current hemorrhage, the patient can be managed in outpatient setting:
Explain the frequency and severity of recurrent bleeding is unpredictable and carries the risk of fetal and maternal complications Advise the woman to seek immediate hospital care if contractions or vaginal bleeding occurs
Ensure emergency transport access to hospital
Admit if active bleeding
If inpatient care is indicated:
Cesarean delivery should be considered where there is maternal or fetal instability despite vigorous management, regardless of gestational age; otherwise, cesarean delivery at 37 weeks should be performed. Vaginal delivery may be considered in low-risk women with low-grade placenta praevia.

*http://www.sahealth.sa.gov.au/wps/wcm/connect/b1c6
* Llewellyn-Jones – Fundamentals of Obstetrics and Gynaecology – Elsevier Mosby – 9th Edition
* Royal College of Obstetrics and Gynaecology – Green-top Guideline No.27

How well did you know this?
1
Not at all
2
3
4
5
Perfectly
15
Q

Which one of the following is unlikely to predispose to postpartum hemorrhage?

A. Uterine fibroids.
B. Multiple gestation.
C. Von Wille brand disease of the mother.
D. Oligohydramnios.
E. Prolonged labor.

A

D. Oligohydramnios

Predisposing factors to postpartum hemorrhage include:

-Uterine fibroids
-Multiple pregnancy
-Polyhydramnios
-Maternal history of bleeding disorders
-Prolonged labor
-Retained placenta
-Instrumental delivery
-Pregnancy-induced hypertension and pre-eclampsia
-Past history of postpartum hemorrhage
-Multiparity
-Obesity

Oligohydramnios does not increase the risk of postpartum hemorrhage.

How well did you know this?
1
Not at all
2
3
4
5
Perfectly
16
Q

A 28-year-old woman, who is 22 weeks pregnant in her second pregnancy, presents for evaluation of a vulval ulcer. The swab taken confirms the diagnosis of herpes simplex type II (HSV-II) infection. When she is informed, she becomes quite surprised as neither she, nor hes husband has ever had this infection before and insists to know the source of the infection. Furthermore, she is very concerned about her baby’s wellbeing and asks whether her condition may affect it. Which one of the following is the most appropriate advice in this regard?

A. Most of neonates with neonatal herpes present with mucocutaneous lesions.
B. Although treatment with antiviral agents such as aciclovir will reduce the rate recurrence of the disease, they cannot be used during pregnancy because of their adverse effects on the fetus.
C. The primary infection is commonly asymptomatic.
D. Unless she has had a new sexual partner recently, this problem could not have been acquired sexually.
E. The risk of neonatal herpes is much higher with recurrent maternal infection compared with primary infection.

A

C. The primary infection is commonly asymptomatic

Unlike what is usually thought, genital herpes infection is asymptomatic in 75% of the cases. Therefore, this woman may have contracted the infection from his asymptomatic husband.

Herpes simplex virus type II exclusively passes on through skin-to-skin contact during sexual activity; therefore having a sexual partner infected with HSV-II is essential for contracting the disease. Herpes simplex virus type I, on the other hand can be transmitted from non-sexual skin contacts e.g. kissing.

With herpetic infection during pregnancy, antiviral therapy can be considered. Studies have shown that aciclovir is the safest to use in pregnancy.

The risk of fetal infection is significantly higher in primary infection, because in recurrences the fetus has already received IgG against HSV, passively from the mother. It should be born mind that neonatal infection with recurrent infection is also possible, but far less likely compared with primary infection. If primary HSV is contracted before 30 weeks, the risk of shedding HSV during a normal birth is 7 % with an overall risk of ≤ 3 % for neonatal HSV disease.

*https://www.sahealth.sa.gov.au/wps/wcm/connect/91b
* Australian society for infectious diseases – Management of Perinatal Infections (2014)

How well did you know this?
1
Not at all
2
3
4
5
Perfectly
17
Q

Which one of the following is not helpful in prevention of transmission of HIV infection from a pregnant woman to her baby?

A. Maternal antiretroviral therapy.
B. Peripartum intravenous zidovudine.
C. Elective cesarean section.
D. Neonatal antiretroviral treatment.
E. Breastfeeding.

A

E. Breastfeeding

Interventions to prevent perinatal transmission of HIV include:
-Maternal antiretroviral therapy
-Peripartum intravenous zidovudine
-Elective cesarean section
-Neonatal antiretroviral treatment
-Bottle feeding

Breast feeding is contraindicated in HIV positive mothers because of increased chance of transmission of HIV infection.

Without intervention, the risk per cent of HIV transmission to the fetus is approximately 20-30%. With intervention this rate the risk decrease to less than 2%.

*https://www.sahealth.sa.gov.au/wps/wcm/connect/72e

How well did you know this?
1
Not at all
2
3
4
5
Perfectly
18
Q

A 27-year-old woman, diagnosed with gestational diabetes at 28 weeks pregnancy, has been being managed at a high-risk pregnancy clinic until 38 weeks when she vaginally deliveres a healthy 4-kg baby without any complications. Which one of the following is correct regarding follow-up?

A. 75g oral glucose tolerance test performed between weeks 6 and 12 after delivery.
B. Check fasting blood glucose level in 6 months after delivery.
C. HbA1C as soon as possible.
D. Fasting lipids.
E. No further action is required.

A

A. 75g oral glucose tolerance test performed between weeks 6 and 12 after delivery

Less than 10% of women with GDM remain hyperglycemic after delivery. The management of these women requires ongoing care from a diabetes or medical clinic in collaboration with their general practitioner.

The following are recommendations for women with gestational diabetes after delivery:

-Checking a random blood glucose level the day after delivery
-4-point blood glucose level (BGL) measurement on the day prior to discharge (fasting and two hours post meals for three meals)
-Cease blood glucose monitoring if BGL is within normal range.
-If there is elevated BGL (above 10 mmol/L) contact the medical registrar / diabetes educator – if BGLs are considerably elevated, after-hours contacts are justified.
-It is recommended that women who have had GDM visit their GP for a follow-up oral glucose (75 gr) tolerance test at 6-12 weeks postpartum, and every 1-2 years thereafter. -Lifestyle counselling.
-A summary letter is provided to the woman’s GP for follow-up care.

This woman should have a random BGL the day after delivery, 4-point BGL test the day before discharge, and an oral glucose tolerance test using 75g glucose between weeks 6 and 12 after delivery.

Checking HbA1C and fasting lipids would be required if this patient is found to have type 2 diabetes mellitus.

How well did you know this?
1
Not at all
2
3
4
5
Perfectly
19
Q

A 17-year-old girl presents to your office for consultation because she has never had a period. On examination. she has normal height for her age. She also has normally developed breasts, pubic hair, axillary hair and genitalia. Which one of the following would be the most likely cause of her amenorrhea?

A. Turner syndrome.
B. Prolactinoma.
C. Absent uterus.
D. A hypothalamic lesion.
E. Congenital adrenal hyperplasia.

A

C. Absent uterus

Primary amenorrhea is defined as the absence of menses by the age of 16 in the presence of normal growth and secondary sexual characteristics, or by the age of 14 in the absence of these features. Primary amenorrhea, in the presence of normal sexual development is suggestive of abnormalities such as absent or malformed uterus or absent or septated vagina or obstructed menstrual flow. Ultrasonography is the best initial investigation to find whether uterus is present or absent.

If uterus is present, amenorrhea may be due to menstruation flow obstruction caused by conditions such as imperforated hymen or vaginal septum. If uterus is absent or abnormal, chromosomal abnormalities are the most likely cause.

In the presence of normal breast development, normal pubic and axillary hair growth, normal genitalia and normal height, hypothalamic hypogonadism (option D), pituitary lesions and chromosomal abnormalities such as Turner syndrome (option A) are very unlikely to be the cause of amenorrhea in this girl.

With prolactinoma (option B), patients are expected to have galactorrhea, headache, or visual symptoms along with menstrual abnormalities.

Congenital adrenal hyperplasia (option E) has different clinical picture with manifestations much earlier in life.

  • Medscape - Amenorrhea
  • NSW Health - Amenorrhoea
How well did you know this?
1
Not at all
2
3
4
5
Perfectly
20
Q

A 38-year-old female at 33 weeks’ gestation sustains a road traffic accident at 90 km/hour. In the emergency department and on examination, she is found to be pale, with a heart rate of 112 bpm, blood pressure of 95/55 mmHg and respiratory rate of 18 breaths per minute. Her oxygen saturation is 95% on room air. Fetal heart rate is audible at 102bpm. The uterus is tense and tender. Which one of the following is the most likely diagnosis?

A. Ruptured uterus.
B. Liver laceration.
C. Placental abruption.
D. Ruptured spleen.
E. Placenta previa.

A

C. Placental abruption

This patient has signs and symptoms consistent with clinical diagnosis of placental abruption.

Trauma in last trimester of pregnancy could be dangerous to both the mother and the fetus. Motor vehicle accidents, by force of deceleration, cause placental separation.

Placental abruption is characterized by painful, tender uterus which is often contracting. The condition leads to maternal hypovolemic hypotension and consequent fetal distress represented by fetal bradycardia and repetitive late decelerations.

The amount of vaginal bleeding is not usually an appropriate indicator to severity of placental abruption, because bleeding could be very severe or it may be concealed between uterine wall and placenta in form of a hematoma.

OPTION A : Uterine rupture is characterized by severe abdominal pain and tenderness, cessation of contractions and loss of uterine tone. It is associated with mild to moderate vaginal bleeding and fetal bradycardia or loss of heart sound. Compared to placental abruption, there uterus is less tense and tender.

OPTION B and D : Ruptured spleen and liver laceration present may justify the low blood pressure, tachycardia and fetal bradycardia but not the tense, tender and contracting uterus.

OPTION E : Placenta previa presents with sudden, painless bleeding with bright red blood. There is no uterine tenderness. This diagnosis is not consistent with the clinical picture.

How well did you know this?
1
Not at all
2
3
4
5
Perfectly
21
Q

Which one of the following is the most common cause of postpartum hemorrhage?

A. Uterine atony.
B. Laceration of genital tract.
C. Uterine rupture.
D. Uterine inversion.
E. Coagulopathy.

A

A. Uterine atony

The traditional definition of a primary postpartum hemorrhage (PPH) is a blood loss of 500 mL or more in the first 24 hours. Postpartum hemorrhage can be minor (500-1,000 mL) or major (> 1,000 mL). A major PPH can be further described as moderate (1,000-2,000 mL) or severe (> 2,000 mL).

A widely accepted definition of postpartum hemorrhage (PPH) in many institutions is a blood loss of 600 ml for vaginal delivery and 750 ml for cesarean delivery. The classification of PPH in relation to the amount of blood loss is problematic, largely due to a well-documented underestimation of blood loss. A clinically relevant alternative is a substantial fall in the hematocrit e.g. 10 %.

PPH causes include:
a) Abnormalities of uterine contraction (Tone) 70 %
b) Genital tract trauma (Trauma) 20 %
c) Retained products of conception or invasive placenta (Tissue) 10 %
d) Abnormalities of coagulation (Thrombin) < 1 %

Postpartum uterine atony is by far the most common cause of PPH.

The most common risk factors for uterine atony include:
-Multiple pregnancy
-Polyhydramnions
-Macrosomia
-Prolonged labor
-Multiparity

How well did you know this?
1
Not at all
2
3
4
5
Perfectly
22
Q

A 26-year-old primigravida presented with vaginal bleeding at 16 weeks gestation. She is Rh-negative, but her partner RH-positive. On examination, cervix is dilated and products of conception are visible. Pelvic ultrasound confirms the diagnosis of spontaneous abortion. Which one of the following would be the most appropriate advice regarding Anti-D antibody (RhoGAM)?

A. Give Anti-D at 28 weeks gestation in next pregnancy.
B. Anti-D is not indicated in this situation.
C. Genetic analysis should be performed before making any decision.
D. Give anti-D now.
E. Advice that Anti-D antibodies would develop within 7 days.

A

D. Give anti-D now

Rhesus (Rh) negative women who deliver an Rh positive baby or who are otherwise exposed to Rh positive red blood cells are at risk of developing anti-Rh antibodies (RhD) and should receive RhD antibody (RhoGAM®) in current pregnancy.

Rh positive fetuses/neonates of these mothers are at risk of developing hemolytic disease of the fetus and newborn, which can be lethal or associated with serious morbidity.

TOPIC REVIEW

An Rh-negative mother has no antibodies against Rh (D) antigen. If she is pregnant and the fetus is Rh-positive any mixing up the fetus’ blood to the mother’s will trigger an immune response by lymphocytes and will lead to production of anti-Rh antibodies in the mother’s blood (anti-D antibody). RhoGAM® is anti-D IgG passive antibody that will eliminate the D-antigen from the mother’s blood, before the mother’s immune system start to sensitize. It is administered intramuscularly (IM).

RhoGAM is available is two forms:
1. CLS-250 IU – one dose contains 50μg of anti-D antibody
2. CLS-265 IU – one dose contains 125μg of anti-D antibody

Each 300μg of anti-D antibody neutralizes 15 mL of fetal packed red cells (30 mL of whole blood)

RhoGAM should be administered within the first 72 hours after the precipitating even, however, with much less success rate it can be given up to 9-10 days.

INDICATIONS

  1. First trimester indications (up to and including 12 weeks)
    -Spontaneous abortion (complete, incomplete, or missed abortion) probably excluding threatened abortion
    -Induced abortion (medically or by D&C)
    -Ectopic pregnancy
    -Chorionic villous sampling (this procedure is performed at 10-12 weeks)
    -Molar pregnancy – chorionic villi may contain D-antigen

MANAGEMENT - in the first 12 weeks the maximum amount of fetal blood that can mix with mother’s is 2.5 mL of RBC (5 mL whole blood). A single dose of CLS 250-IU is sufficient for neutralization of circulating fetal D-antigen. Multifetal pregnancies need extra doses. In a singleton pregnancy Kleihauer-Betki test is not indicated because the amount of fetomaternal blood mix is not significant and a single dose CLS – 250 will be enough.

  1. Second and third trimester indications
    -Spontaneous or induced abortions of all kind after 12 weeks
    -Amniocentesis (this procedure is performed > 15 weeks)
    -Cordocentesis (this procedure is performed >20 weeks)
    -Fetal blood sampling
    -Fetal death
    -External cephalic version of breech presentation (successful or unsuccessful)
    -Blunt abdominal trauma in pregnancy considered sufficient to cause fetomaternal haemorrhage
    -Antepartum haemorrhage (revealed or concealed) in the second or third trimester (e.g. placental abruption, placenta praevia, etc)

MANAGEMENT - in the second or third trimester CLS 625 (containing 125μg anti D-antibody) should be used. A Kleihauer-Betki test is indicated in the second or third trimester events to assess the need for extradoses of CLS 625 IU. Ideally the sample for the test should be collected within 15 minutes of the precipitating event, but if not possible it can be performed up to 72 hours.

*RhoGAM will remain in maternal circulation for up to 6 weeks.

**RhoGAM should not be given to women in which anti-D antibody has already formed.

Routine administration of RhoGAM - in the absence of a precipitating event, RhoGAM (CLS – 625 IU) is routinely given to all Rh-negative pregnant women at 28 weeks and 34 weeks of pregnancy.

Within the first 72 hours postpartum all R-negative women whose baby is Rh-positive should receive a dose of CLS-625 IU and undergo quantification of fetomaternal blood mix using Kleihauer-Betki test to evaluate whether extra doses of CLS-625 IU are indicated.

*http://www.australianprescriber.com/magazine/23/2/
*https://www.nhmrc.gov.au/_files_nhmrc/publications
* Royal Australian and New Zealand College of Obstetricians and Gynaecologists – College Statement C-Obs 6 (2015)

How well did you know this?
1
Not at all
2
3
4
5
Perfectly
23
Q

A 31-year-old G2P1 woman presented to the maternity unit at 38 weeks gestation and in labour. Her previous pregnancy led to caesarean section and delivery of a healthy baby. The current pregnancy had been uneventful without any remarkable problems in antenatal visits except first trimster nausea and vomiting. On arrival, she had a cervical dilation of 4 cm and the fetal head was at -1 station. After 5 hours, the cervical length and fetal head station are still the same despite regular uterine contractions. Suddenly, there is sudden gush of blood of approximately 1000 mL and the fetal heart rate drops to 80 bpm on CTG. Which one of the following could be the most likely cause?

A. Lower genital tract lacerations.
B. Placenta previa.
C. Placenta accreta.
D. Uterine atony.
E. Ruptured uterus.

A

E. Ruptured uterus

With previous caesarean section in history and prolonged active phase of the first stage of the labour, uterine rupture would be the most likely cause of the bleeding.

Maternal manifestations of uterine rupture are variable and include the following :

1- Constant abdominal pain – pain may not be present in sufficient amount, character, or location to suggest uterine rupture and pain may be partially or completely masked by regional analgesia.
2- Signs of intraabdominal hemorrhage (a strong indicator) - Although hemorrhage is common, the signs and symptoms of intraabdominal bleeding in cases of uterine rupture, especially those cases not associated with prior surgery, may be subtle.
3- Vaginal bleeding – Vaginal bleeding is not a cardinal symptom, because it may be modest, despite major intraabdominal hemorrhage.
4- Maternal tachycardia and hypotension
5- Cessation of uterine contractions
6- Loss of station of the fetal presenting part
7- Uterine tenderness

Fetal bradycardia (as seen in this case) is the most common and characteristic clinical manifestation of uterine rupture. Variable or late decelerations may precede the bradycardia, but there is no fetal heart rate pattern pathognomonic of rupture. Furthermore, fetal heart rate changes alone have low sensitivity and specificity for diagnosing uterine rupture.

Postpartum uterine rupture is characterized by pain and persistent vaginal bleeding despite use of uterotonic agents. Hematuria may occur if the rupture extends into the bladder.

NOTE - A history of previous uterine surgery (e.g. cesarean section) typically alerts the obstetrician to the possibility of uterine rupture in symptomatic women.

When uterine rupture is suspected, immediate cesarean section should be performed to save both the mother and the baby. Definite diagnosis of uterine rupture is always made after laparotomy.

*http://www.sahealth.sa.gov.au/wps/wcm/connect/0f2c
*http://www.uptodate.com/contents/rupture-of-the-un

How well did you know this?
1
Not at all
2
3
4
5
Perfectly
24
Q

A 50-year-old woman presents to your GP practice because of persistent vaginal bleeding for the last 3 days. Her last menstrual period occurred one year and a half ago. She mentions that just before the bleeding she felt reduction in hot flushes she has been experiencing for the past year. She is sexually active and has had regular pap smears with a normal one 6 months ago. Which one of the following is the most likely diagnosis?

A. Endometrial cancer.
B. Cystic glandular endometrial hyperplasia.
C. Atrophic vaginitis.
D. Cervical cancer.
E. An episode of ovarian follicular activity.

A

E. An episode of ovarian follicular activity

When menopause occurs, estrogen levels fall. Decreased estrogen levels result in increased production of follicular stimulating hormone (FSH) and very high levels of circulating FSH. At times, these high levels of FSH cause a remaining follicle to become active, resulting in menstruation. At such occasions, produced estrogen by the active follicle leads to decreased symptoms of menopause such as hot flushes, vaginal dryness, etc.

OPTION A : Endometrial cancer could be a possibility but it is very uncommon in premenopausal women or those women within the first 2-3 years of their menopause.

OPTION B : Cystic glandular hyperplasia is a result of long-term exposure of endometrium to unopposed estrogen such as in women with anovulatory cycles (e.g., in PCOS). There is no clue in history to suggest endometrial hyperplasia.

OPTION C : Atrophic vaginitis presents differently with decreased vaginal lubrication and dyspareunia, vaginal discharge or bleeding, dysuria, etc. Although not impossible, it is uncommon for vaginal atrophy to occur within the first 2-3 years of menopause.

OPTION D : With a normal cervical screening 6 months ago, the cervical cancer is very unlikely.

  • AMC Handbook of Multiple Choice Questions – page 528
How well did you know this?
1
Not at all
2
3
4
5
Perfectly
25
Q

Which one of the following is highly suggestive of polycystic ovarian syndrome (PCOS)?

A. Elevated FSH.
B. Elevated LH.
C. Elevated testosterone levels.
D. Decreased free testosterone.
E. Hyperprolactinemia.

A

C. Elevated testosterone levels

Polycystic ovarian syndrome (PCOS) is associated with the following:

  1. Clinical or biochemical hyperandrogenism. Clinical features suggestive of PCOS include hirsutism, acne, deepened voice, etc
  2. Menstrual dysfunction, irregularity or lack of ovulation.
  3. Polycystic ovaries on the ultrasound.

Only the first two mentioned features will suffice for establishing the diagnosis of PCOS, because cystic structures on ultrasound are often not seen, particularly in women who have been on treatment.

The following hormonal changes are seen in PCOS:
-Serum FSH (follicle stimulating hormone) is either normal or low.
-Elevation of LH (luteinizing hormone)
-A normal LH/FSH ratio in premenopausal women is about 1:1, while in PCOS this ration may be 2:1 or 3:1. It should be noted that approximately 30% of patients with established diagnosis of PCOS will have a LH/FSH ration of 1:1.
-Serum estrogen level is either decreased or normal. Overall, estrogen level is very nonspecific for diagnosis of PCOS.
-Serum free testosterone is the first-line investigation in patients suspected of having PCOS. Hyperandrogenism is one of the essential criteria to establish the diagnosis of PCOS.
-Prolactin level is usually normal or mildly elevated, possibly due to chronic estrogen exposure.

Of the options, an elevation of testosterone is diagnostic for PCOS.

*https://www.mja.com.au/journal/2011/195/6/assessme
*https://www.nhmrc.gov.au/_files_nhmrc/publications
*http://emedicine.medscape.com/article/256806-wor

How well did you know this?
1
Not at all
2
3
4
5
Perfectly
26
Q

A 21-year-old woman, who underwent dilation and curettage for termination of an unwanted pregnancy 4 days ago, presents to the emergency department with fever and purulent vaginal discharge. Her pregnancy was the outcome of sex with an stranger, while intoxicated in a party. Which one of the following organisms is most likely to be the cause of her problem?

A. Mycoplasma hominis.
B. E.coli.
C. Chlamydia trachomatis.
D. Chlamydia trachomatis and vaginal pathogens.
E. Neisseria gonorrhea.

A

D. Chlamydia trachomatis and vaginal pathogens

With the presence of fever and purulent vaginal discharge, this woman has inflammatory pelvic disease. The predominance of microbial etiology depends on whether the PID is sexually-acquired or not:

  1. Non-sexually acquired PID – This type is usually caused by mixed pathogens originating from vaginal flora, including anaerobes, facultative bacteria, Mycoplasma hominis and sometimes E.coli. This type of PID often occurs postpartum or following instrumentation or gynecological surgeries.
  2. Sexually acquired PID – Chlamydia trachomatis and Neisseria gonorrhea are the two most common causes of pelvic inflammatory disease (PID) in sexually active women. In Australia chalmydia is much more common.

In patients presenting for termination of pregnancy, especially those with risk factors for Chlamydia, such as not having a constant sexual partner, preoperative swabs have shown that Chlamydia infection is present in up to 15%. For this patient combination of vaginal pathogens and Chlamydia trachomatis would be the most likely etiology.

If the symptom was purulent vaginal discharge, and no fever was present, the diagnosis would have been cervicitis (and not PID). Chlamydia trachomatis is the most common cause of cervicitis. Vaginal pathogens do not cause cervicitis.

NOTE - for the exam purpose remember the following:

  • The most common cause of post-procedural PID is vaginal pathogens
  • The most common cause of post- procedural PID in women, who are high risk for STDs as well is Vaginal pathogens PLUS Chlamydia
  • The most common cause of PID, unrelated to surgical procedures is Chlamydia trachomatis


* Therapeutic Guidelines – Antibiotic; available from: http://tg.org.au * AMC Handbook of Multiple Choice Questions – page 526
*http://emedicine.medscape.com/article/256448-overv

How well did you know this?
1
Not at all
2
3
4
5
Perfectly
27
Q

A 40-year-old woman with history of tubal ligation presents with complaint of irregular heavy menstrual bleeding. She is known to have multiple uterine fibroids. Other significant point in history is dilation and curettage 6 months ago with no improvement in her symptoms, or no diagnosis for a cause. On examination, the uterus has the size of a 12-week pregnant uterus. Laboratory studies are significant for a hemoglobin level of 85 g/L. Which one of the following is the next best step in management?

A. Total abdominal hysterectomy.
B. Endometrial ablation.
C. Myomectomy.
D. Combined oral contraceptive pills.
E. NSAIDs.

A

A. Total abdominal hysterectomy

Uterine leiomyomas (fibroids) are benign tumors of uterus. Since histological confirmation of the clinical diagnosis in not necessary in most cases, asymptomatic uterine leiomyomas only require follow-up without any specific intervention.

Symptoms and complications demand intervention. Hysterectomy is the definitive treatment. Other alternatives include myomectomy with various techniques, endometrial ablation, uterine artery embolization and myolysis.

Hysterectomy is indicated in the following situations:

a) Women with acute hemorrhage who do not respond to other therapies

b) Women who have completed childbearing and have current or increased future risk of other diseases (cervical intraepithelial neoplasia, endometriosis, adenomyosis, endometrial hyperplasia, or increased risk of uterine or ovarian cancer) that would be eliminated or decreased by hysterectomy

c) Women who have failed prior minimally invasive therapy for leiomyomas

d) Women who have completed childbearing and have significant symptoms, multiple leiomyomas, and a desire for a definitive treatment

Total abdominal hysterectomy is the procedure of choice if hysterectomy is planned.

For women, who desire to preserve the ability of child bearing, a course of gonadotropin releasing hormone (GnRH) agonists, followed by myomectomy is the treatment of choice.

NOTE - Leiomyomas are the most common indication for hysterectomy, accounting for 30% of hysterectomies in white women and over 50% of hysterectomies in black women.

This woman, who does not wish to have more children and has undergone tubal ligation, and has bothersome symptoms and anemia, total abdominal hysterectomy is the most appropriate treatment option.

OPTION B : Endometrial ablation is inferior to hysterectomy. With endometrial ablation, amenorrhea is achieved; however, leiomyomas remain untreated.

OPTION C : Myomectomy is not advisable, unless further fertility is desired. There is a likelihood of recurrence that may necessitate further surgeries. Furthermore, when the uterus is enlarged with multiple leiomyomas, it will be very difficult to remove them all. Remaining leiomyomas can grow over time and produce the symptoms again.

OPTION D : Hormonal methods including combined contraceptive pills, progesterone-only method, or diazole, has been used with anecdotal effects on some symptoms such as menorrhagia. Some have controversial efficacy and some with proven efficacy have undesirable adverse effect profiles that limit their use.

OPTION E : NSAIDs have not been extensively studied in leiomyomas-related menorrhagia.
NSAIDs do not appear to reduce blood loss in women with leiomyomas, but they decrease painful menses and can be useful for this purpose.

  • UpToDate - Uterine fibroids (leiomyomas): Treatment overview
How well did you know this?
1
Not at all
2
3
4
5
Perfectly
28
Q

You are giving advice to a young couple about contraception. After explanation of different methods, they choose to use ‘rhythm method’ (calendar calculation). The woman has menstrual cycles of between 26 and 29 days. Which one of the following abstinence periods would be the correct advice?

A. From day 4 to day 17.
B. From day 6 today 17.
C. From day 8 today 19.
D. From day 9 today 19.
E. From day 12 to day 17.

A

B. From day 6 today 17

Regardless of the length of the menstrual cycle, ovulation occurs 14 days (2 weeks) before menstruation. In other words, luteal phase has always a fixed duration of 14 days; therefore, in this woman ovulation occurs on day 12 at earliest and on day 15 at latest. After ovulation, there is a window of 24-36 hours, during which the egg can be fertilzied by sperm; therefore, intercourse should not be resumed until the latest day when ovulation is possible plus an additional 48 hours (15 + 2=17).

In the presence of adequate and normal cervical mucus, sperm can survive up to 6 days. To calculate the day from which abstinence should be started, 6 should be subtracted from the earliest date of possible ovulation (12 here) (12-6=6).

The appropriate advice for this couple will be abstinence between days 6 and 17.

NOTE - calculation of the abstinence period is by the following formula:

-Beginning of the abstinence = subtract 6 days from the earliest day of the cycle on which ovulation can occur
-End of abstinence = Add 2 days to the latest day of the cycle on which ovulation may occur

For example, in a patient with menstrual cycles between 26 and 32 days:

-Possible days of ovulation: earliest=26-14=12th day of menstrual cycle; latest=32-14=18th day of menstrual cycle
-Beginning of the abstinence = 12-6=6th day of menstrual cycle
-End of abstinence =18 +2 =20th day of menstrual cycle

  • AMC Handbook of Multiple Choice Questions – pages 525-526
How well did you know this?
1
Not at all
2
3
4
5
Perfectly
29
Q

A 32-year-old woman presents with complaint of mucopurulent greenish vaginal discharge. Which one of the following organisms is the most likely cause of this presentation?

A. Candida albicans.
B. Chlamydia trachomatis.
C. Gardnerella vaginalis.
D. Mycoplasma hominis.
E. E.coli.

A

B. Chlamydia trachomatis

Of the given options, only Chlamydia trachomatis can cause a thick mucopurulent greenish yellow vaginal discharge. Trichomonas vaginalis (not an option here) can also present with thin frothy yellow-green vaginal discharge that is offensive. Neisseria gonorrhea can also present with mucopurulent vaginal discharge.

OPTION A : The discharge in candida vaginitis is thick, white and non-offensive.

OPTION C and D : Gardnerella vaginalis is the most common cause of bacterial vaginosis. Vaginal discharge of bacterial vaginosis is thin, grey-white and has an offensive (fishy) odor. Mycoplasma hominis is another organism found in bacterial vaginosis.

OPTION E : E.coli may colonize vagina but does not result in infections presenting with green vaginal discharge.

  • Australian STI management guideline - Vaginal Discharge * Medscape - Cervicitis
How well did you know this?
1
Not at all
2
3
4
5
Perfectly
30
Q

A 65-year-old woman presents to your clinic with complaints of a brownish vaginal discharge for the past 2 months. On examination, atrophic vagina is evident. Which one of the following is the most like diagnosis?

A. Vaginal atrophy.
B. Endometrial cancer.
C. Cervical cancer.
D. Chlamydia infection.
E. Endometrial hyperplasia.

A

A. Vaginal atrophy

With vaginal discharge of brownish color in a 65-year-old woman, endometrial cancer should always be the first diagnosis to exclude; however, the question asks about the most likely cause of the presentation and not the most important diagnosis to consider.

The brown color of vaginal discharge is often caused by blood. The source of bleeding can be either the uterine cavity or the vagina. Of all women with postmenopausal vaginal bleeding, only 5-10% were found to have endometrial cancer, while approximately 60% had atrophic vaginitis. Of the options, vaginal atrophy due to estrogen-deficient status of menopausal women is the most likely explanation for this presentation.

Estrogen deficiency in postmenopausal years results in urogenital atrophy.

Symptoms of urogenital atrophy include:
-Vaginal dryness
-Vaginal burning or irritation
-Decreased vaginal lubrication during sexual activity
-Dyspareunia, including vulvar or vaginal pain (at the introitus or within the vagina)
-Vulvar or vaginal bleeding (e.g. postcoital bleeding, fissures)
-Vaginal discharge (leucorrhea or yellow and malodorous)
-Pelvic pressure or a vaginal bulge
-Urinary tract symptoms (e.g. urinary frequency, dysuria, urethral discomfort, hematuria)

TOPIC REVIEW

The differential diagnoses of vaginal bleeding in postmenopausal women is less broad than that for abnormal bleeding in premenopausal women. A series of 1138 women aged 41 to 91 years with postmenopausal bleeding reported the following etiologies and prevalence:

**-Vaginal/endometrial atrophy: 50% **
-Polyps: 12%
-Endometrial cancer: 10%
-Endometrial hyperplasia: 9.8%
-Hormonal effects: 7%
-Cervical cancer: < 1%
-Other (e.g., hydrometra, pyometra, hematometra): 2%

  • Family Practice Notebook - Postmenopausal Bleeding
  • UpTodate - Post menopausal uterine bleeding
How well did you know this?
1
Not at all
2
3
4
5
Perfectly
31
Q

Which one of the following options is the most common presenting symptom in women with atrophic vaginitis?

A. Painless vaginal bleeding.
B. Endometritis.
C. Difficult intercourse.
D. Postcoital bleeding.
E. Scanty purulent discharge.

A

C. Difficult intercourse

Atrophic vaginitis, also referred to as vulvovaginal atrophy, urogenital atrophy, or vaginal atrophy, is characterized by dryness, inflammation, and thinning of the epithelial lining of the vagina and lower urinary tract due to estrogen deficiency. It typically occurs in menopausal women, but can occur in women of any age who experience a decrease in estrogenic stimulation of urogenital tissue. Premenopausal woman are in hypo-estrogenic state in postpartum period, lactation, and when on anti-estrogenic medication. In such conditions, atrophic vaginitis can also occur.

Symptoms of urogenital atrophy include:
-Vaginal dryness
-Vaginal burning or irritation
-Decreased vaginal lubrication during sexual activity
-Dyspareunia, including vulvar or vaginal pain (at the introitus or within the vagina)
-Vulvar or vaginal bleeding (e.g., postcoital bleeding , fissures)
-Vaginal discharge (leukorrhea or yellow or brown discharge that can be malodorous)
-Pelvic pressure or a vaginal bulge
-Urinary tract symptoms (e.g. urinary frequency, dysuria, urethral discomfort, hematuria)

Symptoms of vaginal atrophy are usually progressive and worsen with time. Vaginal dryness and lack of lubrication on sexual arousal is the earliest and the most common symptom of atrophic vaginitis. This is the first signs of estrogen insufficiency in urogenital system. Urinary symptoms present later as the estrogen deficiency continues.

Option A and B: Painless vaginal bleeding and endometritis are not clinical features associated with atrophic vaginitis.

  • AAFP - Diagnosis and Treatment of Atrophic Vaginitis
How well did you know this?
1
Not at all
2
3
4
5
Perfectly
32
Q

Which one of the following is the most common site of endometriosis?

A. Round ligament.
B. Uterosacral ligament.
C. Ovary.
D. Bladder.
E. Broad ligament.

A

C. Ovary

Endometriosis is defined as the presence of endometrial glands and stroma at extrauterine sites. These ectopic endometrial implants are usually located in the pelvis, but can occur nearly anywhere in the body.

The most common sites of endometriosis, in decreasing order of frequency, are:
-Ovaries
-Posterior cul-de-sac
-Broad ligament
-Uterosacral ligament
-Rectosigmoid colon
-Bladder
-Distal ureter

Less common, yet possible, sites are vagina, cervix, rectovaginal septum, caecum, ileum, inguinal canals, abdominal or perineal scars, and umbilicus. Rare sites include breast, pancreas, liver, gallbladder, kidney, urethra, extremities, vertebrae, peripheral nerves, lungs, spleen, diaphragm and central nervous system.

Ovaries are the most common site of endometriosis.

How well did you know this?
1
Not at all
2
3
4
5
Perfectly
33
Q

Which one of the following is the least likely site of endometriosis?

A. Ovaries.
B. Cervix.
C. Uterine wall.
D. Pouch of Douglas.
E. Bladder.

A

B. Cervix

Endometriosis is defined as the presence of endometrial glands and stroma at extrauterine sites. These ectopic endometrial implants are usually located in the pelvis, but can occur nearly anywhere in the body.

The most common sites of endometriosis, in decreasing order of frequency, are:
-Ovaries
-Posterior cul-de-sac (AKA Pouch of Douglas/Rectouterine Pouch)
-Broad ligament
-Uterosacral ligament
-Rectosigmoid colon
-Bladder
-Distal ureter

Less common, yet possible, sites are vagina, cervix, rectovaginal septum, caecum, ileum, inguinal canals, abdominal or perineal scars, and umbilicus. Rare sites include breast, pancreas, liver, gallbladder, kidney, urethra, extremities, vertebrae, peripheral nerves, lungs, spleen, diaphragm and central nervous system.

Ovaries are the most common site of endometriosis.

How well did you know this?
1
Not at all
2
3
4
5
Perfectly
34
Q

A 47-year-old woman presents to your clinic with heavy irregular periods every 2 to 4 months for the past 12 months. Before that, her periods were normal, occurring every 32 days. Physical examination is unremarkable. Blood tests show a hemoglobin level of 110 g/L (normal: 115-165). Which one of the following is the most likely cause of this presentation?

A. Endometrial cancer.
B. Endometrial polyps.
C. Endometriosis.
D. Anovulatory cycles.
E. Uterine fibroids.

A

D. Anovulatory cycles

By far, anovulatory cycles are the most common cause irregular heavy periods. After ovulation, the follicle turns into the progesterone-secreting corpus luteum. The progesterone stabilizes the endometrium. If pregnancy fails to take place, corpus luteum regresses and the progesterone declines. When progesterone falls, endometrial shedding and menstrual flow occurs at the predicted time.

Without ovulation, unopposed estrogen continues to cause endometrial growth. This results in unpredictable shedding of a hypertrophied endometrium and irregular, often heavy vaginal bleeding.

OPTION A : Anovulatory cycles have different etiologies, but polycystic ovary syndrome and approaching menopause are the most common causes. Endometrial cancer in premenopausal woman is not common.

OPTION B, C, and E : Endometrial polyps, endometriosis and uterine fibroids can cause heavy menstrual bleeding (menorrhagia) in a predictable fashion. Menstrual irregularities are not a common feature of these conditions.

  • Clinical Care Standards - Heavy Menstrual Bleeding Clinical Care Standard
  • AAFP - Evaluation and Management of Abnormal Uterine Bleeding in Premenopausal Women
  • UpToDate - Approach to abnormal uterine bleeding in nonpregnant reproductive-age women
How well did you know this?
1
Not at all
2
3
4
5
Perfectly
35
Q

A30-year-old woman on oral contraceptive pills (Microgynon 30®) presents to your clinic for a repeat prescription. On examination, you find out that she has a blood pressure of 160/100mmHg, confirmed by a second reading 20 minutes later. She mentions that her husband and she are planning to become pregnant in 6 months. Which one of the following is the best advice for her?

A. She can continue the same OCP but should use methyldopa for hypertension.
B. She should change to an OCP preparation with lower dose of estrogen.
C. She should stop OCP, use condoms for contraception and reassess her blood pressure in 3 months.
D. She should stop the OCP and be started on methyldopa for hypertension.
E. She should stop the OCP and be started on a diuretic or ACE inhibitor for hypertension.

A

C. She should stop OCP, use condoms for contraception and reassess her blood pressure in 3 months

Eestrogen-containing hormonal contraception methods are NOT absolutely contraindicated in hypertensive women, whether the hypertension has existed before or developed as result of the hormonal contraception use. With hypertension, the patient should be started on appropriate antihypertensive agent if she insists on using the method.

This woman, however, is willing to stop the OCP; hence, cessation of the estrogen-containing method and using an alternative is the best advice. Since this woman is planning to conceive, using condoms for contraception in the next 6 month, during which she does not want to become pregnant, would be the best advice among the options.

If OCPs are ceased, she does not need to be started on antihypertensive drugs at this stage because with cessation of estrogen, there is a good chance that her blood pressure returns to normal. Antihypertensive medications will be indicated if blood pressure remains high despite stopping the OCP.

If use of antihypertensive medications is inevitable, methyldopa should be considered as first line because it is safe to use during pregnancy.

ACE inhibitors and diuretics are contraindicated during pregnancy and should be avoided.

*http://www.aafp.org/afp/2010/0915/p621.html
*http://www.gpnotebook.co.uk/simplepage.cfm?ID=7721

How well did you know this?
1
Not at all
2
3
4
5
Perfectly
36
Q

A 51-year-old menopausal woman presents with frequent hot flushes disturbing her sleep and occupational performance. She has the history of breast cancer diagnosed 10 years ago. The tumor was positive for estrogen receptors on pathology. Which one of the following would be the most appropriate management?

A. Cyclical estrogen and progesterone.
B. Progesterone-only HRT.
C. Paroxetine.
D. Black cohoch.
E. Mefenamic acid.

A

C. Paroxetine

Hot flush is a sudden feeling of warmth that is generally most intense over the face, neck, and chest. The duration is variable but on average lasts about 3-4 minutes. It is often accompanied by sweating that can be profuse and followed by a chill. Hot flushes are seen in perimenopausal women as one of the most common complaints, and is the only indication to start hormone replacement therapy (HRT). HRT is the most effective treatment for severe hot flushes and is a reasonable choice in the absence of contraindications.

OPTION A and B : Hormonal therapy including estrogen and progesterone is avoided in patients with history of breast cancer, although the effect of progesterone on breast cancer is not established.

Women with hot flushes and contraindications to hormonal therapy may benefit from selective serotonin reuptake inhibitor (first-line) such as paroxetine, or other medications such as gabapentin or venlafaxine.

SSRI are widely used in Australia for management of hot flushes when there is contraindication to hormone therapy. It also improves mood symptoms. Paroxetine is the most common prescribed SSRI (and the only one approved by FDA in US). SSRIs are not as effective as HRT, but definitely more effective than placebo. It is important to note that paroxetine, fluoxetine, duloxetine, and bupropion should not be used in patients who are currently on tamoxifen because it interferes with the conversion of tamoxifen to its active metabolite endoxifen.

OPTION D : Black Cohosh acts by unknown mechanism and is used for hot flushes. Many studies have found no significant benefit over placebo.

OPTION E : Mefenamic acid may help for short-term control of menopausal symptoms; however, its long term use is controversial and not recommended.

How well did you know this?
1
Not at all
2
3
4
5
Perfectly
37
Q

A 28-year old woman presents with her husband complaining of inability to conceive after 18 months of regular sexual activity. During the discussion on an etiology, further work-up and the treatment, she asks about the possible outcome for treatment of different etiologies. Which one of the following conditions has the most favorable treatment outcome?

A. Azoospermia.
B. Tubal obstruction.
C. Polycystic ovarian syndrome.
D. Pelvic tuberculosis.
E. Turner syndrome.

A

C. Polycystic ovarian syndrome

Treatment of infertility due to ovulation problems has had the most satisfactory outcomes by far. Polycystic ovarian syndrome (PCOS), also known as Stein-Leventhal syndrome is characterized by hyperandrogenism, small cysts in ovaries, acne and hirsutism as well as problems of ovulation leading to anovulatory cycles due to hormonal imbalance.

70% of females with anovulatory cycles start ovulation on medical management and of those, 50% conceive within 6 to 9 months.

OPTION A : Low or zero sperm counts, poor sperm motility and dysmorphic sperms can all cause infertility. Fertility drugs for promotion of sperm production, artificial insemination with donor sperm and intracytoplasmic injection of sperm can be used as treatment modalities, but success rate is low (less than 25%).

OPTION B and D : Obstruction of or damage to the Fallopian tubes follows pelvic inflammatory disease, prior surgery, or tuberculosis and prevents the sperm from reaching to the ovum. Laparoscopic surgery is used to treat tubal obstructions or adhesions. Success rates of conception vary widely from a minimum of 10% to a maximum of 70% depending on the severity of the blockage.

OPTION E : Turner syndrome is associated with ovarian dysgenesis and infertility. There is no option for treatment of gonadal dysgenesis and reversal of infertility. Success rate is very low even after egg donation due to high rate of miscarriages.

*http://www.ncbi.nlm.nih.gov/pmc/articles/PMC270066 *http://emedicine.medscape.com/article/27

How well did you know this?
1
Not at all
2
3
4
5
Perfectly
38
Q

A 28-year-old unmarried woman presents to your clinic with slight left-sided lower abdominal pain for the past 2 days. On examination, she has a blood pressure of 125/90 mmHg, pulse rate of 90 bpm and temperature of 37.3°C. She is otherwise healthy. Abdominal exam elicits no tenderness, rebound or guarding. Ultrasonographic assessment shows a 6 cm solid mass lateral to the uterus on the left side. Which one of the following is the most likely diagnosis?

A. Mucinous cystadenoma.
B. Ovarian teratoma.
C. Endometriosis.
D. Corpus luteum cyst.
E. Ovarian cancer.

A

B. Ovarian teratoma

Ovarian cystic teratomas and ovarian dermoid cysts are encapsulated tumors with mature tissue or organ component. They are composed of well-differentiated derivations from at least two of the three germ cell layers, namely ectoderm, mesoderm, and endoderm; therefore, they may contain developmentally mature skin components such as hair follicles, sweat glands, pockets of sebum, blood, fat, bone, nail, teeth, eyes, cartilage, thyroid tissue, etc.

Real organoid structures such as teeth, fragments of bone, etc. may be present in approximately 30% of cases.

The diameter of an ovarian teratoma is typically less than 10 cm and very rarely more than 15 cm.

Uncomplicated ovarian dermoids are often asymptomatic and discovered incidentally. There is, however, an increased risk of ovarian torsion that may present with acute pelvic pain.

Ultrasound shows a solid mass in the adnexal (next to the uterus). Teratomas are common in young women around the age of 30 years and the most common pelvic tumors in women younger than 20 years.

OPTION A : Mucinoid cystadenoma, as the name implies, has cystic nature and is not solid. This tumor is at the benign end of the spectrum of mucin-containing epithelial ovarian tumors. On ultrasonography, large cystic adnexal mass that is multilocular with numerous septations is seen.

OPTION C : Ovarian endometriosis has a different appearance on ultrasonography including the acoustic enhancement of ovaries with homogenous low-level internal echoes as a result of the hemorrhagic debris. On the other hand, the predominant feature of endometriosis is dysmenorrhea.

OPTION D : Corpus luteal cyst is a type of functional ovarian cyst that forms when a corpus luteum does not regress, either normally due to pregnancy or without the pregnancy taking place. Corpus luteum cyst is the most common pelvic mass within the first trimester. When corpus luteal cysts are associated with pregnancy, most of them will involute by the end of the second trimester.

On ultrasonography, they might have a range of sonographic appearances depending on the stage of evolution and the age of the associated intracystic hemorrhage, but general characteristics are as follows:
-Diffusely thick wall
-Peripheral vascularity
-Diameter less than 3 cm (with the wall)

OPTION E : Although ovarian cancer is seen as an adnexal solid tumor, it is a remote possibility in a 28-year-old woman.

(PICTURES OF ULTRASOUNDS of TERATOMA, MUCINOID CYSTADENOMA, CORUS LUTEUM CYST in PAGE 934-935)

How well did you know this?
1
Not at all
2
3
4
5
Perfectly
39
Q

A 26-year-old woman presents to your clinic on the 6th day of her menstrual cycle with mild lower abdominal pain. She lives with her male partner and has no children. On examination, she has a blood pressure of 110/70 mmHg, pulse rate of 90bpm, and temperature of 37.5°C. No adnexal mass is palpated; however, on vaginal exam cervical motion tenderness is noted. Which one of the following would be the next best step in management?

A. Transvaginal ultrasonography.
B. Cervical swabs for culture.
C. Urine analysis and culture.
D. Thyroid stimulating hormone.
E. Abdominal CT scan.

A

B. Cervical swabs for culture

Cervical motion tenderness (also called cervical excitation or chandelier sign) suggests a pelvic pathology. Classically, it is present in pelvic inflammatory disease (PID) and ectopic pregnancy (EP). Sometimes the test is used to differentiate PID from appendicitis.

This patient is menstruating and less likely to have an EP; therefore, PID remains the most likely diagnosis. EP is less likely.

Because of the relatively poor specificity and sensitivity of clinical findings in PID, the Center for Disease Control and (CDC) has established minimal criteria for diagnosis of PID.

In a female patient PID is the diagnosis if: -She is at risk for sexually transmitted disease (STD) AND -She has lower abdominal or pelvic pain without no other identifiable cause for her illness other than PID, AND -on pelvic examination there is (1) cervical motion tenderness or (2) uterine tenderness or (3) adnexal tenderness

In such patients, based on diagnosis of PID, empiric antibiotics should be started after cervical swabs are taken for culture and DNA probe.

Since EP is an important differential diagnosis in a patient with cervical motion tenderness, EP should be excluded. This is correct that in the presence of normal menstruation EP is very unlikely, but excluding pregnancy would be a good choice if it was among the options.

Ultrasonography is the next best step in management if ectopic pregnancy is suspected based on history, clinical findings and a positive pregnancy test.

*http://emedicine.medscape.com/article/256448-diffe
*http://emedicine.medscape.com/article/256448-worku

How well did you know this?
1
Not at all
2
3
4
5
Perfectly
40
Q

A 31-year-old woman comes to your clinic complaining of unpredictable vaginal bleeding for the past 6 months. She also mentions that recently the amount of bleeding has increased. Which one of the following is the most appropriate next step in management?

A. Full blood exam (FBE).
B. Transvaginal ultrasonography.
C. Serum beta HCG.
D. Thyroid stimulating hormone (TSH).
E. Endometrial sampling.

A

A. Full blood exam (FBE)

Abnormal uterine bleeding (AUB) affects 9-14% of women. AUB generally can be divided into anovulatory and ovulatory patterns. In ovulatory pattern, the bleeding is regular but heavy, whereas in anovulatory pattern the bleeding is irregular and unpredictable.

Ovulatory pattern is usually caused by uterine problems such as leiomyomas, endometriosis, adenomyosis, polyps, etc. Anovulatory pattern is a result of hormonal problems such as polycystic ovarian syndrome (PCOS), hypothyroidism, hyperthyroidism, hyperprolactinemia, and Cushing syndrome.

The approach to women with ovulatory versus anovulatory pattern is different. It is recommended that if one pattern of bleeding cannot be told from the other, the patient be assessed as having irregular bleeding because this pattern includes investigation for endometrial hyperplasia/cancer for more diagnostic safety.

In approach to abnormal uterine bleeding in reproductive years, the first thing to exclude is pregnancy. It is done using an office urine pregnancy test and not serum beta HCG) (option C)

The next step, after exclusion of pregnancy is checking whether the patient has developed anemia due to blood loss with a full blood exam (FBE). Transvaginal ultrasound (option B) is also a very important investigation in this patient because she has anovulatory pattern of AUB, bringing up endometrial hyperplasia or even cancer as a possibility for which transvaginal ultrasound can be used as a screening tool. Endometrial sampling (option E) using curettage follows if any endometrial abnormalities are found on ultrasound.

A thyroid function test (option D) should only be ordered if there are indicators of thyroid disease. Testing for coagulation disorders such as Von Willebrand disease is recommended if indicated.

  • Cancer Australia - Abnormal Vaginal Bleeding in Pre- and Peri-menopausal Women
  • AAFP - Evaluation and Management of Abnormal Uterine Bleeding in Premenopausal Women * Approach to diagnosis and management of abnormal uterine bleeding
How well did you know this?
1
Not at all
2
3
4
5
Perfectly
41
Q

A 29-year-old woman with past history of pregnancy with an anencephalic fetus presents to your clinic for preconception counselling. Which one of the following is the most appropriate advice regarding folic acid supplementation for her?

A. She should use supplemental folic acid 0.4 to 0.5 mg per day.
B. She should use supplemental folic acid 10 mg per day.
C. She should increase consuming more green vegetables.
D. She should use supplemental folic acid 1 mg per day.
E. She should use supplemental folic acid 4 to 5 mg per day.

A

E. She should use supplemental folic acid 4 to 5 mg per day

Folic acid deficiency of mother can result in fetal neural tube defects and for this reason every woman should be started on folic acid from at least one month before conception and continue during the first trimester. In addition, all women should be encourage to include green vegetables in their diets, at least during this period.

Current recommendation is daily use of folic acid 0.5 mg daily or a multivitamin containing at least 0.4 mg of folic acid.

Women should take 10 times the minimal dose (4 to 5 mg per day) if:

  1. There is a family, or personal history of neural tube defects (NTDs) or a previous pregnancy with NTD
  2. Use of medications that affect folic acid absorption e.g. antiepileptic medications
  3. Diabetes (type I/II)
  4. Mother’s BMI>35 kg/m2

According to National Health and Medical Research Council (NHMRC), the father’s personal history of NTDs is another condition requiring prescription if high dose folic acid.

This woman has previous history of a pregnancy associated with NTD; hence, she should be started on folic acid 4 to 5 mg per day from at least one month before conception and 3 months into the pregnancy.

NOTE - the above recommendations apply for women with all current nutritional status. For example, if a woman is vegetarian, just assuming that she is receiving adequate amount of folic acid through green vegetables should not result in altered management of folic acid supplementation for her.

*http://www.kemh.health.wa.gov.au/development/manua
*https://www.nhmrc.gov.au/_files_nhmrc/file/nics/ma

How well did you know this?
1
Not at all
2
3
4
5
Perfectly
42
Q

A 27-year-old woman presents for a pre-pregnancy consultation. She is vegetarian. Which one of the following would be the best recommendation with regard to folic acid supplementation?

A. She does not need folic acid supplementation because she is receiving adequate amount through her diet.
B. She does not need folic acid supplementation but she should increase consumption of green vegetables.
C. She should be started on folic acid 0.5 mg per day.
D. She should be started on folic acid 0.1 mg per day.
E. She should be started on folic acid 5 mg per day

A

C. She should be started on folic acid 0.5 mg per day

Folic acid deficiency of mother can result in fetal neural tube defects and for this reason every woman should be started on folic acid from at least one month before conception and continue during the first trimester. In addition, all women should be encourage to include green vegetables in their diets, at least during this period.

Current recommendation is daily use of folic acid 0.5 mg daily or a multivitamin containing at least 0.4 mg of folic acid.

Women should take 10 times the minimal dose (4 to 5 mg per day) if:

  1. There is a family, or personal history of neural tube defects (NTDs) or a previous pregnancy with NTD
  2. Use of medications that affect folic acid absorption e.g. antiepileptic medications
  3. Diabetes (type I/II)
  4. Mother’s BMI>35 kg/m2

According to National Health and Medical Research Council (NHMRC), the father’s personal history of NTDs is another condition requiring prescription if high dose folic acid.

The above recommendations apply for women with all current nutritional status, and being a vegetarian does not exclude her. Therefore, she should be started on folic acid supplementation 0.4 to 0.5 mg per day. She should also be advised to include more green vegetables in her diet if she already has not.

*http://www.kemh.health.wa.gov.au/development/manua
*https://www.nhmrc.gov.au/_files_nhmrc/file/nics/ma

How well did you know this?
1
Not at all
2
3
4
5
Perfectly
43
Q

A 32-year-old vegetarian woman presents for a pre-pregnancy consultation. She is epileptic and has been on phenytoin for the past 3 years. Which one of the following would be the best recommendation regarding folic acid supplementation?

A. She does not need folic acid supplementation because she is receiving adequate amount through her diet.
B. She does not need folic acid supplementation but she should increase consumption of green vegetables.
C. She should be started on folic acid 0.5 mg per day.
D. She should be started on folic acid 5 mg per day.
E. She should be started on folic acid 0.1 mg per day.

A

D. She should be started on folic acid 5 mg per day

Folic acid deficiency of mother can result in fetal neural tube defects and for this reason every woman should be started on folic acid from at least one month before conception and continue during the first trimester. In addition, all women should be encourage to include green vegetables in their diets, at least during this period.

Current recommendation is daily use of folic acid 0.5 mg daily or a multivitamin containing at least 0.4 mg of folic acid.

Women should take 10 times the minimal dose (4 to 5 mg per day) if:

  1. There is a family, or personal history of neural tube defects (NTDs) or a previous pregnancy with NTD
  2. Use of medications that affect folic acid absorption e.g. antiepileptic medications
  3. Diabetes (type I/II)
  4. Mother’s BMI>35 kg/m2

According to National Health and Medical Research Council (NHMRC), the father’s personal history of NTDs is another condition requiring prescription if high dose folic acid.

The above recommendations apply to women with all current nutritional status. Being a vegetarian does not exclude her or change the recommendations. Since she is on antiepileptic drugs that decrease the absorption of folic acid, she should be started on folic acid supplementation 4 to 5 mg per day. She should also be advised to include more green vegetables in her diet if she already has not.

*http://www.kemh.health.wa.gov.au/development/manua
*https://www.nhmrc.gov.au/_files_nhmrc/file/nics/ma

How well did you know this?
1
Not at all
2
3
4
5
Perfectly
44
Q

A 41-year-old woman presents with her cervical screening results reporting CIN2. Colposcopic examination is arranged and performed but reveals no gross abnormality. Which one of the following would be the next best step in management?

A. Cone biopsy.
B. LEEP.
C. Repeat pap smear in 12 months.
D. Reassure the patient as it could be a false positive result.
E. Repeat pap smear in 6 months.

A

A. Cone biopsy

Patients with CIN2 cervical screening result (equivalent to HSIL in modified Bethesda system) should be referred for colposcopy. If the lesion cannot be visualized on colposcopy, performing a cone biopsy would be the next best step in management.

Cervical squamous cell abnormalities in different nomenclatures, i.e. CIN (Cervical Intraepithelial Neoplasia) and Australian Modified Bethesda system are defined and compared in the following table:
(see photo below)

CIN 2 and 3 are equivalents of HSIL.

Women with HSIL on cervical screening should be referred to specialist for colposcopy. If colposcopy is unsatisfactory, the next step to consider is cone biopsy.

Cone biopsy is an excisional technique in which a significant length of tissue along the cervical canal (from 1.5–3 cm) is removed. Traditionally, cone biopsy has been performed using the ‘cold-knife’ technique.

The specific indications for cone biopsy are:

  1. Failure to visualize the upper limit of the cervical transitional zone (TZ) in a woman with a HSIL on her referral cervical smear i.e. unsatisfactory colposcopy
  2. Suspicion of an early invasive cancer on cytology, biopsy or colposcopic assessment
  3. The suspected presence of an additional significant glandular abnormality (i.e. AIS) on cytology or biopsy (i.e., a mixed lesion)
  • Australian Government, Department of Health – Cancer screening
  • AAFP - Management of Histologic Abnormalities of the Cervix
How well did you know this?
1
Not at all
2
3
4
5
Perfectly
45
Q

A 26-year-old woman comes to you with a cervical screening test result, reporting high-grade squamous intraepithelial lesion (HSIL). She is 34 weeks’ pregnant. Which one of the following is the most appropriate management at this stage?

A. Loop excision.
B. Cone biopsy.
C. Continuation of the pregnancy.
D. Offer termination of the pregnancy now.
E. Refer for colposcopy.

A

E. Refer for colposcopy

Based on recommendations from National Health and Medical Research Council (NHMRC), management of cervical neoplasia in pregnancy is as follows:

LSIL (CIN 1)
Women with low-grade cytologic lesions should be managed in the same way as for non-pregnant women with such lesions, with a repeat smear after 12 months, if they have had a normal cervical screening in the past 2-3 years. Otherwise, they should be referred to a specialist for colposcopy.

HSIL (CIN 2,3)
With a high-grade squamous intraepithelial lesion (HSIL) (CIN 2,3) the woman should be referred for colposcopy.

Colposcopy is safe during pregnancy. The colposcopic evaluation of the cervix may be more difficult due to vaginal laxity preventing complete visualization of the transformation zone. The increased vascularity due to pregnancy may also be difficult to interpret.

Experienced colposcopists will not usually perform a biopsy if they are confident that they have excluded an invasive cancer. If no lesion is identified at colposcopy, it is advisable to request a review of all the cytological slides. If the diagnosis of a HSIL is confirmed, a second opinion from another colposcopist with wide experience in the colposcopic evaluation of pregnant women is recommended.

With HSIL, it will be prudent to review the woman at approximately 20–24 weeks with cytology and colposcopy to determine, as far as possible, that she does not have an invasive lesion.

Definitive treatment of a high-grade lesion, with the exception of invasive cancer, may be deferred safely until after pregnancy.

Invasive carcinoma
A diagnosis of cervical cancer in pregnancy poses major challenges to the patient, her family, and the treating physician(s). The general principles in management of invasive cervical carcinoma are as follows:

  1. If the fetal lungs are mature at the time of diagnosis, or if the fetus is at/near the gestational age of expected lung maturity, immediate delivery and definitive treatment of the mother is the preferred approach (antenatal corticosteroid therapy is given if indicated).
  2. If the pregnancy is previable and the patient decides not to continue the pregnancy, immediate therapy of the mother should be initiated.
  3. In all other patients, decisions regarding timing of treatment and delivery require careful consideration of the stage of disease, the trimester in which the diagnosis is made, and the preferences of the affected woman and her family regarding the pregnancy.
  • National cervical screening program
  • UpToDate - Cervical cancer in pregnancy
How well did you know this?
1
Not at all
2
3
4
5
Perfectly
46
Q

A 31-year-old woman gave birth to a baby of normal weight through vaginal delivery. The delivery was complicated by a small perianal tear that was taken care of without stitching. Today on the fifth day postpartum, she presents with heavy bright red vaginal bleeding. She mentions that the lochia was in scant amounts after delivery compared to her previous pregnancy. On examination, she has a temperature of 38.8°C. The uterus is mildly tender to palpation. Which one of the following could be the most likely diagnosis?

A. Endometritis.
B. Infection of the perianal tear.
C. Retained products of conception.
D. Cervical tear.
E. Uterine rupture.

A

C. Retained products of conception

The presence of secondary postpartum hemorrhage (between 24 hours and 12 weeks postpartum) that is bright red and fever suggests retained products of conception (RPOC).

After separation of placenta, the basal portion of the decidua remains. This decidua divides into two layers: the superficial layer that is shed and the deep layer regenerates new endometrium, which covers the entire endometrial cavity by the 16th postpartum day. Normal shedding of blood and decidua is referred to as lochia rubra (red, red brown), and lasts for the first few days following delivery. Vaginal discharge then becomes increasingly watery, called lochia serosa (pinkish brown) lasting for 2 to 3 weeks. Ultimately, the discharge becoms yellowish-white, the lochia alba.

Scant amount of lochia in the first few days after delivery suggests that the placental site may have not undergone involution probably due the RPOC. Usually, retained products undergo necrosis with fibrin deposition and may eventually form a so-called placental polyp. As the scar of the polyp detaches from the myometrium, brisk hemorrhage may occur.

The necrotic products may also become infected and cause uterine infection presented by fever, lower abdominal pain and uterine tenderness.

OPTION A : Endometritis can cause fever, offensive-odor lochia and abdominal pain/tenderness. Endometritis is the most common cause of postpartum fever. It occurs within the first 5 days postpartum with the peak incidence between days 2 and 3 postpartum. Although vaginal bleeding can be a feature at occasions, bright red bleeding makes endometritis less likely.

OPTION B : Genital lacerations are another cause of postpartum fever. The peak incidence of wound infection is between days 4 and 5. Although the temporal appearance of the fever favors wound infection as a cause, this diagnosis is less likely because wound infection do not affect the normal course of lochia, and does not have heavy bright red bleeding as an expected feature. Moreover, there are no clues to wound infection such as erythema, tenderness or discharge in history.

OPTION D : Cervical tear can be another cause of bleeding or fever, but cervical tear tends to present with primary rather than secondary postpartum hemorrhage (occurring after the first 24 hours postpartum). An infected overlooked minor cervical laceration can cause fever but is not very likely to cause bright red bleeding. Furthermore, genital tract lacerations do not affect lochia.

OPTION E : Uterine rupture is very unlikely to occur after 24 hours of delivery.

*http://www.uptodate.com/contents/retained-products
*http://www.sahealth.sa.gov.au/wps/wcm/connect/dc33

How well did you know this?
1
Not at all
2
3
4
5
Perfectly
47
Q

A 21-year-old woman presents to your clinic asking for emergency contraception. Which one of the following is the most important initial question to ask her before further management?

A. Previous use of OCP.
B. History of headaches.
C. The date of the unprotected sex.
D. The date of the last menstrual period (LMP).
E. Family history of hypercoagulable state.

A

C. The date of the unprotected sex

Emergency contraception (also known as postcoital contraception, the morning-after pill, or plan B) refers to the use of drugs or a device as an emergency measure to prevent pregnancy in women who have had recent unprotected intercourse, including those who have had a failure of another method of contraception.

Although conception is possible on only a few days of the menstrual cycle, emergency contraception is offered regardless of the timing of the menstrual cycle due to uncertainty in timing of ovulation; therefore, LMP is not a significant determining factor for considering emergency contraception.

Of the options, the most important question to ask is the date of unprotected sex. Most methods are useful within a short time of unprotected sex. The window to use an emergency contraception method is simplified to up to 5 days (120 hours) post-coitus. There is a linear relationship between efficacy and the time from intercourse. The sooner a method is used the lesser the chance of pregnancy would be.

There is no medical contraindication to using pharmacological emergency contraception methods. Contraindications to use of IUCD are the same for routine use of intrauterine devices.

*http://www.uptodate.com/contents/emergency-contrac
*http://kemh.health.wa.gov.au/development/manuals/O

How well did you know this?
1
Not at all
2
3
4
5
Perfectly
48
Q

The anomaly of the child, illustrated in the following photograph, is more likely to be due to which one of the following drugs taken by the mother during the pregnancy?

A. Phenytoin.
B. Sodium valproate.
C. Levodopa.
D. Carbamazepine.
E. Lamotrigine.

A

B. Sodium valproate

Previously, women with epilepsy were informed that their risk for fetal malformations was increased 2- to 3 folds, but more recent data suggest that the risk may not be as great as once thought, particularly for newer agents. More recent studies showed that the risk of fetal malformation following intrauterine exposure to anticonvulsants that are currently in use is approximately the same or only slightly elevated compared to children without exposure.

Sodium valproate is one important exception to these findings. Women treated with sodium valproate are at significantly increased risk of fetal malformations.

The most frequently reported anomalies are:
-Orofacial clefts
-Cardiac anomalies
-Neural tube defects

One study showed that major malformations occurred in 9% of fetuses with first-trimester valproate exposure; however, different data suggest the rate to be as high as 16%.

  • PubMed - Teratogenic effects of antiepileptic drugs
  • William’s Obstetrics – 2014 - McGraw Hill – pages 246-7
How well did you know this?
1
Not at all
2
3
4
5
Perfectly
49
Q

Which one of the following is unlikely to cause fetal growth restriction?

A. Cytomegalovirus infection.
B. Trisomy 13.
C. Thalassemia minor with hemoglobin of 80 g/L.
D. Mother with essential hypertension on treatment with methyldopa.
E. Lupus nephritis.

A

C. Thalassemia minor with hemoglobin of 80 g/L

Of the options, thalassemia minor has not been shown to be associated with negative pregnancy outcomes such as fetal growth restriction. Although a hemoglobin level of 80 g/L may appear low at the first look, it should be noted that in pregnant women the anemia is defined as a hemoglobin level of less than 100 g/L (not 120 g/L in non-pregnant women). The reason is that plasma volume and hematocrit increase by 50% and 30% respectively, resulting in diluted plasma.

OPTION A : Cytomegalovirus (CMV) is the most common prenatal viral infection. Fetal growth restriction is a known complication of intrauterine infection with CMV.

OPTION B : Trisomy 21 (Down syndrome), trisomy 18 (Edward syndrome), Trisomy 13 (Patau syndrome) and Turner’s syndrome are congenital causes of fetal growth restriction. Trisomy 13 is characterized by profound mental retardation, cyclopia, proboscis, holoprosencephaly and severe orofacial clefts.

OPTION D : Maternal hypertension is another important cause of fetal growth restriction. Studies suggest that treatment of hypertension during pregnancy and consequent drop in maternal mean arterial pressure is associated with a lower birth weight. In fact, it is postulated that mild to moderate hypertension of the mother should be left untreated to prevent treatment-induced fetal growth restriction.

OPTION E : Hypertension can be seen in 10-20% of pregnant women with systemic lupus erythematosus, especially if the kidneys are involved (lupus nephritis). Maternal hypertension can result in fetal growth restriction.

TOPIC REVIEW

Causes of intrauterine growth restriction:

Medical
-Chronic hypertension (!)
-Preeclampsia
-Diabetes mellitus
-Systemic lupus erythematous
-Chronic renal disease
-Inflammatory bowel disease
-Severe hypoxic lung disease

Maternal
-Smoking
-Alcohol use
-Cocaine use
-Warfarin
-Phenytoin
-Malnutrition
-Prior history of pregnancy with intrauterine growth restriction
-Residing at altitude above 5000 feet

Infections
-Syphilis
-CMV
-Toxoplasmosis
-Rubella
-Hepatitis B
-Herpes simplex 1,2
-HIV

Congenital
-Trisomy 21
-Trisomy 18
-Trisomy 13
-Turner’s syndrome

  • RACGP - Intrauterine growth restriction: Diagnosis and management
  • IJRM - The pregnancy outcome in patients with minor β-thalassemia
  • CMR - Lupus and Pregnancy: Complex Yet Manageable
How well did you know this?
1
Not at all
2
3
4
5
Perfectly
50
Q

A 58-year-old woman presents to your practice with complaints of hot flushes and mood swings for the past 12 months. Her menstrual periods ceased 2 years ago. She has the history of breast cancer at the age of 35 that was treated successfully. Which one of the following is be the most appropriate management for her symptoms?

A. Clonidine.
B. Fluoxetine.
C. Hormone replacement therapy.
D. Estrogen patch.
E. No treatment at this age.

A

B. Fluoxetine

Menaopausal hormone replacement therapy (HRT) is the most effective treatment for vasomotor symptoms (e.g. hot flushes) of menopause. Menopause is associated with several complications, but vasomotor symptoms are the only indication to start HRT in a menopausal woman. However, personal history of a estrogen-dependent cancer makes HRT of any form an unfavorable treatment option. For those in whom HRT is not an appropriate option either due to a medical condition or the patient’s preferences, the selective serotonin reuptake inhibitors (SSRIs) and selective serotonin norepinephrine reuptake inhibitors (SNRIs) are first choices for treatment of vasomotor symptoms.

The following are recommendations by Royal Australian and New Zealand College of Obstetrics and Gynaecology (RANZCOG) regarding menopausal hormone therapy (MHT):

Recommendation1 - The primary indication for the use of MHT is the alleviation of distressing menopausal vasomotor symptoms.

Recommendation 2 - In women with primary ovarian insufficiency, MHT should be continued until the normal age of the menopause.

Recommendation 3 - MHT is also effective and appropriate for the prevention of osteoporosis related fracture in at risk women within 10 years of the menopause.

Recommendation 4 - The risk of VTE and stroke increases with oral MHT but the absolute risk is very small before age 60 years.

Recommendation 5 - In women within 10 years of the menopause MHT does not increase the risk of coronary heart disease.

Recommendation 6 - Combined MHT use for more than 5 years may be associated with an increased risk of breast cancer. This risk appears to be related to the use of a progestogen and duration of therapy.

Recommendation 7 - Estrogen-only MHT does not increase risk of breast cancer.

Recommendation 8 - Current safety data do not support the use of MHT in breast cancer survivors.

Recommendation 9 - Estrogen-only therapy is appropriate for women who have undergone hysterectomy.

Recommendation 10 - Eestrogen plus progestogen should be used in women with an intact uterus.

Recommendation 11 - The dose and duration of therapy should be consistent with treatment goals.

*http://www.uptodate.com/contents/menopausal-hot-fl
*https://www.ranzcog.edu.au/doc/hormone-replacement

How well did you know this?
1
Not at all
2
3
4
5
Perfectly
51
Q

A 32-year-old woman is referred to you at 36 weeks gestation for management of eclampsia. While examining, the patient develops a seizure in front of you. Which one of the following is the most appropriate immediate management?

A. Phenytoin.
B. Intravenous diazepam.
C. Ensuring airway patency.
D. Magnesium sulphate.
E. Induction of labor.

A

C. Ensuring airway patency

In approaching a patient, especially in an emergency setting such as this one, the most immediate management is checking and stabilizing airway, breathing and circulation (ABC), regardless of the underlying etiology.

Ensuring and maintaining airway patency should always be the first priority, followed by sufficient breathing and oxygenation, and circulation.

Magnesium sulphate is the anticonvulsant of choice for eclampsia. Intravenous diazepam can be given while magnesium sulphate is prepared and the seizure is prolonged.

Arrangements for delivery should be decided once the woman’s condition is stable. In the meantime, close fetal monitoring should be maintained.

  • http://www.ranzcog.edu.au/doc/somanz-hypertensive-
    *http://www.kemh.health.wa.gov.au/development/manua
How well did you know this?
1
Not at all
2
3
4
5
Perfectly
52
Q

A 29-year-old woman presents with two episodes of bleeding, one hour apart at 39 weeks gestation. She has no abdominal pain or any other complaints. Physical exam is inconclusive. An ultrasound scan reveals placenta previa. Which one of the following is the most appropriate next step in management?

A. Admit her for supervision.
B. Discharge her home.
C. Induction of labor.
D. Cesarean section.
E. Vaginal examination.

A

D. Cesarean section

Most women who initially present with symptomatic placenta previa respond to supportive therapy, and do not require immediate delivery. Even a large bleed does not preclude conservative management. Management of placenta previa when mother and fetus are stable is conservative until 38 weeks pregnancy when delivery should be performed by cesarean section. Since this woman is already at 39 weeks gestation, cesarean section without further need to document fetal lung maturity would be the most appropriate option.

Admission for supervision (option A) was the appropriate option at earlier gestational ages (< 37 weeks) where expectant management (if mother and fetus were stable) was considered until adequate fetal maturity is ensured.

Vaginal delivery is contraindicated in placenta previa most of the time, so induction of labor (option C) is inappropriate. Digital vaginal examination (option E) should be avoided in patients with placenta previa.

*http://www.sahealth.sa.gov.au/wps/wcm/connect/b1c6
* https://www.ranzcog.edu.au/doc/rcog-placenta-praev

How well did you know this?
1
Not at all
2
3
4
5
Perfectly
53
Q

A 37-year-old woman presents to your practice for contraception advice. She has otosclerosis and uses hearing aids. She had 3 pregnancies, during which her hearing deteriorated rapidly. Which one of the following is the most appropriate method to recommend?

A. Low-dose combined oral contraceptive pills.
B. High-dose combined oral contraceptive pills.
C. Progestin-only pills.
D. Condoms.
E. Intrauterine contraceptive device (IUCD).

A

E. Intrauterine contraceptive device (IUCD)

Otosclerosis is an often inherited condition, which is the most common cause of progressive deafness in young adults. It involves hardening of the bones of the middle ear resulting in progressive hearing loss that is often irreversible. In most cases of otosclerosis, the smallest bone (stapes) is involved. The bone becomes calcified or hardened by overgrowth of new bone. Treatment for otosclerosis is by a surgical procedure called stapedectomy, duirng which the top portion of the stapes is removed and a small prosthesis is inserted instead.

Although not supported by solid evidence, there have been suggestions that the combined oral contraceptives (COCs) can worsen otosclerosis, especially if the patient has experienced onset or deterioration during pregnancy or steroid use. For this reason and depsite lack of evidence, avoiding systemic hormonal methods may be appropriate. Of the given non-hormonal methods, IUCD provides more contraception benefit and less failure rates and can be advised as an acceptable method for her.

*https://www.ncbi.nlm.nih.gov/pubmed/19121641

How well did you know this?
1
Not at all
2
3
4
5
Perfectly
54
Q

A 27-year-old woman presents with severe nausea and vomiting at 8 weeks pregnancy. She cannot take solid foods but is able to drink small sips of liquids. She is concerned that she might have gastroenteritis, because her partner has recently had it. Which one of the following is the next best investigation in this situation?

A. Pelvic ultrasound.
B. Urinalysis and culture.
C. Stool exam and culture.
D. Quantitative β-HCG.
E. Abdominal ultrasound.

A

B. Urinalysis and culture

Although this woman might have gastroenteritis, the first thing to consider, given the gestational age, is hyperemesis gravidarum/nausea and vomiting of pregnancy. The mean onset of hyperemesis gravidarum/nausea and vomiting of pregnancy symptoms is at 5 to 6 weeks of gestation, peaking at about 9 weeks, and usually abating by 16 to 20 weeks of gestation; however, symptoms may continue until the third trimester in 15-20% of pregnant women and until delivery in 5%. Sixty percent of women are asymptomatic six weeks after onset of nausea. If vomiting persists beyond a few days postpartum, other etiologies should be investigated.

Diagnosis of hyperemesis gravidarum is mainly clinical; however, investigations are needed both for assessment of the patient’s status and excluding other possible causes of nausea and vomiting.

The standard initial evaluation of pregnant women with persistent nausea and vomiting includes measurement of weight, vital signs, orthostatic blood pressure, serum electrolytes, and urine ketones and specific gravity and urine microscopy and culture.

Attending to hydration status and electrolyte imbalance is of paramount importance before proceeding to investigations for and underlying cause. A spot urine dipstick at the office/ED or a formal urinalysis, microscopy, and culture is as well as electrolyte assays is the first step while arrangements for other investigations are made. An obstetric ultrasound exam (pelvic ultrasound) should be performed to look for gestational trophoblastic disease or multiple gestation as possible underlying causes. At this gestational age, transvaginal ultrasound is the preferred method. Pelvic ultrasound allows quick visualization of the female pelvic organs and structures including the uterus, cervix, vagina, fallopian tubes, ovaries, and in pregnant women, the fetus and the gestational sac.

OPTION C : Stool exam and culture may be considered for evaluation of the selected patients if gastroenteritis is suspected as the cause of the clinical picture.

OPTION D : With conditions such as multiple or molar pregnancy, quantitative serum beta HCG is higher than normal pregnancies. However, the most common cause for a β-HCG level unproportionate to gestational age is dating errors. If such discrepancy is found on quantitative β-HCG test, ultrasound will be required for accurate estimation of gestational date; therefore, this test is neither necessary nor useful as initial investigation.

OPTION E : Abdominal ultrasound is inferior to transvaginal ultrasound for evaluation of female reproductive organs and the fetus.

  • South Australian Perinatal Practice Guideline Nausea and Vomiting in Pregnancy and Hyperemesis Gravidaru
  • UpToDate - Nausea and vomiting of pregnancy: Clinical findings and evaluation
How well did you know this?
1
Not at all
2
3
4
5
Perfectly
55
Q

A 24-year-old woman, who underwent an episiotomy during labor four days ago, has presented with severe vaginal pain. On examination, an 8-cm hematoma is noted at the site of the episiotomy. The woman is hemodynamically stable. Which one of the following is the most appropriate next step in management?

A. Aspirate the mass.
B. Explore the hematoma.
C. Reassure the patient as this will resolve spontaneously.
D. Antibiotics.
E. Oral analgesics and review in one week.

A

B. Explore the hematoma

Most puerperal hematomas arise from bleeding lacerations related to operative deliveries or episiotomy, but it is also possible for a hematoma to result from spontaneous injury to a blood vessel in the absence a of laceration/incision of the surrounding tissue.

The most common locations for puerperal hematomas are vulva, vagina/paravaginal area, and retroperitoneum.

The diagnosis of most puerperal hematomas is based upon the presence of characteristic symptoms and findings on physical examination including:

  1. Vulvar hematomas - usually present with rapid development of a severely painful, tense, compressible mass covered by skin with purplish discoloration. A vulvar hematoma may be an extension of a vaginal hematoma that has dissected through loose subcutaneous tissue into the vulva.
  2. Vaginal hematomas - often present with rectal pressure; however, hemodynamic instability due to bleeding into the ischiorectal fossa and paravaginal space may be the first sign of a vaginal hematoma, and can result in hypovolemic shock. On physical examination, a large mass protruding into the vagina is often seen.
  3. Retroperitoneal hematomas - they extend between the folds of the broad ligament and may be asymptomatic initially. Due to the significant amount of blood that can accumulate in the retroperitoneal space, these patients often present with tachycardia, hypotension, or even shock. Patients with a retroperitoneal hematoma usually do not present with pain unless the hematoma is associated with trauma. Palpation of an abdominal mass or fever can also be signs of a retroperitoneal hematoma.

Treatment of a hematoma depends on the size and location:
Non-expanding hematomas that are <3cm can be managed conservatively with analgesics and ice packs.

Management of an expanding hematoma or one greater than 3cm is with surgical exploration under anesthesia. An incision is made and the hematoma is evacuated. The surgical site should not be sutured. Vagina is often packed for 12-24 hours. An indwelling urinary catheter may be indicated.

This patient has a large haematoma (>3cm) that needs surgical excision and evacuation.

Aspiration of the hematoma is not an appropriate treatment. If surgical intervention is indicated excision and evacuation is the preferred option, followed by vaginal packing for 12- 24 hours.

*https://www.sahealth.sa.gov.au/wps/wcm/connect/149

How well did you know this?
1
Not at all
2
3
4
5
Perfectly
56
Q

A 28-year-old primigravida woman presents to the maternity unit at 38 weeks gestation after spontaneous rupture of membrane. On vaginal examination, the cervix is 5 cm dilated and the membranes are ruptured. Meconium-stained liquor is noted in the vagina. A cardiotocography (CTG) unveils a fetal heart rate of 130 bpm, variability of 5 bpm and variable decelerations. Which one of the following would be the most appropriate next step in management?

A. Urgent cesarean section.
B. Fetal Scalp blood sampling.
C. Continue CTG monitoring.
D. Start infusion of oxytocin to facilitate the labor.
E. High forceps delivery.

A

B. Fetal Scalp blood sampling

Meconium can be found in the gastrointestinal tract of the fetus from as early as 10 -16 weeks gestation. Although 75% of meconium is water, the remaining 25% consists of gastric secretions, bile salts, mucous, vernix, lanugo, blood, pancreatic enzymes, free fatty acids and squamous cells. Meconium staining of the amniotic fluid (MSAF) occurs in approximately 10-15% of labors. MSAF rarely occurs before 30 weeks gestation. In a preterm fetus, MSAF can suggest possible infection or hypoxia. Neonatal aspiration of MSAF can become a serious problem if pulmonary hypertension and severe hypoxemia develop. Such conditions require expert neonatal intensive care.

Meconium stained liquor is one of the conditions considered as intrapartum risk factors for the fetus’ wellbeing. With meconium present, continuous cardiotocography (CTG) should follow for ongoing fetal assessment. Further Actions depend on CTG results:

Reassuring CTG: a CTG is reassuring if the following criteria are met:
a. Baseline fetal heart rate (FHR) is between 110-160 bpm
b. Variability of FHR is between 5-25 bpm
c. Decelerations are absent or early

The significance of the presence or absence of accelerations is unclear and are exclude for interpretation.

With a reassuring CTG, watchful waiting for labor to progress is recommended, unless there are other contraindications. Continuous CTG and close monitoring of the mother and fetus is essential.

Non-reassuring CTG: a CTG is non-reassuring if any of the following is present:
a. Baseline FHR is between 100-109 bpm or between 161-170 bpm
b. Variability of FHR is reduced (3-5 bpm for >40 minutes)
c. Decelerations are variable without complicating features

Abnormal CTG: a CTG is abnormal if at least two of the features described in non-reassuring CTG are present OR one or more of the following features exists:

a. Baseline FHR is <100 bpm or >170bpm
b. Variability is absent or <3 bpm
c. Decelerations are prolonged for >3 minutes OR late OR have complicated variables

With a non-reassuring or abnormal CTG, fetal blood sampling should follow unless contraindicated.

Contraindications to fetal blood sampling include the following:

-Clear evidence of serious fetal compromise (e.g., complete absence of reassuring features) – in such conditions urgent preparation to expedite birth should be made.
-Prolonged deceleration
-Mobile presenting part
-Unknown presentation
-Face presentation
-Undilated cervix (< 3 cm)
-Active second stage of labor
-Fetal hereditary bleeding disorders (e.g., suspected fetal thrombocytopenia, hemophilia)
-Maternal infection (e.g., HIV, hepatitis, herpes simplex virus, suspected intrauterine sepsis)
-Prematurity (gestation < 34 weeks) as delayed birth due to the procedure may be associated with an increase in adverse outcomes because the small “at risk” fetus will have a lower threshold than a term infant for fetal compromise.

This fetus has a non-reassuring CTG and no contraindications to fetal scalp blood sampling; therefore, this procedure is the most appropriate next step in management.

Proceeding to delivery is indicated if fetal blood sampling shows genuine fetal compromise.

  • WA Health Department – Obstetrics and Gynaecology Guidelines
  • RANZCOG Intrapartum Fetal Surveillance Clinical Guideline
How well did you know this?
1
Not at all
2
3
4
5
Perfectly
57
Q

A 31-year-old woman is admitted to the Maternity Unit for labor. She has had regular antenatal visits and her pregnancy has been uneventful thus far. During the labor, meconium liquor passage is noted. Cardiotocography shows a fetal heart rate (FHR) of 149 bpm. There are no decelerations or accelerations. There is beat-to-beat variability of 15 bpm. Vaginal exam is normal. Which one of the following is the most appropriate next step in management?

A. Fetal scalp blood sampling as there is a 10% chance of hypoxia.
B. Fetal scalp blood sampling as there is a 50% chance of hypoxia.
C. Fetal scalp blood sampling as there is a 75% chance of hypoxia.
D. Proceeding to immediate cesarean section.
E. The CTG is normal and close monitoring until delivery is all required for now.

A

E. The CTG is normal and close monitoring until delivery is all required for now

Meconium can be found in the gastrointestinal tract of the fetus from as early as 10 -16 weeks gestation. Although 75% of meconium is water, the remaining 25% consists of gastric secretions, bile salts, mucous, vernix, lanugo, blood, pancreatic enzymes, free fatty acids and squamous cells. Meconium staining of the amniotic fluid (MSAF) occurs in approximately 10-15% of labors. MSAF rarely occurs before 30 weeks gestation. In a preterm fetus, MSAF can suggest possible infection or hypoxia. Neonatal aspiration of MSAF can become a serious problem if pulmonary hypertension and severe hypoxemia develop. Such conditions require expert neonatal intensive care.

Meconium stained liquor is one of the conditions considered as intrapartum risk factors for the fetus’ wellbeing. With meconium present, continuous cardiotocography (CTG) should follow for ongoing fetal assessment. Further Actions depend on CTG results:

Reassuring CTG: a CTG is reassuring if the following criteria are met:
a. Baseline fetal heart rate (FHR) is between 110-160 bpm
b. Variability of FHR is between 5-25 bpm
c. Decelerations are absent or early

The significance of the presence or absence of accelerations is unclear and are exclude for interpretation.

With a reassuring CTG, watchful waiting for labor to progress is recommended, unless there are other contraindications. Continuous CTG and close monitoring of the mother and fetus is essential.

Non- reassuring CTG: a CTG is non-reassuring if any of the following is present:
a. Baseline FHR is between 100-109 bpm or between 161-170 bpm
b. Variability of FHR is reduced (3-5 bpm for >40 minutes)
c. Decelerations are variable without complicating features

Abnormal CTG: a CTG is abnormal if at least two of the features described in non-reassuring CTG are present OR one or more of the following features exists:

a. Baseline FHR is <100 bpm or >170bpm
b. Variability is absent or <3 bpm
c. Decelerations are prolonged for >3 minutes OR late OR have complicated variables

With a non-reassuring or abnormal CTG, fetal blood sampling should follow unless contraindicated.

Contraindications to fetal blood sampling include the following:

-Clear evidence of serious fetal compromise (e.g., complete absence of reassuring features) – in such conditions urgent preparation to expedite birth should be made.
-Prolonged deceleration
-Mobile presenting part
-Unknown presentation
-Face presentation
-Undilated cervix (<3 cm)
-Active second stage of labor
-Fetal hereditary bleeding disorders (e.g., suspected fetal thrombocytopenia, hemophilia)
-Maternal infection (e.g., HIV, hepatitis, herpes simplex virus, suspected intrauterine sepsis)
-Prematurity (gestation < 34 weeks) as delayed birth due to the procedure may be associated with an increase in adverse outcomes because the small “at risk” fetus will have a lower threshold than a term infant for fetal compromise.

How well did you know this?
1
Not at all
2
3
4
5
Perfectly
58
Q

A 25-year-old woman presents to the emergency department at 14 weeks pregnancy with complaint of urine retention for the past 18 hours. She had an obstetric ultrasound at 10 weeks pregnancy confirming the date. In the emergency department, a transvaginal ultrasound is performed confirming the gestational age and revealing a retroverted uterus. On abdominal examination, there is a tender mass midway below the umbilicus. Which one of the following is the most likely diagnosis?

A. Ectopic pregnancy.
B. Ovarian fibroid.
C. Red degeneration of uterine fibroid.
D. Ovarian cyst.
E. Incarcerated uterus.

A

E. Incarcerated uterus

The presentation is classic for incarcerated gravid uterus.
The term incarcerated gravid uterus refers to a pregnant uterus entrapped in the pelvis by the sub-promontory sacrum.

In very early pregnancy, the uterus is retroverted (retroflexed) in up to 20% of women. As the uterus enlarges during the first trimester, the fundus normally rises from the hollow of the sacrum to an anterior ventral position and corrects the retroversion. However, in rare cases, the fundus becomes wedged below the sacral promontory, where it continues to enlarge. Concomitantly, the cervix becomes displaced upwards against or above the symphysis pubis and pushes against the urethra and bladder, interfering with normal voiding.

Eventually, the posterior pelvis becomes too small to accommodate the increasing size of the fundus; this typically occurs when the pregnancy progresses beyond 20 weeks of gestation. At this point, the anterior lower uterine wall begins to thin and balloon into the upper abdomen to form a sacculation. Concomitantly, both the bladder and the cervix are pulled up into the abdominal cavity, towards the umbilicus. The cervix can stretch to 10 cm or more in length, such that the internal os becomes located above the symphysis pubis, and occasionally above the bladder.

History and symptoms:

Typically, the patient presents at 14 to 16 weeks of pregnancy with symptoms related to pressure on the anatomic structures adjacent to the entrapped enlarging uterus. The most common symptoms are pain and progressive difficulty voiding. The pain could be abdominal, suprapubic, or felt in the back; or may be just a discomfort or feeling of fullness in the pelvis.

Urinary symptoms include dysuria, frequency, sensation of incomplete emptying, stress incontinence, and urinary retention (most common). Pressure on rectum can cause rectal pressure, tenesmus and worsening constipation. Vaginal bleeding can infrequently occur.

Symptoms can be intermittent, resolving for a period of time and then returning weeks later.

Exam finding:
1. Severe anterior displacement of the cervix behind the pubic symphysis, making the physician unable to visualize the cervix with a speculum or palpate the external os on pelvic examination
2. The vagina is angulated anteriorly and a large, soft, smooth, nontender mass (the incarcerated uterus) fills the cul-de-sac
3. Initially, the uterus may be difficult to palpate abdominally, and later in pregnancy the fundal height may lag behind that expected for gestational age. The abdomen may appear full or distended due to an overdistended bladder.

Ultrasonography is the most appropriate initial investigation when the condition is suspected.

*http://www.uptodate.com/contents/incarcerated-grav

NORMAL GRAVID UTERUS vs INCARCERATED UTERUS
How well did you know this?
1
Not at all
2
3
4
5
Perfectly
59
Q

Which one of the following drugs has the highest rate of congenital malformations if used in pregnancy?

A. Carbamazepine.
B. Phenytoin.
C. Sodium valproate.
D. Levetiracetam.
E. Lamotrigine.

A

C. Sodium valproate

Sodium valproate carries the highest teratogenicity rate among all antiepileptic drugs. The potential congenital defects caused by sodium valproate are shown in the following table: (see photo)

  1. Neural tube defects - Spina bifida, anencephaly
  2. Heart defects - Congenital ventricular septal defect, aortic stenosis, patent ductus arteriosus, aberrant pulmonary artery
  3. Limb defects - Radial ray defect, polydactyly (more than 5 fingers), oligodactyly (less than 5 fingers), absent fingers,overlapping toes, camptodactyly (fixed flexion deformity of one or more proximal interphalangeal joints), split
    hand, ulnar or tibial hypoplasia,
  4. Genitourinary defects - Hypospadias, renal hypoplasia, hydronephrosis, duplication of calyceal system
  5. Brain anomalies - Hydranencephal (1), porencephaly(2), arachnoid cysts, cerebral atrophy, partial agenesis of corpus callosum,agenesis of septum pellucidum, lissencephaly (3) of medial sides of occipital lobes, Dandy-Walker anomaly (4)
  6. Eye anomalies - Bilateral congenital cataract, optic nerve hypoplasia, tear duct anomalies, microphthalmia, bilateral iris defects, corneal opacities
  7. Respiratory tract defects - Tracheomalacia , lung hypoplasia, severe laryngeal hypoplasia, abnormal lobulation of the right lung, right oligemic (less blood flow) lung
  8. Abdominal wall defects - Omphalocele
  9. Skin abnormalities - Capillary hemangioma, aplasia cutis congenital of the scalp.

All other medications mentioned carry the risk of congenital malformations at a lower rate compared to valproic acid.

(1) Also called hydrancephaly is absence of cerebral hemispheres and replacement of the vacant parts of the cranium with cerebrespinal fluid.

(2) A congenital disorder characterised by presence of cysts or cavities within the cerebral hemisphere.

(3) Literally meaning ‘smooth brain’ is caused by defective central nervous migration during 12th and 24th weeks of gestation resulting in lack in development of the brain folds (gyro) and grooves (sulci)

(4) Dandy-Walker syndrome is a congenital malformation of the cerebellum with the key feature being complete absence of the cerebellar vermis.

*http://www.medscape.com/viewarticle/724671
*http://www.australianprescriber.com/magazine/31/3/
*http://www.gpnotebook.co.uk/simplepage.cfm?ID=6511

How well did you know this?
1
Not at all
2
3
4
5
Perfectly
60
Q

Sarah, 18 years old, is in your office for a visit because she is concerned about her delayed puberty and absence of menses. She has never had a period before. On examination, she has no pubic or axillary hair, and her breasts are at Tanner I stage. Which one of the following is the most appropriate next step in management?

A. Pelvic ultrasonography.
B. Serum follicular stimulating hormone (FSH) and luteinizing hormone (LH).
C. Bone age assay.
D. Serum thyroid stimulating hormone (TSH).
E. Karyotype.

A

B. Serum follicular stimulating hormone (FSH) and luteinizing hormone (LH)

This scenario represents a case a primary amenorrhea as wells as delayed puberty evident by complete lack of breast development and axillary and pubic hair.

The most important step in assessment of primary amenorrhea is always a thorough history and complete physical exam to check if secondary sexual characteristics are present. Breast development reflects estrogen exposure and is the best indicator that puberty has begun. In fact, the diagnostic approach to primary amenorrhea starts with breast assessment. Breasts at Tanner III stage or higher are equivalent to presence of secondary sexual characteristics (Tanner III or higher in females older than 13 years). With developing or developed breasts, the next thing to check is whether a normal uterus is present. This is achieved by physical exam (if possible) and a pelvic ultrasound.

With Tanner I breasts, secondary sexual characteristics are absent. In such cases, investigations start with measurement of FSH either alone or with LH for assessment of hypothalamus – pituitary – ovary (HPO) axis.

This patient has not started breast development; therefore, the next step in management is measuring the FSH (and LH) for evaluation of HPO axis. TSH and prolactin are also routinely assessed as a part of hormonal assay in delayed puberty. Exclusion of pregnancy should always be considered as well.

See photo below

OPTION A : Pelvic ultrasonography is considered first choice of investigation in females with primary amenorrhea and developed secondary sexual characteristics.

OPTION C : Bone age assay is used to establish a diagnosis of constitutional growth delay (CGD). CGD is a diagnosis of exclusion, meaning that all possible causes of delayed puberty should be excluded by thorough history, physical examination, and investigation. Some conditions to exclude are hormonal deficiencies, occult systemic illness, or syndromes associated with impaired growth and development. After exclusion of all possible causes of CGD, radiographic study of the left hand and wrist to assess skeletal maturation is critical in diagnosing constitutional growth delay. Typically, in children with CGD, the bone age begins to lag behind chronologic age during early childhood and is delayed in adolescence by an average of 2-4 years.

OPTION E : Karyotyping is considered in patients with syndromic features such as short stature and amenorrhea or where hypergonadotropic hypogonadism (e.g., Turner syndrome) is suspected in the presence of an elevated FSH above the prepubertal reference range.

TOPIC REVIEW

Primary amenorrhea is defined as the absence of menses at age 15-16 in the presence of normal growth and secondary sexual characteristics (with a breast of at least Tanner II) or 13-14 in the absence of secondary sexual characteristics.

Primary amenorrhea is usually the result of a genetic or anatomical abnormality. In a large case series of primary amenorrhea, the most common etiologies were:

*Gonadal dysgenesis including turner syndrome - 43%
*Mullerian dysgenesis (absence of vagina, sometimes with absence of uterus) – 15%
*Physiological delay of puberty (constitutional delay of puberty, chronic systemic disease, acute illness) – 14% *Polycystic ovarian syndrome (PCOS) – 7%
*Isolated gonadotropin-releasing hormone (GnRH) deficiency – 5%
*Transverse vaginal septum – 3%
*Weight loss / anorexia nervosa / exercise induced – 2%
*Hypopituitarism – 2%
*Rare causes - < 1% (imperforated hymen, complete androgen insensitivity syndrome, hyperprolactinemia/prolactinoma), other pituitary tumors, congenital adrenal hyperplasia, hypothyroidism, central nervous system defects, craniopharyngioma, and Cushing’s disease)

Investigation:

According to latest guidelines, indications for investigating a cause of primary amenorrhea include:
-An adolescent who has not had menarche by age 15-16 years
-Adolescent who has not had menarche and more than 3 years have elapsed since thelarche
-An adolescent who has not had a menarche by age 13-14 years and no secondary sexual development
-An adolescent who has not had menarche by age 14 years and:
(1)There is a suspicion of an eating disorder or excessive exercise, or
(2) There are signs of hirsutism, or
(3) there is suspicion of genital outflow obstruction

The following tests are considered basic for all patients with primary amenorrhea:
-Pregnancy test (to exclude pregnancy prior to first menstruation)
-Pelvic ultrasound (complementary to physical exam)
-FSH (and LH)
-Thyroid stimulating hormone (TSH)
-Prolactin

For the exam purposes though, one should know how to prioritize these tests.

In assessment of the female patients with primary amenorrhea two critical questions should be answer:

  1. Are secondary sexual characteristics present or absent?

A physical examination will evaluate secondary sexual characteristics such as breast development, axillary and pubic hair, and growth. Breasts are an endogenous assay for estrogen. Breast development consistent with a Tanner stage of II or greater indicates the presence of estrogen and ovarian function (although it could be insufficient or prematurely failed). With breasts present, the next step is always checking for the presence or absence of a uterus, its anatomy and possible defects, vagina, and hymen. If there is no breast development, FHS (and LH) comes first.

  1. Is a uterus absent or present?

Ultrasonography is the best method to reach an answer for this question. Both transvaginal (if the patient not virgin) and transabdominal ultrasonography should be performed to assess whether mullerian structures are absent or present.

EXAM TIP

The following are characteristic features of some important conditions associated with primary amenorrhea:

Gonadal dysgenesis (including Turner)
1. No or abnormally developed breasts
2. Present uterus (but streak ovaries)
3. Often normally-developed axillary and pubic hair
4. Increased serum FSH

Androgen insensitivity
1. Normally-developed breasts (Tanner II or higher)
2. Absent uterus
3. Absent axillary and pubic hair

Hypothalamic-pituitary failure
1. No or abnormally developed breasts (Tanner stage inconsistent with predicted age)
2. Present uterus
3. Decreased serum FSH

Mullerian dysgenesis
1. Normal breast development
2. Normal axillary and pubic hair
3. Normal hormonal assay (including FSH, LH, prolactin, TSH)
4. Absent uterus (and upper part of vagina)

Transverse vaginal septum/imperforated hymen
1. Normal breast development (Tanner II or higher)
2. Normal axillary and pubic hair
3. Normal hormonal assay (including FHS, LH, prolactin, TSH)
4. Cyclical pelvic pain and/or suprapubic mass

  • UpToDate - Evaluation and management of primary amenorrhea
  • American Family Physician - Disorders of Puberty: An Approach to Diagnosis and Management * MSD Manual - Amenorrhea
How well did you know this?
1
Not at all
2
3
4
5
Perfectly
61
Q

A 16-year-old girl is brought to your clinic by her mother for evaluation because she has not started menstruation yet. She is a dedicated ballet dancer. On examination, she has a body mass index (BMI) of 16. Of all secondary sexual characteristics, only breast buds consistent with a Tanner stage II are present. You order serum FSH, LH, TSH and prolactin levels all of which are in normal range. Which one of the following is the most appropriate next step in management?

A. Pelvic ultrasonography.
B. Refer to an adolescent health center.
C. Reassure that it could be a normal variant of puberty.
D. Perform a Pap smear.
E. Karyotyping.

A

A. Pelvic ultrasonography

The case scenario describes a girl with primary amenorrhea. Primary amenorrhea is defined as the absence of menses by the age of 15-16 years in the presence of normal growth and secondary sexual characteristics or by the age of 3-14 years in the absence of secondary sexual characteristics. According to this definition, this girl has primary amenorrhea and should be thoroughly investigated.

Etiologies of primary amenorrhea diverse but generally can be considered as anatomical or hormonal.

In a large case series of primary amenorrhea, the most common etiologies were:

*Gonadal dysgenesis including turner syndrome - 43%
*Mullerian dysgenesis (absence of vagina, sometimes with absence of uterus) – 15%
*Physiological delay of puberty (constitutional delay of puberty, chronic systemic disease, acute illness) – 14%
*Polycystic ovarian syndrome (PCOS) – 7%
*Isolated gonadotropin-releasing hormone (GnRH) deficiency – 5%
*Transverse vaginal septum – 3%
*Weight loss / anorexia nervosa / exercise induced – 2%
*Hypopituitarism – 2%
*Rare causes - < 1% (imperforated hymen, complete androgen insensitivity syndrome, hyperprolactinaemia/prolactinoma), other pituitary tumours, congenital adrenal hyperplasia, hypothyroidism, central nervous system defects, craniopharyngioma, and Cushing’s disease)

There are a few points in the history that can narrow down the diagnosis. Firstly, this girl has Tanner II breast development indicating that she has been exposed to estrogen; however, at 16 years age, more developed breasts with higher Tanner stages would be expected.

Puberty in girls starts with breast development. Growth spurt then occurs and is followed by axillary and pubic hair development. Menstruation often does not take place until two years from the onset of puberty.

Another point to consider is the fact that this girl has no pubic or axillary hair. This finding can be either due to arrested or delayed progression of puberty which once has been started (as evident by breast buds), or incomplete androgen insensitivity in which breast development occurs but the uterus and axillary and pubic hair are absent. In this girl who is a professional ballet dancer with a BMI of 16, the former is more likely.

Low body weight, excessive physical exercise, and starvation are associated with hypogonadotropic hypogonadism (central hypogonadism). This is a very common finding among patients with anorexia nervosa or bulimia nervosa, but is also seen in female athletes as a cause of primary or secondary amenorrhea. If it occurs prior to complete puberty, it could impede puberty from normal progression.

In hypogonadotropic hypogonadism, the characteristic finding is a decreased serum FSH (and LH). Since this patient has normal hormonal assays, an ultrasonography should be performed for assessment of possible uterine anomalies or those of genital tract, resulting in outflow obstruction, some of which include: mullerian agenesis, transverse vaginal septum, Asherman’s syndrome, and imperforated hymen.

One interesting point in the question is that, in fact, with breasts at Tanner II stage, ultrasound should have been preceded the hormonal assays because, for instance, if this girl was found on ultrasound to have no uterus, there was no need to measure FSH and other hormones, and the diagnostic pathway then should be continued with karyotyping (option E) and testosterone assays to investigate complete or incomplete androgen insensitivity as the most likely diagnosis.

Referral to adolescent health centre (option B) may be indicated somewhere down the line once other causes of primary amenorrhea other than those related to excessive exercise, weight loss and eating disorders are excluded with high certainty. It is not appropriate at this stage when other probable causes of primary amenorrhea are not yet fully investigated.

This girl lags behind normal puberty and cannot be reassured (option C) unless the cause is ensured to be constitutional delay of puberty. Interestingly and unlike the common belief, constitutional delay of puberty is not that common in females compared to males.

OPTION D : Screening with Pap smear starts at 25 years of age. It is not indicated for this girl now.

How well did you know this?
1
Not at all
2
3
4
5
Perfectly
62
Q

A mother has brought her 16-year-old daughter to your practice for evaluation with the complaint that she has not started her periods yet. On examination, her height is 140 cm. She has breast buds consistent with stage II Tanner. There is axillary and pubic hair growth appropriate for her age. She gives no history of galactorrhea. Which one of the following is the most likely cause to her presentation?

A. A normal variant in sexual development.
B. Hypothalamic hypogonadism.
C. Mullerian agenesis.
D. Ovarian dysgenesis.
E. Pituitary tumour.

A

D. Ovarian dysgenesis

The main complaint presented in the scenario is primary amenorrhea. Primary amenorrhea is defined as the absence of menses at age 15-16 in the presence of normal growth and secondary sexual characteristics, or 13-14 in the absence of secondary sexual characteristics. According to this definition, this girl has primary amenorrhea and should be thoroughly investigated.

Etiology of primary amenorrhea is diverse but generally can be considered as anatomical or hormonal.

In a large case series of primary amenorrhea, the most common etiologies were:

  • Gonadal dysgenesis including turner syndrome - 43%
  • Mullerian dysgenesis (absence of vagina, sometimes with absence of uterus) – 15%
  • Physiological delay of puberty (constitutional delay of puberty, chronic systemic disease, acute illness) – 14%
  • Polycystic ovarian syndrome (PCOS) – 7%
  • Isolated gonadotropin-releasing hormone (GnRH) deficiency – 5%
  • Transverse vaginal septum – 3%
  • Weight loss / anorexia nervosa / exercise induced – 2%
  • Hypopituitarism – 2%
  • Rare causes - < 1% (imperforated hymen, complete androgen insensitivity syndrome, hyperprolactinaemia/prolactinoma), other pituitary tumors, congenital adrenal hyperplasia, hypothyroidism, central nervous system defects, craniopharyngioma, and Cushing’s disease)

By far, the gonadal dysgenesis (ovarian dysgenesis) including Turner syndrome is the most common cause of primary amenorrhea in female patients.

Gonadal dysgenesis is caused by complete or partial deletion of one X chromosome with the classic Karyotype being (45,OX). The female may also have a mosaic genotype – (45,Ox/46,xx). Short stature and a square appearance is the cardinal common clinical feature of Turner syndrome.

In older adolescents and adults, presenting symptoms of the syndrome usually involves problems of puberty and fertility as well as short stature. Pubic and axillary hair growth occurs at a normal age, but is not an indicator that puberty will progress normally. Breast development may be absent if ovarian failure occurs before puberty, or partially developed if it occurs at some time after puberty has begun. Up to 13% of girls with Turner syndrome have spontaneous breast development or menses; some of these are XO/XX mosaics, with normal gonadotropin responses to luteinizing hormone–releasing hormone (LHRH). Overall, approximately 30% of girls with Turner syndrome have some spontaneous pubertal development.

Turner syndrome should be considered in individuals with primary or secondary amenorrhea and in adult women with unexplained infertility, particularly when such individuals also have a short stature.

Considering the fact that gonadal dysgenesis is the most common cause of primary infertility, and more importantly, with the short stature, gonadal (ovarian) dysgenesis is most likely to underlie the condition. A Tanner II stage breast advocates the diagnosis; however, even more developed breasts do not exclude the diagnosis. Primary amenorrhea and short stature remain the most important pointers towards ovarian dysgenesis.

OPTION A: A 17-year-old amenorrheic girl with a height of 140 is far beyond parameters of normal puberty and cannot be reassured as just having a normal variation in puberty.

OPTION B and C : Mullerian agenesis and hypothalamic hypogonadism are not associated with short stature.

OPTION E : Pituitary tumors are the cause of primary amenorrhea in less than 0.5% of cases. Short statue is not a characteristic feature.

  • Medscape - Turner Syndrome
  • UpToDate - Clinical Manifestations and diagnosis of Turner syndrome
  • UpToDate - Evaluation and management of primary amenorrhea
How well did you know this?
1
Not at all
2
3
4
5
Perfectly
63
Q

A 16-year-old girl presents with primary amenorrhea. On examination, she has satisfactory development of breasts and axillary and pubic hair for her age. She is virgin and does not allow you to perform a vaginal exam for further assessments to exclude an imperforated hymen. Which one of the following would be the most appropriate next step in management?

A. CT scan of the pelvis.
B. Ultrasound scan of the pelvis.
C. Convince her to a vaginal exam.
D. Refer her to another doctor.
E. Serum follicle stimulating hormone (FSH).

A

B. Ultrasound scan of the pelvis

Physical exam is an important part in assessment of females with primary amenorrhea. It should include examination for assessment of presence and development of secondary sexual characteristics, genitalia and clitoris, and a vaginal exam to check for anomalies of the vagina and hymen if feasible. A vaginal exam may reveal an imperforated hymen, absence of vagina (in vaginal agenesis), or short vagina (in mullerian agenesis).

Ultrasonography of the pelvis is another very important part in assessment of every patient with primary amenorrhea, particularly if breasts are over Tanner II stage.
With ultrasound the following are checked:
-Presence or absence of the uterus
-Possible mullerian anomalies
-Ovaries
-Presence of a transverse vaginal septum
-Evidence of menstrual flow obstruction

Since this patient does not consent to vaginal examination, pelvic ultrasonography can provide details as a surrogate, and additionally, further information regarding pelvic structures. Therefore, in this situation, a pelvic ultrasound is the next best in management.

Ideally, both transvaginal and transabdominal sonography should be performed unless the patient’s virginity or refusal precludes the former. It should be noted that if the patient had consented to vaginal exam, the need for ultrasonography was not eliminated.

NOTE - vaginal examination should be restricted to women who are or have been sexually active.

With satisfactory development of breasts and pubic and axillary hair, it is very likely that the cause of this girl’s amenorrhea is mullerian agenesis. Vaginal transverse septum and imperforated hymen are also possible; however, with these conditions cyclical pelvic pain and/or discomfort and/or suprapubic mass would be expected.

OPTION A : CT scan of the pelvis adds nothing to ultrasound findings at this stage. It is more expensive and unnecessarily poses the patient to radiation exposure.

OPTION C : Convincing the patient to the examination as long as ultrasound can provide almost the same efficacy is not appropriate.

OPTION D : Referral to another doctor just because the patient does not consent to vagial exam is not an approriate action.

OPTION E : Serum FSH is unlikely to provide further clarification. With reassuring development of secondary sexual characteristics, FSH is very likely to be within the normal range. Measuring FHS might be indicated somewhere down the road but not at this stage.

  • UpToDate - Evaluation and management of primary amenorrhea
  • NSW Health – Amenorrhoea
How well did you know this?
1
Not at all
2
3
4
5
Perfectly
64
Q

A 16-year-old ballet dancer girl is brought to your office by her mother because she has not had any periods until now. On examination, the girl has a body mass index (BMI) of 16, and breasts at Tanner II stage. No axillary or pubic hair is present. Which one of the following should come next in management?

A. Reassurance.
B. Hormonal assays.
C. Abdominal pelvic ultrasound.
D. Combined oral contraceptive pills (COCPs).
E. Referral to adolescent health center.

A

B. Hormonal assays

Puberty in girls starts with breast development. Growth spurt then occurs and is followed by axillary and pubic hair development. Menstruation often does not take place until two years after the onset of puberty.

There are a few points in the history that can narrow down the diagnosis. Firstly, this girl has Tanner II breast at the age of 16 which shows lack of breast development as the first sign of puberty (Tanner II breasts after the age of 13 means absence of breast development).

Another point to consider is the fact that this girl has no pubic or axillary hair. This finding can be either due to arrested or delayed progression of puberty, or androgen insensitivity presented with breast development, but absence of a uterus and axillary and pubic hair. In this girl, who is a professional ballet dancer with a BMI of 16, the former is more likely.

In approaching the female patients with primary amenorrhea, the diagnostic approach depends on the presence or absence of secondary sexual characteristics. In patients with absence of such characteristics, hormonal studies including FSH, LH, TSH, and prolactin is the most appropriate next step in investigations, while in the presence of such characteristics, a pelvic ultrasound for uterine or menstrual outflow abnormalities come first.

This girl has amenorrhea and absence of secondary sexual characteristics; therefore, hormonal assay is the most appropriate next step in management.

Low body weight, excessive physical exercise, and starvation are associated with hypogonadotropic hypogonadism (central hypogonadism). This is frequently observed in patients with anorexia nervosa or bulimia nervosa, but also can be seen in female athletes. Hypogonadotropic hypogonadism may result in primary or secondary amenorrhea. If it occurs during puberty, it could impede puberty from normal progression.

Although a decreased serum FSH will be the expected finding associated with the condition, with breast buds present, the most appropriate next step in management is a pelvic ultrasound. Along with physical examination, a pelvic ultrasound is always the very first initial assessment of patients with primary amenorrhea who have secondary sexual characteristics (evident by Tanner II or higher stages of breast development). Ultrasound helps to check for the presence of a uterus, mullerian agenesis, imperforated hymen, etc. as the potential causes of primary amenorrhea. Ideally, both transvaginal and transabdominal pelvic ultrasound should be performed unless virginity precludes transvaginal ultrasound.

OPTION A : This girl lags behind normal puberty and cannot be reassured unless the cause is ensured to be constitutional delay of puberty.

OPTION C : Pelvic ultrasound would be the correct answer if secondary sexual characteristics were present.

OPTION D : Combined oral contraceptives (COCs) have no role in diagnosing or treatment of this girl.

OPTION E : Referral to adolescent health center may be indicated later once other causes of primary amenorrhea other than those related to excessive exercise, weight loss, and eating disorders are excluded with high certainty. It is not appropriate at this stage when other probable causes of primary amenorrhea are not yet fully investigated.

  • American Family Physicians - Amenorrhea: An Approach to Diagnosis and Management
  • NSW Health - Amenorrhoea
How well did you know this?
1
Not at all
2
3
4
5
Perfectly
65
Q

A 32-year-old woman presents to the emergency department with vaginal bleeding at 28 weeks pregnancy. After prompt investigations, the diagnosis of placental abruption is made. Of the following options, which is not expected to be caused by the condition?

A. Tense and tender uterus.
B. Blood pressure of 180/110mmHg.
C. Shock out of proportion to the amount of vaginal bleeding.
D. Fetal head mobile above the pelvic brim.
E. Fetal demise.

A

B. Blood pressure of 180/110mmHg

Placental abruption occurs when a normally implanted placenta (not in the lower uterine segment) separates from the uterine wall before delivery of the fetus. Separation can be partial or complete. The condition is associated with vaginal bleeding of varying extent and a tense tender uterus (option A) .

Commonly, the bleeding is overt and external (80% of cases); however, at occasions the bleeding may be concealed with blood accumulating between the uterine wall and the placenta. This can lead to uterine hematoma and an increased size of the fundal height. Concealed bleeding, if severe, can result in hemorrhagic shock not justified by the amount of external bleeding (option C) . Fetal head can be felt in the pelvic brim or above it (option D) , depending on the gestational age. An already engaged head can lose its position if placental abruption occurs.

Placental abruption is a serious condition capable of putting both the mother and the fetus in jeopardy. In utero fetal demise (option E) is a major concern.

There are several conditions associated with increased risk of placental abruption, with hypertension being one, but placental abruption does not result in hypertension.

*http://www.uptodate.com/contents/placental-abrupti
*http://www.aafp.org/afp/2004/1001/p1303.html

How well did you know this?
1
Not at all
2
3
4
5
Perfectly
66
Q

A 25-year-old woman, on oral contraception pills, presents to your office because she is concerned about 3 episodes of painless vaginal bleeding in the past week. She has had regular cervical screening tests with normal results to date, with the last one taken 12 months ago. Which one of the following is the most likely cause of her postcoital bleeding?

A. Chlamydia infection.
B. Cervical cancer.
C. Endometrial cancer.
D. A cervical polyp.
E. Cervical ectropion.

A

E. Cervical ectropion

Postcoital bleeding can certainly be caused by Chlamydia cervicitis, a cervical polyp or cervical carcinoma. However, the most likely cause in this instance is a cervical ectropion, where the single layer of columnar epithelium of the endocervix has extended onto the ectrocervix and is exposed to trauma during coitus.

Cervicitis and ectropion are the two most common causes of postcoital bleeding in women younger than 30 years. Although Chlamydia infection (option B) and cervicitis can be a cause as well, absence of other symptoms of Chlamydia infection such as discharge makes such diagnosis less likely than an ectropion cervix.

Cervical polyps (option D) are more common in women of reproductive age, especially those over 30-40 years.

Cervical cancer (option B) and endometrial cancer (option C) are far less likely at this age. Cervical intraepithelial neoplasia does not cause postcoital bleeding per se. However, such lesions often reside in ectropion and can bleed with contact during sexual intercourse.

Other possible causes are pregnancy and its complications such as abortion, HRT, and hormonal contraception.

TOPIC REVIEW
TABLE age-specific common causes irregular uterine bleeding

  • Cancer Australia - Abnormal vaginal bleeding in pre- and peri-menopausal women
  • AMC Handbook of Multiple Choice Questions – page 530
How well did you know this?
1
Not at all
2
3
4
5
Perfectly
67
Q

An 18-year-old girl presents to your office with painful menses for the past 12 months. She has been prescribed NSAIDs initially and then OCPs after the NSAIDs failed to control her symptoms. OCP has been unable to control the painful menses as well. Which one of the following would be the most appropriate next step in management?

A. Transabdominal ultrasound.
B. Transvaginal ultrasound.
C. CT scan of the pelvis and abdomen.
D. Dilation and curettage.
E. Laparoscopy.

A

B. Transvaginal ultrasound

The main complaint in this patient is with painful periods, also termed dysmenorrhea.

Dysmenorrhea can be divided into two broad categories of primary and secondary.
Primary dysmenorrhea is defined as recurrent, crampy pain occurring with menses in the absence of identifiable pelvic pathology.
Secondary dysmenorrhea is menstrual pain associated with underlying pelvic pathology such as endometriosis.

Primary dysmenorrhea usually begins in adolescence after the establishment of ovulatory cycles. Primary dysmenorrhea is caused by myometrial activity and consequent uterine ischemia. This myometrial activity is modulated and augmented by prostaglandin synthesis; therefore, non-steroidal anti-inflammatory drugs (NSAIDs), by inhibition of prostaglandin synthesis, are the first-line treatment option.

Oral contraceptive pills (OCPs) are used as second-line if NSAIDs fail. OCPs can also be used as the treatment of choice (first-line) if contraception is desired as well. OCPs prevent menstrual pain by suppressing ovulation and decreasing uterine prostaglandin levels. An additional mechanism is the reduction of menstrual flow after several months of use.

If treatment with either of these medications fails after 2 or 3 menstrual cycles, a course of treatment with the other modality should be used. Treatment with both hormonal contraceptives and NSAIDs may be effective in women who do not respond to either drug alone.

Adolescents who fail to respond to first- or second-line treatment, have recurrent symptoms, or have worsening symptoms should be re-evaluated for the possible causes of secondary dysmenorrhea such as endometriosis, uterine leiomyomas, polyps, or other pelvic pathologies.

In this patient, treatment has failed despite use of both NSAIDs and OCP; therefore, secondary dysmenorrhea should be considered and investigated. When pelvic pathology is suspected, abdominal and transvaginal ultrasonography should be used as first-line investigation. They are inexpensive, effective, and readily available. Transvaginal ultrasound is more accurate and the preferred option if possible.

OPTION A : Transabdominal ultrasound is inferior to transvaginal ultrasound for evaluation of dysmenorrhea; but it can be used instead of transvaginal ultrasound if transvaginal is not possible e.g., in virgin girls.

OPTION C : CT scan and MRI are not routinely used for assessment of secondary dysmenorrhea.

OPTION D and E : Dilation and curettage or laparoscopy might be indicated at some stage after initial evaluation points towards a specific pathology such as endometriosis or endometrial problems, but not as an initial step.

  • MedScape - Dysmenorrhea
  • UpToDate - Primary dysmenorrhea in adolescents
  • AAFP - Diagnosis and Initial Management of Dysmenorrhea
How well did you know this?
1
Not at all
2
3
4
5
Perfectly
68
Q

A 34-year-old primigravida woman presents to the emergency department at 36 weeks gestation after she passed approximately 200ml of clotted blood per vagina at home 45 minutes ago. Currently, there is no active vaginal bleeding. On examination, she has a pulse rate of 120 bpm and blood pressure of 90/60 mmHg. Abdominal examination reveals a very tense uterus and moderate tenderness on palpation. Fetal heart sound is audible at a rate of 140 bpm. Which one of the following is the most appropriate next step in management?

A. Intravenous fluids.
B. Ultrasound.
C. Cardiotocography (CTG).
D. Induction of labor.
E. Immediate cesarean section.

A

A. Intravenous fluids

The clinical picture of a tense and tender uterus in the presence of vaginal bleeding is suggestive of placental abruption as the most likely diagnosis. The important clue is tachycardia and hypotension unproportionate to the amount of vaginal bleeding indicating that the bleeding is concealed. In concealed bleeding blood accumulates between the placenta and the uterine wall.

In the presence of hypotension and tachycardia, fluid resuscitation with intravenous isotonic fluids is the most appropriate next step in management.

Blood typing and cross match should also be performed and packed cells reserved in case of severe bleeding requiring blood transfusion.

An ultrasound scan (option B) is then indicated for assessment of the placental and fetal status. Continuous CTG (option C) should be performed for fetal monitoring. Labor induction option D) or cesarean section (option E) are indicated if there is maternal or fetal jeopardy.

*http://www.sahealth.sa.gov.au/wps/wcm/connect/b1c6
*http://3centres.com.au/guidelines/complications-in
*http://www.kemh.health.wa.gov.au/development/manua

How well did you know this?
1
Not at all
2
3
4
5
Perfectly
69
Q

A 29-year-old woman with history of schizophrenia for the past nine years presents to your practice, accompanied by her husband, because of amenorrhea of 2 months duration. She is currently stable on clozapine. Which one the following is most important step in management?

A. Ultrasound.
B. Measurement of serum follicle stimulating hormone (FSH).
C. Measurement of serum luteinizing hormone (LH).
D. Urine pregnancy test.
E. Full blood count (FBC).

A

D. Urine pregnancy test

Universally, the most common cause of secondary amenorrhea is pregnancy. Hence, in every woman of reproductive age presenting with amenorrhea, the first thing to check is a urine pregnancy test.

An ultrasound or measuring FSH and LH may be used later (if indicated) for assessment of the condition, once pregnancy has been excluded.

FBC is used to monitor adverse outcomes of clozapine such as neutropenia or agranulocytosis. It is not indicated for evaluation of amenorrhea.

  • NSW Health – Amenorrhoea
How well did you know this?
1
Not at all
2
3
4
5
Perfectly
70
Q

A 20-year-old woman is planning to conceive and has presented for your advice. She has the history of juvenile myoclonic epilepsy, and has been stable on sodium valproate. Despite your full explanations about the risks of the teratogenicity of antiepileptic drugs, she insists that she wants to become pregnant and asks you to show her a way. Which one of the following would be the most appropriate advice?

A. Continue sodium valproate.
B. Discontinue sodium valproate.
C. Switch to carbamazepine.
D. Switch to lamotrigine.
E. Switch to phenytoin.

A

A. Continue sodium valproate

When approaching a woman, who wishes to become pregnant while on antiepileptic drugs, two major groups of epilepsies should be distinguished because they typically respond differently to different drugs.

Partial epilepsies respond to most antiepileptic drugs, but for idiopathic generalized epilepsies, especially in juvenile myoclonic epilepsy, seizure can be controlled with a reasonably low dose of sodium valproate.

Option D: Although lamotrigine may be helpful, it is not as effective as sodium valproate, and even sometimes worsens the myoclonic seizures of juvenile myoclonic epilepsy. Therefore, substitution of sodium valproate with lamotrigine is NOT the right advice.

Topiramate and levetiracetam may be effective in idiopathic generalized epilepsy, while carbamazepine , phenytoin and gabapentin may worsen some seizure types, especially myoclonic and absence seizures. For some women with idiopathic generalized epilepsies, there may be no effective alternative to sodium valproate. Cessation of sodium valproate is associated with recurrence of the generalized seizures, especially juvenile myoclonic epilepsy; therefore, it is not advisable to discontinue the drug.

On the other hand, sodium valproate has the highest teratogenicity potential among antiepileptic drugs. The Australian Pregnancy Register has reported the risk to be as high as 16% for the first trimester. Sodium valproate should therefore be avoided in women of reproductive age. If a patient is willing to become pregnant, she should be fully informed of the risk of teratogenicity and the decision is left to her. If she decides to accept the risks, and pregnancy is unavoidable, the lowest effective dose should be used.

If sodium valproate dose has been lowered to a minimum during pregnancy to reduce teratogenesis, the prepartum effective dose may need to be re-established before the onset of labor. This is a time of increased seizure risk, especially in patients with idiopathic generalized epilepsy who are very sensitive to sleep deprivation. Breastfeeding is safe during valproate therapy.

  • Australian Prescriber - Antiepileptic drugs in pregnancy and lactation
How well did you know this?
1
Not at all
2
3
4
5
Perfectly
71
Q

A pregnant elementary school teacher, aged 24 years, presents to your GP office after she found that one of the students in her class has been recently diagnosed with rubella by his GP; the diagnosis however was not confirmed by serologic studies. Her last vaccination against rubella was when she was 12 years. Which one of the following would be the next best step in management?

A. No action is required; reassure her.
B. Check rubella serology.
C. Confirm the diagnosis of rubella in the sick child with rubella serology testing.
D. Give her a booster dose of MMR vaccine.
E. Advise her to terminate the pregnancy.

A

B. Check rubella serology

All pregnant women with contact with a person, who has confirmed diagnosis of rubella infection or is suspicious of having the disease, should be offered serologic testing (both IgM and IgG) for rubella as the most appropriate next step in management. However, if the patient had been screened for rubella IgG in the first trimester and had a positive IgG with titers >10 IU/ml, no further serologic testing would have been indicated.

OPTION A : Taking no action is incorrect as serologic testing is indicated in all pregnant patients after contact to a rubella or rubella-like infection, unless the woman has been documented to be immune in the current pregnancy using IgG titers as mentioned earlier.

OPTION C : There is no need for serologic testing in the index case (the child in this scenario).

OPTION D : Rubella-containing vaccines (MMR, MMRV) are live attenuated vaccines and contraindicated throughout pregnancy.

Advising the patient to terminate the pregnancy is recommended if there is a with definite diagnosis of rubella infection (confirmed by serology) in the first trimester, as it is associate with fetal anomalies in almost 85% of cases.

TOPIC REVIEW

Rubella, also called German measles, is usually a mild infectious disease in children and adults. It is clinically difficult to diagnose due to transient clinical features that are also common to some other viral infections.

Rubella has an incubation period of 14-23 days. The infectivity period starts from one week before until 4 days after onset of rash.

Rubella is asymptomatic in 25 to 50 % of cases. If there are symptms, they may include:
-Low grade fever
-Transient erythematous rash
-Lymphadenopathy involving post-auricular and sub-occipital nodes
-Occasionally arthritis and arthralgia (commonly observed in women of child-bearing age)
-Rarely neurological disorders and thrombocytopenia
-Rash - the rash characteristically begins on the face and spreads to the trunk and extremities. It will usually resolve within three days in the same order in which it appeared (face first and then body)

During pregnancy, maternal viremia may occur 5 to 7 days after exposure with spread of the virus throughout the body as well as transplacental infection of the fetus.

Vertical transmission (transplacental) from mother to fetus can only occur during viremia of a primary infection. The effect of maternal infection on the fetus depends on the gestational age:

  • <8 weeks: up to 85% of fetuses will be infected and all will have clinical manifestations of the congenital rubella syndrome.
  • 8≤ but <12 weeks: 50-80% of fetuses will be infected and of those 65-85% will be clinically affected.
  • 13-16 weeks: 30% of the fetuses will be infected and of those, 1/3 will have sensorineural deafness.
  • 16-19 weeks: 10% of the fetuses will be infected. Clinical features are rare, yet deafness can be a possibility.
  • > 19 weeks: there is no apparent risk.

NOTE - reinfection is associated with far less risk of fetal infection (~5%.)

Abnormalities associated with congenital rubella syndrome include:
-Central nervous system dysfunction (10-25%, intellectual impairment, developmental delay, microcephaly)
-Eye abnormalities (10-25%, cataracts, retinopathy, glaucoma, strabismus, micropthalmos)
-Sensorineural deafness (60-75%)
-Cardiac abnormalities (10-20%, PDA, PA stenosis)
-Intrauterine growth restriction, short stature
-Inflammatory lesions of the brain, liver, lungs and bone marrow

All pregnant women, who have contact with rubella or clinical features consistent with rubella – like illness should be screened for the presence of rising antibody titre (IgG) and rubella specific IgM, regardless of previous history of vaccination or infection. Interpretation of the results and corresponding action is as follows:

SEE TABLE BELOW

If maternal infection is confirmed, antenatal testing is recommended at least 6 weeks after known maternal infection. Rubella PCR, rubella culture and fetal IgM can be performed following chorionic villus sampling (CVS) / amniocentesis or cordocentesis; however, due to the very high risk of fetal infection and consequent fetal anomalies, if maternal infection occurs in the first trimester, termination of pregnancy should be recommended.

NOTE - In all pregnant women, serologic testing for rubella (IgG only) should be offered and performed in the first antenatal visit. With titers ≥10 IU/ml, risk of reinfection is minimal. However, if the titers are ≤15, vaccination after delivery should be performed.

*https://www.sahealth.sa.gov.au/wps/wcm/connect/d81
*https://www.asid.net.au/documents/item/368

How well did you know this?
1
Not at all
2
3
4
5
Perfectly
72
Q

A kindergarten teacher presents to your clinic at 10 weeks pregnancy. Recently, she has found that one of the children in her class has rubella infection. Which one of the following is the most appropriate next step in management?

A. Exclude the sick child and reassure her.
B. Terminate the pregnancy before 13 weeks.
C. Check anti-rubella IgM and IgG levels.
D. Tell her to come back if she becomes symptomatic.
E. Give her MMR vaccine.

A

C. Check anti-rubella IgM and IgG levels

All pregnant women should be offered serologic testing for rubella as the most appropriate next step in management if there is exposure to rubella infection through contact with an infectious case. This is true for all pregnant women regardless of the past history of immunization or infection with rubella. However, if the patient was screened for rubella IgG in the first trimester and had a positive IgG with titers >10 IU/ml no further serologic testing would have been indicated.

OPTION A : Exclusion of the child until full recovery or at least four days after the onset of the rash is recommended, but this will not affect the management plan for the woman. She is at risk of contracting rubella and its detrimental impacts on her fetus, and cannot be reassured until investigations exclude rubella infection.

OPTION B : Termination of the pregnancy is recommended for pregnant woman with established diagnosis of rubella within the first trimester of pregnancy, confirmed with serology studies. Rubella infection in the first trimester is associated with severe fetal anomalies in 85% of cases.

OPTION D : Telling her to come back if she is symptomatic is inappropriate. She should undergo serologic studies now.
OPTION E : Rubella-containing vaccines (MMR, MMRV) are live attenuated vaccines and are contraindicated during pregnancy.

TOPIC REVIEW

Rubella, also called German measles, is usually a mild infectious disease in children and adults. It is clinically difficult to diagnose due to transient clinical features that are also common to some other viral infections.

Rubella has an incubation period of 14-23 days. The infectivity period starts from one week before until 4 days after onset of rash.

Rubella is asymptomatic in 25 to 50 % of cases. If there are symptms, they may include:
-Low grade fever
-Transient erythematous rash
-Lymphadenopathy involving post-auricular and sub-occipital nodes
-Occasionally arthritis and arthralgia (commonly observed in women of child-bearing age)
-Rarely neurological disorders and thrombocytopenia
-Rash - the rash characteristically begins on the face and spreads to the trunk and extremities. It will usually resolve within three days in the same order in which it appeared (face first and then body)

During pregnancy, maternal viremia may occur 5 to 7 days after exposure with spread of the virus throughout the body as well as transplacental infection of the fetus.

Vertical transmission (transplacental) from mother to fetus can only occur during viremia of a primary infection. The effect of maternal infection on the fetus depends on the gestational age:

  • <8 weeks: up to 85% of fetuses will be infected and all will have clinical manifestations of the congenital rubella syndrome.
  • 8≤ but <12 weeks: 50-80% of fetuses will be infected and of those 65-85% will be clinically affected.
  • 13-16 weeks: 30% of the fetuses will be infected and of those, 1/3 will have sensorineural deafness.
  • 16-19 weeks: 10% of the fetuses will be infected. Clinical features are rare, yet deafness can be a possibility.
  • > 19 weeks: there is no apparent risk.

NOTE - reinfection is associated with far less risk of fetal infection (~5%.)

Abnormalities associated with congenital rubella syndrome include:
-Central nervous system dysfunction (10-25%, intellectual impairment, developmental delay, microcephaly)
-Eye abnormalities (10-25%, cataracts, retinopathy, glaucoma, strabismus, micropthalmos)
-Sensorineural deafness (60-75%)
-Cardiac abnormalities (10-20%, PDA, PA stenosis)
-Intrauterine growth restriction, short stature
-Inflammatory lesions of the brain, liver, lungs and bone marrow

All pregnant women, who have contact with rubella or clinical features consistent with rubella – like illness should be screened for the presence of rising antibody titre (IgG) and rubella specific IgM, regardless of previous history of vaccination or infection. Interpretation of the results and corresponding action is as follows:
(SEE TABLE BELOW)

If maternal infection is confirmed, antenatal testing is recommended at least 6 weeks after known maternal infection. Rubella PCR, rubella culture and fetal IgM can be performed following chorionic villus sampling (CVS) / amniocentesis or cordocentesis; however, due to the very high risk of fetal infection and consequent fetal anomalies, if maternal infection occurs in the first trimester, termination of pregnancy should be recommended.

NOTE - In all pregnant women, serologic testing for rubella (IgG only) should be offered and performed in the first antenatal visit. With titers ≥10 IU/ml, risk of reinfection is minimal. However, if the titers are ≤15, vaccination after delivery should be performed.

*https://www.sahealth.sa.gov.au/wps/wcm/connect/d81
*https://www.asid.net.au/documents/item/368

How well did you know this?
1
Not at all
2
3
4
5
Perfectly
73
Q

A 24-year-old woman presents at 39 weeks gestationy with complaint of intermittent watery vaginal discharge, which started last night after she had sex with her husband. On speculum examination, pooling of straw-colored fluid in the posterior vaginal fornix is noted. The cervical os is closed. Furthermore, there is leak of liquor from cervical os when she is asked to cough or strain. Which one of the following best explains these clinical findings?

A. Retained semen from the last night sex.
B. Vaginal infection.
C. Premature rupture of membranes (PROM).
D. Urine leakage.
E. Cervical insufficiency.

A

C. Premature rupture of membranes (PROM)

The presentation is classic for premature rupture of membranes (PROM), probably caused by trauma during intercourse.

By definition, premature rupture of membranes (PROM) is defined as rupture of embryonic membranes before the labor regardless of the age of pregnancy. If PROM occurs before 37 weeks gestation, it is termed as premature PROM (PPROM).

The classic presentation of rupture of the membranes (ROM), regardless of gestational age, is with a sudden gush of watery fluid per vagina; however, many women, such as in this scenario may complain of continuous or intermittent leakage of fluid or the sensation of wetness within the vagina or on the perineum. Presence of liquor flow from the cervical os or pooling in the posterior vaginal fornix is pathognomonic.

If there is doubt, further tests can be conducted to clinch the diagnosis:

Nitrazine test – the normal vaginal pH is between 4.5 and 6.0, whereas amniotic fluid is more alkaline, with a pH of 7.1 to 7.3. Nitrazine paper will turn blue when the pH is above 6.0; however, the presence of contaminating substances such as blood, semen, alkaline antiseptics can also cause nitrazine paper to turn blue, giving a a false-positive result. Bacterial vaginosis alkalinizes the vagina as well.

Amnisure test - a non-invasve and approximately 99% accurate for definite diagnosis of PROM. It is based on immunoassay, can be done at any gestational age and does not need a speculum exam.
NOTE - The above tests are currently used in practice in Australia.

Ferning on microscopy – using a swab, fluid is obtained from the posterior fornix or vaginal sidewalls. Once the fluid is dried on the slide, the slide is checked for ferning (arborization) under low-power microscope. The presence of ferning indicates PROM. It is noteworthy that vaginal blood can obscure presence of ferns, and that cervical mucus can result in false-positive result if the cervical os has been swabbed. Ferning test is no more performed in Australia.

NOTE - ultrasonography is not used merely for diagnosis of PROM; however, in rare cases when the diagnosis cannot be certain using history, physical exam, and mentioned tests ultrasonography may help. An example is when the patient’s history suggests PROM, but physical findings findings are not consistent with the diagnosis. Ultrasonography, however, is a part of investigation plan for assessment of fetal well-being, the position of the fetus, placental location, estimated fetal weight and presence of any anomalies in PROM and PPROM.
Retained semen will have a different appearance and will not result in findings in the clinical scenario.

OPTION B : Infections will be associated with characteristic features including purulent cervical discharge, malodorous vaginal discharge, etc. Pooling of clear fluid in the posterior fornix is pathognomonic for ROM.

OPTION E : Urine leakage is common during the pregnancy, but it is inconsistent with such a clinical scenario.

OPTION E : Although cervical insufficiency is associated with increased vaginal discharge, absence of other findings on speculum exam such as cervical dilation, and bulging membranes makes this diagnosis unlikely.

  • https://www.ranzcog.edu.au/Statements-Guidelines/Obstetrics/Preterm-Prelabour-Rupture-of-Membranes
How well did you know this?
1
Not at all
2
3
4
5
Perfectly
74
Q

In which one of the following women groups, endometrial hyperplasia is most likely to be found?

A. An ovulating woman.
B. An obese diabetic woman.
C. A woman on cyclic combined oral contraceptive pills.
D. A woman on Depo-Provera® for treatment of endometriosis.
E. A woman with an intrauterine device.

A

B. An obese diabetic woman

Endometrial hyperplasia is characterized by proliferation of endometrial glands. The condition may progress to or coexist with endometrial carcinoma. Endometrial hyperplasia virtually always results from chronic exposure of endometrial tissue to estrogen (sometimes in excess amounts) unopposed by the counterbalancing effects of progesterone.

Excess estrogen exposure can be exogenous or endogenous. Exogenous estrogen exposure includes postmenopausal estrogen therapy and tamoxifen, while endogenous exposure may result from obesity anovulatory menstrual cycle, or estrogen-secreting tumors.

Of the options, diabetes and obesity are associated with increased risk of endometrial hyperplasia. Diabetes mellitus type 2 is associated with increased levels of insulin-like growth factor (IGF). IGF can cause endometrial proliferation. Increased body mass index (BMI) causes excessive peripheral conversion of androgens to estrogen in adipose tissue.

NOTE - Risk factors for endometrial hyperplasia are essentially the same.

Risk factors for endometrial cancer/hyperplasia and their relative risk are summarized in the following table:

Risk factor (Relative Risk, %)

-Unopposed estrogen therapy (2-10%)
-Increasing age (1.4%) (50-70 years)
-Late menopause (>55 years) (2%)
-Nulliparity (2%)
-Chronic anovulation (e.g. in PCOS) (3%)
-Obesity (2-4%)
-Diabetes mellitus (2%)
-Tamoxifen therapy (2%)
-Lynch syndrome (hereditary nonpolyposis colorectal cancer) (22-50%) (lifetime risk)
-BRCA1/BRCA2 gene mutation (20%)
-Early menarche (Not specified)
-Estrogen secreting tumor (Not specified)
-Family history of endometrial, ovarian, breast, or colon cancer (Not specified)

OPTION A : Ovulating women has cyclical exposure to progesterone. This leads to reduces risk of endometrial hyperplasia. For the same reason, women on cyclical combined oral contraceptive are unlikely to be at increased risk of endometrial hyperplasia.

OPTION C : Combined oral contraceptive pills (COCs) contain progesterone that counterbalances the effects of estrogen. Women on COCs do not have an increased risk of endometrial hyperplasia/cancer.

OPTION D : Depo-Provera® is injectable progesterone. Progesterone counterbalances the effect of estrogen on endometrial hyperplasia. It is a protective factor rather than risk factor.

OPTION E : Intrauterine devices have not been shown to be associated with increased risk of endometrial hyperplasia.

  • UpToDate - Classification and diagnosis of endometrial hyperplasia
  • RCOG - Management of Endometrial Hyperplasia
How well did you know this?
1
Not at all
2
3
4
5
Perfectly
75
Q

Which one of the following is the most likely finding in a pregnant woman with placental abruption?

A. Decreased fetal movements.
B. Preterm labor.
C. Vaginal bleeding.
D. Uterine tenderness.
E. Shock.

A

C. Vaginal bleeding

Placental abruption is defined as the premature separation of the placenta from the uterus and presents with bleeding, uterine contractions, and fetal distress. Placental abruption is a significant cause of third-trimester bleeding and is associated with fetal and maternal mortality and morbidity. The condition should be considered as a differential diagnoses in all pregnant women with vaginal bleeding in the second half of the pregnancy.

Vaginal bleeding is the most common presenting symptom reported by 80% of women with placental abruption. In 20% of women with placental abruption, vaginal bleeding is concealed; therefore, absence of vaginal bleeding does not exclude placental abruption.

Frequency of symptoms in women with placental abruption is as follows:
-Vaginal bleeding – 80%
-Abdominal or back pain and uterine tenderness – 70%
-Fetal distress – 60%
-Abnormal uterine contractions (e.g. hypertonic, high frequency) – 35%
-Idiopathic premature labor – 25%
-Fetal death – 15%

Exam findings may include:
-Vaginal bleeding
-Uterine contractions and/or tenderness
-Shock
-Absence of fetal heart sounds
-Increased fundal height do to an expanding hematoma

Shock is seen in class 3 placental abruption that represents 24% of all cases.

TOPIC REVIEW

Placental abruption can be classified clinically, based on the extent of separation (partial versus complete) or location of the separation (marginal versus central). Clinical classification of placental abruption, the prevalence of each class and its clinical features of each class is as follows:

Class 0 – asymptomatic Presentation
A diagnosis of class 0 is made retrospectively by finding an organized blood clot or a depressed area on a delivered placenta. Class 1 – Mild (represents approximately 48% of all cases)

Presentation
-No vaginal bleeding to mild vaginal bleeding
-Slightly tender uterus
-Normal maternal blood pressure and heart rate
-Absence of coagulopathy
-No fetal distress

Class 2 – moderate (represents approximately 27% of all cases)

Presentation
-No vaginal bleeding to moderate vaginal bleeding
-Moderate to severe uterine tenderness with possible tetanic contractions
-Maternal tachycardia with orthostatic changes in blood pressure and heart rate
-Fetal distress
-Hypofibrinogenemia (i.e. 50-250 mg/dL)

Class 3 - Severe (represents approximately 24% of all cases)

Presentation
-No vaginal bleeding to heavy vaginal bleeding
-Very painful tetanic uterus
-Maternal shock
-Hypofibrinogenemia (ie, < 150 mg/dL)
-Coagulopathy
-Fetal death

  • Medscape - Abruptio Placentae Clinical Presentation
How well did you know this?
1
Not at all
2
3
4
5
Perfectly
76
Q

A 28-year-old 10-week pregnant woman presents 24 hours after eruption of a rash, characteristic of measles. The history and exam findings establish the diagnosis with certainty. Which one of the following is the most appropriate next step in management?

A. Give MMR vaccine.
B. Administer immunoglobulin.
C. Contact tracing.
D. Check her antibody status.
E. No action is required.

A

C. Contact tracing

The measles (rubeola) virus is a single-stranded RNA virus of the family Paramyxoviridae.

Measles is highly infectious. The main route of transmission is by respiratory airborne droplets; however, it can rarely be passed on to others by means of articles soiled with respiratory droplets.
Measles has an incubation period of 10-14 days.

Initially, the illness presents with the following:
-Fever
-Malaise
-Cough
-Coryza (inflammation of the mucous membranes of the nose)
-Conjunctivitis
-Koplik’s spots (white spots, each surrounded by a red ring, found on the buccal mucosa)

Two to 4 days after the initial symptoms, a maculopapular rash develops. The rash typically starts from face and upper neck and later spreads to involve the whole body. The infectivity periods starts 2 days before the onset of the rash and continues for 4 days after the eruption.

Measles is often a severe disease. Complication may occur, and include otitis media in 7% and bronchopneumonia in 6% of immunocompetent patients. Acute encephalitis is a rare complication occurring in between 2 and 10 per 10,000 reported cases with measles in the general population, with an associated mortality rate of 10 -15 %. Around 15-40 % of survivors will have permanent brain damage.

In pregnant women measles can be associated with increased risk of the following:
-Preterm labor
-Spontaneous abortion
-Fetal / neonatal loss
-Maternal mortality

This woman has established diagnosis of measles and is already at increased risk of above-mentioned complications. There is no active treatment for her except conservative management; however, since measles is a ‘notifiable’ disease, notification should be made as soon as possible as the most appropriate next step in management.

OPTION A : MMR is vaccine has no role in management of established measles. Furthermore, MMR is a live attenuated vaccine and is contraindicated throughout the pregnancy.

OPTION B : Normal human immunoglobulin (NHIG) is used for prophylaxis in contacts of an index case with measles in whom MMR vaccine is contraindicated, e.g., pregnant woman.

OPTION D : Checking antibody status in this patient with established measles is of no use, and will not change the management plan.

OPTION E : Taking no action while measles is a notifiable disease is not appropriate. A mentioned before all cases of measles should be reported.

  • SA Health - Measles and measles contacts in pregnancy
How well did you know this?
1
Not at all
2
3
4
5
Perfectly
77
Q

A 27-year-old pregnant woman presents to your practice at 17 weeks gestation with complaint of eruption of a rash, following a 2-day history of malaise, low grade fever and rhinorrhea. You suspect measles and order serology tests for her. IgM against measles is positive; IgG is negative. Which one of the following is the most appropriate next step in management?

A. MMR vaccine.
B. Serologic testing of the fetus for measles.
C. Administration of natural human immunoglobulin (NHIG) within 6 days.
D. Contact tracing.
E. Repeat the serology in 2 weeks.

A

D. Contact tracing

The scenario describes a typical case of measles infection in a pregnant woman, confirmed by serologic studies. Positive IgM suggests acute infection, while negative IgG shows that the infection is in its early phase and seroconversion has not occurred yet.

After contracting measles, conservative management of the symptoms and its potential complications is the only management plan therapeutically.
Another very important issue is ‘notification’ and contact tracing. Measles is a notifiable disease and healthcare professionals are mandated to report cases of measles to the authorized public health units. Contact tracing is the main objective of this notification.

Control of measles relies on early diagnosis and notification of the cases, prompt isolation of infectious cases, and timely and effective identification of contacts, with provision of advice and post-exposure prophylaxis, as appropriate.

Since measles is primarily transmitted by air-borne means, a contact is defined as anyone, who has or may have shared the same air-space (enclosed area) for any length of time with an infectious case. Contact tracing should aim to identify those most susceptible to measles or at greater risk from infection, including infants, immunocompromised people and pregnant women.

In general, contacts may be prioritized in the following order, recognizing that it may not be feasible for the PHU to identify and arrange post-exposure prophylaxis for all susceptible contacts, given the constraints of time, resources and logistics:

*The household and other settings where people share communal facilities (for example, in a hospital, boarding school or military barracks)

*Early childhood education and care services, school or other educational settings where people share a classroom with the case

*People who shared a waiting area at the same time as the case (for example, patients in a healthcare facility’s waiting room and any people accompanying these patients) and people who used the waiting area or who were seen in the same consulting room for up to 30 minutes after the case left. The latter period is based on the recognition that normal room ventilation systems ensure that levels of airborne viruses and virus survival diminish rapidly.

*Work settings where work colleagues have shared the same work area or communal facilities at the same time as a case.

OPTION A : MMR vaccine is used for prevention of measles including post-exposure prophylaxis. It is not useful once measles is contracted. Even for prophylaxis, MMR was contraindicated throughout pregnancy. For those with contact to a case of measles, MMR vaccine within 72 hours of contact may have a protective effect. All measles-containing vaccines (MMR, MMRV) are contraindicated throughout pregnancy.

OPTION B : Serologic testing of the fetus is not useful because circulating maternal antibodies cross the placenta into fetal ciculation. A positive test does not mean that the fetus is infected.

OPTION C : NHIG is used as a post-measles exposure prophylactic means for those persons with contraindications to MMR vaccine such as pregnant women, premature babies, etc. It is not useful for treatment of established measles.

OPTION E : Repeating measles-specific serologic test is not necessary in this case because both symptoms and IgM levels indicate measles as the diagnosis. In general no test is indicated, unless the result has an impact on further management or is of prognostic value. Repeating serologic tests in a patient with established measles is futile.

*http://www.health.gov.au/internet/main/publishing.

How well did you know this?
1
Not at all
2
3
4
5
Perfectly
78
Q

A 27-year-old woman presents to the primary care center where you work at 26 weeks gestation after she has been noticing intermittent leakage of watery liquor per vagina for the past eight hours, especially after straining, coughing or sneezing. You perform a speculum vaginal exam which reveals clear fluid in the posterior vaginal fornix, as well as flow of liquid through the cervical os. Further evaluation, establishes preterm premature rupture of the membranes (PPROM) as the diagnosis. No uterine contraction is felt. There is a tertiary hospital 50 km away. Which one of the following is the most appropriate next step in management?

A. Administration corticosteroids.
B. Commencement of tocolysis.
C. Transferring her to the tertiary hospital.
D. Sending her home on oral antibiotics and warning signs.
E. Admission for observation.

A

A. Administration corticosteroids

The scenario is a classic presentation of preterm premature rupture of membranes (PPROM). Rupture of membrane harbingers labor in term or near term women. If ROM does not end up in established labor in 4 hours, it is termed premature ROM (PROM). In other words, PROM is defined by ROM befire the laor onset. If it occurs before 37 + 0 days weeks, PPROM is the preferred entity, as approach to PROM and PPROM are different.

The classic presentation of rupture of the membranes (ROM), regardless of the gestational age, is with a sudden gush of watery fluid per vagina or continuous or intermittent leakage of fluid or the sensation of wetness within the vagina or on the perineum. Presence of liquor flow from the cervical os or pooling in the posterior vaginal fornix is pathognomonic.

PPROM is associated with the following risks:
-Preterm labor
-Cord prolapse
-Placental abruption
-Chorioamnionitis
-Fetal pulmonary hypoplasia and other features of prematurity
-Limb positioning defects
-Perinatal mortality

Diagnosis of women with PPROM, as usual, begins with history and physical exam. Fetal heart rate auscultation and monitoring (+/- CTG) is another important step. A speculum vaginal exam should be performed to exclude cord prolapse, to assess the cervical length, and visualize pooling of liquor in the posterior fornix. High and low vaginal swabs should be obtained. Samples should be taken for nitrazine test.

Ultrasound may be used for estimation of amniotic fluid volume if the diagnosis is not certain, as well as for assessment of fetal wellbeing, estimated fetal weight, location of placenta, etc.

Once the diagnosis is certain the following should be considered as components of a management plan:

1. Maternal corticosteroids
Corticosteroids are effective in reducing adverse perinatal outcomes such as respiratory distress syndrome, intraventricular hemorrhage, and necrotizing enterocolitis. In case respiratory distress develops, duration of neonatal respiratory support is significantly reduced if corticosteroids have been administered.

Corticosteroids are indicated in the following situations:
-Gestational age is between 23+0d and 34+6d weeks if preterm labor is a concern
-Preterm birth is planned or expected within the next 7 days

Recommended regimens are IM betamethasone in two doses of 11.4 mg, 24 hours apart, to the woman. If betamethasone is unavailable, IM dexamethasone in two doses of 12 mg, 24 hours apart, is given.

When the gestational age is less than 32 + 6d, a single repeat dose of corticosteroid may be given seven days or more after the first dose if the woman is still considered to be at risk of preterm labor. Up to 3 repeat doses might be considered.

**2. Antibiotics **
The rationale for antibiotic prophylaxis is that infection appears to be both a cause and consequence of PPROM and is related to preterm delivery. The objective of antibiotic therapy is to reduce the frequency of maternal and fetal infection and delay the onset of preterm labor.

Antibiotic prophylaxis is indicated in all women with PPROM after low and high vaginal swabs are taken. In the absence of chorioamnionitis recommended antibiotic prophylaxis of choice is erythromicin given for 10 days intravenously.

Chorioamnionitis is a feared complication of PPROM. The diagnosis of chorioamnionitis relies on the clinical presentation and may be difficult in its early manifestations. The clinical picture may include maternal fever with 2 or more of the following:
-Increased white cell count (> 15 x 109 / L)
-Maternal tachycardia (> 100bpm)
-Fetal tachycardia (>160bpm)
-Uterine tenderness
-Offensive smelling vaginal discharge
-C-Reactive Protein > 40

3. Tocolytics
Chorioamnionitis is an absolute contraindication to tocolysis, while PPROM in the absence of intrauterine infection is only relatively contraindicated. The rationale for PPROM being so is that it is often difficult to exclude infections in the presence of PPROM because at times uterine contractions can be the only presenting symptom.

However, where contractions are present, nifedipine, as a tocolytic, may be started to prolong pregnancy for 48 hours while corticosteroid cover is established if there are no other signs of chorioamnionitis.

If not in labor or labor is not anticipated immediately, transfer to a tertiary care facility should be considered if possible. This ensures prompt care of the premature neonate if preterm labor and delivery occurs.

In this woman with PPROM at 26 weeks gestation, administration of corticosteroids as outlined above is the most appropriate next step in management.

For the moment, there are no signs and symptoms suggesting intrauterine infection; nonetheless, prophylactic antibiotics should be started to prevent infections as well as to delay labor.

OPTION B : Tocolysis using nifedipine was indicated if the woman was in labor. Cessation of labor for at least 48 hours provides a window for corticosteroid to establish its effects. In the absence of uterine contractions suggesting labor, tocolysis is not needed.

OPTION C : Transferring this woman to a tertiary hospital is an appropriate option after first doses of corticosteroid and antibiotic are given. Being in a tertiary hospital ensures optimal neonatal care in case of premature delivery.

OPTION D : Discharging this woman home on oral antibiotics is not an appropriate option because she needs investigations and fetal monitoring as well as close observation for dvelopment of any signs of infection and preterm labor.

OPTION E : Admission to a primary care center with no neonatal ICU (NICU) adds no benefit to the outcome.

*https://www.ranzcog.edu.au/Statements-Guidelines/Obstetrics/Preterm-Prelabour-Rupture-of-Membranes

How well did you know this?
1
Not at all
2
3
4
5
Perfectly
79
Q

A 23-year-old woman presents to your clinic at 26 weeks gestation after a sudden gush of watery fluid while cooking. Examination establishes the diagnosis of preterm premature rupture of membranes (PPROM). Fetal assessment using ultrasound and cardiotocography (CTG) is reassuring. Vaginal examination using a speculum shows fluid in the posterior vaginal fornix and a 1cm dilated cervical os. Uterine contractions are absent. There are two hospitals in the vicinity: a primary hospital and a tertiary hospital, 50km and150 km away, respectively. After administration of antibiotics and corticosteroids, which one of the following would be the most appropriate step in management of this patient?

A. Give him the warning signs and send her home.
B. Send her to the secondary care hospital 50km away.
C. Send her to the tertiary care chospital 150km away.
D. Tocolysis.
E. Labor induction.

A

C. Send her to the tertiary care chospital 150km away

PROM is defined as rupture of fetal membranes before labor at any gestational age. If PROM occurs before, it is termed premature PROM (PPROM). The main concerns in PPROM are preterm labor, which is associated with neonatal complications due to prematurity if labor establishes, and intrauterine infections with chorioamnionitis being of greatest significance.

If PPROM occurs before 34+6d, corticosteroids should be given to the mother in an attempt to decrease neonatal complications such as respiratory distress syndrome, intraventricular hemorrhage, and necrotizing enterocolitis.

Intravenous prophylactic antibiotics are given in anticipation of chorioamnionitis as a major complication of PPROM. Currently, intravenous erythromicin is the common practice is Australia as it is safe and mildy tocolytic.

Women with PPROM are at significantly elevated risk of preterm labor. The premature baby is required to be cared for in a center with optimal neonatal care in a neonatal intensive care unit (NICU). This can be found in a tertiary hospital. Therefore, if a woman with PPROM is not in labor and the labor is not anticipated immediately, the most appropriate next step will be transferring her to a tertiary hospital with NICU.

OPTION A : Expectant management at home may be considered for all man after 72 hours of hospitalization if there is:
-No further fluid leakage.
-Singleton pregnancy AND
-Cephalic presentation > 23 weeks AND
-Easy access to the hospital

This woman just had PPROM and has not been intensively evaluated in a hospital. Moreover, the nearest tertiary hospital is 150km away. Therefore, she does not meet criteria for expectant home care.

OPTION B : Secondary care hospitals do not have NICU and add no benefit to the care of the patient in this regard.

OPTION D : If the patient was in labor, evident by presence of uterine contractions and a dilating cervix, use of tocolytic agents were indicated to stop labor for at least 48 hours. This could provide a window for corticosteroids to exert their effects, as well as providing time for transfer to a tertiary care facility. Tocolysis is contraindicated in the presence of intrauterine infection, but only relatively contraindicated in PPROM; therefore, in the absence of intrauterine infection benefits of tocolysis outweigh risks of prematurity in women with (P)PROM and with uterine comtractions.

OPTION E : Induction of labor is not indicated due to prematurity unless:

-Signs of chorioamnionitis are present
-There is anterpartum hemorrhage
-The woman requests so
-There is neonatal jeopardy.

  • https://www.ranzcog.edu.au/Statements-Guidelines/Obstetrics/Preterm-Prelabour-Rupture-of-Membranes
How well did you know this?
1
Not at all
2
3
4
5
Perfectly
80
Q

Which one of the following drugs can potentially cause the most harmful neurological defects in a fetus if given during pregnancy?

A. Amphetamine.
B. Morphine.
C. Cocaine.
D. Amitriptyline.
E. Alcohol.

A

C. Cocaine

The role of cocaine in producing congenital malformations is controversial. Some studies suggest that congenital birth defects including those of face, nervous system and urinary system are possible. These problems, however, are very rare and somewhat negligible. Although cocaine is not associated with congenital anomalies, it has been shown, especially in late pregnancy, to be associated with increased risk of fetal intracranial hemorrhage with the worst possible outcome among the options.
The following features are definitely associated with maternal cocaine use during the pregnancy:
-IUGR
-Placental abruption
-Still birth
-Premature delivery
-Need for neonatal resuscitation -Developmental delay
-Intraventricular hemorrhage

OPTION A : The role of amphetamines in fetal malformations are controversial. While some studies have shown no teratogenic effects associated with amphetamines, other suggest that orofacial clefts are possible effects of amphetamine use during pregnancy; the incidence, however, has been exceedingly rare. Other complications of amphetamines during pregnancy include:
-Intrauterine growth restriction (IUGR)
-Preterm delivery
-Increased neonatal mortality and morbidity
-Hypertension and pre-eclampsia

OPTION B : Morphine or other opiates have a number of complications. One example is transitory CNS depression. This is not of major concern if promptly being taken care of.

OPTION D : Tricyclic antidepressants (e.g. amitriptyline) are not associated with congenital anomalies or concerning complications of the fetus or neonate and are rather safe for use during pregnancy.

OPTION E : Alcohol, if used in excess amounts can cause foetal alcohol syndrome which has developmental delays as part the complications spectrum; however, intracranial hemorrhage associated with cocaine use has a much worse outcome compared to alcohol.

TOPIC REVIEW

The following table lists the most important drugs associated with congenital malformations: (see photo)

1) An extremely rare congenital disorder presenting with limb malformation (dysmelia)

*http://www.merckmanuals.com/professional/gynecolog
* Llewellyn-Jones Fundamentals of Obstetrics and Gynaecology – Elsevier –Mosby - 9th Edition

How well did you know this?
1
Not at all
2
3
4
5
Perfectly
81
Q

A 22-year-old woman presents with complaint of amenorrhea of one year duration. Her menses were regular before. She has no other complaints or symptoms. Physical examination is inconclusive. Pelvis ultrasonography is unremarkable. Thyroid function tests show a TSH of 3.5mU/L (0.5-5mU’/L). Which one of the following is most likely to be the cause of her amenorrhea?

A. Mullerian agenesis.
B. Ovarian dysgenesis.
C. Hormonal dysfunction.
D. Hyperthyroidism.
E. Subclinical hyperthyroidism.

A

C. Hormonal dysfunction

Secondary amenorrhea is defined as cessation of periods after they have started. As opposed to primary amenorrhea is defined as lack of menstrual flow by age 14 and absence of secondary sexual characteristics or lack of menstrual flow by age 16 and presence of secondary sexual characteristics

Since this woman’s case is the cessation of previously present periods, she has secondary amenorrhea. Mullerian agenesis (option A) and ovarian dysgenesis (option B) result in primary amenorrhea. With previous menstruation, these two conditions are unlikely. Of the uterine causes, only Asherman’s syndrome can be the cause of secondary amenorrhea.

Hyperthyroidism can cause menstruation abnormalities, particularly amenorrhea or oligomenorrhea; however, a normal TSH excludes hyperthyroidism (option D) as the etiology of her amenorrhea. Subclinical hyperthyroidism is defined as a subnormal TSH while T3 and T4 are within normal range. This woman has a normal TSH level and does not have subclinical hyperthyroidism **(option E).

Of the options, the only explanation can be hormonal dysfunction. A hormonal dysfunction in this case could be due to conditions such as a pituitary tumor, hyperprolactinemia, ovarian insufficiency, or polycystic ovary syndrome (PCOS).

  • UpToDate - Evaluation and management of secondary amenorrhea
How well did you know this?
1
Not at all
2
3
4
5
Perfectly
82
Q

A 37-year-old woman presents with secondary amenorrhea of 7 months duration. She has a body mass index (BMI) of 24. On laboratory studies, she has a FSH level of 55 U/L (2- 8U/L in luteal phase; >25U/L in menopause), LH of 54 U/L and estradiol the lower limit of normal. Serum prolactin level is also normal. A urine pregnancy test excludes pregnancy. On ultrasonography, each ovary contains 3-4 cysts. She desires to become pregnant in the future. Which one of the following would be the most appropriate management of her condition?

A. Progestin-only pills (POP).
B. Menopausal hormone replacement therapy (HRT).
C. Combined oral contraceptives (COCs).
D. Metformin.
E. Danazol.

A

B. Menopausal hormone replacement therapy (HRT)

In this woman, FSH and LH are within menopausal range (cut-off point> 40 U/L) and estradiol in the normal lower limit. Prolactin level is normal and excludes hyperprolactinaemia as a cause of her amenorrhea. The clinical picture and laboratory values are highly suggestive of ovarian failure.

Primary ovarian insufficiency (POI), commonly referred to as premature ovarian failure (POF), is defined as ovarian failure before 40 years of age. The incidence is 1 in 250 by age 35 and 1 in 100 by age 40.

In its full-blown presentation, ovarian failure is associated with amenorrhea, symptoms of estrogen deficiency, and increased gonadotropin levels (In the menopausal range) before the age of 40 years. The term ‘failure’ means that ovarian function is not normal, but it does not necessarily imply total cessation of ovarian function. Patients diagnosed with primary ovarian insufficiency may intermittently produce estrogen, ovulate or even conceive and have a normal pregnancy and delivery in 5-10% of cases.

Measuring serum FSH level is the core study to establish the diagnosis of POI/POF after pregnancy has been ruled out. By convention, two FSH levels in the menopausal range for the specific assay (>40 U/L by radioimmunoassay), measured at least 1 month apart, are diagnostic of POI/POF.

Measurement of serum LH is also important. In most cases of spontaneous POI/POF, FSH is higher than LH. If autoimmune oophoritis is present, FSH may be only mildly elevated, sometimes below the cutoff of 40 U/L, while LH is markedly elevated.

All women with POI/POF should be started on menopausal hormone replacement therapy (HRT) to prevent menopause complications and this should be continued at least until the average age of normal menopause (50-51 years). The main objective of early HRT is prevention of bone loss and early osteoporosis. Other measures to consider are supplementation of calcium and vitamin D.

Standard postmenopausal HRT does not provide effective contraception and spontaneous ovarian activity may resume; therefore, women with POI/POF, who require contraception, should use oral contraceptives for both hormone replacement and contraception.

This woman desires pregnancy; therefore, the most appropriate option for her is postmenopausal HRT. OCP would have been the correct answer if she also had asked for contraception.

  • Medscape - Ovarian Insufficiency
  • Clinical manifestations and diagnosis of spontaneous primary ovarian insufficiency (premature ovarian failure)
  • Management of spontaneous primary ovarian insufficiency (premature ovarian failure)
How well did you know this?
1
Not at all
2
3
4
5
Perfectly
83
Q

Which one of the following foods should be avoided during pregnancy?

A. Liver.
B. Smoked salmon.
C. Beans.
D. Fruits.
E. Rice.

A

Smoked salmon

Liver, beans, fruits and rice can all be safely used during pregnancy. Salmon and other sea foods are generally safe to use, unless they are smoked and ready-to-use. Smoked salmon and trout are susceptible to colonization with Listeria monocytogenes even in the refrigeration temperature and should be avoided during pregnancy.

Soft cheese is another source of Listeria, unless it is cooked above 65°C and served hot.

*http://www.babycenter.com.au/c4209/food-and-drink-
*http://www.nutritionaustralia.org/sites/default/fi
*https://www.nhmrc.gov.au/_files_nhmrc/publications
Time spent:

How well did you know this?
1
Not at all
2
3
4
5
Perfectly
84
Q

Lisa is in your office at 8 weeks pregnancy for an antenatal visit. She asks you about foods that can be harmful to her pregnancy. Which one of the following she should avoid during pregnancy?

A. Soft cheese.
B. Tinned salmon.
C. Fish rich is iodine.
D. Lobster.
E. Liver.

A

Soft cheese

Of the options, soft cheese should be avoided due to the risk of listeriosis, unless it is thoroughly cooked and heated.

Listeria monocytogenes can cause invasive diseases including meningitis, meningoencephalitis, or bacteremia in immunosuppressed patients, individuals at the extremes of age including neonates and elderly adults, and pregnant women. Listeriosis can be associated with miscarriage and early neonatal sepsis.
Some foods, such as soft cheese, are more prone to contamination by Listeria, which can even grow at refrigeration temperature. Such foods should be avoided during pregnancy.

During pregnancy it is recommended that the following dairy products be avoided:
-Soft, semi-soft and surface ripened cheeses (e.g. brie, camembert, ricotta, feta and blue cheese)
-Soft serve ice cream
-Unpasteurized dairy foods - almost all dairy foods produced in Australia must be pasteurized, however some specialty imported cheeses may be unpasteurized.

NOTE - Soft cheese is safe if it is cooked above 65 ̊C Sand served hot e.g. ricotta and spinach cannelloni, cheese topping on pizza.

Ready-to-eat fish, such as smoked salmon or trout is also considered high-risk for Listeria contamination and should be avoided during pregnancy, but canned salmon or appropriately cooked salmon can be is used safely.

Liver, fish rich in iodine (if not smoked or ready-to-use), and lobster can be safely used if appropriately cooked and served.

*http://www.nutritionaustralia.org/sites/default/fi
*http://www.babycenter.com.au/c4209/food-and-drink
*https://www.nhmrc.gov.au/_files_nhmrc/publications

How well did you know this?
1
Not at all
2
3
4
5
Perfectly
85
Q

Kate is 22 years old and a patient in your clinic for the past 3 years. She was diagnosed with hypothyroidism 12 months ago, and was started on levothyroxine 100 mcg, daily. Today, she has presented with complaints of hot flushes and irregular periods for the past 4 months. Her periods were previously regular occurring every 28-30 days. Which one of the following hormonal assessments is more important to do next?

A. Estrogen.
B. Follicular stimulating hormone (FSH).
C. Luteinizing hormone (LH).
D. Thyroid stimulating hormone (TSH).
E. Prolactin.

A

D. Thyroid stimulating hormone (TSH)

The scenario represents a case of irregular periods and hot flushes. In approach to menstruation abnormalities, thyroid hormone problems should always be considered. In fact with Kate’s history, the diagnostic approach is completely clear. Kate is on thyroid hormones for treatment of hypothyroidism. Excess exogenous thyroid hormones either used for treatment can have led to hyperthyroidism, which in turn has resulted in menstruation abnormalities. The most common menstruation abnormalities associated with hyperthyroidism are oligomenorrhea and amenorrhea; however, menorrhagia is also possible.

Also, hyperthyroidism is a well-known cause of non-menopausal hot flushes. Hypertension is another cause.

Another possibility is that her hypothyroidism is still undertreated. Hypothyroidism, however, is most likely to present with menorrhagia. Additionally, the presence of hot flushes goes against hypothyroidism.
In this case, the next best step in approach to Kate’s problem, after taking a complete history, performing a thorough physical exam, and excluding pregnancy is to order a TSH.

Estrogen (option A), FSH (option B) , LH , and prolactin (option E) should be considered next if TSH is normal and thyroid problems are excluded because premature ovarian failure is another possibility to consider next.

  • UpToDate - Overview of the clinical manifestations of hyperthyroidism in adults
How well did you know this?
1
Not at all
2
3
4
5
Perfectly
86
Q

Regarding oxytocin, which one of the following is true?

A. It has antidiuretic effect.
B. It is a steroid.
C. It is produced by the posterior pituitary.
D. It has an action on most involuntary muscles.
E. It causes milk production.

A

A. It has antidiuretic effect

Oxytocin has an antidiuretic effect through an unknown mechanism.

OPTION B and C : Oxytocin is a neurohypophysial peptide mostly produced in hypothalamus and stored in the posterior pituitary; however, recent studies showed that this hormone is also produced in uterus, placenta, amnion, corpus luteum, testis and heart.

OPTION D : The most prominent action of oxytocin is on smooth muscles of the uterus and breast. The most widely known effect of oxytocin on the uterine smooth muscles is uterine contraction. Its effect on smooth muscles of the breast is milk ejection during lactation. Oxytocin does not have an action on most of involuntary muscles such as those in digestive system or respiratory tract, or smooth muscles within the wall of vessels; at least not while within physiologic limits.

OPTION E : Although oxytocin is the main hormone in milk ejection process and milk-let-down reflex, it has no known role in milk production.

*http://www.ncbi.nlm.nih.gov/pmc/articles/PMC239673
*http://www.ncbi.nlm.nih.gov/pubmed/11274341
*http://physrev.physiology.org/content/81/2/629

How well did you know this?
1
Not at all
2
3
4
5
Perfectly
87
Q

Which one of the following is not an effect of oxytocin?

A. Mammalian glands stimulation.
B. Uterine stimulation.
C. Labor induction.
D. Progesterone release.
E. Antidiuretic effect.

A

D. Progesterone release

Oxytocin stimulates the smooth muscles of the uterus and breast. The most widely known effect of oxytocin on the uterine smooth muscles is uterine contraction and labor induction. Its effect on smooth muscles of the breast is milk ejection during lactation. It has no role in milk production.

Also, oxytocin has antidiuretic effect through an unknown mechanism.
Progesterone release is not an effect of oxytocin.

*http://www.ncbi.nlm.nih.gov/pmc/articles/PMC239673 *http://www.ncbi.nlm.nih.gov/pubmed/11274341
*http://physrev.physiology.org/content/81/2/629

How well did you know this?
1
Not at all
2
3
4
5
Perfectly
88
Q

A 17-year-old girl is brought to your practice for evaluation because her periods have not started yet. She also complains of abdominal pain every month and about the same time. On examination, she has fully developed breasts and axillary and pubic hair. She does not consent to a vaginal exam. Which one of the following could be the most likely diagnosis?

A. Imperforated hymen.
B. Absent vagina.
C. Mullerian agenesis.
D. Turner syndrome.
E. Normal variant.

A

A. Imperforated hymen

This girl has presented with primary amenorrhea as her chief complaint.

A girl is expected to have her periods until the age of 16 years. Primary amenorrhea is defined as absence of periods after the age 15-16 years in the presence of secondary sexual characteristics, or after the age of 13-14 years in the absence of such characteristics. The most common cause of primary amenorrhea is gonadal dysgenesis such as in Turner syndrome, followed by müllerian agenesis as the second most common cause. Patients with müllerian agenesis typically present with primary amenorrhea in the presence of normal growth and development in adolescence.

On physical examination, patients with müllerian agenesis have normal height, secondary sexual characteristics, body hair, and external genitalia, but a vagina is either absent, or present as a short blind-ended structure without a cervix at the vaginal apex. Patients with müllerian agenesis have a normal 46,XX karyotype and a normal hormonal profile. Most patients have a rudimentary nonfunctioning uterus.
In other words, cyclical endometrial shedding does not occur in most of women with müllerian agenesis; however, in 2 to 7% cases, there is a uterus with functioning endometrium. In such women, chronic or cyclic abdominopelvic pain occurs secondary to hematocolpos (accumulation of blood in the vagina), hematosalpinx (accumulation of blood in the ovarian tubes), hematometra (accumulation of blood in the uterus), or endometriosis.

Imperforated hymen can be another cause of primary amenorrhea. In contrast to most patients with müllerian agenesis, patients with an imperforate hymen will not have the typical fringe of hymenal tissue.

The patient with a low transverse vaginal septum will have a normal hymen with more proximal obstruction of the vaginal canal. In addition to presenting with primary amenorrhea, imperforated hymen and transverse vaginal septum are often associated with symptoms of cyclic abdominal or pelvic pain and a pelvic mass due to the obstructed outflow tract and associated hematocolpos.

With findings of fully developed sexual characteristics, absent periods, and cyclical pain, an imperforated hymen is the most likely diagnosis both clinically and statistically.

OPTION B and C : Absent vagina can be a presentation of a müllerian agenesis. Absent vagina /müllerian dysgenesis can present similar to imperforated hymen in 2 to 7% of cases in whom a functioning uterus exists. However, imperforated hymen is more likely statistically.

OPTION D : In Turner syndrome, usually no menstruation occurs. In fact, the gonads are dysplastic and malfunctioning (streak gonads). The patients often have secondary sexual characteristics (often delayed), but there is no cyclical abdominal pain because they do not menstruate. Short stature is commonly noted.

OPTION E : Absence of periods after the age of 16 years is always alarming and cannot be considered a normal variant.

  • UpToDate - Evaluation and management of primary amenorrhea
How well did you know this?
1
Not at all
2
3
4
5
Perfectly
89
Q

A 53-year-old woman presents to your practice with vaginal bleeding after sexual intercourse 7 hours ago. She has not had any menses for the past 18 months. Her last cervical screening test, performed 12 months ago, was normal. She does not complain of any other symptoms and is otherwise healthy. Which one of the following could be the most likely underlying cause of postcoital bleeding in this woman?

A. Endometrial cancer.
B. Cervical cancer.
C. Cervical polyp.
D. Cervical ectropion.
E. Vaginal atrophy.

A

E. Vaginal atrophy

Postmenopausal vaginal bleeding, including post coital bleeding, can have many causes. Some menopausal women may experience ‘rouge ovulation’ which is an episode of ovulation and followed by menstruation after more than a year without episodes. Hormone replacement therapy (HRT) is another important cause of post-menopausal vaginal bleeding. Apart from those, vaginal bleeding in postmenopausal women should be taken very seriously and investigated promptly.

Fortunately, the most common causes of post-menopausal vaginal bleeding are benign and far less serious. Of these causes, atrophic vaginitis due to ongoing estrogen deficiency has been shown to be the most common cause of vaginal bleeding, especially post coital bleeding, in menopausal women.

It should be noted the although serious condition such as endometrial cancer or hypertrophy and cervical cancer are less common and comprise up to 20% of cases presenting with post-menopausal bleeding, these conditions should always considered first and excluded using prompt investigations.

Guidelines by the RACGP recommend that any post coital bleeding in a post-menopausal woman should be investigated with a co-test (HPV and LBC) and referral for gynecological assessment to rule out cervical cancer.

OPTION A : Although endometrial cancer can be another possibility, compared to atrophic vaginitis, it is not as common; nonetheless, assessment to exclude endometrial cancer should be undertaken with transvaginal ultrasound and endometrial biopsy if there is an abnormally thick endometrium.

OPTION B : With a normal pap smear 12 months ago, cervical cancer is very unlikely.

OPTION C and D : Cervical ectropion and cervical polyps can be other causes of postcoital bleeding. Cervical ectropion is more commonly a result of high estrogen levels such as during ovulatory phase in younger women, pregnancy and in those women on oral contraceptive pills. Cervical ectropion in postmenopausal women is not very common.

TOPIC REVIEW
Common causes of irregualr vaginal bleeding, including postcoital bleeding, are summarized in the following table: (see table)

  • AAFP - Diagnosis and Treatment of Atrophic Vaginitis
How well did you know this?
1
Not at all
2
3
4
5
Perfectly
90
Q

Robert and Janet are in your clinic for consultation. Robert complains of malaise, mild headache and a generalized rash in form of both intact and unroofed vesicles, as well as macules and papules. Three weeks ago, Alan, their 7-year-old was diagnosed with chickenpox. Based on the history and the appearance of the rash, you establish the diagnosis of chickenpox for Robert. Janet is 13 weeks pregnant and is concerned if the infection can affect her and her baby too. Janet is not sure if she had chickenpox in childhood. Which one of the following is the most appropriate next step in management of Janet?

A. Start her on oral acicolvir.
B. Give her varicella zoster vaccine (Varivax).
C. Give her varicella zoster immunoglobulin (VZIG).
D. Arrange for checking her varicella zoster antibody (IgG) status.
E. Start her on oral famcilovir.

A

D. Arrange for checking her varicella zoster antibody (IgG) status

In approaching a pregnant woman, who has been exposed to varicella zoster virus (VZV) by contact to a case of chicken pox, the first question to be answered is whether the pregnant woman has had a history of chickenpox. If she remembers chickenpox infection, no further action is required. If she has had not the infection or the history is uncertain, the next step in management would be serologic tests for VZV (IgG). If the test is positive, the woman is immune. If not, management depends on the time elapsed since exposure:

  1. If the presentation is within the first 96 hours after exposure, passive immunization with VZV immunoglobulin (VZIG) is the most appropriate next step in management.
  2. If the presentation is after 96 hour, VZIG is of no use; however, the following women should still receive prophylaxis with the antiviral agents aciclovir (first line) or
    valaciclovir:
    *In the second half of pregnancy
    *With a history of an underlying lung disease
    *Immunocompromised
    *Smokers

Since Janet is not certain about contracting chicken pox before and there is no history of vaccination against VZZ, the next best step for her would be checking her immunity status against VZV by VZV IgG (D is correct).

OPTION A : Oral aciclovir (or valaciclovir as a second option) is used for women with established VZV infection who present within the 24 hours after the rash appears or for those who present later and are at risk of complicated VZV infection indicated by: neurological symptoms, fever longer than 6 days, eruption of new pocks after 6 days, VZV pneumonia, or those who are immunocompromised.

OPTION B : Varivax contains live attenuated virus and is contraindicated throughout pregnancy.

OPTION C : VZIG would be the correct option if IgG levels show that Janet is not immune to VZV.

OPTION E : while there is adequate data that aciclovir and valaciclovir are safe to use during pregnancy, safety of famciclovir for use in a pregnant has yet to be established; hence, famciclovir is not as safe.

  • Australian Society of Infectious Diseases - Management of Prenatal Infections
  • South Australia Health Department - Varicella Zoster (chicken pox) in Pregnancy Clinical Guideline
How well did you know this?
1
Not at all
2
3
4
5
Perfectly
91
Q

A 35-year-old woman is brought to the emergency department after she sustaind ankle twisting. After initial management of her current problem, you realize that she is 10 weeks along her second pregnancy. On further questioning, she admits to heroin addiction. She also says that ‘Doc, I sometimes need to get high on meth. My favorite wings to fly are cocaine though, but since I cannot afford it, I a bit when I manage to crash a party’. She drinks a bottle of Whisky every day. During the past few weeks, she has started to worry about not being able to be a good mother. For this reason she is taking diazepam at night. She says that she managed to illegally get a bottle of these sleeping pills. Which one of the following in her history is most likely to cause fetal malformations?

A. Diazepam.
B. Alcohol.
C. Amphetamine.
D. Cocaine.
E. Heroine.

A

B. Alcohol

This woman is taking a bottle of Whisky every day. A small glass of Whisky (1.5 Oz) is equivalent of a standard drink. A bottle definitely exceeds 12 standard drinks; therefore this woman’s fetus would be at significant risk of fetal alcohol syndrome (FAS) which is associated with the following congenital malformations:

-Low-set ears
-Midfacial hypoplasia
-Elongated philtrum
-Upturned nose
-Microcephaly
-Skeletal and cardiac malformations

OPTION A : The health risks of benzodiazepines in pregnancy have not been clearly established. There have been inconsistent reports of teratogenic effects associated with fetal exposure to benzodiazepines. Regular benzodiazepine use in pregnancy may be associated with a neonatal abstinence syndrome, which may be of delayed onset..

OPTION C : Amphetamine use in controlled doses during pregnancy is unlikely to pose a substantial teratogenic risk, but women who use amphetamines are at higher risk of a range of obstetric complications such as reduced birth weight — many of these outcomes are not specific to amphetamines but influenced by other drug use and lifestyle factors in addition to amphetamine use. Exposure to amphetamines in utero may influence prenatal brain development, but the nature of this influence and potential clinical significance are not well established.

3,4- methylenedioxymethamphetamine (MDMA), commonly known as ecstasy, is an amphetamine derivative. Existing evidence suggests that use of ecstasyduring first trimester poses a potential teratogenic risk; therefore, it is strongly recommended that use of ecstasy be avoided during this dangerous period of organogenesis (between week two and week eight post conception or between week four and ten of last menstrual period)

OPTION D : The role of cocaine in congenital malformations is controversial, and the malformations caused by cocaine are extremely rare. However, if it leads to fetal intracranial haemorrhage the outcome would be devastating.

OPTION E : Opiate addiction carries the significant risk of several perinatal complications, but there is no associated congenital malformation.

  • Llewellyn-Jones Fundamentals of Obstetrics and Gynaecology – Elsevier –Mosby - 9th Edition
    *http://www.merckmanuals.com/professional/gynecol
How well did you know this?
1
Not at all
2
3
4
5
Perfectly
92
Q

A 54-year-old woman presents to your clinic, with complaint of a vulvar lesion associate with severe itching. Her problem started nearly 6 months ago and has progressed in severity. She has been menopausal since the age 50 years. On examination, atrophic white plaques are evident as well as excoriation marks over and around the vulva. The labia minora is slightly atrophic and there is interoitus stenosis. Vaginal examination is normal. The appearance of the vulva is shown in the accompanying photograph. Which one of the following is the most appropriate option for treatment of this patient?

A. Vaginal estrogen cream.
B. Clobetasol cream.
C. Hydrocortisone cream.
D. Topical antifungals.
E. Oral prednisone.

A

B. Clobetasol cream

The clinical findings of pruritic white vulvar plaques, in the absence of vaginal symptoms and signs, make genital lichen sclerosus (LS) the most likely diagnosis.

LS is a chronic inflammatory dermatosis of unknown etiology. The disease most commonly affects the genitalia, but it can occur at any skin site (extragenital lichen sclerosus). Balanitis xerotica obliterans is an outdated termed used to describe the LS of glans penis. Lichen sclerosus can occur in males or females of any age. In female, however, there are two peaks of onset during prepubertal and perimenopausal/postmenopausal ages, both of which are estrogen-deficient states; however, the relationship between LS and estrogen deficiency is unclear. In males it is more common between the puberty and age 60 years.

Typical clinical presentation is with white, often atrophic, plaques associated with pruritus and pain that result in genital scarring and adhesions. The most common complaint in vulvar lichen sclerosus is progressive pruritus that can be followed by dyspareunia, dysuria, or genital bleeding. Female genital lesions may be confined to the labia majora but usually involve, and eventually obliterate the labia minora and stenose the interoitus. Often, an hourglass, butterfly, or figure-8 pattern involves the perivaginal and perianal areas, with minimal involvement of the perineum in between.

Penile LS is associated with sexual/urinary dysfunction. It is usually preceded by pruritus but may present with sudden phimosis of previously retractable foreskin, and urinary obstruction can result.
Extragenital lichen sclerosus (LS) may be asymptomatic (approximately one third of cases) or it may itch or be tender.

NOTE - With genital LS in women, there is an increased risk of squamous cell carcinoma (<5%). Progression to malignancy in genital LS in men is rare.

First-line therapy for LS includes patient education and super-potent topical corticosteroids (e.g. clobetasol propionate). Prolonged use of potent corticosteroids does not result in atrophy of the labia minora and clitoris, owing to the resistant nature of their modified mucous membranes. However, perianal and hair-baring areas (e.g. labia majora) can atrophy within 2-3 weeks of use. Intralesional corticosteroid injections are also considered at occasions.

Second-line therapies include the calcineurin inhibitors, tacrolimus and pimecrolimus, which can be a helpful adjunct to topical corticosteroids for maintenance.

Third-line therapies that could be considered in treatment-resistant genital lichen sclerosus could include topical or oral retinoid, steroid injections, systemic cyclosporine (topical use has not shown effective), methotrexate, or hydroxyurea. For extragenital lichen sclerosus, phototherapy or methotrexate could be considered.

OPTION A : Vaginal estrogen cream is the first-line treatment option for women with atrophic vaginitis and not effective against LS. Normal vaginal exam excludes atrophic vaginitis as the diagnosis.

Although extensively used in the past, topical estrogen, topical testosterone, topical progesterone, and hormone replacement therapy are not recommended anymore due to lack evidence for effectiveness.

OPTION C : Hydrocortisone is a weak corticosteroid and not appropriate for treatment of LS where potent to very potent (super potent) topical corticosteroids are indicated.

OPTION D : Topical antifungal agents were the correct answer if the diagnosis would have been vulvovaginal candidiasis. Candidiasis is associated with inflamed skin and mucosa and cheesy- white vaginal discharge that are not present in this woman.

OPTION E : Oral (systemic) predniso(lo)ne is not used for treatment of LS due to their adverse effect profile that is not outweighed by the benefits.

  • Medscape - Lichen Sclerosus
  • Localized provoked vestibulodynia (vulvodynia): assessment and management
How well did you know this?
1
Not at all
2
3
4
5
Perfectly
93
Q

A 39-week pregnant woman is admitted to the Labor Unit due to commencement of labor pain and contractions. The woman is placed in left lateral decubitus position and started on supplemental oxygen by face mask. An intravenous line is established and isotonic fluids and cyntocinon drip is started according to the protocol. A CTG is performed that shows a fetal heart rate of 140 which drops to 70 bpm and returns back to the base line of 140 bpm in two minutes. Which one of the following is the most appropriate immediate management?

A. Stop cyntocinon.
B. Fetal scalp blood sampling.
C. Immediate cesarean section.
D. Titrate and increase the dose of cyntocinon.
E. Continuous CTG monitoring.

A

Stop cyntocinon

The scenario represents a case of prolonged fetal heart rate deceleration during labor. Prolonged deceleration is defined as a drop in the fetal heart rate below the baseline that lasts from 90 seconds up to five minutes. Prolonged bradycardia is defined as a fetal heart rate <100 bpm lasting more than five minutes.

The following are the possible causes of prolonged deceleration or prolonged bradycardia:
* Maternal hypotension
* Cord prolapse or compression
* Uterine hypertonia
* Scar dehiscence
* Placental abruption
* Rapid fetal descent

Management of prolonged decelerations or prolonged bradycardia includes the following steps:
-Reposition the woman – e.g. lateral position
-Checking maternal blood pressure and administration of bolus IV fluids in case of hypotension.
-Discontinuation of oxytocin or decreasing rate of infusion (if in progress)
-Check the maternal blood pressure.
-Checking the maternal pulse to differentiate maternal pulse rate from the fetal heart rate (FHR).
-If indicated, performing a vaginal exam to exclude cord prolapse or rapid cervical dilatation if the bradycardia persists.
-Consideration of applying a fetal scalp electrode.
-Assessment of abdominal tone to exclude a hypertonic uterus.
-Preparation e for assisted delivery (vacuum or forceps delivery) or emergency cesarean section if bradycardia does not resolve.

It is very likely that prolonged decelerations have been caused by excess uterine contraction as a result of oxytocin (cyntocinon) infusion; therefore, cessation of cyntocinon is the most important immediate action to consider. CTG monitoring option E should also be continued throughout the labor for assessment of fetal status.

OPTION B : Fetal scalp blood sampling is considered if measures such as repositioning the woman, cessation of cyntocinon, blood pressure correction or other measures fail to correct the situation. This will determine if there is fetal jeopardy necessitating immediate delivery.

OPTION C : Urgent cesarean section or assisted vaginal delivery may be required if fetal compromise persists despite more conservative measures.

Increasing the dose of cyntocinon , by increasing the contractions, will escalate the fetal condition and is an incorrect option.

  • WA Health Department – Obstetrics and Gynaecology Guidelines
  • RANZCOG Intrapartum Fetal Surveillance Clinical Guideline
How well did you know this?
1
Not at all
2
3
4
5
Perfectly
94
Q

A 38-year-old pregnant woman, G1P0, is brought to the emergency department after spontaneous rupture of membranes. She is admitted to the labor unit. After three hours, there is still no uterine contractions present; hence, she is started on cyntocinon. After four hours, reassessment shows that there are adequate uterine contractions. A CTG shows a base line fetal heart rate of 143 bpm dropping to 60 bpm and slowly returning to the baseline in four minutes. On vaginal exam, the fetal head is in LOA position, the station is 1+ and there is 1+ molding. The cervix is dilated at 9 cm. Which one of the following is the most appropriate next step in management?

A. Forceps delivery.
B. Vacuum delivery.
C. Addition of one liter of normal saline.
D. Cessation of cyntocinon.
E. Emergent cesarean delivery.

A

D. Cessation of cyntocinon

With adequate uterine contractions, head engagement and appropriate descent, the labor is progressing appropriately; however, with FHR dropping to 60 bpm (<100 bpm) there is prolonged decelerations that make the CTG abnormal.

CTG is considered abnormal if there are at least two of the following features on CTG:
* Baseline FHR is between 100-109 bpm or between 161-170 bpm
* Variability of FHR is reduced (3-5 bpm for >40 minutes)
* Decelerations are variable without complicating features
OR
There is any of the following:
* Baseline FHR is <100 bpm or >170bpm
* Variability is absent (<3 bpm)
* Decelerations are prolonged for >3 minutes OR late OR have complicated variables

When there is CTG abnormalities, the first steps to consider are reversing the possible causes of decelerations :
* Maternal hypotension
* Cord prolapse or compression
* Uterine hypertonia
* Scar dehiscence
* Placental abruption
* Rapid fetal descent

Management of prolonged decelerations or prolonged bradycardia includes the following steps:

-Reposition the woman – e.g. lateral position
-Checking maternal blood pressure and administration of bolus IV fluids in case of hypotension.
-Discontinuation of oxytocin or decreasing rate of infusion (if in progress)
-Check the maternal blood pressure.
-Checking the maternal pulse to differentiate maternal pulse rate from the fetal heart rate (FHR).
-If indicated, performing a vaginal exam to exclude cord prolapse or rapid cervical dilatation if the bradycardia persists.
-Consideration of applying a fetal scalp electrode.
-Assessment of abdominal tone to exclude a hypertonic uterus.
-Preparation e for assisted delivery (vacuum or forceps delivery) or emergency cesarean section if bradycardia does not resolve.

Uterine hyperstimulation and contractions due to excess cyntocinon is a common cause of prolonged deceleration; therefore, cessation of cyntocinon is the most appropriate immediate step once prolonged decelerations are encountered on CTG. It is very likely that prolonged decelerations have been caused by excess uterine contraction as a result of oxytocin (cyntocinon) infusion; therefore, cessation of cyntocinon is the most important immediate action to consider.

It is also important to check the maternal blood pressure as the mother’s hypotension is also a cause of prolonged deceleration. In case of hypotension, intravenous fluids should be administered (option C) to correct maternal hypotension. CTG monitoring should be continued for assessment of fetal status and response to treatment.

Assisted vaginal delivery using forceps or vacuum (option A and B) or emergent cesarean delivery (option E) are measures to consider if the above conservative steps fail to correct the underlying cause(s) of fetal compromise.

* RANZCOG Intrapartum Fetal Surveillance Clinical Guideline * WA Health Department – Obstetrics and Gynaecology Guidelines
How well did you know this?
1
Not at all
2
3
4
5
Perfectly
95
Q

A 24-year-old woman presents to a GP clinic with concerns about breastfeeding. She gave birth to a healthy male child 20 days ago through an uneventful vaginal delivery. She is worried that she is not producing milk to feed her baby and feels he is always hungry. Which one of the following conditions could be the most likely cause of decreased mild production?

A. Poor positioning of the baby.
B. Exhaustion of the mother.
C. Short duration of feeding at each session.
D. Decreased frequency of breastfeeding.
E. Insufficient fluid intake by the mother.

A

D. Decreased frequency of breastfeeding

Although most women are capable of producing more milk than their infants require, more than half of breastfeeding mothers perceive that their milk supply as inadequate. Results of an online survey of 12 World Health Organization Western Pacific offices revealed that the second most commonly reported reason women did not breastfeed was “not having enough milk” (17%).

For some mothers, this is a perceived rather than real issue. In such cases, providing information on what constitutes ‘normal’ newborn feeding behavior may provide reassurance. For example, it is normal for infants to have frequent but small amount feeds during a particular time of the day. A change to an infant’s feeding pattern, such as increased frequency of feeds, or a feeling of softer breasts, may concern the mother; however, these may be normal changes that are unrelated to decreased supply.

Of the causes that lead to genuine lowered milk production, decreased frequency of breastfeeding is the most common one. In the human mammary gland, lactation is under autocrine control, in which the frequency and degree of milk removal by expression regulate an inhibitory peptide present in the milk. This means that if the milk is not removed, this inhibitory peptide accumulates and subsequently decreases the synthesis of milk. If the milk is frequently removed, this inhibitory peptide does not accumulate, and milk synthesis increases.

*https://www.racgp.org.au/afp/2016/august/overcoming-challenges-faced-by-breastfeeding-mothers/
* https://emedicine.medscape.com/article/979458-overview#a4

How well did you know this?
1
Not at all
2
3
4
5
Perfectly
96
Q

Which one of the following women is most likely to develop pre-eclampsia during pregnancy?

A. A 40-year-old woman, gravida 6, para 5, with no previous history of any pregnancy-related complications.
B. A 16-year-old primigravida woman.
C. A 35-year-old woman with primary hypertension.
D. A 25-year-old woman who is gravida 4, para 3.
E. A 25-year-old primigarvida with family history of pre-eclampsia in her mother and sister.

A

C. A 35-year-old woman with primary hypertension

Pre-eclampsia is defined as hypertension and proteinuria after 20 weeks of pregnancy.
The following table summarizes the established risk factors for development of pre-eclampsia and their associated increased risks:
(See table below)

According to the above table, a woman with chronic hypertension is five time more likely to develop preeclampsia compared to other options.

OPTION A : In a 40-year-old woman with no previous obstetrical history, the risk of preeclampsia is almost doubled.

OPTION B : Nulliparity almost triples the risk of preeclampsia. Age 16 years or younger has been considered as a risk factor but recent studies failed to establish a meaningful relationship.

OPTION D : This woman has no known risk factor for preeclampsia.

OPTION E : In this woman, the family history of preeclampsia in her mother and sister, triples the risk of preeclampsia for her.

How well did you know this?
1
Not at all
2
3
4
5
Perfectly
97
Q

In the event of chord prolapse during labor, which one of the following findings is more likely on CTG monitoring?

A. Sinusoidal pattern.
B. Early decelerations.
C. Variable decelerations.
D. Late decelerations.
E. Fetal tachycardia.

A

C. Variable decelerations

The most common CTG abnormality associated with umbilical cord prolapse is variable decelerations. Other less common CTG findings include persistent fetal bradycardia and prolonged decelerations of over one minute.

  • RCOG - Greentop Guidelines - Umbilical Cord Prolpase
How well did you know this?
1
Not at all
2
3
4
5
Perfectly
98
Q

Dorothy, 21 years of age, presents to your GP clinic for advice regarding commencement of combined oral contraceptive pills (COCPs). You take a careful medical and family history, which is significant for gastro-esophageal reflux disease (GERD) and premenstrual headaches. She has smoked 15-20 cigarettes a day for the past four years and drinks 2 to 3 glasses of wine every day. Her family history includes breast cancer of her mother diagnosed at the age of 53 years and hypertension in her father. On examination, her blood pressure is 130/78 mmHg. She has a body mass index (BMI) of 28. She wants to know if there is anything wrong with her taking OCPs. In counselling her, which one of the following will you discuss further as a possible absolute contraindication to COCPs?

A. Her drinking history.
B. Her family history of breast cancer.
C. Her premenstrual headaches.
D. Her BMI.
E. Her smoking history.

A

C. Her premenstrual headaches

According to the UK Medical eligibility criteria for contraceptive use, the following are considered absolute risk factors for COCPs:

  • Breastfeeding and ≤6 weeks postpartum
  • Smoker ≥35 year and ≥ 15 cigarettes/day
  • Presence of multiple risk factors for CVD including older age, smoking, diabetes, hypertension
  • Hypertension with systolic ≥160mmHg or diastolic ≥95mmHg
  • Vascular disease
  • Major surgery with prolonged immobilization
  • Current or past history of venous thromboembolism (VTE)
  • Known thrombogenic mutations (Factor V Leiden, Prothrombin mutation, Protein S, Protein C and Antithrombin deficiencies)
  • Migraine with aura
  • Current or past history of Ischemic Heart Disease (IHD);
  • Complicated valvular heart disease
  • Diabetes complicated by nephropathy, retinopathy or vascular disease
  • Breast cancer
  • Severe Liver disease including cirrhosis hepatocellular adenoma and hepatoma
  • Raynaud’s with lupus anticoagulant
  • SLE with antiphospholipid antibodies

In the history, Dorothy has mentioned headaches before her menses. The relation of the headaches to her periods makes migraine a possibility, as one of the triggers for migraine is hormonal changes in the premenstrual period. If that is the case, further scrutinizing the history is required for characteristic features of aura. If her headaches are associated with aura, COCPs will be absolutely contraindicated for her.

OPTION A : Alcohol use is not a contraindication to COCPs use.

OPTION B : While personal history of breast cancer is an absolute contraindication to COCPs, family history of breast cancer is not associated with increased risk of breast cancer in COCPs users; hence, the history of breast cancer in Dorothy’ mother does not preclude use of COCPs for her.

OPTION D : Overweightness and obesity alone are not contraindications to use of combines oral contraceptive pills, unless associated with other cardiovacular risk factors such as smoking, diabetes, etc.

OPTION E : COCPs are contraindicated in woman who are 35 years or older AND smoke ≥ 15 cigarettes/day. Although Dorothy smokes 15-20 cigarettes a day, she is 21 years; therefore, her smoking history is not an absolute contraindication.

  • RANZCOG - Combined Hormonal Contraceptives
How well did you know this?
1
Not at all
2
3
4
5
Perfectly
99
Q

A 55-year-old woman presents to your GP practice with complaints of dyspareunia, dysuria, and vaginal itch. She has been menopausal since the age of 50 years. She relates that sexual intercourse has been becoming progressively painful for the past 6 months. She describes the pain as burning. The vaginal itching and dysuria has developed in the past few weeks. Her past medical history is unremarkable, and she is currently on no medications except for supplemental vitamins. She denies any hot flushes. On examination, atrophy of the labia minora is evident, but labia majora and other parts of the vulva are normal. Vaginal exam reveals a rather stenotic vagina with pale and atrophic walls but no vaginal or cervical inflammation or discharge. Which one of the following would be the most appropriate first-line treatment for her?

A. Estrogen cream.
B. Topical antifungal agents.
C. Conization.
D. Hydrocortisone cream.
E. Topical antibiotics.

A

A. Estrogen cream

This woman has atrophic changes of the vagina 5 years after menopause. For her, atrophic vaginitis is the most likely diagnosis.

Atrophic vaginitis results from estrogen deficiency and is experienced by almost 50% of postmenopausal women. Symptoms include itching, burning, dryness, and irritation, all of which can lead to dyspareunia. On the other hand, a decline in estrogen alters the vaginal flora, leading to bacterial overgrowth. This can cause bacterial vaginosis and vaginal discharge.

Declining estrogen also affects the urinary tract, leading to thinning of the bladder and urethral linings. This can result in chronic dysuria and an increased incidence of urinary tract infections. Estrogen creams are the mainstay of treatment for atrophic vaginitis. As this may take a while to come to effect, lubricants can be used to facilitate sexual intercourse.

OPTION B : Antifungal agents were the treatment of choice if vaginal candidiasis would have been the diagnosis. The condition most commonly presents with itching. Burning and dyspareunia can also be present; however, an inflamed vaginal with or without cheesy white discharge rather than an atrophic vagina would have been expected on examination.

OPTION C : Conization has no role in treatment of atrophic vaginitis. It is often used to treat premalignant and malignant cervical lesions.

OPTION D : Potent to very potent topical corticosteroids are used, as first-line therapy, for treatment of lichen sclerosus (LS). This woman is not likely to have LS, as LS does not involve vagina. On the other hand, hydrocortisone is a weak corticosteroid and not effective in treatment of LS.

OPTION E : Topical antibiotics (e.g., metronidazol) may be used for concomitant bacterial infection or vaginosis in atrophic vaginitis, but it is not effective for atrophic vaginitis itself.

  • MJA – Management of common vulval conditions
  • RACGP - Localized provoked vestibulodynia (vulvodynia): assessment and management
  • Medscape – Treating Atrophic Vaginitis
How well did you know this?
1
Not at all
2
3
4
5
Perfectly
100
Q

Hanna is a 32-year-old patient of yours, who have decided to start combined oral contraceptive pills (COC) for contraception, but she is worried about the risk of cancer because she has heard from her friends and read on different websites that COC increase the risk of some cancers. In consulting her regarding long-term use COC, which one of the cancers will you mention to be of highest risk for her if she takes COC in the long run?

A. Endometrial cancer.
B. Colon cancer.
C. Breast cancer.
D. Cervical cancer.
E. Ovarian cancer.

A

D. Cervical cancer

COCs cease ovulation and is associated with decreased risk of ovarian cancer in women who have never used COCs. This protective effect is increased with the length of time COCs are used, and last up to 30 years after cessation. This effect is caused by reducing the number of ovulations a woman experiences in her lifetime, and consequently reducing exposure to naturally occurring female hormones as a risk factor.

The risk of colorectal cancers is also decreased by 15% to 20% in women who have ever taken COCs compared to women never on COC. This effect is caused by reduction in bile acids in women on COCs.

Another cancer with **decreased risk associated with COCs use is endometrial cancer **. There is at least 30% reduction in risk of developing endometrial cancer in women who have ever used COCs compared to those who have not. The risk reduces more with prolonged use of COCs. This reduction is even more significant in long-time users of oral COCs who are smokers, obese, or physically inactive. Such protective effect is caused by suppression of endometrial proliferation as a result of naturally occurring sex hormones when a woman uses COCs (synthetic sex hormones).

COCs, however, are associated with an increased risk of cervical cancer. Women who have used oral COCs for 5 or more years have a higher risk of cervical cancer than women who have never used COCs. The longer a woman uses COCs, the greater the increase in her risk of cervical cancer will be. One study found a 10% increased risk for less than 5 years of use, a 60% increased risk with 5–9 years of use, and a doubling of the risk with 10 or more years of use. This risk declines over time after cessation of COCs. Such effect is probably due to changes in the susceptibility of cervical cells to persistent infection with high-risk HPV types which are virtually the cause of all cervical cancers. Another hypothesis is that women on COCs have more frequent unprotected sex which increases the risk of HPV infection.

COCs are also associated with a slight increase in risk of breast cancer compared to women who have never taken COCs. The risk declines after cessation, and after 10 years of cessation, will be the same as that of women who have never used COCs. Generally, the benefits of preventing unwanted pregnancy and its harms outweighs the slightly increased risk of breast cancer associated with COCs.

In consulting Hanna, she should be told the main concern would be the increased risk of cervical cancer if COCs are taken longer than 5 years as the main concern, and the fact that there is a just slightly increased risk of breast cancer according to studies. COCs would not be appropriate for her in the presence of any personal history of breast cancer but even with breast cancer history in her first degree relatives, she can still take COCs.

It should be explained to her than in fact OCPs decrease her risk of developing ovarian, endometrial and colon cancers.

  • National Cancer Institute – Oral contraceptives and cancer risk
  • Cancer Council - Oral contraceptives
How well did you know this?
1
Not at all
2
3
4
5
Perfectly
101
Q

Miranda is a 33-year-old patient of your, who has presented for cervical screening test today. Her previous tests were all normal. She is married, has never been pregnant, and started her sexual relationships from the age of 17 years. She has been using oral contraceptive pills (OCPs) for the past 8 years as the means of contraception. She does not smoke but
drinks alcohol on social occasions. She is obese with a BMI of 31 kg/m2. In consulting her, which one of the following will you mention as the most significant risk factor for development of cervical cancer in the future?

A. Sexual relationship.
B. Alcohol.
C. Obesity.
D. Prolonged use of OCPs.
E. Nulliparity.

A

D. Prolonged use of OCPs

Some conditions are associated with increased risk of developing cervical cancer. Of these, the most important one is infection with high risk human papilloma virus (HPV) types, such as HPV type 16 and 18. Virtually, all the cases of cervical cancers are caused by HPV infection. In the absence of HPV infection, no cervical cancer occurs. Other risk factors predispose to the development of cervical cancer only in the presence of HPV infection.

All women, who have ever been sexually active, are at an average risk of developing cervical cancer through receiving the HPV infection either by sexual intercourse or skin to skin contact. However, sexual relationship without contracting HPV infection is not a risk factor for cervical cancer.

OCP use in another important risk factor for cervical cancer. Women, who have used oral OCPs for 5 years or more \, have a higher risk of cervical cancer than women who have never used OCPs.

The longer a woman uses oral contraceptives, the greater the increase in her risk of cervical cancer. One study found a 10% increased risk for less than 5 years of use, a 60% increased risk with 5–9 years of use, and a doubling of the risk with 10 or more years of use. Such effect is probably due to changes in the susceptibility of cervical cells to persistent infection with high-risk HPV types. Another hypothesis is that women on OCPs have more frequent unprotected sex, with consequent increased the risk of HPV infection. This should be mentioned to Miranda as the most significant risk factor among other options.

Alcohol (option B) have not shown to be associated with an increased risk of cervical cancer. Although obesity and overweightness have been implicated for increasing the risk of cervical cancer, such association is not significant.

Nulliparity (option E) is a protective factor rather than a risk factor. Studies suggest that giving birth to 5 or more children may slightly increase the cervical cancer for women who have HPV infection.

*TOPIC REVIEW
*
Important risk factor for cervical cancer are as follows:

*Persistent infection with high-risk HPV types (the most significant risk factor – without HPV infection, there is no risk of cervical cancer even in the presence of multiple other factors)
*Smoking
*Lack of regular cervical screening tests
*Age – cervical cancer risk increases after the age of 35 years Prolonged use of OCPs (more than 5 years) *Immunosuppression
*Previous screening abnormalities or previous cervical cancer
*Multiparity (5 or more)
*Exposure to diethylstilbestrol (DES)

  • RACGP – Guidelines for preventive activities in general practice (The Red Book) – cervical cancer
  • Cancer Australia – Cervical cancer: what are the risk factors for cervical cancer?
How well did you know this?
1
Not at all
2
3
4
5
Perfectly
102
Q

A 27-year-old woman comes to antenatal clinic at 10 weeks gestation. It is her first pregnancy. She smokes 2 packs of cigarettes a day and drinks 5-7 standard drinks every day. Recently, she has started using crack cocaine and heroin. She mentions that she is going through a lot these days because her employer found out about her addiction and fired her. She is very anxious and worried about her future and cannot sleep well. This is why she uses diazepam every night. Which one of the following in the history may have the worst effect on the fetus?

A. Cocaine.
B. Heroin.
C. Cigarette smoking.
D. Diazepam.
E. Alcohol.

A

A. Cocaine

Although cocaine is not associated with congenital anomalies, its use is associated with increased risk of fetal intracranial hemorrhage with the worst possible outcome if it occurs.

OPTION B : Heroine and other opiates can be associated with the following complications:

  • Low birth weight usually associate with intrauterine growth restriction(IUGR)
  • Preterm labor
  • Drug withdrawal for the neonate (neonate abstinence syndrome)
  • Increased risk of sudden infant death syndrome (SIDS)

OPTION C : Smoking is associated with multiple complications usch as IUGR, preterm labor, limb reduction, and GI malformations; however, none of them is as life-threatening as fetal intracranial hemorrahge should it occur.

OPTION D : The health risks of benzodiazepines in pregnancy have not been clearly established. There have been inconsistent reports of teratogenic effects associated with fetal exposure to benzodiazepines. Regular benzodiazepine use in pregnancy may be associated with a neonatal abstinence syndrome, which may be of delayed onset.

OPTION E : Although alcohol use is associated with fetal alcohol syndrome (FAS), the amount needed for occurrence of FAS often has to be more than 12 standard drinks (120gr) per day. With only 5 to 7 standard drinks per day, the likelihood of FAS is insignificant.

NOTE - There is a long list of therapeutic medications and recreational drugs that can cause neonatal malformations. Malformations are caused by teratogenic effects of these drugs. Bear in mind that malformations are different from perinatal complications. For example, marijuana can be associated with several perinatal risks, but there are no associated congenital malformations. Alcohol, on the other hand, not only causes complications such as intrauterine growth restriction (IUGR), but it use can also give rise to neonatal alcohol syndrome (FAS), characterized by congenital malformations such as midfacial hypoplasia, upturned nose, long philtrum and low-set ears.

  • Llewellyn – Jones Fundamentals of Obstetrics and Gynaecology Elsevier – Mosby 9th Editionhttp://www.merckmanuals.com/professional/gynecology_and_obstetrics/drugs_in_pregnancy/drugs_in_pregnancy.html
  • http://www.sahealth.sa.gov.au/wps/wcm/connect/fad9
How well did you know this?
1
Not at all
2
3
4
5
Perfectly
103
Q

A couple present to your clinic for evaluation of infertility because they have failed to conceive after one year of unprotected sexual intercourse in a timely fashion at mid-cycles. The female is 45 years old and has three children from her former spouse. She has regular periods at 30-day intervals. The male is 50 years old and has never fathered a child. They have been married for five years now. Which one of the following could be the most likely cause of the infertility?

A. Maternal age.
B. Paternal age.
C. Semen abnormalities.
D. Anovulatory cycles.
E. Pelvic adhesions from previous pregnancies.

A

C. Semen abnormalities

Infertility is defined by the World Health Organisation (WHO) as the inability of a couple to conceive after 12 months of regular unprotected intercourse in women less than 35 years of age; and after six months of regular unprotected intercourse in women 35 years of age or older; or the inability to carry pregnancies to live birth. Based on this definition, this couple are infertile.

Infertility can be caused by a male factor, female factor, or both. Causes of infertility can be categorized as follows:

Female infertility factors:
* Hormonal disorders
* Damaged or blocked fallopian tubes
* Endometriosis
* Excessively thick cervical mucus

Male infertility factors:
*Semen abnormalities (volume, PH, count, morphology, motility, vitality, etc.)
*Any obstruction in the course of sperms out and/or ejaculation failure.

In general, advances maternal age is the most important factor influencing infertility. This is followed by male infertility as the second most important factor.
In this case, the female partner has already given birth to three children via vaginal delivery. There has been no risk factor in the history, such as caesarean section or instrumentation, to make pelvic adhesions from previous pregnancies (option E) a likely explanation to the infertility.

She is 45 years old and, as mentioned before, advanced maternal age (option A) has the most important role in infertility due to factors such as poor follicle reserve and anovulatory cycles, decreased quality of the egg, and undesirable endometrium for the fertile egg to implant. This woman has regular periods at 30-day intervals indicating that she very likely to be still ovulating because anovulatory cycles are associated with irregular periods as the most common symptom.

Although having ovulatory cycles does not exclude problems such as poor-quality eggs or endometrium for fertility due to advanced age, the fact that the male partner has never fathered a child makes male-infertility, and more specifically semen abnormalities, the most likely explanation for infertility in this couple.

Paternal age (option B) is a risk factor for semen abnormalities, but again since he has never fathered a child before (even at younger ages), semen abnormalities independent of the paternal age could be a better and more likely explanation.

For this couple, the first investigation to consider should be a semen analysis. Once male infertility, as the most likely explanation in this scenario, is excluded, attention should be turned to female factors.

  • RACGP – AFP – We’re having trouble conceiving
How well did you know this?
1
Not at all
2
3
4
5
Perfectly
104
Q

Mary, 27 years old, is admitted to the Maternity Ward after her labor pain started. After amniotomy, she is placed in the left lateral position and on supplemental oxygen by nasal canula, and intravenous fluids and Syntocinon® (oxytocin) infusion is started. A while later and during fetal heart auscultation, fetal heart rate (FHR) of 70 bpm is noted. CTG is applied which reveals a baseline fetal heart rate of 140 bpm dropping to 70 bpm periodically with each episode of bradycardia lasting approximately three minutes. Which one of the following is the most appropriate next step in management?

A. Fetal scalp blood sampling.
B. Stop Syntocinon.
C. Immediate cesarean delivery.
D. Continuous CTG monitoring.
E. Reposition to supine.

A

B. Stop Syntocinon

The FHR pattern on CTG is typically interpreted as reassuring or non-reassuring. The presence of a reassuring pattern indicates that there is a minimal possibility of fetal acidemia at that point in time. It does not predict the future fetal status because tracing pattern can change.
A CTG pattern is reassuring if all of the following are present:

  • A baseline fetal heart rate of 110 to 160 bpm
  • Absence of late or variable FHR decelerations
  • Moderate FHR variability (6 to 25 bpm)
  • Age-appropriate FHR accelerations

Any other CTG pattern except the above is considered non-reassuring and prompt action should be taken. When a non-reassuring CTG is encountered, management includes the following steps:

  • Calling for assistance
  • Administration of oxygen via a tight-fitting face mask
  • Changing the maternal position to left lateral or knee chest (to remove the pressure of the gravid uterus from the inferior vena cava and improve maternal circulation and consequently placental circulation)
  • Administration of fluids (bolus)
  • Discontinuation of any uterine stimulant (e.g. oxytocin)
  • Continuous CTG monitoring
  • Performing a vaginal examination (to exclude cord prolapse) and fetal scalp stimulation
  • Determination and correction of the cause of the non-reassuring CTG pattern if possible
  • Considering tocolysis if indicated
  • Determining whether operative intervention (e.g., cesarean delivery) is warranted and, if so, how urgently it is needed

In this scenario, the presence of fetal bradycardia (FHR<110 bpm) makes the CTG non-reassuring and expeditious action and investigation (as mentioned above) is warranted. Mary has already been placed in the left lateral position and is receiving oxygen and intravenous fluids. Of the options, cessation of oxytocin infusion is the most appropriate next step in management because it is likely that oxytocin has led to uterine hyperstimulation. Uterine hyperstimulation is defined as tachysystole (more than five active labor contractions in 10 minute) or uterine hypertonus (contractions lasting more than two minutes or contractions occurring within 60 seconds of each other) in the presence of fetal heart rate abnormalities.

OPTION A : Fetal scalp blood sampling is indicated to assess the presence and severity of fetal acidosis if CTG remains non-reassuring despite initial measures.

OPTION C : Immediate cesarean delivery is considered if there are persisting CTG abnormalities indicative of fetal jeopardy. Unless conditions dictate otherwise, a fetal scalp blood sample is performed to evaluate fetal acidemia prior to making such decision.

OPTION D : Continuous CTG monitoring is indicated in all cases of non-reassuring CTGs after initial measures are undertaken. CTG monitoring is a very important step in this case, but does not take precedence over cessation of oxytocin.

OPTION E : Placing Mary in supine position will cause the pregnant uterus to compress the inferior vena cava and reduce the venous return which will lead to decreased maternal cardiac output. This will result in decreased blood flow to the placenta and deteriorate the fetus’s condition.

  • RANZCOG Intrapartum Fetal Surveillance Clinical Guideline
How well did you know this?
1
Not at all
2
3
4
5
Perfectly
105
Q

A 25-year-old woman presents at 20 weeks gestation for an antenatal check-up. Laboratory studies reveal a platelet count of 90,000/mm3 with no other abnormalities in test results. She has no symptoms and physical examination is unremarkable. Which one of the following could be the most likely diagnosis?

A. Idiopathic thrombocytopenia.
B. Immune thrombocytopenia.
C. Gestational thrombocytopenia.
D. Disseminated intravascular coagulation (DIC).
E. Preeclampsia.

A

C. Gestational thrombocytopenia

Thrombocytopenia is the second most common hematological finding during pregnancy after anemia. Thrombocytopenia can have many etiologies, some of which are not specific to pregnancy.

Causes of thrombocytopenia in pregnancy include:

  1. Pregnancy-related
    * Acute fatty liver
    * Gestational thrombocytopenia
    * HELLP syndrome
    * Hypertensive disorders such as preeclampsia/eclampsia
  2. General
    * Autoimmune conditions (SLE, antiphospholipid syndrome)
    * Bone marrow disorders
    * Disseminated intravascular coagulation
    * Drugs
    * Heparin induced thrombocytopenia
    * Hypersplenism
    * Inherited, Type IIB Von Willebrand disease
    * B12, or folate deficiency
    * Immune (idiopathic) thrombocytopenia
    * Pseudothrombocytopenia
    * Secondary immune thrombocytopenia due to viral infections (e.g., HIV, Hep C, CMV, EBV)
    * Thrombotic microangiopathies ( TTP/HUS)

Of these, gestational thrombocytopenia is the most common cause of thrombocytopenia in pregnancy, followed by ITP as the second most common cause.

Gestational thrombocytopenia (incidental thrombocytopenia) occurs in approximately eight percent of all pregnancies and accounts for more than 70% of cases with thrombocytopenia in pregnancy. Although the pathophysiology of gestational thrombocytopenia is unknown, it is thought to be related to hemodilution, increased platelet consumption, and increased platelet aggregation (as a result of increased levels of thromboxane A2). Platelet count may be lower in women with twins perhaps due to a greater increase in thrombin generation.

In summary, gestational thrombocytopenia is defined by the following five criteria:
1. Mild and asymptomatic thrombocytopenia
2. No past history of thrombocytopenia (except possibly during a previous pregnancy) 3. Occurrence during late gestation (from mid-second trimester to third trimester)
4. No association with fetal thrombocytopenia
5. Spontaneous resolution after delivery

This woman has an isolated and asymptomatic mild thrombocytopenia. This picture is consistent with either gestational thrombocytopenia or ITP. Given the high prevalence of gestational thrombocytopenia, it could be the most likely diagnosis.

OPTION A and B : Immune thrombocytopenia and idiopathic thrombocytopenia are in fact two names for one condition characterized by isolated thrombocytopenia of autoimmune origin that can present similar to gestational thrombocytopenia; however, gestational thrombocytopenia is much more common during pregnancy and more likely to be the underlying cause of this woman’s presentation.

OPTION D : DIC is a serious life-threatening condition presenting with low-platelet count, intravascular coagulation, and thrombus formation potentially in every organ system. The two most common conditions associated with DIC during pregnancy are placental abruption and fetal demise. Laboratory studies show low levels of all of coagulation factors. An asymptomatic isolated thrombocytopenia is not a picture consistent with DIC.

OPTION E : Although thrombocytopenia is a common feature of pregnancy-related hypertensive disorders such as pre-eclampsia or HELLP syndrome. With pre-eclampsia other manifestations clinical or laboratory findings would be expected (e.g., proteinuria, hypertension, elevated liver enzymes, headache, abdominal pain, etc.) An isolated mild thrombocytopenia in a completely asymptomatic pregnant woman is unlikely to have been caused by pre-eclampsia.

NOTE - Compared to non-pregnant women with thrombocytopenia, pregnant women with thrombocytopenia tend to have fewer bleeding complications due to the procoagulant state induced by increased levels of fibrinogen, factor VIII and von Willebrand factor, suppressed fibrinolysis and reduced protein S activity. There are several other pregnancy-related conditions that can also lead to thrombocytopenia. Thrombocytopenia in pregnancy is a common reason for hematology consultation.

  • Medscape – Thrombocytopenia in Pregnancy * UpToDate – Thrombocytopenia in pregnancy
How well did you know this?
1
Not at all
2
3
4
5
Perfectly
106
Q

A 31-year-old woman presents with complaints of 10 kg weight loss and heavy periods for the past six months. However, her periods were and still are regular occurring at intervals of 32 days lasting for five days with no increased pain or discomfort. The number of pads she uses indicates blood loss of more than 80 mL per cycle. She has no known medical condition. The rest of the physical examination, including pelvic exam is inconclusive. An office urine pregnancy test is negative. Which one of the following would be the most appropriate investigation to consider first?

A. Transvaginal ultrasonography.
B. Abdominal ultrasonography.
C. Thyroid stimulating hormone (TSH) level.
D. Prolactin level.
E. Follicle stimulating hormone (FSH) level.

A

A. Transvaginal ultrasonography

This woman has presented with an altered pattern of menstrual bleeding in from of increased menstrual flow at regular intervals, namely ‘menorrhagia’. Menorrhagia is a form of abnormal uterine bleeding (AUB).

Normal menstrual periods last 3–6 days and is associated with blood loss of up to 80 ml. Menorrhagia is defined as menstrual periods lasting more than 7 days and/or involving blood loss greater than 80 mL.

AUB affects 9-14% of women. AUB generally can be divided into anovulatory and ovulatory patterns. Ovulatory pattern of AUB is characterized by abnormal volume or length of blood loss at regular intervals, whereas in anovulatory pattern the bleeding is irregular and unpredictable (no regular periods)

Ovulatory pattern is usually caused by uterine problems such as leiomyomas, endometriosis, adenomyosis, polyps, etc. as opposed to anovulatory pattern that is a result of hormonal problems such as polycystic ovarian syndrome (PCOS), hypothyroidism, hyperthyroidism, hyperprolactinemia, or Cushing syndrome.

The approach to women with ovulatory versus anovulatory pattern is different. It is recommended that if the pattern cannot be specified, the patient be assessed as having irregular bleeding because this pattern includes investigation for endometrial hyperplasia/cancer for more diagnostic safety.

In approach to every woman of reproductive age presenting with AUB, the most important to consider and exclude first is pregnancy regardless of the pattern. If anemia is suspected based on the history or clinical findings, a full blood exam (FBE) is indicated. A serum TSH to exclude thyroid disease is indicated if the history and/or physical examination raise suspicion against thyroid disease, especially hypothyroidism. Routine use of TSH in every woman presenting with AUB is not recommended.

Other investigations depend on the pattern and whether it is ovulatory or anovulatory. Since this woman has menorrhagia, indicating ovulatory AUB and the high possibility of structural anomalies such as fibroids, adenomyosis, polyps, etc., a transvaginal ultrasound is very likely to pick up the etiology and is the most important initial diagnostic approach to consider first and go with. Abdominal ultrasound is not as accurate as transvaginal ultrasound for this purpose yet can be used if transvaginal ultrasound is not possible (e.g., adolescent girls).

NOTE - A very significant finding in the history is the weight loss. For significant weight loss in this woman in the presence of AUB, the two most important differential diagnoses to think of are hyperthyroidism and cancer. Of these two, the greatest concern is cancer, especially endometrial cancer. Even if cancer is the case, transvaginal ultrasound is the initial option of choice for assessment of endometrial thickness and other abnormalities of the uterine cavity and adjacent structures. In the presence of any endometrial abnormality, the patient will require endometrial curettage and biopsy. Hyperthyroidism is more likely to be associated with amenorrhea or oligomenorrhea than menorrhagia; however, thyroid function tests can be considered as well once cancer as a main concern is safely excluded.

Hormonal assay like FSH (option E), LH, and prolactin (option D) are investigations to consider in patients with anovulatory AUB, where hormonal derangements are the most important underlying etiology to consider and think of.

  • Cancer Australia - Abnormal Vaginal Bleeding in Pre- and Post-menopausal Women
  • Clinical Care Standard - Heavy Menstrual Bleeding Clinical Care Standard
How well did you know this?
1
Not at all
2
3
4
5
Perfectly
107
Q

Angelina, accompanied by her mother, is in your office for consultation. She is 15 years old, and is concerned because she has not started her periods yet. She says: “I am about the same height as everybody else in my class at school. Their periods have started but mine not.” Which one of the following is the most appropriate initial question to ask her?

A. When she had her breast buds developed.
B. When she had her pubic hair.
C. Whether she gets cyclic abdominal pain.
D. Whether she had accelerated growth.
E. When she had her axillary hair.

A

A. When she had her breast buds developed

The complaint presented in the scenario is primary amenorrhea. Primary amenorrhea is defined as the absence of menses after the age of 16 in the presence of normal growth and secondary sexual characteristics, or 13-14 in the absence of secondary sexual characteristics. In case breast budding occurs before the age of 10, amenorrhea is defined as absent periods within the next 5 years.

Puberty in girls start with breast development followed by the growth spurt, growth of axillary and pubic hair, and finally menstruation. Breast above Tanner II stage indicates the commencement of puberty and exposure to estrogen.

In this case, it is very important to know whether puberty has started and if so, how long has she been through puberty. This can be known by simply asking if she has had any breast developments as the most appropriate question. Other question are relevant if puberty has already begun indicated by breast development.

  • NSW Health – Amenorrhoea
  • RACP – Investigating primary and secondary amenorrhoea
How well did you know this?
1
Not at all
2
3
4
5
Perfectly
108
Q

A 26-year-old woman with history of chronic immune thrombocytopenic purpura (ITP) presents to your clinic and seeks advice regarding pregnancy. She has a platelet count of 70000/mm3. Which one of the following is the correct statement in counselling her?

A. She should have splenectomy before pregnancy.
B. She should avoid pregnancy in the next 2 years.
C. She can become pregnant.
D. If she becomes pregnant, the mode of delivery should be cesarean section.
E. She cannot become pregnant while the platelet count is below 70000/mm3.

A

C. She can become pregnant

While uncommon, immune thrombocytopenia (ITP) is an important cause of thrombocytopenia in pregnant women either as a pre-existing condition or occurring at any time during pregnancy.

ITP developed in pregnancy is important to be distinguished from ‘incidental’ or gestational thrombocytopenia, which, is responsible for up to 80% of cases of thrombocytopenia in pregnant women, or from more serious conditions such as HELLP syndrome.

Gestational thrombocytopenia usually causes an approximately 10% decrease in platelet counts and is usually characterized by counts that are above 70,000/mm3. When platelets are lower than that, it is mostly ITP.

A diagnosis of ITP is usually made in 1-4% of cases of thrombocytopenia. ITP, either pre-exiting or developed during pregnancy, does not prevent a woman from becoming pregnant or safely delivering a healthy baby. The best advice for this woman is that she can become pregnant and ITP does not preclude vaginal delivery; however, the pregnancy is categorized as high risk and she will need extra care and probably treatment, especially around the delivery.

In pregnant women with ITP, treatment is usually not necessary as long as the platelet count is above 30,000, unless there is bleeding or easy bruising. For pregnant women with symptomatic platelet counts of above 30,000, or women with platelet count of below 30,000 regardless of symptoms, treatment either with prednisolone or intravenous immunoglobulin (IVIG) is required due to significant concern of bleeding, especially uterine bleeding.

It is important to maintain a platelet count that allows the pregnant woman with the condition to go through delivery. This safe count is at least 50,000. For regional anesethesia, a count between 70,000 and 100,000 is often demanded.

  • Medpage Today - Clinical Challenges: ITP in Pregnancy
  • Immune Thrombocytopenia in Pregnancy
How well did you know this?
1
Not at all
2
3
4
5
Perfectly
109
Q

Adele is a 23 years old primigravida woman who has a spontaneous abortion at 12 weeks gestation 2 weeks ago. She has presented to you because she wants to have another pregnancy as soon as possible as she thinks this could help her husband and her get over with their loss. She asks you when she can become pregnant again. Which one of the following would be the best advice?

A. She should not become pregnant for at least 6 months and should be on OCPs for now.
B. She can start trying to conceive again immediately if she feels fit for that.
C. She can conceive after 12 months.
D. She can conceive after 3 months.
E. She can conceive after 2 menstrual cycles.

A

B. She can start trying to conceive again immediately if she feels fit for that

Despite the emotional pain of pregnancy loss, most couples go onto having another pregnancy. It is recommended that women with pregnancy loss wait for one menstrual period after a pregnancy loss and then try the next pregnancy when they are feeling emotionally strong and ready.

For some, this could be 2 months, for others 2 years. It is generally recommended that women with pregnancy loss can conceive immediately after they feel fit both physically and emotionally to have another pregnancy. For this woman, next pregnancy can be planned once she feels emotionally ready.

The following are recommendations for women who are planning conceiving again after a pregnancy loss:

  • Reducing the chance of another pregnancy loss by starting the next pregnancy as healthy as possible. Some measures are quitting smoking and alcohol, achievement of a healthy weight through exercise and diet and adoption of healthy lifestyle in general.
  • Checking on any long term health problems such as depression, high blood pressure, or diabetes. Medicines might need to be altered or more emphasis placed on non- medical treatments after appropriate advice from health professionals.
  • Having a health check-up before the next pregnancy.

For couples with a lost pregnancy, peer support can be invaluable. Other mothers who have suffered pregnancy loss can have a real understanding of a woman’s experience as she goes through another pregnancy.

  • RANZCOG – Pregnancy loss
How well did you know this?
1
Not at all
2
3
4
5
Perfectly
110
Q

A 27-year-old woman is in labor at 39 weeks gestation when passage of meconium is noted. A cardiotocography (CTG) is arranged that shows a fetal heart rate (FHR) of 149 bpm, a beat-to-beat variability of 15, no acceleration, and no deceleration. Which one of the following should be the next best step in management?

A. Fetal scalp blood sampling as there is a 10% chance of hypoxia.
B. Fetal scalp blood sampling as there is a 50% chance of hypoxia.
C. Fetal scalp blood sampling as there is 75% chance of hypoxia.
D. Emergency cesarean section.
E. Close monitoring until delivery as there is no abnormality.

A

E. Close monitoring until delivery as there is no abnormality

Baseline fetal heart rate (FHR) is the mean level of the FHR when this is stable, excluding accelerations and decelerations. It is determined over a period of 5-10 minutes, expressed as beats per minute (bpm). Preterm fetuses tend to have values towards the upper end of the normal range.

Baseline variability is the minor fluctuation in baseline FHR. It is assessed by estimating the difference between the highest peak and lowest trough of fluctuation in one-minute segments of the fetal hear rate trace. Baseline variability is categorized as follows:
* Normal variability: 6-25 beats per minute
* Reduced variability: 3-5 beats per minute
* Absent variability: <3 beats per minute
* Increased (salutatory) variability: > 25 beats per minute.

Accelerations are transient increases in FHR of 15 bpm or more above the baseline that last at least 15 seconds. Accelerations in preterm fetuses may be of lesser amplitude and shorter duration.

Decelerations are transient episodes of decreased FHR below the baseline of more than 15 bpm lasting at least 15 seconds. The specific features of the deceleration inform the classification. Decelerations should never be described as ‘unprovoked’; the fetus will not decelerate its heart rate without physiological provocation. Uterine activity, even in its mildest form, will result in decelerations in a fetus whose oxygenation is already compromised.

Decelerations are categorized as:

  1. Early decelerations - benign and associated with the sleep cycle and often in the range of 4-8 cm of cervical dilatation. They are caused by head compression and in general are a normal physiological response to a mild increase in intracranial pressure. Importantly they are uniform in shape and start and finish with the contraction. They may be said to mirror the contraction.
  2. Variable decelerations - repetitive or intermittent decreasing of FHR with rapid onset and recovery. Time relationships with contraction cycle may be variable but most commonly occur simultaneously with contractions. The significance of variable decelerations depends on the overall clinical picture and specific features of the decelerations themselves, as well as other features of the CTG. Variable decelerations in association with other non-reassuring or abnormal features change the category of the deceleration to ‘complicated’.
  3. Prolonged decelerations - a decrease of FHR below the baseline of more than 15 bpm for longer than 90 seconds but less than 5 minutes.
  4. Late decelerations - defined as uniform, repetitive decreasing of FHR with, usually, slow onset mid to end of the contraction and nadir more than 20 seconds after the peak of the contraction and ending after the contraction. Late decelerations are caused by contractions in the presence of hypoxia. This means that they will occur with each contraction and the fetus is already hypoxic. There will be no features of a well oxygenated fetus, like early or typical variable decelerations, normal baseline variability or shouldering. They start after the start of the contraction and the bottom of the deceleration is more than 20 seconds after the peak of the contraction. Importantly, they return to the baseline after the contraction has finished. In the hypoxic fetus, this will include decelerations of less than 15 bpm (and occasionally less than 5 bpm).

A CTG is considered abnormal and demands further action or investigations if :

There is at least two of the following:
* Baseline FHR is between 100-109 bpm or between 161-170 bpm
* Variability of FHR is reduced (3-5 bpm for >40 minutes)
* Decelerations are variable without complicating features

OR

There is any of the following:
* Baseline FHR is <100 bpm or >170bpm
* Variability is absent (<3 bpm)
* Decelerations are prolonged for >3 minutes OR late OR have complicated variables

This CTG has none of the above and is completely normal; therefore, close monitoring until the delivery is all that is needed at this stage.

  • WA Health Department – Obstetrics and Gynaecology Guidelines
  • RANZCOG Intrapartum Fetal Surveillance Clinical Guideline
How well did you know this?
1
Not at all
2
3
4
5
Perfectly
111
Q

A 29-year-old woman presents to your practice with complaints of annoying pain and cramping with her periods as well as slight increase in her menstrual flow. She has had this problem since she started menstruating and investigations revealed no apparent cause for the presentation. Recently, she was advised to take combined oral contraceptives (COCs) for both contraception and treatment of her dysmenorrhea and was started on microgynon 30 mcg. Her dysmenorrhea, however, persists. Which one of the following would be the next best step in management?

A. Start her on NSAIDs during her menses.
B. Increase the dose of estrogen.
C. Decrease the dose of estrogen.
D. Prescribe progestogen-only pills (POP).
E. Advise Mirena®.

A

A. Start her on NSAIDs during her menses

Based on the history and the normal investigation, this woman has primary dysmenorrhea. The characteristic symptoms of primary dysmenorrhea include lower abdominal or pelvic pain with or without radiation to the back or legs, with initial onset six to 12 months after menarche. Pain typically lasts 8 to 72 hours and usually occurs at the onset of menstrual flow. Other associated symptoms may include low back pain, headache, diarrhea, fatigue, nausea, or vomiting. For primary dysmenorrhea to be the diagnosis, other underlying causes such as uterine leiomyoma, endometriosis, etc. should have been excluded by thorough investigations.

A Cochrane review of 73 randomized controlled trials, demonstrated strong evidence to support nonsteroidal anti-inflammatory drugs (NSAIDs) as the first-line treatment for primary dysmenorrhea. The choice of NSAID should be based on effectiveness and tolerability for the individual patient, because no NSAID has been proven more effective than others. NSAIDs should be taken 1 to 2 days before the anticipated onset of menses and continued for 2 to 3 days into the periods.

Oral, intravaginal, and intrauterine hormonal contraceptives have been recommended for management of primary dysmenorrhea; however, the evidence supporting their effectiveness is limited. Such methods are often considered second-line for treatment of dysmenorrhea. In patients who also desire contraception a 2-3 months trial of oral contraceptives is considered first. Both 28-day and extended-cycle oral contraceptives are reasonable options in women with primary dysmenorrhea who also desire contraception. If unresponsive, NSAIDs are added.

112
Q

A 23-year-old woman presents to your clinic with amenorrhea for 6 weeks. Which one of the following, if found on history or exam, will make you decide to proceed to emergency Intervention?

A. Vaginal bleeding.
B. Lower abdominal pain.
C. Lower abdominal tenderness.
D. Shoulder tip pain.
E. Rebound tenderness.

A

D. Shoulder tip pain

Abdominal pain in women may be related to pathology in the pelvic organs. Ovarian cysts, uterine fibroids, tuboovarian abscesses, and endometriosis are common causes of lower abdominal pain in women. In women of reproductive age, special attention to pregnancy complications, including ectopic pregnancy and loss of pregnancy is critical in forming an appropriate differential diagnosis. The possibility of pregnancy modifies the likelihood of disease and significantly changes the diagnostic approach.

In every woman of reproductive age, abdominal pain, especially in the presence of a positive pregnancy test, diagnoses such as ectopic pregnancy (EP) or miscarriage should come on top of the list of differential diagnoses and investigated thoroughly.

Specific clinical findings point to different conditions. Vaginal bleeding can be seen in EP as well as almost all types of abortions and failed pregnancies. Lower abdominal pain and tenderness is often another shared finding in the mentioned conditions. Lower abdominal rebound with minimal or no guarding is the most common abdominal findings in women with EP even when it is still unruptured. Such finding warrants immediate investigation and action because, for example, rupture of an EP can rapidly result in shock and catastrophic outcomes.

Of the option, however, shoulder tip pain is the most important sign demanding urgent action. This sign is not common but if present in suspected EP, demands urgent surgical action because indicates free blood in the peritoneum and peritoneal irritation. The mechanism such pain is produced is thought to be irritation of the diaphragm and the phrenic nerve by the free blood in the peritoneal cavity. Then pain can radiate to the shoulder tip especially on the right side.

NOTE - Generally, abdominal rebound tenderness indicates acute abdomen and surgical emergency. In specific circumstances such as in EP, however, the rebound tenderness could have been caused by an unruptured EP that can be taken care of non-surgically (e.g., methotrexate). Other indications of surgical acute abdomen include rigidity and peritonism.

  • RANZCOG – Ectopic pregnancy and miscarriage
  • Medscape – Ectopic Pregnancy
  • World Journal of Emergency Medicine - An unusual emergency department case: ruptured ectopic pregnancy presenting as chest pain
113
Q

A 32-year-old woman with history of primary infertility presents to your clinic with a 6-month history of secondary amenorrhea. Which one of the following hormones, if significantly elevated, would signify that pregnancy has already occurred?

A. Serum prolactin (PRL).
B. Serum leuteinizing hormone (LH).
C. Serum follicle stimulating hormone (FSH).
D. Serum progesterone.
E. Serum estradiol.

A

B. Serum leuteinizing hormone (LH)

Estradiol, prolactin and progesterone all are elevated during the pregnancy, but they may be also raised in other pathological conditions without pregnancy.

FSH is suppressed throughout pregnancy due to the negative feedback signaled by elevated eestradiol.

During the pregnancy β-HCG reaches it maximum plasma levels around 11-12 weeks of gestation and remains high throughout the pregnancy.

LH, FSH, TSH and HCG have identical alpha subunits. But only LH has a beta subunit (physiologically active component of LH) with similar structure to that of HCG; therefoe, virtually in all LH assays during gestation the LH will be reported high. Although LH levels of 100-150 IU/L may be seen in other conditions such as polycystic ovarian syndrome (PCOS), levels above 200 IU/L are highly indicative of pregnancy.

114
Q

Janet, 32 years of age, who is 33-week pregnant, presents to the Emergency Department with lower abdominal pain starting few hours ago and increasing in intensity. Her pregnancy so far has been uncomplicated, and she is not on any medications apart from supplements. Few days ago, she had an upper respiratory infection associated with sneezing, bouts of cough, runny nose, and mild fever from which she is recovering. She denies vaginal bleeding. On examination, her vitals are within normal limits. Abdominal exam reveals tenderness over the right lower part of her uterus, but no abnormal uterine contractions are note. A cardiotocograph (CTG) is normal with a fetal heart rate of 155 bpm. Which one of the following could be the most likely explanation to this presentation?

A. Diastasis recti abdominis.
B. Rectus sheath hematoma.
C. Placental abruption.
D. Placenta previa.
E. Vasa previa.

A

B. Rectus sheath hematoma

The scenario represents a rather sudden-onset abdominal pain in a pregnant woman. The history is unremarkable except for preceding sneeze and bouts of cough and abdominal tenderness in the absence of vaginal bleeding and fetal compromise which could be justified best by rectus sheath hematoma. Rectus sheath hematoma in pregnancy is uncommon or even rate yet the best explanation for this scenario.

Etiologies of recuts sheath hematoma are as follows:
* Anticoagulation therapy
* Severe cough
* Pregnancy
* Previous or recent abdominal surgery
* Abdominal trauma
* Chronic kidney disease
* Steroid/immunosuppressive therapy
* Vigorous uncoordinated rectus muscle contraction
* Some general medical condition

Janet already has two of these etiologies for the condition: pregnancy and cough as the most common inciting factor.

Rectus sheath hematoma can mimic acute abdomen; therefore, it is of paramount importance that other causes of acute abdomen be considered and fully investigate through history, physical exam, and relevant investigations. With pregnancy, other pregnancy-related conditions presenting with abdominal pain such as placental abruption should be highly suspected and excluded. For Janet however placental abruption is less likely of a diagnosis. Placental abruption can cause uterine tenderness and overt vaginal bleeding or even without vaginal bleeding if bleeding forms a retroplacental hematoma (concealed bleeding). There is often fetal compromise on CTG monitoring. The CTG in this scenario is within physiologic limits making placental abruption less likely. On the other hand, sneezing and bouts of cough are very unlikely to result in placental abruption. What is elicited as tenderness of the uterus is in fact overlying abdominal wall tenderness due to the hematoma.

Diastasis recti abdominis (DRA) (option A) is a midline separation at the linea alba of the rectus muscles of the abdominal wall. A diastasis is a palpable midline gap of more than 2.5 cm or any visible bulging on exertion or when the intrabdominal pressure is increased (Valsalva maneuvers, coughing, straining, etc.) DRA often develops around the umbilicus but can also occur anywhere between the xiphoid and the pubic bone. DRA can happen in varying degrees in pregnancy due to abdominal musculature stretch weakness from maternal hormonal changes and increased tension by the growing uterus. DRA presents as a painless midline swelling which increases in size and accentuates with increased abdominal pressure such as with coughing. The presence of tenderness and its locations in this scenario (RLQ) make DRA less likely of a diagnosis.

Placenta previa (option D) and vasa previa (option E) could present with painless vaginal bleeding. CTG usually shows compromised fetal parameters. With abdominal pain and normal CTG, these are less likely to be the cause of such presentation.

  • BMC – Journal of medical case report - Spontaneous rectus sheath hematoma in pregnancy and a systematic anatomical workup of rectus sheath hematoma: a case report
115
Q

Which one of the following is the most important factor for labor to start and progress?

A. Head molding.
B. Rupture of membranes.
C. Cervical effacement.
D. Cervical dilation.
E. Uterine contractions.

A

E. Uterine contractions

Labor is a physiologic process during which the fetus, membranes, umbilical cord, and placenta are expelled from the uterus. Labor has 3 stages:

  1. First stage of labor
    Begins with regular uterine contractions and ends with complete cervical dilatation at 10 cm. First stage of the labor divides into 2 phases:

a) Latent phase:
Begins with mild, irregular uterine contractions that soften and shorten the cervix. Contractions become progressively more rhythmic and stronger.
b) Active phase:
Usually begins at about 3-4 cm of cervical dilation and is characterized by rapid cervical dilation and descent of the presenting fetal part

  1. Second stage of labor
    Begins with complete cervical dilatation and ends with the delivery of the fetus
  2. Third stage of labor
    The period between the delivery of the fetus and the delivery of the placenta and fetal membranes. Delivery of the placenta often takes less than 10 minutes, but the third stage may last as long as 30 minutes.

For the labor to start and progress, uterine contractions are the most important inciting factor precipitating the chain of events at different stages of labor such as cervical dilation (option D) and cervical effacement (option C) . Rupture of membrane (option A) happens due to the force imposed on over-stretched amniotic membranes by the uterine contraction.

Skull sutures and fontanelles are flexible in fetus to accommodate brain growth. Head molding happens as a result of the fetal head passing through the birth canal during labor. The flexibility of the sutures may cause slight temporary disfigurement of the skull which is referred to as head molding. Head molding cause for it. (option A) is an effect of the labor not a cause for it.

  • MSD Manual – Management of Normal Labor
  • Medscape – Normal Labor and Delivery
116
Q

Alison is 33 years old and has come to you for contraception advice. Two years ago, while on combined oral contraceptive pills (COCP), she developed deep vein thrombosis; therefore, she was switched to a progestogen-only pill (POP). Last year, she came off the pill to start a family, but her pregnancy turned out ectopic, and aborted 6 months ago. She wants to start contraception again as she is not planning any pregnancies in near future. Which of the following would be the correct advice in this regard?

A. She cannot take POP.
B. She can start taking POP.
C. She can start taking COCP, but she will need prophylactic anticoagulation.
D. She can take POP after 12 months of her ectopic pregnancy.
E. She can start taking COCP.

A

B. She can start taking POP

The stem in question asks about the best approach to contraception after an ectopic pregnancy. Important to note in history is a contraindication to the use of contraceptives, in this case, developing deep venous thrombosis after COCP use, which makes it an absolute contraindication to use of COCPs (option C and E) in the future.

There is controversy regarding the use of POPs, with some studies reporting an increase in the incidence of ectopic pregnancy by reducing fallopian tube activity of cilia and altered tubal motility. The same mechanism is postulated to be responsible for the increased incidence of ectopic pregnancies following emergency contraception with POPS (progestogen- only emergency contraception, (POEC).

For the recommendations of contraception after contraception, women should be advised that any method of contraception can be safely initiated immediately after surgical or medical treatment for an ectopic pregnancy. Therefore, in this case, the woman can be safely advised to use POP. Telling her she cannot take POP (option A) is incorrect.

Telling her to start POP after 12 months (option D) is not recommended because contraception can be started immediately. An additional contraceptive (barrier or abstinence) may be required if hormonal contraception started 5 days or more after miscarriage (can be omitted if used immediately or within 5 days after the miscarriage.
All contraceptives are effective in preventing intrauterine and ectopic pregnancies. However, LARC (long-acting reversible contraceptive) is most effective in the reduction of any type of pregnancy including ectopic.

Women with a history of ectopic pregnancy are at risk of another one and they should be informed of the small risk of ectopic pregnancy while on contraceptive use if a pregnancy occurs. In this context, they should be offered LARCs since they are the most effective method in reducing all pregnancies including ectopic ones.

  • RANZCOG - FSRH Guideline Contraception after pregnancy
  • RCH - Oral hormonal contraception in special circumstances
  • Harrison-Woolrych M, Woolley J. Progestogen-only emergency contraception and ectopic pregnancy. BMJ Sexual & Reproductive Health. 2003 Jan 1;29(1):5-6.
117
Q

Mary is in your GP clinic for consultation. She is 35 years old and is annoyed by her periods which are regular yet heavy and prolonged. She has been already assessed with transvaginal ultrasound and blood tests with normal results except for mild iron-deficiency anemia. She is married and has no plans to start a family, not at least in one year. Which one of the following would be the most appropriate treatment option for her?

A. Progestogen-only pill (POP).
B. Combined oral contraceptive pills (COCP).
C. Mirena.
D. Implanon.
E. NSAIDs.

A

C. Mirena

Heavy menstrual bleeding (HMB) is the most common form of abnormal vaginal bleeding (AUB) in women of reproductive age and is defined as an excessive blood loss that impairs the woman’s quality of life either physically, emotionally, or socially. It is benign, and not associated with pregnancy or any other gynecological or systemic disease.

HMB can have a significant impact on quality of life and lead to time work, and fatigue related to iron deficiency anemia. Moreover, HMB can have a significant burden on healthcare resources. HMB is a common reason for referral to a gynecologist.

Treatment for HMB can be medical or surgical. Hysterectomy has traditionally been regarded as the ‘definitive’ treatment, but surgical options such as hysterectomy and the less invasive endometrial ablation are associated with risks and complications. Medical options enable women to retain their fertility and avoid the risks of surgery. The UK NICE guidelines on HMB recommend the following medical treatments: hormonal (levonorgestrel-releasing intrauterine system (LNGIUS), combined oral contraceptives, and progestogens), and non-hormonal (non-steroidal anti-inflammatory drugs (NSAIDs) and antifibrinolytics).

The choice of medication depends upon its appropriateness, likely acceptability to a woman, and whether she requires contraception. Combination contraception methods, in the form of a pill, the vaginal ring, and the transdermal patch, have all been shown to regulate the menstrual cycle in premenopausal women, with the added benefit of reducing MBL.

Management of HMB has to take into account the woman’s preferences, any comorbidities, the presence or absence of fibroids (including size, number, and location), polyps, endometrial pathology, or adenomyosis, and other symptoms such as pressure and pain.

The most appropriate treatment option for this woman is Mirena (C is correct), as she is young and has heavy menstrual bleeding. As a part of long-acting reversible contraceptives (LARCs), which provide highly effective ‘set-and-forget’ contraception and are suitable for provision in general practice. They have very few contraindications and can be recommended as first-line contraceptive options across the reproductive lifespan, from adolescence to perimenopause. Mirena is 99.7-99.9% effective and lasts up to 5 years with efficacy for contraception and HMB.

OPTION D : Implanon (etonogestrel single-rod contraceptive implant) provides highly effective (99.95%) contraception for up to three years. However, troublesome bleeding is the most common side effect leading to discontinuation of the implant. This makes it less desirable for this woman.

OPTION C : COCP could be a choice but compared to LNGIUS is less effective.

OPTION A : Progesterone only pill is associated with irregular and unpredictable blood loss and is not usually recommended as a treatment for HMB.

OPTION E : NSAIDs can be used to treat heavy menstrual bleeding but is not a contraceptive.

  • NICE guidelines UK for heavy menstrual bleeding, 2018
  • AJGP - Long-acting reversible contraceptives: New evidence to support clinical practice
118
Q

A 49-year-old woman presents with heavy irregular periods every 2 to 3 months lasting 7-10 days for the past seven years. She undergoes a diagnostic dilation and curettage (D&C) in the premenstrual period. Which one of the following findings on histological examinations, if present, is the most likely cause of her problem?

A. Normal secretory endometrium.
B. Cystic glandular hyperplasia.
C. Atypical hyperplasia.
D. An endometrial polyp.
E. Endometrial cancer.

A

B. Cystic glandular hyperplasia

Endometrial exposure to unopposed estrogen leads to endometrial hyperplasia and irregular endometrial shedding and periods.
Endometrial hyperplasia can be seen in the following different forms:

  • Simple hyperplasia (cystic without atypia) aka cystic glandular hyperplasia
  • Complex hyperplasia (adenomatous without atypia)
  • Atypical simple hyperplasia (cystic with atypia)
  • Atypical complex hyperplasia (adenomatous with atypia)

Of the above forms, simple hyperplasia, also termed cystic glandular hyperplasia (CGH) is the most common form of endometrial hyperplasia seen in perimenopausal women.
Endometrial hyperplasia can present with:
* Bleeding between periods
* Heavy and/or prolonged periods
* Vaginal discharge
* Abdominal pain

Statistically, the most common cause of heavy and/or irregular periods in a peri-menopausal woman is anovulatory cycles, giving rise to endometrial hyperplasia, mostly CGH form.

OPTION A : The clinical picture is not caused by a normal secretory endometrium.

OPTION C and E : Endometrial cancer and atypical hyperplasia can result in irregular heavy menstrual bleeding but they are less common compared with CGH.

OPTION D : Endometrial polyp can be a cause of heavy menstrual bleeding but firstly, it is less common compared to CGH and secondly, it only causes menorrhagea and not irregular periods.

119
Q

A 48-year-old woman presents with complaint of heavy irregular bleedings for the past five months. Her periods were completely regular before, occurring every 30 days and lasting for 6 days. Her last pap smear was performed 6 months ago with normal result. Physical examination is unremarkable. Which one of the following is the most likely cause of her presentation?

A. Endometriosis.
B. Endometrial polyps.
C. Endometrial carcinoma.
D. An ovulatory cycles.
E. Submucosal fibroids.

A

D. An ovulatory cycles

This woman is most likely experiencing perimenopausal erratic bleeding due to fluctuating levels of estrogen and progesterone. Failure to ovulate causes unopposed estrogen effect and endometrial hyperplasia that when undergoes shedding leads to metrorrhagia, menorrhagia or both.

Endometrial polyps (option B) , endometriosis (option A) and fibroids (option E) are associated with increased volume of bleeding, namely metrorrhagia, but not irregular patterns of bleeding.

Although endometrial carcinoma (option C) should always be borne in my mind in perimenopausal women, statistically it is less likely of a cause compared to anovulatory cycles as the underlying etiology of irregularities in bleeding pattern during the perimenopausal period.

120
Q

A 50-year-old woman presents to you complaining of hot flushes disturbing her daily life. Her medical history includes hysterectomy due to uterine fibroid and an episode of deep vein thrombosis 18 months ago. She asks for hormone replace therapy (HRT). Which one of the following would be the best treatment option for her?

A. Low-dose estradiol valerate.
B. Low-dose combined estrogen and progesterone.
C. Progesterone.
D. Estrogen dermal patch.
E. Estrogen implants.

A

D. Estrogen dermal patch

Estrogen-only HRT is more effective than combined HRT and indicated whenever endometrial cancer caused by unopposed estrogen is not a concern, such as in this woman who does not have a uterus.

Oral estrogen is known to stimulate the coagulation pathway and is associated with increased risk of thromboembolism; hence, systemic estrogen is not a good option for her. However, estrogen dermal patches have not been shown to be associated with increased risk of venous thromboembolism and can be safely use for this woman. Estrogen implants are the second-line option if the transdermal patches fail to control the symptoms.

TOPIC REVIEW

Combination HRT is associated with a 2- to 5-fold relative risk of venous thromboembolism (VTE). The risk is 1.2 -1.5 in estrogen HRT. Natural progesterone is not associated with increased risk of VTE, whereas, synthetic progesterones such as medroxy progesterone acetate (Depo Provera®) has been shown to be associated with a slight increase in risk of VTE.

Studies comparing oral and transdermal estorgen preparations have demonstrated that transdermally administered estrogen has little, if any, effect in increasing prothrombotic substances (due to first-pass effect in the liver) and may have beneficial effects on proinflammatory markers, including C-reactive protein, prothrombin activation peptide, and antithrombin activity. Also, in contrast to oral estrogen, transdermal estrogen may have a suppressive effect on tissue plasminogen activator and plasminogen activator inhibitor activity.

121
Q

Which of the following breast diseases has been shown to be associated with vitamin D deficiency?

A. Fibroadenoma.
B. Periductal mastitis.
C. Ductal carcinoma in situ.
D. Intraductalpapilloma.
E. Ductal ectasia.

A

C. Ductal carcinoma in situ

Studies suggest that vitamin D deficiency has been associated with an increased risk of cancer, especially colon cancer, prostate cancer, and breast cancer. Vitamin D may play a role in controlling normal breast cell growth and may be able to stop
breast cancer cells from growing.

  • Medscape - Vitamin D Linked to Enhanced Breast Cancer Survival
122
Q

Eve, 19 years old, comes to your office seeking advice on contraception. She is planning a trip to Thailand with her new boyfriend. She has no contraindication to contraception. Which one of the following would be the most appropriate advice?

A. Condoms.
B. Intrauterine contraceptive device (IUCD).
C. Oral contraceptive pills (OCPs).
D. Oral contraceptive pills and condoms.
E. Diaphragm and spermicide gel.

A

D. Oral contraceptive pills and condoms

For the following groups, it is recommended that contraception be adivsed with combined oral contraceptives (COCs) if not contraindicated PLUS condoms to prevent sexually transmissbles infections (STIs) such as chalmidya, gonorrhea, HIV, etc.:

  • Young women (<25 years)
  • Women older than 25 years with a new partner
  • Women older than 25 year with two or more partners in the last year
  • Women older than 25 years whose regular partner has multiple partner

Eve falls is younger than 25 years and must be recommended to use both COCs and condomes. Addittionally, she has started a new relationship, and this stresses more on such recommendation as the best advice.

123
Q

A 25-year-old woman presents for your advice regarding choice of oral contraceptive pills after delivery of her male baby two weeks ago. She used combined oral contraceptive pills before her last pregnancy. She is non-smoker and breastfeeds her baby on demand. Which one of the following is the best choice in this situation?

A. Restarting combined oral contraceptive pills.
B. Start progesterone-only pills
C. No contraception needed.
D. Advise condoms.
E. Refer to a gynecologist.

A

C. No contraception needed

Elevated prolactin levels and a reduction in gonadotropin-releasing hormone from hypothalamus during lactation suppress ovulation. This results in a reduction in leuteinizing hormone (LH) release and inhibition of follicular maturation. The duration of suppression varies and is influenced by the frequency and duration of breastfeeding and the length of time since birth.

Lactational amenorrhea (LAM) is an excellent method of contraception provided that all the following criteria are met:
* The woman remains amenorrheic
* Less than 6 months since giving birth
* The baby is fully breastfed (or breastfed with very infrequent supplements)

NOTE - if the woman is not breastfeeding or breastfeeds but not on a regular basis, ovulation recurs within 45 days but it is possible after 21 days. Ovulation may occur in the absence of menstruation.

The following contraception methods can beused in postpartum period in a lactating woman:
Condoms - can be used immediately; spermicide gels are not recommended.

Minipill (POP) - Can be started in breastfeeding women. Although it does not have an adverse effect on breast milk, because of a theoretical concern about the effect of sex steroids on the neonate the commencement of POP is recommended after 3-4 weeks.

Etonorgestrel implant (Implanon®) - It can be inserted anytime from 6 weeks postpartum. If the woman is amenorrheic, pregnancy should be excluded first. Small amounts of contraceptive steroids are excreted in breast milk and concerns about exposure prior to 6 weeks exist. If it is considered, risks and benefits should be fully discussed with the woman.

Progesterone intramuscular injection (Medroxy progesterone acetate e.g. Depo-Provera® or Depo-Ralovera®) – can be started anytime postpartum; however, it is recommended by WHO that it not be used until 6 weeks postpartum, unless more appropriate methods are not avaiable. Pregnancy should be excluded if >21 days.

NOTE - this method may be considered in women, who are breastfeeding and are less than 6 weeks postpartum if risk of subsequent pregnancy is high and other forms of contraception are unacceptable.

Mirena® (Levonorgestrel) IUD, or the Copper Intrauterine Devices (Cu-IUD) - Unless the IUD can be inserted within 48 hours postpartum, it should be delayed until 4 weeks after childbirth. Prior to insertion after 4 weeks, pregnancy should be excluded. Women who have had a cesarean section should not have an IUD inserted prior to 6 weeks postpartum due to the increased risk of perforation. Mirena® is effective within 7 days, and Cu-IUDs are effective immediately.

This woman is regularly breastfeeding her baby on demand; therefore, she probably does not need further contraception at least while breastfeeding in the first 6 months, but in reality there is an slight risk of pregnancy (about 2%) if menstruation resumes.

NOTE - Postpartum women should not have intercourse within the first 2 weeks after giving birth because of the risk of air embolism or infections. Overall, intercourse is recommended to be deferred to until 6 weeks postpartum to avoid various complications. If a women, who does not fullfill criteria for LAM, decides to start having sexual intercourse before that time offer condoms for the first 3-4 weeks and POP afterwards.

124
Q

A 27-year-old woman presents to your practice on the 10th day of her menstrual cycle because she missed her pills on 6th and 7th days and had unprotected intercourse on 8th and 9th days. She is on 35mcq ethinylestradiol. She wants your advice regarding contraception because she does not want to become pregnant. Which one of the following would be the most appropriate advice?

A. She should take the last missing pill now and continue the same OCP as usual.
B. She should stop taking pills from the current package and starts a new one from now.
C. Give emergency contraception now using Postinor® (2 doses 12 hours apart).
D. Stop OCP and wait for 4 weeks and check if she is pregnant.
E. She should take the last missed pill now and continue the same OCP as usual and use condoms or abstain for 7 days.

A

C. Give emergency contraception now using Postinor® (2 doses 12 hours apart).

The following are the latest recommendations regarding missed oral contraceptive pills and emergency contraception if sexual intercourse occurs within the ‘missed pill’ period:

If one pill (active) is missed, anywhere in the pack (i.e. more than 24 and up to 48 hours late):

  • The last pill missed should be taken now, even if it means taking two pills in one day.
  • The rest of the pack should be taken as usual.
  • No additional contraception is needed.
  • The seven-day break is taken as normal.

Emergency contraception Emergency contraception is not needed if just one pill has been missed. However, it should be considered if other pills have been missed recently, either earlier in the current packet, or at the end of the previous packet.

If two or more pills are missed (i.e. more than 48 hours late):

  • The last missed pill should be taken now, even if it means taking two pills in one day.
  • Any earlier missed pills should be left.
  • The rest of the pack should be taken as usual and additional precautions (e.g. condoms or abstinence) should be taken for the next seven days.

The next step then depend on where in the packet the pills are missed

  • -If the pills are missed in the first week of a pack (pills 1-7): emergency contraception should be considered if the patient had unprotected sex in the pill-free interval or the first week of the pill packet. She should finish the packet and have the usual pill-free interval.
  • -If the pills are missed in the second week of a pack (pills 8-14): there is no need for emergency contraception as long as the pills in the preceding seven days have been taken correctly. The packet should be finished and the usual pill-free interval taken.
  • -If the pills are missed in the third week of a pack (pills 15-21): the next pack of pills should be started without a break – i.e. the pill-free interval is omitted. If taking a packet with dummy/placebo pills, these should be discarded, and the new packet started. Emergency contraception is not required.

NOTE - If more than seven pills are missed, the woman should start again as if starting for the first time. (Exclude pregnancy, and start a new pack on the first day of the next menstrual period.)

—–

This woman has two missed pills on days 6 and 7 (within the first) week; therefore, in order to prevent pregnancy she should have emergency contraception using Postinor® (750mcq of levonorgestrel, 2 tablets, 12 hours apart) as the most important step. Postinor is effective if given within the 72 hours (up to 96 hours) of unprotected sexual intercourse. She should also continue with the rest of the package as well as using condoms or abstinence for 7 days. As a rule when missed pills (even 1) are within the first 7 pills, emergency contraception is needed if sexual intercourse has been done in the missed pill period.

125
Q

A 32-year-old woman comes to your practice because she forgot to take her 7th and 8th p pills from a pack of OCPs containing 20mcg ethinyloestradiol (EO) and had unprotected intercourse on days 6 and 7. Today is the 10th day of her menstrual cycle. She wants advice regarding contraception because she does not want to become pregnant. Which one of the following would be the best advice for her?

A. She should take the last missing pill now and continue with the same pack of OCP.
B. Give emergency contraception now using Postinor® (2 doses 12 hours apart).
C. She should take the last missed pill now and continue the same OCP as usual and use condoms or abstain for seven days.
D. Stop OCP and wait for 4 weeks and check if she is pregnant.
E. She should stop taking pills from the current package and starts a new one with her new period.

A

B. Give emergency contraception now using Postinor® (2 doses 12 hours apart)

The following are the latest recommendations regarding missed oral contraceptive pills and emergency contraception if sexual intercourse occurs within the ‘missed pill’ period:

If one pill (active) is missed, anywhere in the pack (i.e. more than 24 and up to 48 hours late):

  • The last pill missed should be taken now, even if it means taking two pills in one day.
  • The rest of the pack should be taken as usual.
  • No additional contraception is needed.
  • The seven-day break is taken as normal.

Emergency contraception Emergency contraception is not needed if just one pill has been missed. However, it should be considered if other pills have been missed recently, either earlier in the current packet, or at the end of the previous packet.

If two or more pills are missed (i.e. more than 48 hours late):

  • The last missed pill should be taken now, even if it means taking two pills in one day.
  • Any earlier missed pills should be left.
  • The rest of the pack should be taken as usual and additional precautions (e.g. condoms or abstinence) should be taken for the next seven days.

The next step then depend on where in the packet the pills are missed

  • -If the pills are missed in the first week of a pack (pills 1-7): emergency contraception should be considered if the patient had unprotected sex in the pill-free interval or the first week of the pill packet. She should finish the packet and have the usual pill-free interval.
  • -If the pills are missed in the second week of a pack (pills 8-14): there is no need for emergency contraception as long as the pills in the preceding seven days have been taken correctly. The packet should be finished and the usual pill-free interval taken.
  • -If the pills are missed in the third week of a pack (pills 15-21): the next pack of pills should be started without a break – i.e. the pill-free interval is omitted. If taking a packet with dummy/placebo pills, these should be discarded, and the new packet started. Emergency contraception is not required.

NOTE - If more than seven pills are missed, the woman should start again as if starting for the first time. (Exclude pregnancy, and start a new pack on the first day of the next menstrual period.)

—–

This woman has missed two pills (>48 hours); therefore she should follow the following steps:

  • She should use emergency contraception now – although one pill is missed on day 8, since there has been no complete seven-day coverage, emergency contraception is needed.
  • She should take other pills and use condoms or abstain for 7 days.

The main concern of this woman is preventing the pregnancy so the most important advice would be emergency contraception. Post-coital methods have no effect on an implanted pregnancy. It is recommended that emergency contraception be performed within the first 72 hours post-coitus; however, it can be effective up to 96-120 hours (4-5 days) post-coitus.

126
Q

A 28-year-old woman comes to your practice because of nausea, vomiting and diarrhea after she had a burger in a local restaurant. She is on oral contraceptive pills. She has not missed her pills even when she was severely sick. Which one of the following would be the most appropriate advice to give her regarding contraception and the pills?

A. She should continue the pills and use condom for 7days.
B. She should stop the pills now and starts a new pack for the next cycle.
C. She should discontinue the pills and use a barrier method until diarrhea settles.
D. Give her ciprofloxacin and review in 2 days.
E. She should avoid sexual activity for 3 weeks.

A

A. She should continue the pills and use condom for 7days.

In the following situations, the effectiveness of oral contraceptive pills is reduced and additional measures should be taken:

  • An active pill is forgotten to be taken for more than 24 hours from the last pill.
  • Severe diarrhea or vomiting lasting more than 24 hours. Virtually, this has the same effect of not having the next pill for more than 24 hours after the last.

In situations like these, the woman should take the next pill on the routine schedule but an extra protection method such as barrier methods should be used, or intercourse avoided for 7 days.

127
Q

A 32-year-old woman, with past medical history of deep vein thrombosis, comes to your clinic for advice. She has been taking progesterone only pills (POP) until 4 months ago when she planned to become pregnant. The pregnancy, however, turned out to be ectopic. Fortunately, she survived through. She is not willing to become pregnant again and asks whether she can start the pills again. Which one of the following would be the best advice you can give?

A. She cannot take progesterone only pills.
B. She should use combined oral contraceptive pills.
C. She can take progesterone only pills after 12 months of first ectopic pregnancy.
D. She should take anticoagulants along with combined oral contraceptive pills.
E. Tell her she should get pregnant as that is the only option.

A

A. She cannot take progesterone only pills

With history of DVT, estrogen-containing contraceptives are contraindicated in this woman. This is probably why she was previously started on POP instead of regular combined pills. With the history of ectopic pregnancy, progesterone is contraindicated as well and should never be used again. Other options such as barrier methods could be used in this woman.

  1. Absolute contraindications to using progesterone only pills include:
    * Suspected pregnancy
    * Breast cancer
    * Undiagnosed vaginal bleeding
    * Past history of ectopic pregnancy or high risk for ectopic pregnancy
  2. Relative contraindications to progesterone only pills include:
    * Active viral hepatitis
    * Severe chronic liver disease
    * Malabsorption syndrome
    * Severe arterial disease
    * Successfully treated breast cancer more than 5 years ago concomitant use of hepatic enzyme enducing medications
128
Q

A 20-year-old woman presents to your practice with complaints of episodes of erratic vaginal bleeding for the past four months after she started taking combined oral contraceptive pills (Microgynon 30). She is sexually active and uses condoms for prevention of sexually transmitted infections as well. Which one of the following would be the most appropriate advice?

A. She should switch to progestogen-only pills.
B. She should switch to a new combined pill with ethinylestradiole 50mcg.
C. She should use intrauterine contraceptive device.
D. She should continue the same oral contraceptive pill.
E. She should use Implanon®.

A

B. She should switch to a new combined pill with ethinylestradiole 50mcg

Erratic breakthrough vaginal bleeding is seen in 30% of women who are recently started on combined oral contraceptive pills (COCPs). This percentage, however, decrease to only 10% after 3 months.

This problem is more common with preparations that contain low-dose estrogen. If the history of erratic bleeding is less than three months, only reassurance is required as the condition resolves after three months of use in most women. Once the problem persists beyond three months, the next best step would be increasing the dose of estrogen (e.g. to 50mcg).

129
Q

A 24-year-old woman presents to your clinic for advice regarding contraception. She has been on oral combined oral contraceptive pills for the past two years. Two months ago, she developed deep vein thrombosis and has been on treatment with warfarin. Her mother and sister also have history of deep vein thrombosis. Which one of the following would be the most appropriate advice?

A. She should use male condoms.
B. Continuation of combined oral contraceptive pills.
C. Estrogen patch.
D. Progestogen-only pill.
E. Discontinuation of warfarin.

A

A. She should use male condoms

Any patient with history of DVT should discontinue estrogen-containing oral contraceptive pills.

Guidelines released by the WHO (World Health Organization) advocate that for most women with a family or personal history of deep vein thrombosis (DVT) or pulmonary embolism (PE) the benefits of using a POP outweigh the risks, and a POP can generally be used.

The guideline, however, mentions that the use of a POP is not recommended in the presence of current venous thromboembolism (VTE) unless other methods are not available or are unacceptable. Medicines Adverse Reaction Committee (MARC) considers that POPs are absolutely contraindicated in women with a current thromboembolic process.

Since this woman has a current DVT, for which she is on therapy, not only combined contraceptive are absolutely contraindicated, but progesterone-only pills are better avoided in this situation, and an alternative non-hormonal contraceptive methods such as condoms is used.

NOTE - Without making any distinction between minor and major surgery, WHO guidelines recommend that POPs can be continued for women undergoing any surgical procedure. Where there is prolonged immobilization following surgery, the guidelines mention that it is generally appropriate to continue POP use. Based on this advice, the MARC suggests that when determining the most appropriate oral contraceptive, a POP may also be considered for women with other risk factors that put them at a high risk of VTE with a combined oral contraceptive, especially for those with more than one risk factor (e.g. family history and BMI > 30).

130
Q

Which one of the following is the first-line contraceptive choice for women with epilepsy, who are on enzyme-inducing anti-epileptic drugs?

A. Combined oral contraceptive pills with low-dose estrogen.
B. Levonorgestrel-releasing intrauterine contraceptive device.
C. Barrier methods.
D. Progestin-only pills.
E. Combined oral contraceptive pills with high-dose estrogen.

A

B. Levonorgestrel-releasing intrauterine

There are a few management options for women on enzyme-inducing medications, levonorgestrel-releasing intrauterine contraceptive device (Mirena®) is the first-line contraceptive choice. Other methods to consider are male condom or increasing the dose of estrogen.

Mirena® is a very effective form of contraception. The effect of progesterone is mediated locally and is not affected by enzyme-inducing antiepileptic drugs. Failure rate is 1%, with prompt reversion of fertility once the contraception is withdrawn.

  1. Contraindications to intrauterine devices include:
    * Pregnancy
    * Active pelvic inflammatory disease (PID) or the history of acute PID within the past 3 months
    * Postpartum or post-abortion endometritis in the past 3 months
    * Uterine abnormalities resulting in anatomic distortion
    * Genital bleeding of unknown etiology
    * Untreated vaginitis or cervicitis, including bacterial vaginosis
    * Previously inserted IUD still in place
  2. Relative contraindications:
    * Menorrhagea
    * Dysmenorrhea
    * Very large or very small uterus (>5.5cm or <0.9cm)
    * Anemia
    * Defective immune system
    * Impaired clotting mechanism
    * Valvular heart disease
    * Patient or partner with multiple sexual partners

Combined oral contraceptive pills are not the first-line option for contraception in patients taking enzyme inducing anti-epileptic drugs. If a patient chooses to use combined oral contraceptive pills, those with higher estrogen content should be considered.

Progestin-only pills are not suitable for women who are on enzyme inducing antiepileptic drugs which can cause reduction in circulating levels of progestin..

131
Q

A 38-year-old woman presents to your clinic for advice. She is smoker and has been on progesterone-only pills for the past 3 years. Recently, following an episode of tonic-clonic convulsion, a neurologist has started her on carbamazepine. Which one of the following would be the most appropriate advice regarding contraception?

A. She should stop progesterone-only pill and use condoms.
B. She should cease the progesterone-only pill and start taking combined oral contraceptive pills.
C. She should stop carbamazepine.
D. She can continue both carbamazepine and progesterone-only pills as there is no interaction.
E. The dose of progesterone-only pill should be doubled.

A

A. She should stop progesterone-only pill and use condoms.

Enzyme-inducing drugs can affect the efficacy of oral contraception. Concomitant use of enzyme-inducing drugs may cause a 40-50% decrease in plasma levels of both progesterone and estrogen, resulting in unwanted pregnancy.

Enzyme inducing drugs affect the following:
* Combined oral contraceptive pills (COCs)
* Progestin-only pills
* Progestin implants (Implanon®)

NOTE - the efficacy of Mirena® and Depo Provera® (medroxy progesterone acetate) are not affected by enzyme-inducing drugs.

Carbamazepine is an enzyme inducer that can reduce the efficacy of oral contraceptive pills. Women using COCs , POPs or Implanon® should be advised that the efficacy of these methods may be reduced by enzyme-inducing drugs and that they should use an alternative method, such as male condoms. While there is a significant interaction between these two advising continuing use of both drugs together is not appropriate.

As this woman is smoker and older than 35 years, estrogen-containing contraceptives are contraindicated.

Doubling the dose of the POP is not an appropriate option. Rather the women should be advised to use levonorgestrel (LNG) IUCD (Mirena®), injectable medroxy progesterone (Depo Provera®), or male condoms.

OPTION C : Cessation of carbamzepine is not appropriate as this can result in uncontrolled seizures.

TOPIC REVIEW

Enzyme-inducing drugs and contraception:

  • All women starting enzyme-inducing drugs should be advised to use a reliable contraceptive method unaffected by enzyme inducers (e.g. progesterone-only injectable, copper-bearing intrauterine devices (Cu-IUDs) or the levonorgestrel-containing
    intrauterine system (LNG-IUS))
  • Women who do not wish to change from a combined method while on short-term treatment with an enzyme-inducing drug (and for 28 days after stopping treatment), may choose to continue using a combined oral contraceptive (COC) containing at least 30 mcg ethinylestradiol, the patch or ring along with additional contraception. An extended or tricyclic regimen should be used with a hormone-free interval of 4 days. Additional contraception should be continued for 28 days after cessation of the enzyme inducing drug. Tricycling is taking three packs of COC (3x21) in a row with 4-days hormone-free intervals (3x21)
  • With the exception of the very potent enzyme inducers rifampicin and rifabutin, women who are on an enzyme-inducing drugs, who do not wish to change from COC may increase the dose of COC to a preparation with at least 50mcg ethinyl estradiol (up to 75mcq) and use an extended or tricycling regimen with a pill-free interval of 4 days

NOTE - with rifampicin and rifabutin, advising to increase the dose of the COC is inappropriate. An alternative method should be used.

  • In women using enzyme-inducing drugs with COC, breakthrough bleeding may indicate low serum ethinylestradiol concentrations. If other causes (e.g. Chlamydia) have been excluded, the dose of ethinylestradiol can be increased up to a maximum of 75 mcg (or according to some references 100mcg)
  • Women, who do not wish to use additional contraception, or women on long-term treatment (>2 months) with an enzyme-inducing drug, who do not wish to change to another method, may be offered an increased dose of COC containing at least 50mcq ethinylestradiol during treatment and for 28 days afterwards. An extended or tricycling regimen and pill-free interval of 4 days are recommended but additional contraception is not essential.

Progesterone-only pills, implants, and enzyme-inducing drugs

  • Women using progesterone-only pill (POP) or implant should be advised that the efficacy of these methods may be reduced by enzyme-inducing drugs and that they should use an alternative method.
  • Women using POP or implant with a short-term course of enzyme-inducing drug (<2 months) could be offered an injection of DMPA. Women using enzyme-inducing drugs in the short term, who do not wish to change their contraceptive method, should be advised that they must take additional contraceptive precautions while on the enzyme-inducing drug and for 28 days after cessation of treatment
  • Women who do not wish to change from the progesterone-only pill or implant while on short term treatment with an enzyme-inducing drug or within 28 days of stopping treatment may choose to continue the method together with additional contraceptive precautions (e.g. condoms). Additional precautions should be continued for 28 days after stopping the enzyme-inducing drug.
132
Q

A 50-year-old menopausal female presents with frequent hot flushes disturbing her sleep and her work. Her mother had breast cancer at 50 years of age. She is otherwise in good health, with no significant previous medical or surgical history. Which one of the following is the best management option for her symptoms?

A. Combined hormone replacement therapy.
B. Progesterone.
C. Paroxetine.
D. Blackcohoch.
E. Estrogen dermal patch.

A

A. Combined hormone replacement therapy

Currently, hormone replacement therapy (HRT) is the most effective treatment for troublesome vasomotor symptoms of menopause such as hot flushes. Other benefits include: significant reduction in fracture risk, improvement of vaginal dryness and sexual function, improvement of sleep, decreased muscle aches and pain, and improved quality of life in symptomatic women.

For women with a uterus, the regimen consists of both estrogen and progesterone. Although HRT with estrogen alone has shown to be more effective, progesterone should be added to counteract the effect of unopposed estrogen on uterus and endometrial hyperplasi. However, in a woman without a uterus, estrogen-only regimens are preferred.

Current evidence-based guidelines advise consideration of HRT for troublesome vasomotor symptoms in perimenopausal and early postmenopausal women without contraindications and after individualized discussion of likely risks and benefits. HRT reduces the fracture risk and improve mood and libido, but none of these is an indication for commencement of HRT. Vaginal symptoms alone do not require systemic HRT and can be managed with topical estrogen.

Starting HRT in women over the age 60 years is generally not recommended. For women with premature (age <40 years) or early (<45 years) menopause, current guidelines recommend HRT until the age of 50 for treatment of vasomotor symptoms and bone preservation.

As a history of hysterectomy is not mentioned in the question, it should be assumed that the patient has a uterus; therefore the treatment of choice would be combination therapy with estrogen and progesterone if there are no contraindications.
There is no consensus on absolute contraindications to HRT. However, HRT should be avoided or discontinued in patients with the following conditions:

  • Personal history of breast cancer - HRT may increase the risk of breast cancer recurrence and development of new breast cancers. Family history of breast cancer is not a contraindication. Any breast abnormality should be excluded before starting HRT. Counsel women considering HRT that it may increase their risk of an abnormal mammogram and that combined HRT may increase their risk of breast cancer after 4-5 years of use.
  • A personal history or known risk factor for venous or arterial thromboembolic disease, including stroke and cardiovascular disease. If HRT is decided in such patients, a transdermal preparation with minimal estrogen is preferred.

NOTE - In the absence of a personal or family history of venous thromboembolism, screening for inherited thrombophilias before starting HRT is not indicated.

  • Uncontrolled hypertension.
  • Abnormal vaginal bleeding - HRT should not be started in women with undiagnosed abnormal vaginal bleeding. Combined HRT may often cause unscheduled bleeding in the first six months of use. Persistent or new onset (after six months) unscheduled bleeding requires investigation to exclude pelvic disease.

The following precautions should be taken in using HRT:
* Avoid oral HRT products in the presence of abnormal liver function tests, since these estrogen and progesterone are metabolized in the liver.
* Migraine does not seem to be exacerbated by HRT, but if the patient has migraine, low-dose transdermal preparations are preferred.
* History of endometrial or ovarian cancer - seek specialist review before considering HRT.
* High risk of gallbladder disease - Advise that HRT may increase the risk gallbladder disease. The risk may be lower with transdermal therapy.

While family history of breast cancer is not a contraindication to use HRT, it should be avoided in patients with personal (not family) history of breast cancer with estrogen or progesterone receptors or both being positive.

OPTION B : The safety or efficacy of progesterone alone for treatment of menopausal symptoms in a patient with history of breast cancer is not known.

OPTION C : SSRIs are widely used in Australia for management of hot flushes when HRT is contraindicated. They also help with improving mood symptoms. But they lack other effects of HRT such as increasing libido and predervation of bone density. SSRIs are used for treatment of hot flushes if HRT is contraindicated.

OPTION D : Black Cohosh is sometimes used for hot flushes. The mechanism of potential is unknown. Many studies have found no significant benefit over placebo.

OPTION E : Oral route is the most effective way of taking HRT. This woman has no condition to necessitate the use of dermal patches of estrogen (which delivers less estrogen)

133
Q

A 64-year-old man is brought to the Emergency Department with confusion. He is a known case of hypertension and heart failure and is on indapamide and enalapril for treatment. Laboratory studies are significant for a serum sodium level of 120 mmol/L. Which one of the following would be the most appropriate next step in management?

A. Restrict oral water intake to 500ml/day.
B. Hypertonic saline.
C. Stop indapamide.
D. Stop enalapril.
E. Stop both enalapril and indapamide.

A

B. Hypertonic saline

The clinical presentation in this scenario is most likely to have been caused by hyponatremia. If symptomatic, hyponatremia and hypernatremia present with CNS-related symptoms such as weakness, lethargy, confusion, seizure, coma, and even death.

Hyponatremia can be caused by a variety of conditions including:
* Medications, especially diuretics, ACH inhibitors, Angiotensin receptor blockers (ARBs), etc.
* Dehydration
* Syndrome of inappropriate ADH (SIADH)
* Congestive heart failure
* Renal failure
* Liver failure
* Hormonal diseases e.g. Addison’s disease, hypothyroidism, etc.

Mild hyponatremia is managed with fluid restriction; whereas severe hyponatremia should be treated with prompt sodium replacement. There are debates as to whether normal saline or hypertonic saline should be used, but it is generally accepted that patients with severe neurological symptoms should initially be treated with hypertonic saline. The following conditions, when present, are indications for the administration of hypertonic saline after expert consultation and with great caution:
1. Confusion
2. Seizures
3. Coma

The first step to consider in the management of this patient is dealing with hyponatremia while an investigation for an underlying cause is carried out. With the presence of confusion, cautious use of hypertonic saline would be the next best step in management.

The sodium level is not an indicator of severity; the clinical status is. A very low serum sodium level can be well tolerated if it has occurred chronically, whereas an acute fall may result in severe CNS problems and death. According to Royal ChildreChildren’sal guidelines, lethargy is an indication for the administration of hypertonic saline (3%) in children. It remains unclear whether this sign signifies the need for hypertonic saline use in adults.

Indapamide is a thiazide-like diuretic used for the treatment of hypertension. Hyponatremia and hypokalemia are two well-known adverse effects of thiazide and thiazide-like diuretics. The hyponatremia caused by diuretics can be either hypovolemic or euvolemic. ACE inhibitors, through a not-fully-understood mechanism, can cause both hypovolemic and euvolemic hyponatremia as well. Enalapril and indapamide should also be stopped (option C,D and E) and their use and dose, either as stand-alone medication or in combination, should be reassessed once the patient is stable. Response to the cessation of the medications implicated in hyponatremia is delayed and depends on the half-lives of the drugs. Therefore, stopping the offending drug alone will not help with the rapid resolution of life-threatening neurological symptoms of severe hyponatremia.

OPTION A : Fluid restriction is used for patients with asymptomatic or mildly symptomatic patients with hyponatremia and is not appropriate as a stand-alone therapy for this patient with severe CNS symptoms.

  • Medscape - Hyponatremia in Emergency Medicine * RCH - Hyponatraemia
134
Q

Which one of the following antiepileptic medications does not affect the efficacy of hormonal contraceptive pills?

A. Topiramate.
B. Carbamazepine.
C. Primidone.
D. Levetiracetam.
E. Phenytoin.

A

D. Levetiracetam

Epilepsy does not pose any restrictions on the use of hormonal contraceptive methods, but restrictions may apply if certain antiepileptic drugs (AEDs) are used.

Some AEDs can reduce the efficacy of the following hormonal methods:
1. Combined oral contraceptive pills
2. Progestin-only pills
3. Progestin implants (Implanon®)

NOTE - the efficacy of Mirena® and Depo Provera® (medroxy progesterone acetate) are not affected by enzyme inducing AEDs.

The mechanism of action is through inducing more cytochrome P450 activity and more accelerated clearance of hormonal contraceptives. On the other hand, circulating sex-hormone binding globulins are increases and results in decreased levels of freely circulating contraceptive medications.

The following drugs reduce the efficacy of hormonal contraceptive pills with the mentioned mechanisms:
* Phenobarbital
* Primidone
* Phenytoin
* Carbamazepine
* Oxcarbazepine
* Topiramate

Following antiepileptic drugs are non-inducers and do not affect the efficacy of hormonal contraceptive pills:
* Valproic acid
* Gabapentin
* Levetiracetam
* Tigabine
* Zonisamide
* Pregabalin
* Vigabatrin
* Ethosuximide
* Benzodiazepines
* Lamotrigine

If hormonal methods are considered for contraception while the patient is on enzyme-inducing antiepileptic drugs, an increased dose of estrogen, or alternative methods should be used. Mirena® or Depo Provera® can be an excellent choice if not contraindicated.

NOTE - Combined oral contraceptives (COCs) increases the clearance of lamotrigine and reduce serum lamotrigine levels. Women using lamotrigine should be advised that seizure frequency may increase when initiating COCs while on lamotrigine.

135
Q

A 37-year-old smoker woman presents for contraception advice. She smokes 20 cigarettes a day. Which one of the following is an appropriate option for her?

A. Combined oral contraceptive pills.
B. Condoms.
C. Progestogen-only pills.
D. Diaphragm.
E. Cervical cap.

A

C. Progestogen-only pills

Smoking more than 15 to 20 cigaarettes a day in a woman aged more than 35 year is a contraindication to combined oral contraceptive pills . Progesterone-only preparations (e.g. progesterone-only pills) are options of choice when estrogen-containing preparations are contraindicated. Failure rate per year is 3 to 10%, compared to 2 to 6% for combined pill.

OPTION B : Male condoms are the next best appropriate option with a Pearl index* of 3 compared to 0.5 of POPs.

OPTION D : Diaphragm is dome-shaped rubber cup with a flexible rim that fits over the cervix. It creates barrier to the sperm. The fitness should initially be checked by a medical practitioner. It should remain in place for more than 8 hours after the coitus. Pregnancy rate is 6% in the first year. It is not routinely used anymore in Australia.

OPTION E : Cervical cap is slightly more rigid than diaphragm and is smaller in size. It comes in several sizes. It needs to be fitted by health care practitioner. It can be left in place for 48 hours. Pregnancy rate is over 9%. It is not a favorite and advisable method of contraception anymore.

*The Pearl index, also called the Pearl rate, is the most common technique used in clinical trials for reporting the effectivenss of a birth control method

Pearl-Index = [(number pregnanciesx12)/(number of womenxnumber of months)]100

136
Q

A 23-year-old university student visits your clinic for advice about contraception. She reports migraine-type headaches once or twice a month for the past 3 months. She already has been on combined oral contraceptive pills for the past 2 years. Which one of the following would be the most appropriate advice?

A. Discontinuation of combined oral contraceptive pills.
B. Stopping the combined oral contraceptive pills and starting progesterone only pills (POP).
C. Continuation of combined oral contraceptive pills.
D. Refer her to an obstetrician for further management.
E. Tell her to use barrier methods, as those would be safer.

A

B. Stopping the combined oral contraceptive pills and starting progesterone only pills (POP)

Since this patient has developed migraine headache while on combined oral contraceptives (COCs), the best advice would be discontinuation of COCs and using progestogen-only pills (POPs) after discussion of potential adverse effects with her.

COCs are generally considered safe in women who have migraine without aura under the age of 35; however, if the migraine has started after the initiation of COCs, COCs will be relatively contraindicated.

Migraine with aura remains the absolute contraindication to the use of COCs because of the increased risk of ischemic stroke.

Progestogen-only methods provide a useful contraceptive option for any woman, in whom estrogen is contraindicated, irrespective of age or parity. No harmful effects have been found on long-term follow-up of children whose mother used progesterone-only methods. This makes POPs a good contraceptive option for lactating women as well.

Indications to use progesterone-only methods as the first-line options include:
* Hypertension
* Superficial thrombophlebitis
* History of thromboembolism
* Biliary tract disease
* Thyroid disease
* Epilepsy
* Diabetes mellitus without vascular disease
* Migraine – see the explanations above lactation
* Chloasma

Contraindications to progesterone-only methods are:
* Pregnancy
* Breast cancer
* Undiagnosed genital tract bleeding
* History of or risk factor for ectopic pregnancy
* Active liver disease
* Liver cirrhosis

137
Q

Which one of the following statements is correct regarding post-coitus contraception with Postinor-2®?

A. Nausea and vomiting is a very common complication seen with Postinor-2®.
B. Virilization of a female fetus is likely if Postinor-2 ® fails to prevent pregnancy.
C. Menstruation is likely within 7 days of the treatment.
D. Taking the two doses at the same time is associated with lower risk of failure and less adverse effects.
E. Vaginal spotting (breakthrough bleeding) is a common complication seen within 3 days of treatment.

A

D. Taking the two doses at the same time is associated with lower risk of failure and less adverse effects

Postinor-2 ®, used for post-coital contraception, is a progesterone-only method. The package contains 2 tablets of levonorgestrel 750mcgs. Often one tablet is given within 72 hours of unprotected sexual intercourse and the other 12 hours later; however, taking the two pills of the package at once seems to be more effective and associated with less adverse effects. The failure rate is 2-3%.

NOTE - according to latest recommendation, the efficacy of Postinor-2® significantly declines after 72 hours of coitus, it is recommended that it be given even beyond 72 hours and up to 5 days.If pregnancy occurs there will be no risk of virilization of female fetuses.

OPTION A : Nausea and vomiting are not a usual complication of Postinor-2 ®.

OPTION B : Postinor-2 ® does not cause female fetus virilization if pregnancy occurs.

OPTION E : If Postinor-2 ®.is given before days 8 to 10 of the menstrual cycle, breakthrough bleeding within few days of administration is possible, but if it is given in mid-cycle vaginal bleeding would be unlikely and the timing of period will not be affected unless the method fails and the patient becomes pregnant.

138
Q

A 36-year-old woman comes to your office for advice regarding oral contraception. She is currently on Microgynon 30 and is satisfied with it. She has 2 children, does not smoke and enjoys good health. On examination, you find that her blood pressure is 150/100mmHg. Another reading in 20 minutes shows the sameblood pressure. Which one of the following would be the next best step in management?

A. Continue the same OCP, but start her on antihypertensive medications.
B. Stop OCP and give antihypertensive medications.
C. Use an OCP with lower doses of estrogen.
D. Start her on progestogen-only pills (POPs).
E. Continue the same OCP without further action.

A

D. Start her on progestogen-only pills (POPs)

Evidence suggests that combined oral contraceptive pills (COC) have a small adverse effect on blood pressure with an average increase of 8mmHg for systolic and 6mmHg for diastolic pressure. However, a minority of women may suffer severe hypertension induced by COC.

Unlike COCs, progesterone-only pills do not appear to induce an increase in blood pressure; therefore the next best step in management is switching to POPs.

In women with well-controlled and monitored hypertension ,who are 35 years or younger, a trial of combined oral contraceptives may be appropriate as long as the patient is otherwise healthy, shows no signs of end-organ vascular disease, and does not smoke. If blood pressure remains well controlled several months after the trial is started, combination contraceptive may be continued.

139
Q

A 24-year-old woman is at your office for prescription of oral contraceptive pills. She has the history of epilepsy for the past 5 years and is on carbamazepine. She has not had any episode of seizure in the past 6 months. Which one of the following would be the most appropriate advice?

A. Cease carbamazepine.
B. Reduce the dose of carbamazepine.
C. Prescribe high-dose oral contraceptive pills (microgynon50).
D. Prescribe low-dose oral contraceptive pills (microgynon30) and follow up in 3 months.
E. Advise her to use condoms only.

A

C. Prescribe high-dose oral contraceptive pills (microgynon50)

Since this patient has been stable on carbamazepine, cessation of carbamazepine or dose reduction is not wise, but it should be remembered that some antiepileptic drugs such as carbamazepine can induce liver cytochrome enzyme activity and increase the metabolism of the active gradients of oral contraceptive pills (OCPs), progestin-only pills and progesterone implants. Concomitant use of anti-epileptic drugs (AEDs) reduces the efficacy of oral contraceptive pills by 40-50%. This can cause less effective protection against pregnancy and ‘pill failure’.

With low-dose OCPs (microgynon 30), this reduction in efficacy will result in a 6% failure rate. This is why it is recommended that women on antiepileptic drugs take high-dose OCPs (microgynon 50) to minimize the risk of ‘pill failure’. Even by such increase, there is a small risk of unplanned pregnancy, and barrier methods can be considered along with microgynon 50 after discussion with the patient.

Mirena® is often the best option to offer, as it is unaffected by enzyme-inducing drugs with provision of excellent protection, and should be offered as an option unless IUCDs are contraindicated. Depo Provera® is another good choice.

NOTE - COCs, POPs and Implanon® are affected by enzyme-inducing drugs. These drugs have no imapct on Mirena® and Depo Provera®.

The following antiepileptic drugs can induce cytochrome enzymes and cause pill failure:
* Phenobarbital
* Primidone
* Phenytoin
* Carbamazepine
* Oxcarbazepine
* Topiramate

140
Q

A 30-year-old woman presents to the emergency department with lower abdominal pain and vomiting. She is restless and anxious. On examination she is sweaty and disturbed. Brown vaginal discharge is noted on vaginal examination. Her last menstrual period occured six weeks ago. Vital signs include a blood pressure of 105/55 mmHg, heart rate of 102 bpm and respiratory rate of 20 breaths per minute. Which of the following is the best investigation to establish the diagnosis?

A. CT scan of the abdomen.
B. Serum beta hCG.
C. Transvaginal ultrasound.
D. Full blood count.
E. Urine beta hCG.

A

C. Transvaginal ultrasound

This patient has suspected diagnosis of ruptured ectopic pregnancy that has led to pre-shock.

The most accurate test (not the first) for assessment of suspected ectopic pregnancy is transvaginal ultrasound, which has a sensitivity of 69 to 96% and specificity of 85 to 99%. The first test is alwasy urine pregnancy test to establish a pregnancy. However, this test is a screenign test, meaning that a negative result at 6 weeks excludes ectopic pregnancy with certainty, while a positive result prompts more evaluation, the most important and diagnostic of which is a transvaginal ultrasound scan for detection of gestational sac outside the uterine cavity.

OPTION A : CT scan of abdomen is not needed and should be avoided considering possible pregnancy.

OPTION C : Patients with indeterminate ultrasonography findings require further evaluation with quantitative beta HCG levels. Serum beta HCG and laparoscopy are usually used as secondary tests in women with inconclusive findings on ultrasound.

OPTION D : Full blood count helps to determine the degree of blood loss and in guiding fluid resuscitation or blood transfusion. It is not diagnostic.

OPTION E : Urine beta hCG is the very first diagnostic test when ectopic pregnancy is suspected. With most current pregnancy test kits (sensitive for as little as 25 units per litre) urine may reveal results 3-4 days after implantation; by 7 days (the time of expected period) 98% will be positive. With an ectopic pregnancy a urine beta hCG may be falsely negative in more than 50% of women in early pregnancy; however, at 6 weeks it will be positive in as many as 99% of women, and a negative urine test can safely exclude the ectopic pregnancy.

Another very important test to consider without delay in this situation (not mentioned here) is blood group typing and cross matching, as this patient may need blood transfusion.

141
Q

A 27-year-old woman presents to the Emergency Department with vaginal bleeding and lower abdominal pain. She had her last period 6 weeks ago. She has irregular periods, each lasting variably between 4 and 7 days. On examination, her blood pressure is 110/75 mmHg, pulse rate 98 bpm and respiratory rate 18 breaths per minutes. The rest of the exam is inconclusive. Which one of the following is the next best step in management?

A. Abdominal ultrasonography.
B. Transvaginal ultrasonography.
C. Laparoscopy.
D. Urine pregnancy test.
E. Serum beta hCG.

A

Urine pregnancy test

The triad of amenorrhea, vaginal bleeding, and abdominal pain in women of reproductive age is ectopic pregnancy (EP) until proven otherwise. In order to establish EP as a diagnosis, pregnancy should be diagnosed first. Therefore, the next best step would be a simple urine pregnancy test. In early pregnancy, the urine pregnancy test may be falsely negative; however, 99% of women with an EP will have a positive urine pregnancy later.

Other steps depend on the result of the urine beta hCG test. A negative test at 6 weeks excludes ectopic pregnancy with high certainty, but if positive the next step would be an transvaginal ultrasonography:

  • If the gestational sac is seen within the uterus cavity, EP is excluded.
  • If the gestation sac is seen outside the uterine cavity diagnosis of EP is established. Medical (methotrexate) or surgical (laparoscopy or laparotomy) management should be contemplated as the next step.
  • It the ultrasonography is inconclusive, serial serum beta hCG is considered. In an intrauterine pregnancy, beta hCG is expected to almost doubles every 48 hours. A less than predicted rise in serum beta hCG is suggestive of EP.

NOTE - transvaginal ultrasonography often shows the EP if the beta-HCG is above 1500-2000. An abdominal sonography is likely to detect the EP only if beta-HHCG is above 5000-6500.

  • Diagnostic Imaging Pathways - Ectopic Pregnancy (Suspected)
142
Q

During an ultrasonsography scan for suspected renal stones, a 28-year-old woman is found to have two ovarian cysts on the left side. The cysts are 10 cm and 12 cm in diameter. Which one of the following is the the most appropriate management option?

A. Aspiration.
B. Laparoscopy.
C. Laparotomy.
D. Oophorectomy.
E. Conservative management.

A

B. Laparoscopy

Ovarian cysts are frequently noted incidentally during ultrasonography for other reasons.
The following incidentally-found ovarian cysts can be ignored without any furtehr follow-up:
* Asymptomatic simple cysts ≤3cm in women of reproductive age
* Asymptomatic simple cysts ≤1cm in postmenopausal women

Such cysts are almost always of no clinical importance in asymptomatic women and can be safely ignored.

Simple cysts larger than 3 cm in women of reproductive age or larger than 1 cm in postmenopausal women should be described in ultrasound reports.

Although simple cysts of any size are unlikely to be malignant lesions, it is reasonable to perform yearly sonographic follow-up of cysts, including classic-appearing hemorrhagic cysts larger than 5 cm in premenopausal women and larger than 1 cm in postmenopausal women.

Based on new (2010) recommendations from Radiological Society of North America (RSNA) asymptomatic ovarian cysts in premenopausal women are managed as follow:
* Cysts ≤3 cm are normal physiologic findings that do not need follow-up.
* For cysts >3 cm, but ≤5cm repeat ultrasound in 6 to 12 weeks.
* For cysts>5 cm, but smaller than 7cm, repeat ultrasound in 6 to12 weeks and then yearly follow-up with ultrasound.
* As cysts over 7cm may be difficult to be completely assessed with ultrasonography; therefore, further imaging with magnetic resonance (MRI) or surgical evaluation should be considered. Laparoscopy is the method of choice.

This woman has two cysts larger than 7cm in diameter. According to the mentioned guidelines, laparoscopic removal of these cysts (cystectomy) is the most appropriate management option. Laparoscopy can be used for both diagnosis and treatment.

143
Q

A 20-year-old woman presents to your office with secondary amenorrhea. Pregnancy is excluded by urine pregnancy test. You order a pelvic ultrasound as a part of work-up that reports back a cyst of 1.8 x 1.3 x 1 cm. The cyst is thin-walled and fluid- filled. There is no pain in history. On examination, not tenderness is elicited. Which one of the following is the most appropriate next step in management?

A. Laparoscopy for cytology.
B. Repeat the ultrasound in 6 weeks.
C. Oral contraceptive pills.
D. Do nothing.
E. Oophorectomy.

A

D. Do nothing

Ovarian cysts are frequently noted incidentally during ultrasonography for other reasons.
The following incidentally-found ovarian cysts can be ignored without any furtehr follow-up:
* Asymptomatic simple cysts ≤3cm in women of reproductive age
* Asymptomatic simple cysts ≤1cm in postmenopausal women

Such cysts are almost always of no clinical importance in asymptomatic women and can be safely ignored.

Simple cysts larger than 3 cm in women of reproductive age or larger than 1 cm in postmenopausal women should be described in ultrasound reports.

Although simple cysts of any size are unlikely to be malignant lesions, it is reasonable to perform yearly sonographic follow-up of cysts, including classic-appearing hemorrhagic cysts larger than 5 cm in premenopausal women and larger than 1 cm in postmenopausal women.

Based on new (2010) recommendations from Radiological Society of North America (RSNA) asymptomatic ovarian cysts in premenopausal women are managed as follow:
* Cysts ≤3 cm are normal physiologic findings that do not need follow-up.
* For cysts >3 cm, but ≤5cm repeat ultrasound in 6 to 12 weeks.
* For cysts>5 cm, but smaller than 7cm, repeat ultrasound in 6 to12 weeks and then yearly follow-up with ultrasound.
* As cysts over 7cm may be difficult to be completely assessed with ultrasonography; therefore, further imaging with magnetic resonance (MRI) or surgical evaluation should be considered. Laparoscopy is the method of choice.

Since the cyst is less than 3 cm in diameter, no further action or follow-up is required for the cyst. The cause for the presenting symptom (amenorrhea) should be investigated though.

Aspiration of ovarian cysts, either vaginally or laparoscopically, is less effective and associated with a high rate of recurrence.

Oophorectomy is not an appropriate option for women of reproductive age.

144
Q

A 53-year-old woman, who has been on hormone replacement therapy (HRT) for the past two years asks how frequent she should undergo screening for breast cancer. Which one of the following would be the most appropriate response?

A. Annual breast screening is recommended as HRT increases the risk of breast cancer.
B. Mammogram every two years until the age of 70.
C. Breast cancer screening every 6 months.
D. Mammography every 3 months as long as HRT continues.
E. Self breast examination every month and mammography every 6 months.

A

B. Mammogram every two years until the age of 70

Current Australian guidelines recommend that all asymptomatic women aged 50-69 years undergo screening tests for breast cancer every 2 years.

Currently, there is no evidence that more frequent mammograms improve the ability of early detection of breast cancer in women on HRT. These women should have their screening test similar to other women not on HRT. Women who are started on HRT should be encouraged to join Australia Breast Screen Program.

145
Q

A 14-year-old girl presents to your clinic with a dark rash on her axillae and around her neck for the past few months. She also complains of fatigue and lethargy. She has regular periods which started at age 12. On examination, she has a BMI of 32, and blood pressure of 120/80mmHg. She also has abdominal striae. Which one of the following investigations is most likely to establish the diagnosis?

A. 24-hour urinary cortisol.
B. Fasting blood glucose( FBS).
C. SerumTSH.
D. Pelvic ultrasound.
E. Serum ACTH.

A

B. Fasting blood glucose( FBS)

This girl with a BMI of 34 is obviously obese, and the rash on her axillae and around the neck is acanthosis nigricans, an indicator of insulin resistance. Insulin resistance and increased plasma glucose can also cause lethargy and fatigue, which are present in history. An FBS, therefore, is most likely to establish the diagnosis.

OF note, the oral glucose tolerance test is more sensitive than FBS for the detection of impaired glucose tolerance or early type 2 diabetes.

Obesity, lethargy, acanthosis nigricans, irregular menstruation, and striae are features seen in metabolic syndrome and polycystic ovarian syndrome (PCOS). With regular periods PCOS is unlikely but with irregular periods both PCOS and metabolic syndrome could be possible diagnoses.

This girl has regular menstruation. Her blood pressure is also normal. These two, make Cushing’s syndrome less likely; therefore, serum ACTH and urinary or plasma cortisol levels are not likely to be abnormal. The striae can be caused by obesity and overstretching of the abdominal wall.

Fatigue can also be a feature seen in thyroid hormone problems. Although hypothyroidism has fatigue and obesity as its clinical features, the presence of acanthosis nigricans makes this diagnosis less likely; therefore, THS measurement (option C) probably will not add a diagnostic benefit.

Metabolic syndrome is seen in 25% of patients with PCOS. In patients with PCOS, ultrasonography can show multiple ovarian cysts. PCOS is associated with menstruation abnormalities and is the most common cause of anovulatory cycles and infertility in women. With regular periods, this girl is unlikely to have PCOS, and ultrasound (option D) is unlikely to be helpful.

  • UpToDate - The metabolic syndrome, insulin resistance syndrome , or syndrome X
  • UpToDate - Epidemiology and clinical manifestations of Cushing’s syndrome
146
Q

Which one of the following statements is incorrect regarding Down syndrome screening in a 40-year-old pregnant woman?

A. Dating ultrasound together with second trimester serum screening test has detection rate of 75%.
B. Dating ultrasound together with second trimester serum screening test has detection rate of 97%.
C. Without dating ultrasound, second trimester serum screening test has detection rate of 65%.
D. Chorionic villous sampling has a miscarriage risk of 1 in 100.
E. Amniocentesis is associated with 1 in 200 miscarriage risk.

A

B. Dating ultrasound together with second trimester serum screening test has detection rate of 97%

OPTION A : Correct statement: second-trimester measurement of biochemical markers for screening has a lower sensitivity of 75% if a first-trimester dating ultrasound has established an accurate gestational age and quadruple marker screening test (free beta hCG, alpha-fetoprotein, unconjugated estriol and inhibin A level) is used.

OPTION B : Incorrect statement: combinatoin of second trimester screening tests and a first trimester dating ultrasound has a maximum detection rate of 75%, not 97%.

OPTION C : Correct statemen: with a triple marker screening test (free beta hCG, alpha-fetoprotein and unconjugated oestriol) without first-trimester dating ultrasound the sensitivity declines to as low as 65%.

OPTION D and E : Correct satements: chorionic villous sampling and amniocentesis are associated with miscarriage risks of 1 in 100 and 1 in 200, respectively.

NOTE - In the first-trimester, using a combination of ultrasonography for nuchal translucency, pregnancy associated plasma protein A (PAPP-A) and free beta hCG can be as sensitive as 90%; hence the screening tests of choice.

https://www.rcpa.edu.au/getattachment/2ab50a32-563

147
Q

Which one of the following maternal serum screening markers can be used in both first and second trimesters for screening of Down syndrome?

A. Alpha fetoprotein level.
B. Inhibin A level.
C. Unconjugated estriollevel.
D. Pregnancy-associated plasma protein A level.
E. Free beta HCG level.

A

E. Free beta HCG level

For first-trimester Down syndrome screening, maternal serum free beta hCG (along with pregnancy-associated plasma protein A (PAPPA)) is measured, whereas total HCG is measured for second-trimester screening. Free beta hCG can be used instead of total hCG for second-trimester screening. The difference between the detection rates of free beta hCG and total hCG for Down syndrome remains controversial; however, recent studies showed that application of free beta hCG is associated with a 4% increase in detection of Down syndrome (not certain yet).

Alphafetoprotein (option A) , inhibin A and unconjugated estriol levels (option C) are used for screening Down syndrome solely in the second trimester. ‘Pregnancy-associated plasma protein A level (option D) is used as a screening method only appropriate during the first-trimester.

TOPIC REVIEW

Down syndrome (Trisomy 21) is one of the most common genetic birth defects in Australia, and the most common cause of intellectual disability. The proportion of pregnancies involving women over the age of 35 has increased from about 5% to 20% over the past 20 years, making Down syndrome one of the few congenital defects with an increasing prevalence. The previously quoted incidence of 1:750 has now increased to more than 1:500.

Once a woman is over the age of 35, her risk of a Down syndrome in a pregnancy is 1:300. By the age of 43, her risk increases to about 1:30. It is, therefore, necessary to use screening tools for early detection of Down syndrome in utero.

Tools used for assessment, screening and antenatal diagnosis of Down syndrome include:
SCREENING TESTS
At present there are two recommended sets of antenatal screening tests for Down syndrome:

  1. First trimester screening tests for Down syndrome (9-12weeks) :
    * Ultrasonography for nuchal translucency (>0.5mm) PLUS
    * Maternal serum biomarkers:

Maternal Serum Biomarker in Down syndrome pregnancies are
Free beta hCG - increased
Pregnancy-associated plasma protein A - decreased

NOTE - The combined first-trimester screening is the screening test of choice. First-trimester ultrasound of the nuchal translucency (measurement of the translucent area between the fetal skin and the underlying tissue at the nape of the neck) has a sensitivity of 60-80%, and first-trimester serum screening has a sensitivity of 60-65%. In combination, they can detect 90% of Down syndrome pregnancies (90% sensitivity), while maintaining 95% specificity (i.e. only 5% of normal pregnancies will give a positive screen).

  1. Second trimester screening for Down syndrome (15-18weeks):
    Triple marker screening test
    * Free beta hCG - Increased
    * Alpha-fetoprotein (AFP) - Decreased
    * Unconjugated estriol - Decreased
    * Sensitivity - 65-70%

Quadruple marker screening test
* Free beta hCG - Increased
* Alpha-fetoprotein (AFP) - Decreased
* Unconjugated estriol - Decreased
* Inhibin A level - Increased
* Sensitivity - 70-75%

NOTE - provided that a first-trimester dating ultrasound has determined a good estimate of the gestational age, the sensitivity of second-trimester screening testing can be as high as 70% and 75% with triple and quadruple marker screening tests, respectively. Without first-trimester dating ultrasound, the sensitivity of the second-trimester screening tests can be as low as 65%.

DIAGNOSTIC TESTS
* Chorionic villous sampling – performed at 12-14 weeks of pregnancy and carries a 1:100 risk of pregnancy loss 1:100
* Amniocentesis – performed at 15-18 weeks of pregnancy and carries a 1:200 risk pregnancy loss

Consider and discuss these two tests with the patients where:
1. Screening tests whether in first- or second-trimester suggest Down syndrome.
2. The risk of the mother having a Down syndrome exceeds the risk of fetal loss.

*https://www.rcpa.edu.au/getattachment/2ab50a32-563
* http://www.ncbi.nlm.nih.gov/pubmed/7687460

148
Q

A 35-year old primigravida woman presents to your clinic for her first antenatal check-up. She is 8 weeks pregnant, and is concerned that her baby might have chromosomal abnormalities. She asks for antenatal screening tests. Which one of the following results at this time would indicate further assessment for trisomy 21?

A. Decreased pregnancy-associated plasma protein A (PAPP-A).
B. Decreased hCG.
C. Increased alpha-fetoprotein.
D. Increased unconjugated estriol.
E. Decreased inhibin A.

A

A. Decreased pregnancy-associated plasma protein A (PAPP-A)

The presence of decreased pregnancy-associated plasma protein (PAPP-A) in the first-trimester of pregnancy is an indication for further diagnostic testing for Down syndrome. PAPP-A is the only serum marker that can be used in the first timester of pregnancy in combination with free beta-hCG.

Tools used for assessment, screening and antenatal diagnosis of Down syndrome include:
SCREENING TESTS
At present there are two recommended sets of antenatal screening tests for Down syndrome:

  1. First trimester screening tests for Down syndrome (9-12weeks) :
    * Ultrasonography for nuchal translucency (>0.5mm) PLUS
    * Maternal serum biomarkers:

Maternal Serum Biomarker in Down syndrome pregnancies are
Free beta hCG - increased
Pregnancy-associated plasma protein A - decreased

NOTE - The combined first-trimester screening is the screening test of choice. First-trimester ultrasound of the nuchal translucency (measurement of the translucent area between the fetal skin and the underlying tissue at the nape of the neck) has a sensitivity of 60-80%, and first-trimester serum screening has a sensitivity of 60-65%. In combination, they can detect 90% of Down syndrome pregnancies (90% sensitivity), while maintaining 95% specificity (i.e. only 5% of normal pregnancies will give a positive screen).

  1. Second trimester screening for Down syndrome (15-18weeks):
    Triple marker screening test
    * Free beta hCG - Increased
    * Alpha-fetoprotein (AFP) - Decreased
    * Unconjugated estriol - Decreased
    * Sensitivity - 65-70%

Quadruple marker screening test
* Free beta hCG - Increased
* Alpha-fetoprotein (AFP) - Decreased
* Unconjugated estriol - Decreased
* Inhibin A level - Increased
* Sensitivity - 70-75%

NOTE - provided that a first-trimester dating ultrasound has determined a good estimate of the gestational age, the sensitivity of second-trimester screening testing can be as high as 70% and 75% with triple and quadruple marker screening tests, respectively. Without first-trimester dating ultrasound, the sensitivity of the second-trimester screening tests can be as low as 65%.

DIAGNOSTIC TESTS
* Chorionic villous sampling – performed at 12-14 weeks of pregnancy and carries a 1:100 risk of pregnancy loss 1:100
* Amniocentesis – performed at 15-18 weeks of pregnancy and carries a 1:200 risk pregnancy loss

Consider and discuss these two tests with the patients where:
1. Screening tests whether in first- or second-trimester suggest Down syndrome.
2. The risk of the mother having a Down syndrome exceeds the risk of fetal loss.

*https://www.rcpa.edu.au/getattachment/2ab50a32-563

149
Q

A 37-year-old pregnant woman presents to your practice for consultation regarding the chance of her baby having Down syndrome. She has read somewhere that advanced maternal age is a risk factor for Down syndrome. She is concerned and asks you to advise her on ways she can know whether her baby can be affected. She is 8 weeks pregnant. Which one of the following would you offer as both safest and the most accurate disgnostic tool for exclusion of Down syndrome?

A. Chorionic villus sampling at 10 to 12 weeks of gestation.
B. Ultrasonography at 10 to 12 weeks gestation.
C. Amniocentesis at 16 weeks gestation.
D. Triple marker screen test.
E. Cordocentesis at 18 weeks of gestation.

A

C. Amniocentesis at 16 weeks gestation

Karyotyping of the fetal cells is the only definite method to exclude Down syndrome with almost 100% accuracy. For this reason, fetal cells should be obtained through either of the following:

  1. Chorionic Villus sampling (CVS)
    CVS is an outpatient office procedure performed at 10-12 weeks gestation without anesthesia. Under the guide of ultrasound, a catheter is placed directly into the placental tissue without entering the amniotic cavity. Chorionic villi, which are placental precursors, are aspirated and sent for karyotyping. The chromosomes of chorionic villi are almost always identical to those of the fetus.
    The risk of miscarriage associated with CVS is 0.7%, roughly equivalent to 1 in 100-150 depending on the technique and the operator’s skills.
  2. Amniocentesis
    This office-based procedure is performed after 15 and up to 20 weeks gestation without anesthesia. Under the guide of ultrasound, a needle is placed into the amniotic cavity and amniotic fluid is aspirated. The fluid contains desquamating live fetal cells (amniocytes) which can be used for karyotyping. Furthermore, the amniotic fluid itself can be analyzed for alpha-fetoprotein and acetylcholine esterase as screening parameters in neural tube defects (NTD). NTDs are associated with increased alpha-fetoprotein and elevated acetylcholinesterase activity.
    The risk of miscarriage associated with amniocentesis is 0.5% equivalent to 1 in 200 pregnancies.

Percutaneous umbilical blood sample (PUBS) or cordocentesis

Under ultrasound guidance, blood is aspirated from the umbilical vein. The procedure is performed after 20 weeks gestation without anesthesia. The blood sample can be used for karyotyping, ABG, or titration of IgM and IgG antibodies.

The risk of miscarriage with PUBS is 1-2% equivalent to 1 in 50-100 pregnancies.

Although all of the above methods can obtain fetal cells for karyotyping, the method that carries the least risk is amniocentesis.

Triple marker screen test (maternal serum alpha-phetoprotein, estradiol and β-HCG) and ultrasonography for nuchal fold translucency are screening tests and never used as definitive methods of diagnosis.

*https://www.nhmrc.gov.au/_files_nhmrc/file/your_he
*https://www.rcpa.edu.au/getattachment/2ab50a32-563

150
Q

A 35-year-old woman presents for counselling regarding Down syndrome. She is planning to conceive in the next few months and is concerned because she was told by one of her friends that she is at risk of having a baby with Down syndrome due to advanced age. She wants to discuss this issue with you. Which of the following statements is true regarding second-trimester screening tests for Down syndrome?

A. Elevated alpha-phetoprotein in the second trimester suggests Down syndrome.
B. Elevated unconjugated estriol in these second trimester suggests Down syndrome.
C. Decreased free beta hCG in the second trimester suggests Down syndrome.
D. Decreased inhibin A level in these second trimester suggests Down syndrome.
E. Every screening test should be interpreted in conjunction with the gestational age.

A

E. Every screening test should be interpreted in conjunction with the gestational age

  • Triple marker screen test is done in the window of 15 to18 (up to 20) weeks gestation. Since the reference values are gestational age-specific, accurate dating is important.

Biomarkers measured in triple-marker screening test include:
1. Maternal serum alpha-fetoprotein (AFP)
2. Maternal serum free beta hCG
3. Maternal serum unconjugated estriol

AFP is the major serum glycoprotein of embryo. AFP concentration peaks at 12 weeks in the fetus and the amniotic fluid, then rises in the maternal serum until 30 weeks. Fetal structural defects such as neural tube or ventral wall defects cause spillage of more AFP into the amniotic fluid and maternal blood. Other causes of increased AFP include:
* Multiple pregnancies
* Placental bleeding
* Fetal renal disease
* Sacrococcygeal teratoma

Maternal serum AFP (MS-AFP) is reported in ‘multiples of median’ (MoM). The most common causes of increased (>2.5 MoM) or decreased (<0.85 MoM) MS-AFP on screen test is dating errors. Therefore, the next best step, when the values are not within the normal range, would be ultrasonography for accurate dating.

MS-AFP alone is only 20% sensitive in detection of Down syndrome. With triple marker screen, the sensitivity rises to 70% if accurate dating based on first-trimester dating sonography is performed.
* With Down syndrome (trisomy 21), MS-AFP and estriol are decreased, but β-hCG is increased.
* With Edward syndrome (trisomy 18) all three markers are decreased

Quadruple marker screen:

By adding the forth marker, inhibin A, to triple marker screen, there will be an increase of up to 75% in detection of Down syndrome. Inhibin A is increased in Down syndrome.

None of these markers, however, are predictive of Down syndrome unless they are interpreted in conjunction with accurate gestational age. This is why ultrasonography for accurate dating is an indispensible part of marker screen tests.

Once the test is abnormal with reference to normal values of an accurately estimated gestational age, the next step would be amniocentesis for amniotic fluid AFP, acetylcholinesterase activity and karyotyping.

*https://www.rcpa.edu.au/getattachment/2ab50a32-563

151
Q

Katherine, 28 years old, presents at your office with soreness down below at 37 weeks pregnancy. The physical exam is highly suggestive of genital herpes. She has not had such lesions before and this is the first time she is experiencing such problem. You take swabs for PCR and culture with the results confirming the diagnosis of primary herpes simplex infection. Which one of the following would be the most appropriate next step in management?

A. Vaginal delivery.
B. Prophylatic antiviral therapy.
C. Give acyclovir to the neonate after delivery.
D. Topical lignocaine.
E. Antibiotics.

A

B. Prophylatic antiviral therapy

The risk of transmission of herpes simplex virus to the neonate and neonatal infection significantly increases if a pregnant woman develops primary herpes simplex infection after 30 weeks gestation.

Risk factors for intrapartum herpes simplex infection of the baby include:
* Premature labor
* Premature rupture of membrane
* Primary herpes simplex infection near the time of delivery
* Multiple lesions in the genital area

The most appropriate management includes:
* Checking for herpes simplex infection with PCR testing (cervical swab)
* Prophylactic antiviral therapy of the mother from 36th week until delivery
* Cesarean section delivery

For this pregnant woman, antiviral therapy should be started and cesarean section be performed to minimize the risk of vertical transmission.

  • Australian society for infectious diseases – Management of Perinatal Infections (2014)
    *https://www.sahealth.sa.gov.au/wps/wcm/connect/91b
152
Q

Martha, 42 years of age, has developed a rash characteristic of chicken pox today, just 5 days prior to her expected date of delivery. She is 37 weeks pregnant. Which one of the following is the most appropriate management to prevent neonatal chickenpox?

A. Antiviral therapy.
B. Consultation with an infectious diseases expert.
C. Varicella zoster immunoglobulin to the mother before delivery.
D. Varicella zoster immunoglobulin to the baby after delivery.
E. Follow up for early onset sepsis.

A

D. Varicella zoster immunoglobulin to the baby after delivery

Management of neonates exposed to maternal varicella zoster (maternal chickenpox) is according to the following guidelines:

Presentation of the symptoms >7 days before delivery:
* No varicella zoster immunoglobulin (VZIG) required.
* No isolation required.
* Encourage breastfeeding.
* No other interventions required even if baby has chickenpox at or very soon after birth, unless preterm < 28 weeks gestation or low birth weight <1,000 g.

NOTE - Very preterm neonates (≤ 28 weeks gestation) born with chickenpox should receive intravenous aciclovir 20 mg/kg every 8 hours as a slow infusion (in 1-2 hours)

  1. Maternal chickenpox 7 days before to 2 days after birth
    * Give newborn VZIG 200 IU (one vial) intramuscularly (IM) immediately after birth. VZIG should be given as soon as possible within the first 24 hours of birth but may be given up to 72 hours.
    * Discharge term neonates as soon as possible.
    * No isolation required.
    * Encourage breastfeeding.
  2. Maternal chickenpox > 2 to 28 days after birth
    * If neonate < 28 weeks gestation or 1000 g birth weight, give VZIG (preferably within 96 hours but can be given up to 10 days post-maternal rash.
    * Due to the increased risk of severe varicella in newborns of seronegative women (if the mother has no personal history of infection with VZV), give VZIG to neonates exposed to varicella between 2 to 28 days of age.
    * Discharge term neonates as soon as possible.
    * No isolation required.
    * Encourage breastfeeding.

NOTE - Some experts give VZIG 200 IU (one vial) IM when mothers develop chickenpox to term babies who are more than 2 to 28 days of age but there is little evidence to support this.

This woman has developed chickenpox 5 days before delivery; therefore, the neonate should receive ZIG immediately after birth. Although antiviral therapy, preferrably with acyclovir, will benefit the mother if she presents within 24 hours of appearance of the rash, it will not effectively reduce the risk of transmission to the neonate at this stage of pregnancy.

*http://www.sahealth.sa.gov.au/wps/wcm/connect/a69e * https://www.asid.net.au/documents/item/368

153
Q

Which one of the following is an indication of cesarean delivery in a young woman at 37 weeks gestation?

A. Hepatitis C.
B. Hepatitis B.
C. Primary herpes simplex infection with multiple cervical lesions.
D. Recurrent genital herpes infection before 30 weeks.
E. Vaginal Candidiasis.

A

C. Primary herpes simplex infection with multiple cervical lesions

The following are the indications of cesarean section if a pregnant woman has genital herpes:
* There are active lesions in the cervix or vulva at the time of delivery or within the preceding 4 days
* Rupture of membrane for more than four hours in a mother with history of recent genital herpes

Risk of intrapartum and vertical transmission of herpes simplex to the neonate is high if the mother develops the infection after 30 weeks.

Cesarean delivery is not required in pregnant women with hepatitis C, B, or vaginal Candidiasis; however, invasive procedures such as fetal scalp electrodes and fetal scalp blood sampling should be avoided, because they may increase the risk of neonatal infection with hepatitis B and C.

  • Australian society for infectious diseases – Management of Perinatal Infections (2014)
154
Q

A 10-month-old female infant is brought to you for assessment. She was born via an uneventful vaginal delivery. Her mother was found to have positive titers of anti-hepatitis C virus antibodies. Which one of the following would the most appropriate next step in management of this baby?

A. Wait until 18 months and check the baby’s anti HCV antibody titre.
B. Check HCV RNA serology (PCR) now.
C. Refer to specialist for further management.
D. Start the baby on ribavirin and interferon.
E. Reassure that these antibodies are from the mother’s blood and would be cleared with time.

A

A. Wait until 18 months and check the baby’s anti HCV antibody titre

The Royal Australian and New Zealand College of Obstetricians and Gynecologists recommend that all infants of HCV positive mothers should be screened at 12-18 months of age to determine whether they have been infected. The delay allows full clearance of maternal antibody in the infant and avoiding confusing results. Screening should be performed by antibody and, if positive, HCV RNA by PCR.

Since this child is only 10 months, the next best step would be waiting until she reaches 18 months to let the circulating antibodies cleared. If the antibody titer remains positive beyond this time, HCV RNA serology would be the next step.

Positive HCV RNA serology indicates presence of hepatitis C virus and infection of the infant. If that is the case, the neonate should be referred to a specialist for further management.

  • Australian society for infectious diseases – Management of Perinatal Infections (2014) *http://www.sahealth.sa.gov.au/wps/wcm/connect/1a77
155
Q

A 27-year-old pregnant woman presents to your practice in the first trimester of her pregnancy for routine antenatal care. On investigations, she is found to be positive for hepatitis C virus antibody (HCVAb). On discussing the matter with her, she becomse extremely concerned about transmission of the disease to her baby. Which one of the following is correct about the condition?

A. Cesarean section can prevent the transmission.
B. Fetal scalp blood sampling should be avoided.
C. Breastfeeding is contraindicated for her.
D. The baby should be screened for hepatitis C shortly after delivery.
E. Co-existence of HIV has no effect on HCV transmission.

A

B. Fetal scalp blood sampling should be avoided

Hepatitis C infection is usually contracted from needle sharing. Fifty to 70% of intravenous drug users are hepatitis C positive. Needle sharing is the most common mode of hepatitis C transmission in Australia. Other modes of transmission include transfusion of contaminated blood products, and contaminated instruments e.g. tattooing and body piercing.

If a mother is serologically positive for hepatitis C virus, there is a 5% average risk of vertical transmission to the fetus. If there is co-infection with HIV, the transmission rate will increase to as high as 25%. Measures such as fetal scalp electrode and fetal scalp sampling should be avoided, as these measures increase the risk of fetal infection.

Currently, there is no clear evidence suggesting that cesarean delivery or avoiding breast feeding decreases the rate of transmission from mother to baby; however, it is recommended that if the nipples are cracked and there is bleeding, breast milk be expressed and given to the baby.

NOTE - Ribavirin and interferon are used to treat chronic hepatitis in non-pregnant women, but should be avoided in pregnant women as ribavirin is teratogenic.

All infants of HCV positive mothers should be screened at about 18 months of age (not shortly after devlivery ) to determine whether they have been infected. The delay allows full clearance of maternal antibody from the infant and avoiding confusing results. The mother should be screened by means of antibody titer, and if positive, HCV RNA by PCR. The HIV shoud also be checked if not previously done.

One very important precautionary measure is avoiding intrauterine procedures that can lead to skin breach exposure of the fetus to maternal blood (e.g. fetal scalp blood sampling).

  • Australian society for infectious diseases – Management of Perinatal Infections (2014)
    *http://www.sahealth.sa.gov.au/wps/wcm/connect/1a77
156
Q

Which one of the following is definitely not an adverse effect of long-term combined hormone replacement therapy (HRT)?

A. Myocardial infarction.
B. Breast cancer.
C. Deep vein thrombosis.
D. Pulmonary embolism.
E. Bowel cancer.

A

E. Bowel cancer

The following table (from NHMRC) shows the effect of combined HRT and estrogen-only HRT on incidence of different conditions: (see table)

According to the table, HRT does not increase the risk of bowel cancer.

157
Q

A 14-year-old girl presents to your clinic with complaints of significant fatigue and dark rash in her axillae and around her neck that have been present for the past few months. Her menses are regular. Her family history is only notable for type 2 diabetes mellitus in her grandmother. On examination, she has a BMI of 32, and blood pressure of 120/80mmHg. She has also abdominal striae. Which one of the following could be the most likely diagnosis?

A. Cushing’s syndrome.
B. Addison disease.
C. Metabolic syndrome.
D. Polycystic ovarian syndrome (PCOS).
E. Hypothyroidism.

A

C. Metabolic syndrome

Obesity and the dark rash which is characteristic of acanthosis nigricans are suggestive of insulin resistance. The presence of lethargy and fatigue is also suggestive of hyperglycemia. This presentation can be seen in a variety of conditions such as Cushing’s syndrome, metabolic syndrome, and PCOS.

OPTION A : This girl, however, has normal blood pressure, making Cushing’s syndrome less likely. Between 74% to 87% of patients with Cushing’s syndrome are hypertensive. Abdominal striae can be simply due to obesity and abdominal wall overstretch.

OPTION D : Menstrual irregularities or amenorrhea is a characteristic feature seen in PCOS as well as in obesity, especially centripetal obesity. Patients with obesity and metabolic syndrome might or might not have menstrual abnormalities, but in the absence of
such complaint PCOS is excluded, as impaired menstruation is one of the criteria required for PCOS to be the diagnosis.

OPTION E : Hypothyroidism can also present with obesity and fatigue, but acanthosis nigricans is not a feature seen in hypothyroidism.

OPTION B : Addison disease should always be among differential diagnoses in patients with fatigue. Adrenal deficiency results in increased production of ACTH with melanocyte-stimulating hormone (MSH) as a co-product. The elevated MSH level results in increased melanin synthesis and hyperpigmentation. The hyperpigmentation is generalized but is most prominent and conspicuous in light-exposed areas such as the face, neck, and back of the hands, areas exposed to chronic friction or pressure such as elbows, knees, spine, knuckles, waist (belt), and shoulders (brassier straps), and in areas that are normally pigmented, such as the areola, axillae perineum, and umbilicus. The rash only in the axillae and around the neck makes Addison’s disease less likely, but not excluded from differentials. Cardiovascular symptoms of adrenal insufficiency include postural dizziness or syncope. In most patients the blood pressure is low, but some have only postural (orthostatic) hypotension. Normal blood pressure in this patient makes Addison’s disease less likely, but again not impossible.

By exclusion, metabolic syndrome (Mets) could be the most likely diagnosis in this patient.
Mets is a cluster of risk factors including:

  • Excess abdominal weight
  • Lipid abnormalities
  • Hypertension
  • Elevated glucose levels

According to current guidelines in Australia, metabolic syndrome is defined by the presence 3 of the 5 risk factors summarized in the following table (2001 National Cholesterol Education Program/ATP III):
(See table below)

Although none of these criteria are explicitly mentioned in the scenario, metabolic syndrome is the most likely diagnosis by exclusion.

  • UpToDate - The metabolic syndrome, insulin resistance syndrome, or syndrome X
  • UpToDate - Epidemiology and clinical manifestations of Cushing’s syndrome
158
Q

A 27-year-old woman, previously intravenous drug abuser, presents at 8 weeks pregnancy for evaluation. On assessment, she is found to have positive hepatitis C antibody (HCVAb). Which one of the following would be the next best step in management?

A. Termination of the pregnancy.
B. Maternal blood PCR for hepatitis C, perinatally.
C. Screen the infant at 12 months.
D. Reassure her that the risk of vertical transmission is negligible and nothing needs to be done.
E. Screen the infant at 18 months.

A

B. Maternal blood PCR for hepatitis C, perinatally

It is recommended that pregnant women wth a positive hepatitis C virus (HCV) antibody titer, have a PCR test for HCV RNA and liver function tests because the risk of perinatal transmission depends on the presence of HCV RNA. The PCR detects the presence or absence of the virus and the viral load in the blood, and the genotype.

Liver function tests should be performed at the time of checking HCV RNA status. Since concomitant HIV infection increases the risk of transmission, HIV status should also be checked if not already performed.

With a mother serologically positive for hepatitis C, transmission rate to fetus would be 5%, and higher if the mother becomes infected during pregnancy.

Termination of pregnancy is not advised in this situation; rather the patient should be educated about the risk of transmission of hepatitis C to the infant and screening the infant at 12-18 month of age for hepatitis C. If the infant is found to have hepatitis C, specialist care would be required.

  • Australian society for infectious diseases – Management of Perinatal Infections (2014) * http://www.health.gov.au/internet/publications/pub
159
Q

Jane, 25 years old, presents to you at 14 weeks gestation, and is extremely concerned because this morning she developed a rash characteristic of chickenpox after 2 days of low-grade fever and mild malaise. Serology studies show positive anti- varicella IgM. Which one of the following would be the most appropriate course of action in this situation?

A. Antiviral therapy and pelvic ultrasound.
B. Varicella zoster immunoglobulin.
C. Repeat serology in one week.
D. Give antibiotics.
E. No action is needed.

A

A. Antiviral therapy and pelvic ultrasound

Mary has clinical varicella zoster infection (chickenpox), confirmed by a positive titre of IgM against varicella zoster. Pregnant patients with clinical varicella zoster infection should be managed as follows:

Chicken pox without complications:
* Rash≤24 hours – give oral antivirals (aciclovir)
* Rash>24 hours – no treatment is required.

Chicken pox with complications or in immunocompromised patients:
* Intravenous acyclovir

NOTE - chicken pox is considered complicated if:
* There are respiratory symptoms
* There is hemorrhagic rash or bleeding
* New pocks develop >6 days
* There is persistent fever >6 days
* There are neurological symptoms

This patient should also have ultrasound to check if the baby has developed any anomaly.

OPTION B : Varicella zoster immunoglobulin (VZIG) is the treatment of choice if a pregnant woman comes to contact to a case with active varicella zoster. VZIG is not effective once the clinical disease establishes.

OPTION C : Since the serology is positive for varicella zoster, further serological tests would not be necessary.

OPTION D : Varicella zoster is a viral illness and antibiotics have no role in treatment.

OPTION E : Maternal infection can follow after exposure to infectious cases of chicken pox. Doing nothing or reassurance is not a correct action because the mother’s infection carries the risk of fetal infection and resultant congenital anomalies including:
* Skin scarring
* Eye defects
* Limb hypoplasia
* Prematurity and low birth weight
* Cortical atrophy, mental retardation
* Poor sphincter control
* Early death

  • Australian society for infectious diseases – Management of Perinatal Infections (2014)
    *http://www.sahealth.sa.gov.au/wps/wcm/connect/a69e
160
Q

A 30-year-old woman presents to your clinic at 26 weeks gestation. She is worried because 24 hours ago, she came in contact with a child who had chickenpox. Her pre-pregnancy IgG against chickenpox was negative, and she missed her prenatal chickenpox vaccine. On examination, she is asymptomatic. Which one of the following would be the next best step in management?

A. Check varicella zoster virus IgM level.
B. Give varicella vaccine.
C. Check IgG level.
D. Give varicella zoster immunoglobulin (VZIG).
E. No action is needed because she is asymptomatic.

A

D. Give varicella zoster immunoglobulin (VZIG)

In the absence of a postive IgG antibody in prenatal assessment, the patient should be informed of her susceptibility to varicella infection, and warned about risks of exposure to other individuals, who may have acute varicella.

If a susceptible pregnant woman is exposed to someone with chickenpox, serologic assessment for varicella zoster virus (VZV IgG) should be performed and varicella zoster immunoglobulin (VZIG) given as prophylactic treatment if the patient is within the 96 hours of exposure.

If >96 hours, the following patients should still be treated prophylactically with aciclovir (preferred) or valacilovir after exposure:
* If in the second half of pregnancy
* There is a history of an underlying lung disease
* The woman is immunocompromised
* The woman is smoker

The patient has no history of chickenpox or vaccination against VZV, and a prenatal IgG has been negative. For this patient serology is not required. Since she has presented within 96 hours of exposure, the next step in management would be treating her with prophylactic IVIG (VZIG).

OPTION A : Measuring IgM level could be helpful if there was an atypical presentation of varicella zoster infection. This patient is completely asymptomatic at this stage; hence, checking IgM level would be unnecessary.

OPTION B : Chicken pox vaccine is a live attenuated vaccine and contraindicated throughout pregnancy.

OPTION C : This patient has a recent negative IgG level and has had no infection to make her immune. Her IgG is negative and no IgG measurement is required.

OPTION E : This patient needs follow-up for possible infection. Taking no action is incorrect.

  • https://www.asid.net.au/documents/item/368
    *http://www.sahealth.sa.gov.au/wps/wcm/connect/a69e
161
Q

Which one of the following will best minimize the risk of mother-to-child transmission of human immune deficiency virus?

A. Cesarean section.
B. Use of zidovudine.
C. Elective cesarean section at term and giving zidovudine to the neonate.
D. Combined triple antiviral therapy for the neonate.
E. Induction of delivery as early as possible in pregnancy consistent with maintaining good fetal maturity.

A

C. Elective cesarean section at term and giving zidovudine to the neonate

Perinatal HIV transmission rates have been reduced by more than 70 % through:
* Treatment of the mother and baby with antiretroviral treatment (ART)
* Elective cesarean section
* Not breastfeeding

The Pediatric AIDS Clinical Trial Group (PACTG 076) study showed that administration of zidovudine alone, or in combination with other retroviral drugs, to HIV infected women and their newborn, reduces the risk of perinatal transmission by approximately two thirds (from 25.5 % to 8.3 %). When antiretroviral therapy is combined with elective cesarean section, the risk of vertical transmission falls to 2%.

Rates of transmission are increased in case of:
* Advanced maternal illness
* High maternal viral load (due to advanced infection or viral activity)
* Poor maternal immune status e.g. low CD4 count (also known as T cell count)
* Rupture of membranes > 4 hours before birth
* Preterm labor
* Vaginal delivery
* Procedures that may jeopardize the integrity of natural barriers (e.g. fetal scalp electrodes, vigorous suctioning, injections through unwashed skin)
* Breastfeeding

*http://www.sahealth.sa.gov.au/wps/wcm/connect/72e4

162
Q

A 27-year-old woman comes to your clinic at 16 weeks gestation for review of her blood tests, which is positive for cytomegalovirus (CMV) IgM. She is asymptomatic and clinical examination is unremarkable. Which one of the following options is the most appropriate next step in management?

A. Termination of the pregnancy.
B. Antiviral medications during pregnancy.
C. Amniocentesis.
D. Reassurance.
E. Isolation from the family until delivery.

A

C. Amniocentesis

CMV (a DNA virus) is the most common cause of intrauterine infection and the most common viral cause of birth defects. Primary infection of the mother is often asymptomatic but may present as flue-like symptoms. Up to 50% of pregnant women are CMV IgG seropositive.

Vertical transmission from mother to fetus occurs mainly during the viremia of a primary infection. However, since the result of primary infection is predisposition to a residual life-long latency, fetal infection can occur with reactivation as well.
Transplacental fetal infection is 50% with primary infection, regardless of the gestational age but less than 1% with reactivation of a latent infection.

The infected newborn will be symptomatic in 10 % of cases, but of these 10% almost 90% carry a risk of sequelae including:
* Microcephaly
* Ascites
* Hydrops fetalis
* Oligo or polyhydramnios
* Hepatomegaly
* Pseudomeconium ileus
* Hydrocephalus (ventricular dilation)
* Intrauterine growth restriction (IUGR)
* Pleural or pericardial effusions
* Intracranial calcification
* Abdominal calcification
* Hearing loss

Serologic testing for cytomegalovirus is recommended for the following women in pregnancy:
* History suggestive of CMV illness
* Exposure to known CMV infected individual or blood product
* Immunocompromised
* Abnormalities on routine antenatal ultrasound (usually at 18 weeks)

The serology results are interpreted as follows:
1. A patient with positive IgG but negative IgM has a had past exposure

  1. Changing from an IgG-negative to an IgG-positive state (seroconversion) or a significant rise in IgG indicates a recent primary CMV infection
  2. If the patient has a positive IgM with or without a positive IgG, the result is equivocal and the test should be repeated in 2-4 weeks. The reasons for this include:
    * IgM can remain positive for over one year after an acute infection; therefore, presence of CMV IgM is not helpful for timing of the onset of infection.
    * IgM is only positive in only 75-90% of women with acute infection
    * IgM can revert from negative to positive in women with CMV reactivation or reinfection with a different strain

NOTE - Interpretation of CMV IgM results in pregnancy requires specialist opinion.

Fetal diagnosis is best achieved by a combination of fetal ultrasound, amniocentesis + /- fetal serology; however, the definite diagnosis of fetal infection is by amniocentesis and PCR or the amniotic fluid for CMV. It should be born in mind that positive results do not predict any degree of fetal damage.

Since this woman is 16 weeks pregnant, amniocentesis for definite diagnosis is the most appropriate option. Amniocentesis is perfomed in the rather small window of 15-18 (up to 20) weeks. However, repeating the test in 2-4 weeks was the most appropraite option if it was an option.

  • Australian society for infectious diseases – Management of Perinatal Infections (2014)
  • Murtagh’s General Practice – McGraw Hill – 5th Edition – page 1023
    *http://www.sahealth.sa.gov.au/wps/wcm/connect/9317
163
Q

Which one of the following conditions will require intra-partum antibiotics prophylaxis against group B streptococcus (GBS)?

A. Elective cesarean section at 37 weeks gestation in a woman with positive swab culture for GBS.
B. GBS carrier in the previous pregnancy, but negative carrier state in current pregnancy.
C. Threatened preterm labor with intact membranes.
D. GBS carrier state established by low vaginal swab culture results at 34 weeks.
E. Labor at 38 weeks pregnancy with no maternal risk factor for GBS sepsis in the baby.

A

D. GBS carrier state established by low vaginal swab culture results at 34 weeks

In the following situations, prophylaxis against group B streptococcus is NOT required:
1. Elective cesarean section (no labor, no rupture of membrane), irrespective of carriage state or gestational age.

  1. Threatened preterm labor with intact membranes and labor after 37 weeks with no maternal risk factors for GBS sepsis in the baby (threatened preterm labor is different from preterm labor that needs GBS prophylaxis if the woman is a carrier or the carrier state is not known).
  2. Group beta streptococcus carriage in a previous pregnancy, but negative carriage state in current pregnancy, provided that no other indications for prophylaxis against GBS exists.

Of the given options, only ‘group B streptococcus colonization in current pregnancy at 34 weeks is the only indication for intra-partum antibiotic prophylaxis.

TOPIC REVIEW

Group B streptococcus (GBS or Streptococcus agalactiae) is an encapsulated gram-positive coccus that colonises the gastrointestinal and genital tracts of 15 to 40% of pregnant women. Although GBS colonisation usually remains asymptomatic in these women, maternal colonisation is the critical determinant of infection in neonates and young infants (less than 90 days of age), in whom GBS is the most common cause of bacterial infection. Vertical (mother-to-child) transmission primarily occurs when GBS ascends from the vagina to the amniotic fluid after onset of labor or rupture of membranes, but can occur with intact membranes as well.

To reduce the risk of mortality and morbidity from neonatal GBS infections, prophylactic antibiotics are used in certain groups.

Generally, there are two approaches for deciding as to whether intrapartum antibiotic prophylaxis should be given:

A. Culture-based approach :
Women are screened for GBS by taking low vaginal and rectal swabs. All women with who are culture-positive should receive intrapartum prophylactic antibiotics (penicillin; cefazolin if penicillin-allergic)

B. Risk-factor approach :
Women with the following risk factors should receive prophylactic antibiotics:
* Intrapartum fever ≥38’C
* Delivery before 37 weeks of gestation (preterm labour)
* Rupture of membranes ≥18 hours
* Previous delivery of an infant affected by GBS disease
* GBS bacteriuria (≥10 4 cfu/mL) in the current pregnancy

NOTE - according to recent studies, culture-based approach is superior to risk factor-based one, as most woman whose baby developed early complications of GBS sepsis had no risk factors. For this reason it is recommended that all women be screened at 35-37 weeks’ pregnancy and receive prophylactic antibiotics intrapartum.

Latest recommendations suggest that the following receive intrapartum prophylaxis:
* Positive screening culture for GBS from either vagina or rectum in the current pregnancy
* Positive history of birth of an infant with early-onset GBS disease
* GBS bacteriuria during the current pregnancy
* Unknown culture status (culture not performed or result not available) and either of the following:
-Intrapartum fever (≥100.4oF, ≥38oC)
-Preterm labor (<37 weeks of gestation)
-Prolonged rupture of membranes (≥18 hours)
-Intrapartum NAAT (nucleic acid amplification test) e.g. PCR positive for GBS

Intrapartum antibiotic prophylaxis against GBS is NOT recommended for women with:

  • In a previous pregnancy, a positive GBS rectovaginal culture or GBS bacteriuria and none of the indications for prophylaxis listed above
  • Women with a positive GBS culture who undergo elective caesarean delivery (at any gestational age) without labour or rupture of membranes; these patients however should undergo routine vaginal and rectal screening for GBS at 35 to 37 weeks because onset of labor or rupture of membranes may occur before the planned cesarean delivery.
  • Pregnant women with negative GBS cultures at 35 to 37 weeks of gestation, even if they have one or more of the following intrapartum risk factors: intrapartum fever ≥38oC, preterm labor (<37 weeks of gestation), or prolonged rupture of membranes (≥18 hours). However, the use of broad-spectrum intrapartum antibiotics for the treatment (not prophylaxis) is indicated for febrile women in labor if they have clinical evidence of chorioamnionitis.
  • Queensland Clinical Giudelines - Early onset Group B Streptococcal disease
164
Q

Mary, 33 years old, presents to the Maternity Unit for delivery at 38 weeks gestation. She has had all her antenanal visits until now, but there is no record of GBS status in her file. When you discuss the issue with her, you learn that her general practitioner failed to take swabs for culture. Mary has a 3-year-old son, Michael, who suffered an early GBS sepsis shortly after birth, but survived and enjoys good health now. Mary’s current pregnancy has been uneventful until now. Which one of the following is the most appropriate next step in management?

A. Do vaginal swab for GBS now.
B. Give antibiotics during the labor.
C. Give prophylactic antibiotics to the baby immediately after the delivery.
D. Ask her general practitioner why he has not performed the swabs.
E. Refer the matter to the Medical Board.

A

B. Give antibiotics during the labor

Group B Streptococcus (GBS) is found in the rectum, vagina and urinary tract of 10-30% of healthy Australian women. These women are usually asymptomatic. Giving these women prophylactic intravenous penicillin is an important measure to take for reduction of the risk of neonatal infections with GBS and neonatal sepsis.

The following women should be treated for GBS during labour:
* All women with a history of a GBS-related disease - these women should be given intrapartum antibiotics in all their subsequent pregnancies regardless of the swab culture results
* All women with a GBS positive swab or urine culture result in the current pregnancy
* Premature rupture of membranes for more than 18 hours, or when the time is unknown
* Maternal pre- or intra-partum fever of ≥38°C
* Women with unknown status of GBS colonization

Mary have two indications for intrapartum treatment with antibiotics. The first one is the history of a previous baby affected by GBS. The second one is her unknow GBS status in this pregnancy. Either of these conditions makes intrapartum prophylaxis the best management option for her.

OPTION A : With a previous history of GBS infected baby, no swabs are necessary, and prophylactic antibiotics are indicated regardless of the culture results.

OPTION C. : Best prophylaxis is achieved while the baby is in utero. The objective of prophylaxis is suppressing the maternal colonization before the child passes the birth canal, not afterwards.

OPTION D : Asking Mary’s GP why he has not perform the test does not change the management plan.

OPTION E : Referring the case to the medical board is not the appropriate next step prior to taking care of Mary’s GBS status.

*http://www.kemh.health.wa.gov.au/development/manua
* http://www.health.qld.gov.au/qcg/documents/g_gbs5-
* Australian society for infectious diseases – Management of Perinatal Infections (2014)

165
Q

A 24-year-old pregnant woman comes for her first antenatal visit at 10 weeks pregnancy. Which one of the following investigations will change management at this gestational age?

A. HIV screening.
B. Group B streptococcus.
C. Glucose challenge test.
D. Varicella antibodies.
E. Preeclampsia.

A

A. HIV screening

Among the given options, HIV is the only screening test, the result of which may alter the management of the pregnancy at this gestational age.

It is recommended that all pregnant women undergo HIV screening at the first antenatal visit. If the pregnant woman is found to be HIV positive, antiretroviral therapy should be started immediately to decrease the risk of passing the infection to the fetus.

OPTION B : Screening for group B streptococcus colonization in a pregnant woman is routinely performed between 35-37 weeks. Those with positive culture results should receive intrapartum intravenous penicillin for prevention of neonatal sepsis.

OPTION C : The glucose challenge test lacks both sensitivity and specificity and is no longer part of the diagnostic algorithm. Fasting oral glucose tolerance test with 75mg glucose has substituted this test.

OPTION D : Checking for varicella antibodies is recommended in the first trimester antenatal visit if there is no or uncertain history of prior varicella infection. Negativity of anti-varicella antibody does not add any further steps to the management, unless the pregnant woman comes to contact with a case with varicella zoster or develops the clinical disease.

OPTION E : By definition, preeclampsia occurs after 20th week of gestation.

  • Therapeutic Guidelines – Antibiotic; available on: http://tg.org.au
  • Llewellyn – Jones – Fundamentals of Obstetrics and Gynaecology – Elsevier Mosby 9th Edition
  • Australian society for infectious diseases – Management of Perinatal Infections (2014)
166
Q

A 28-year-old pregnant woman presents to your clinic at 32 weeks gestation with vaginal discharge and itching. A low vaginal swab is taken as a part of assessment. The result is postive for group B streptococcus (GBS). No other abnormal finding is reported. Which one of the following would be the appropriate management regarding the positive GBS culture?

A. No treatment is needed before labor.
B. Give trimethoprim for 3 days.
C. Penicillin.
D. Repeat swab in a week and treat if still positive.
E. Check full blood count.

A

A. No treatment is needed before labor

Group B Streptococcus positive vaginal swabs are not indication for treatment at the time of diagnosis, as recolonization may occur; instead, the mother should receive intravenous penicillin during labor for prevention of neonatal sepsis. It is of great importance to advise this woman to report signs of labor, or rupture of the membranes so that antibiotics can be commenced before delivery.

A negative culture is only current for 5 weeks.

NOTE - Positive vaginal or rectal swabs for GBS do not need treatment, but urine cultures positive for GBS should be treated with oral antibiotics. Cultures should be repeated for assessment of response to treatment.

TOPIC REVIEW

Group B streptococcus (GBS or Streptococcus agalactiae) is an encapsulated gram-positive coccus that colonises the gastrointestinal and genital tracts of 15 to 40% of pregnant women. Although GBS colonisation usually remains asymptomatic in these women, maternal colonisation is the critical determinant of infection in neonates and young infants (less than 90 days of age), in whom GBS is the most common cause of bacterial infection. Vertical (mother-to-child) transmission primarily occurs when GBS ascends from the vagina to the amniotic fluid after onset of labor or rupture of membranes, but can occur with intact membranes as well.

To reduce the risk of mortality and morbidity from neonatal GBS infections, prophylactic antibiotics are used in certain groups.

Generally, there are two approaches for deciding as to whether intrapartum antibiotic prophylaxis should be given:

A. Culture-based approach :
Women are screened for GBS by taking low vaginal and rectal swabs. All women with who are culture-positive should receive intrapartum prophylactic antibiotics (penicillin; cefazolin if penicillin-allergic)

B. Risk-factor approach :
Women with the following risk factors should receive prophylactic antibiotics:
* Intrapartum fever ≥38’C
* Delivery before 37 weeks of gestation (preterm labour)
* Rupture of membranes ≥18 hours
* Previous delivery of an infant affected by GBS disease
* GBS bacteriuria (≥10 4 cfu/mL) in the current pregnancy

NOTE - according to recent studies, culture-based approach is superior to risk factor-based one, as most woman whose baby developed early complications of GBS sepsis had no risk factors. For this reason it is recommended that all women be screened at 35-37 weeks’ pregnancy and receive prophylactic antibiotics intrapartum.

Latest recommendations suggest that the following receive intrapartum prophylaxis:
* Positive screening culture for GBS from either vagina or rectum in the current pregnancy
* Positive history of birth of an infant with early-onset GBS disease
* GBS bacteriuria during the current pregnancy
* Unknown culture status (culture not performed or result not available) and either of the following:
-Intrapartum fever (≥100.4oF, ≥38oC)
-Preterm labor (<37 weeks of gestation)
-Prolonged rupture of membranes (≥18 hours)
-Intrapartum NAAT (nucleic acid amplification test) e.g. PCR positive for GBS

Intrapartum antibiotic prophylaxis against GBS is NOT recommended for women with:

  • In a previous pregnancy, a positive GBS rectovaginal culture or GBS bacteriuria and none of the indications for prophylaxis listed above
  • Women with a positive GBS culture who undergo elective caesarean delivery (at any gestational age) without labour or rupture of membranes; these patients however should undergo routine vaginal and rectal screening for GBS at 35 to 37 weeks because onset of labor or rupture of membranes may occur before the planned cesarean delivery.
  • Pregnant women with negative GBS cultures at 35 to 37 weeks of gestation, even if they have one or more of the following intrapartum risk factors: intrapartum fever ≥38oC, preterm labor (<37 weeks of gestation), or prolonged rupture of membranes (≥18 hours). However, the use of broad-spectrum intrapartum antibiotics for the treatment (not prophylaxis) is indicated for febrile women in labor if they have clinical evidence of chorioamnionitis.
  • Queensland Clinical Giudelines - Early onset Group B Streptococcal disease
  • Australian society for infectious diseases – Management of Perinatal Infections (2014)
167
Q

A 25-year-old woman is found to have a urine culture positive for group B streptococcus at 30 weeks gestation. She denies no urinary symptoms such as frequency, dysuria, or urgency, and is otherwise healthy. Which one of the following would be the next best step in management?

A. Prophylactic penicillin during labour.
B. Nitrofurantoine.
C. Cephalexin.
D. Do vaginal swab in a week and then treat if positive.
E. Follow up to see if she develops fever.

A

C. Cephalexin

At the first look and if the question is not read carefully, one may mistake the diagnosis and management of an asymptomatic bacteriuria in a pregnant woman with the notorious group B streptococcus (GBS) colonization.

In cases of coloniation of pregnant women with GBS, the management includes intravenous antibiotics once the labor starts, as an attempt to significantly decrease the chances of early neonatal GBS infection and sepsis with consequent catastrophic outcomes.

In this case, however, this pregnant woman has asymptomatic bacteriuria. and should be treated with appropriate antibiotics once diagnosed. The recommedned antibiotic regimens include:
1. First-line: cephalexin 500mg orally, 12 hourly for 10 days (category A)
2. Second-line: nitrofurantoin 50mg orally, 6 hourly for 10 days (category A)
3. Third-line: amoxicillin + clavulanate 500+125mg orally, 12-hourly for 10 days (category B1)

There is increased resistance to amoxicillin. This drug should only be used if the susceptibility test is proven.

Women with confirmed bacteriuria should have repeat cultures sent at each antenatal visit or monthly to monitor for recurrent bacteriuria. Ongoing prophylaxis for the duration of the pregnancy should be offered to women with persistent bacteriuria.

Group B streptococcus positive vaginal swabs are not treated in the antenatal period, as group B streptococcus can recolonize at any time after treatment.

  • Therapeutic Guidelines – Antibiotic; available on: http://tg.org.au
    *http://www.kemh.health.wa.gov.au/development/manua
  • http://www.health.qld.gov.au/qcg/documents/g_gbs5-
168
Q

A 51-year-old woman comes to your GP clinic to discuss the choice of hormone replacement therapy for ‘hot flushes’ and other menopausal symptoms. Which one of the following statements is correct regarding continuous combined hormone replacement therapy?

A. It increases the risk of breast cancer.
B. It does not increase the risk of breast cancer.
C. It increases the risk of endometrial cancer.
D. It decreases the risk of endometrial cancer.
E. It provides primary prevention from coronary artery disease.

A

A. It increases the risk of breast cancer

According to Australian National Health and Medical Research Council (NHMRC) guidelines, estrogen-only hormone replacement therapy does not increase the risk of breast cancer; however combined hormone replacement does. The risk is directly related to the duration of HRT.

The following table (from NHMRC) shows the effect of combined HRT and oestrogen-only HRT on incidence of different conditions:

169
Q

A 5-year-old boy is brought to the emergency department by ambulance after he had a generalized tonic-clonic seizure at home 20 minutes ago. Upon arrival at the hospital, he is assessed immediately. He is not having a seizure now but is lethargic and confused. His blood pressure is 105/65 mmHg, pulse rate 76 bpm, respiratory rate 16 breaths/min, and temperature 37.°C. His mucous membranes are not dry, skin turgor is normal, and capillary refill time is 2 seconds. The rest of the examination, including cardiovascular, respiratory, and neurological, is completely normal. Pathology results are as follows:
* FBE: Normal
* Random blood sugar: 8.3 mmol/L (4 - 11.1 mmol/L)
* Sodium: 120 mmol/L (135-145 mmol/L)
* Potassium: 4.1 mmol/L (3.5 – 5.5 mmol/L)
* Bicarobonate: 24 mmol/L (22-32 mmol/L)
* Creatinine: 80 μmol/L (60-110 μmol/L)
* Urea: 4.5 mmol/L (2.5-7.1 mmol/L)
* Calcium: 2.5 mmol/L (2.2-2.7 mmol/L)

Which of the following could be the most likely cause of this presentation?

A. Acute renal failure.
B. Addison’s disease.
C. Congestive heart failure.
D. Hyponatremia due to dehydration.
E. SIADH.

A

E. SIADH

Afebrile seizures in children can have a variety of etiologies including space-occupying intracranial lesions, CNS infections (e.g., meningitis, encephalitis), metabolic derangements (e.g., hypoglycemia, hyponatremia, and hypernatremia, hypercalcemia, uremia, epileptic disorders, medications, head trauma, etc.

The abnormal finding in the lab values of this child is serum sodium of 120 mmol/L indicating hyponatremia as the most likely cause of the seizure episode in this child. Both hyponatremia and hypernatremia can present with lethargy, headache, altered mental status, seizure coma, and even death.

Hyponatremia can be caused by a variety of conditions including:
* Medications, especially diuretics, ACH inhibitors, Angiotensin receptor blockers (ARBs), etc.
* Dehydration
* Syndrome of inappropriate ADH (SIADH)
* Congestive heart failure
* Renal failure
* Liver failure
* Hormonal diseases e.g. Addison’s disease, hypothyroidism, etc.

Hyponatremia, depending on the volume state, is classified as hypovolemic hyponatremia, hypervolemic hyponatremia, and euvolemic hyponatremia.

In hypovolemic hyponatremia, volume depletion of any cause such as dehydration results in a surge in the secretion of antidiuretic hormone (ADH). ADH, by retaining water and its dilutional effect results in hyponatremia. Since this child has no clinical findings suggestive of dehydration, hyponatremia due to dehydration (option D) is not an unlikely diagnosis.

Hypervolemic hyponatremia is when the presence of excess total body water dilutes sodium. This kind of hyponatremia can be seen in congestive heart failure (CHF), renal failure, and chronic liver disease.

In renal failure, sodium loss and water retention caused by faulty kidneys result in hyponatremia. This child has normal creatinine and urea, and acute renal failure (option A) is far less likely to be the underlying etiology for hyponatremia and seizure in this child.

CHF, another cause of hypervolemic hyponatremia, is associated with low cardiac output and blood pressure. This activates several neurohormonal pathways to preserve arterial blood volume and pressure. The primary mechanism for such compensation is the increased activity of arginine vasopressin (AVP). AVP increases free-water reabsorption in the renal collecting ducts; therefore, the blood volume is increased and plasma sodium is diluted. In this child, however, the absence of clinical manifestations of CHF in the history and physical examinations makes CHF (option C) to be an unlikely underlying cause of hyponatremia.

In euvolemic hyponatremia, the total amount of water in the body is normal. The most common cause of euvolemic hyponatremia is the syndrome of inappropriate secretion of antidiuretic hormone (SIADH).

Addison’s disease can be another cause of hyponatremia. Addison’s disease presents with an adrenal crisis if it occurs acutely. Chronic symptoms result from cortisol deficiency, aldosterone deficiency, and excess ACTH. Cortisol deficiency manifests with weakness and fatigue, weight loss, anorexia, nausea and vomiting, diarrhea or constipation, flank or abdominal pain, hypoglycemia, and hyperthermia.

Clinical manifestations of aldosterone deficiency include hyponatremia, hyperkalemia, acidosis, tachycardia, and hypotension. Suggestive symptoms are postural hypotension and salt cravings.

With low sodium but normal potassium in this child and the absence of other clinical and laboratory manifestations, Addison disease (option B) is unlikely to cause hyponatremia and consequent seizure.

Given the fact that there is no abnormal laboratory value except the hyponatremia, and by the exclusion of other options as above, SIADH is the most likely diagnosis in this child. Excess ADH in SIADH results in increased retained water in the body. With excess water, sodium is diluted and hyponatremia occurs. Hyponatremia is the most common clinical presentation in patients with SIADH.

Any of the following can cause SIADH:
* CNS pathologies - Guillain-Barre syndrome, subarachnoid hemorrhage, head trauma, meningitis, encephalitis
* Lung pathologies - TB, sarcoidosis, pneumonia, pneumothorax, atelectasis, asthma
* Malignancies - Small cell lung cancer, nasopharyngeal cancers, mesotheliomas, gastrointestinal cancers, pancreatic cancer, lymphoma, sarcoma
* Medications - Antidepressants, antipsychotics, anticonvulsants, anti-inflammatory drugs, ecstasy, cyclophosphamide
* Other - prolactinoma, Waldenstrom macroglobulinemia

The mainstay of treatment in SIADH is the treatment of symptomatic patients with water restriction with or without sodium replacement, depending on the severity of symptoms, while a meticulous investigation is carried out for an underlying etiology. Although this child does not have overt symptoms or clinical findings indicating the presence of any of the above conditions, for now, there might be an occult pathology such as an infection or malignancy.

  • RACGP – AFP – The suspect – SIADH
  • RACGP – AFP – Addison disease
  • RCH – Hyponatraemia
  • Medscape - Hyponatremia in Emergency Medicine
170
Q

Which one of the following risk factors is not associated with increased risk of group-B streptococcus (GBS) neonatal sepsis?

A. Preterm labor.
B. GBS bacteriuria in current pregnancy.
C. Previous baby with GBS-related early onset sepsis.
D. Rupture of membranes of 10 hours duration.
E. Maternal temperature greater than or equal to 38°C within 24 hours of giving birth.

A

D. Rupture of membranes of 10 hours duration

Risk factors for early neonatal GBS sepsis include:

  • Maternal fever more than or equal to 38°C either intrapartum or within 24 hours of giving birth
  • Group-B streptococcus bacteriuria and colonization in current pregnancy
  • Rupture of membranes prior to birth for more than 18 hours
  • Preterm labor (less than 37 weeks)
  • Previous GBS related early onset sepsis

Rupture of membranes of 10 hours duration is not associated with increased risk of group B streptococcus neonatal sepsis.

*http://www.kemh.health.wa.gov.au/development/manua
* http://www.health.qld.gov.au/qcg/documents/g_gbs5-
* Australian society for infectious diseases – Management of Perinatal Infections (2014)

171
Q

A newly married24-year-old woman is found to have multiple fibroids, palpable up to the umbilicus. She has been trying to get pregnant, but every attempt has failed so far. Laboratory investigations show hemoglobin of 82 g/L. Which one of the following would be the most appropriate management?

A. Open myomectomy after correcting her anemia with blood transfusion.
B. GnRH analogues for 3 months followed by myomectomy.
C. OCP for 3 months followed by myomectomy.
D. Immediate myomectomy.
E. Hysterectomy.

A

B. GnRH analogues for 3 months followed by myomectomy

Gonadotropin-releasing hormone (GnRH) agonists are the most effective medical therapy for uterine myomas. These drugs work by increasing the release of gonadotropins, followed by desensitization and down-regulation to a hypogonadotropic hypogonadal state that clinically resembles menopause.

Most women will develop amenorrhea, improvement in anemia, and a significant reduction (35 to 60%) in uterine size within 3 months of initiating the therapy (most of size reduction occurs in the first month of use).

Surgical removal of myomas then follows. The only surgical procedure that preserves fertility and effectively ameliorates fibroid-related symptoms is myomectomy.

For women with poor prior infertility, in whom the only finding is a distorted uterine cavity from the presence of one or more uterine fibroids (most commonly submucous fibroids), myomectomy can promote fertility and successful pregnancy outcome.

Combined hormonal contraceptives and progesterone are commonly prescribed to regulate abnormal uterine bleeding, but appear to have limited efficacy in the treatment of uterine leiomyomas.

Hysterectomy would be a wrong option for this woman and almost every woman of reproductive age (unless they are not willing to have more children).

  • AAFP - Uterine Fibroid Tumors: Diagnosis and Treatment
  • An Evidence-based Guideline for the Management of Uterine Fibroids
172
Q

A 27-year-old pregnant lady at 13 weeks’ gestation presents to your clinic after she came to contact with a child with parvovirus infection four days ago. She is concerned that her baby may become affected. You order serum IgM and IgG antibody levels for parvovirus, both of which are negative. Which one of the following would be the most appropriate next step in management?

A. Repeat serologic tests in two weeks.
B. Ultrasonography.
C. Reassure her as the antibody titres are negative.
D. Arrange for intrauterine blood transfusion.
E. Arrange for fetal umbilical cord blood sampling.

A

A. Repeat serologic tests in two weeks

Parvovirus B19 is a single-stranded DNA virus. It is the causative virus for erythema infectiosum, also known as fifth disease or slapped cheek syndrome.

Maternal infection with parvovirus B19 is associated with the risk of transplacental fetal infection throughout the pregnancy. Fetal infection may result in fetal parvovirus syndrome. The syndrome is characterized by anemia-hydrops with cardiac failure and possibly death.

The earlier the exposure occurs, the more likely the development of fetal parvovirus syndrome would be. With third trimester infection still births are common.

Women who have been exposed to parvovirus in early pregnancy should be informed of the possible risks to the fetus and offered serology for parvovirus B19 specific IgG:
* If parvovirus specific IgG is positive reassure that pregnancy is not at risk
* If parvovirus specific IgG is negative, serology for IgM should be performed.

After infection with parvovirus, IgM is expected to become positive in 1 to 3 weeks and remain high until 8-12 weeks. IgG levels start to rise. IgG levels start to rise 2 to 4 weeks after the infection.

This woman has a negative IgG titer indicating that she is not immune to the infection. Although her IgM titer is negative, this does not exclude infection because it takes approximately 1 to 3 weeks for IgM to become positive after infection. IgM will remain high for 8 to 12 weeks. In such cases, it is recommended that serologic tests be repeated in 2 weeks when the IgM may become positive while IgG starts to rise.

Positive IgM titers confirm maternal parvovirus infection. If that is the case, the next step would be fetal monitoring with ultrasound for development of hydrops at 1-2 weeks intervals for the next 6-12 weeks (needs referral). Once the fetus is found to have hydrops, fetal umbilical cord sampling and intrauterine blood transfusion are considered.

Positive IgG and negative IgM indicates maternal immunity to parvovirus.

The following table summarizes interpretation of serologic tests results and recommended further actions:

*http://www.sahealth.sa.gov.au/wps/wcm/connect/04ab

173
Q

Which one of the following is the most common symptom of endometriosis?

A. Dysmenorrhea.
B. Dysuria.
C. Infertility.
D. Deep dyspareunia.
E. Painful abdominal bloating.

A

A. Dysmenorrhea

Endometriosis can presents with the following features:
* Dysmenorrhea (moderate to severe) – 60 to 80% of cases
* Chronic pelvic pain – 40-50% of cases
* Dysuria and hematuria – 5% of cases
* Infertility – 30 to 50% of cases
* Deep dyspareunia – 40 to 50% of cases
* Dyschezia, tenesmus, and painful abdominal bloating - 10 to 40% of cases
* Premenstrual spotting lasting 1-2 days
* Heavy menstrual bleeding – 10-20%
* Dysmenorrhea has been by far the most common symptoms in patients with endometriosis.

  • Australian Prescriber - Medical management of endometriosis
174
Q

Which one of the following investigations can help in diagnosing endometrioses earliest?

A. Transvaginal ultrasound.
B. Pelvic examination.
C. Laparoscopy.
D. Hysteroscopy.
E. Colposcopy.

A

C. Laparoscopy

The definite diagnosis of endometriosis can be made by laparoscopy. This method can detect endometriosis at the earliest stages.

OPTION A : Ultrasonography can only visualize endometriomas (tumors of endometrial tissue), not endometriosis.

OPTION B : Pelvic examination does not often help with the diagnosis.

OPTION D and E : Hysteroscopy and colposcopy are not helpful in diagnosing endometriosis, as the former can just visualize inside the uterine cavity and the latter only the vagina and the cervix.

  • Medscape - Endomteriosis
175
Q

Which one of the following statements is incorrect regarding endometriosis?

A. Laparoscopy is the best diagnostic test.
B. It is the commonest cause of chronic pelvic pain in women in most developed countries.
C. The usual delay in diagnosis is 8 to 10 months with onset in adolescence.
D. Medical treatment for endometriosis usually does not improve fertility.
E. Excisional surgery is the treatment of choice for infertility caused by endometriosis.

A

C. The usual delay in diagnosis is 8 to 10 months with onset in adolescence

Option A is correct - Endometriosis is a highly variable condition and diagnosis can be difficult. Confirmation of diagnosis requires laparoscopy in most situations.

Option B is correct - Endometriosis is the most common cause of chronic pelvic pain in most developed countries.

Option C is incorrect - usual delay in diagnosis is about 8 to 10 years and not 8 to 10 months.

Option D is correct - There is strong evidence that medical treatment only relieves symptoms and does not improve fecundity.

Option E is correct - Excisional surgery for endometriosis is usually the treatment of choice as it improves the symptoms, confirms the diagnosis and improves the quality of life in 60-80% of patients.

  • Medscape - Endomteriosis
176
Q

A 24-year-old Aboriginal woman presents to the antenatal clinic at 19 weeks gestation. This is the best time for which one of the following?

A. Ultrasonography.
B. Maternal serum screening for Down syndrome.
C. Amniocentesis.
D. Chorionic villous sampling.
E. Rubella screening.

A

A. Ultrasonography

Ultrasound for identification of physical abnormalities, including neural tube defects is best performed between 18- 20 weeks of gestation, and should be considered in this visit.

OPTION B : Maternal serum screening for Down syndrome is best performed between 15-18 weeks of gestation. The screening tests include alpha fetoprotein, oestriol, and free beta hCG in the maternal blood (triple screening test). Accurate date of gestation is needed for reliable interpretation of the results. This patient is young and the risk of having a baby with Down sydrome is not significant. Even otehrwise, sonography can be used for assessment of nuchal translucency as well as other fetal parameters.

OPTION C : Amniocentesis is best performed between 15-18 weeks of gestation and carries a risk of miscarriage of 1 in 200. It is a very accurate measure for diagnosing chromosomal anomalies including Down syndrome.

OPTION D : Chorionic villous sampling is best performed between 10-12 weeks of gestation. The procedure carries a miscarriage risk of 1 in 100. It is also accurate for diagnosis of chromosomal anomalies.

OPTION E : Screening for rubella is best performed before conception and not during the pregnancy.

  • http://www.racgp.org.au/your-practice/guidelines/n
  • http://www.babycenter.com.au/a557390/18—20-week-
177
Q

A 31-year-old woman G2P1 presents at 10 weeks’ gestation for antenatal visit. She is found to have a twin pregnancy. Her previous pregnancy was complicated with placental abruption at 34 weeks. Which one of the following is the next best step in management in addition to routine antenatal care?

A. Vitamin supplementation.
B. Cervical suture.
C. Increased iron and folic acid supplementation.
D. Hospital admission after 34 weeks.
E. Serial CTGs after 34 weeks.

A

C. Increased iron and folic acid supplementation

This woman has a high-risk pregnancy due to a multiple pregnancy and the history of placental abruption in the previous pregnancy, each of which alone is significant enough to warrant extra care of the patient. Risks and care associated with and needed for easc condition is as follows:

Placental abruption: A pregnancy following a placental abruption (PA) must be considered a high risk pregnancy, not only in terms of increased risk of recurrence, but also due to excess risk of small for gestational age, preterm labor and birth and pregnancy-induced hypertension (PIH), irrespective of recurrence of placental abruption. Therefore, all pregnancies following a pregnancy complicated by placental abruption should be offered close antenatal surveillance. A significant risk factor for PA is abruption in a previous pregnancy. It is estimated that a woman with a previous PA has a 5-10% chance of recurrent PA in subsequent pregnancies. With two or more PAs, the risk increases to 19-25%.

It is recommended that once a woman had a PA, close surveillance be performed in the subsequent pregnancies starting from 2-3 weeks (up to 3 months) before the gestational age the PA occurred. There are limited measures to prevent subsequent PAs to develop except intervention in modifiable risk factors (e.g. hypertension, smoking, amphetamine and cocaine).

Multiple pregnancies: multiple pregnancy is associated with several potential maternal and fetal complications. The summary of recommendations for multiple pregnancies is reviewed in the following table:

(PICTURE OF TABLE IN PAGE 1096-1097)

For this woman several measures should be contemplated, starting with increasing the dose of folate and iron supplementation as the best initial step at this time. Multifetal pregnancies are associated with increased risk of iron and folate deficiencies. Iron deficiency anemia is associated with increased risk of preterm delivery and low ferritin level with prematurity. Folate deficiency may result in neural tube defect and anaemia (megaloblastic).

Ultrasonographic assessment is recommended to start at 12-13 weeks gestation where it can demonstrate whether the twins are monochorionic or dichorionic. If initial ultrasonography reveals a dichorionic twin pregnancy serial sonographies should be carried out at 20, 28, 32 and 36 weeks. For monochorionic twin pregnancies the time of further subsequent sonographies will be at 16,19,22,26,30,33 and 36 weeks.

CTG should be performed after 34 weeks on a weekly basis. CTG and serial sonography, as mentioned above, provide the best method for monitoring discordant growth, with evaluation of fetal well-being by use of CTG monitoring, biophysical profiles, and Doppler studies.

Previously it was suggested that patients such as this woman be admitted around the time the previous PA occurred, but currently there is not enough evidence to support a policy of routine hospitalization for bed rest in multiple pregnancy. No reduction in the risk of preterm birth or perinatal death is evident, although there is a suggestion that fetal growth is improved. In women with just multiple pregnancy and no other complications, this measure is not only useful, but it may result in increased risk of very preterm birth. Until furthe evidence is available to the contrary, the policy cannot be recommended for routine clinical practice.

  • http://www.ncbi.nlm.nih.gov/pubmed/11279677
  • http://apps.who.int/rhl/pregnancy_childbirth/anten
  • https://www.rcog.org.uk/en/guidelines-research-ser
  • https://www.rcog.org.uk/globalassets/documents/gui
    *http://www.kemh.health.wa.gov.au/development/manua * http://www.ncbi.nlm.nih.gov/pubmed/11279677
  • http://apps.who.int/rhl/pregnancy_childbirth/anten
  • http://3centres.com.au/guidelines/complications-in
178
Q

A 29-year-old woman is planning to become pregnant. She is extremely concerned about rubella infection during pregnancy and its impacts on her baby because 2 years ago her elder sister had to undrgo an abortion after she was diagnosed with rubella infection. She remembers that she was vaccinated for measles, mumps and rubella when she was a child. Which one of the following statements is correct regarding rubella infection?

A. A history of immunization for rubella is enough and no further rubella testing is needed.
B. Rubella vaccine can be given safely during pregnancy.
C. In Australia, 25% of women aged 15-40 years are immune to rubella.
D. Serology testing of rubella antibodies titer should be ordered now.
E. Still birth following infection of the mother in the second trimester is common.

A

D. Serology testing of rubella antibodies titer should be ordered now

Every woman who is planning for pregnancy should be tested for rubella serology (IgG only), regardless of her previous immunization status. The serology results should be documented.

OPTION B : Rubella vaccine is a live attenuated vaccine and is contraindicated during pregnancy, as there is theoretical risk of harm to the developing fetus; however, rubella vaccine has not been proven to cause problems for the fetus so far.

OPTIOIN C : In Australia, 95% of women aged 15-40 years of age are rubella IgG seropositive and immune to rubella.

OPTION E : If the fetus is infected during the second trimester, still birth will be an extremely rare event.

TOPIC REVIEW

Rubella, also called German measles, is usually a mild infectious disease in children and adults. It is clinically difficult to diagnose due to transient clinical features that are also common to some other viral infections.
Rubella has an incubation period of 14-23 days. The infectivity period starts from one week before until 4 days after onset of rash.

Rubella is asymptomatic in 25 to 50 % of cases. In some cases prodromal symptoms may be evident, such as:
* Low grade fever
* Transient erythematous rash
* Lymphadenopathy involving post-auricular and sub-occipital nodes
* Occasionally arthritis and arthralgia (commonly observed in women of child-bearing age)
* Rarely neurological disorders and thrombocytopenia
* Rash - the rash characteristically begins on the face and spreads to the trunk and extremities. It will usually resolve within three days in the same order in which it appeared (face first and then body)

During pregnancy, maternal viremia may occur 5 to 7 days after exposure with spread of the virus throughout the body as well as transplacental infection of the fetus.

Vertical transmission (transplacental) from mother to fetus can only occur during viremia of a primary infection. The effect of maternal infection on the fetus depends on the gestational age:
* <8 weeks: up to 85% of fetuses will be infected and all will have clinical manifestations of the congenital rubella syndrome.
* 8≤ but <12 weeks: 50-80% of fetuses will be infected and of those 65-85% will be clinically affected.
* 13-16 weeks: 30% of the fetuses will be infected and of those, 1/3 will have sensorineural deafness.
* 16-19 weeks: 10% of the fetuses will be infected. Clinical features are rare, yet deafness can be a possibility.
* >19 weeks: there is no apparent risk.

NOTE - reinfection is associated with far less risk of fetal infection of approximately 5%.

Abnormalities associated with congenital rubella syndrome include:
* Central nervous system dysfunction (10-25%, intellectual impairment, developmental delay, microcephaly)
* Eye abnormalities (10-25%, cataracts, retinopathy, glaucoma, strabismus, micropthalmos)
* Sensorineural deafness (60-75%)
* Cardiac abnormalities (10-20%, PDA, PA stenosis)
* Intrauterine growth restriction, short stature
* Inflammatory lesions of the brain, liver, lungs and bone marrow

All pregnant women, who have contact with rubella or clinical features consistent with rubella – like illness should be screened for the presence of rising antibody titre (IgG) and rubella specific IgM, regardless of previous vaccination history or infection. Interpretation of the results and corresponding action is as follows:
(See photo below)

If maternal infection is confirmed, antenatal testing is recommended at least 6 weeks after known maternal infection and is best performed after the 20th week of gestation. Rubella PCR, rubella culture and fetal IgM can be performed following chorionic villus sampling (CVS) / amniocentesis or cordocentesis; however, due to the very high risk of fetal infection and consequent fetal anomalies, if maternal infection occurs in the first trimester, termination of pregnancy should be recommended.

NOTE - In all pregnant women, serologic testing for rubella (IgG only) should be offered and performed in the first antenatal visit. With titers ≥10 IU/ml, risk of reinfection is minimal. However, if the titers are ≤15, vaccination after delivery should be performed.

*https://www.sahealth.sa.gov.au/wps/wcm/connect/d81 * https://www.asid.net.au/documents/item/368

179
Q

A 20-year-old pregnant woman presents to your practice at 10 weeks pregnancy. She is worried because 3 days ago she came to contact with one of her friends from work, who had rubella. She has never been vaccinated against rubella, nor contracted the disease. On examination, she is asymptomatic with no signs or symptoms. Which one of the following is the next best step in management?

A. Check serum IgM and IgG titres for rubella.
B. Give intravenous immunoglobulin.
C. Give MMR Vaccine.
D. Give only rubella vaccine.
E. Terminate the pregnancy.

A

A. Check serum IgM and IgG titres for rubella

All pregnant women with suspected rubella or exposure to rubella should be serologically tested (IgM/IgG), irrespective of a history of previous vaccination, clinical rubella or a previous positive rubella antibody result.

In Australia, over 90% of women are immune against rubella. This can be due to either previous vaccination or infection. In some cases, infection occurs without clinical significance; this means that even though this woman denies previous rubella infection, she might have been infected without even knowing, therefore immune to rubella. This would be evident by an IgG titer of equal to or greater than 10 IU/ml. On the other hand, a positive IgM titer maskes a recent acute infection likely. A positive IgM demands the test to be repeated.

After exposure of a pregnant woman to a case with rubella infection, the next best step is always rubella serology testing regardless of previous history of immunization, infection or previous IgG titres. One exception is an IgG level of equal to or greater than 10 IU/ml documented in current pregnancy. In such case, serologic studies can be skipped and the patient be reassured.

Rubella infection during pregnancy is potentially teratogenic with the risk of fetal infection being greatest if maternal infection occurs during the first trimester. Fetal damage occurs in up to 85% cases of maternal rubella infection in the first 8 weeks and 50-80% if infection occurs within the third month of pregnancy.

Risk of fetal damage declines to about 10 to 20% by 16 weeks and would be rather rare beyond this time.

Vaccination against rubella (option C and D) is contraindicated during pregnancy and there is no single rubella vaccine availabel. The value of immunoglobulin is controversial. Therapeutic abortion is generally recommended after proven infection during the first trimester. The doctor must counsel the patient regarding the risks associated with congenital rubella syndrome, but if the patient, by taking the risks, prefers to continue the pregnancy, her wish should be followed.

TOPIC REVIEW

Rubella, also called German measles, is usually a mild infectious disease in children and adults. It is clinically difficult to diagnose due to transient clinical features that are also common to some other viral infections.
Rubella has an incubation period of 14-23 days. The infectivity period starts from one week before until 4 days after onset of rash.

Rubella is asymptomatic in 25 to 50 % of cases. In some cases prodromal symptoms may be evident, such as:
* Low grade fever
* Transient erythematous rash
* Lymphadenopathy involving post-auricular and sub-occipital nodes
* Occasionally arthritis and arthralgia (commonly observed in women of child-bearing age)
* Rarely neurological disorders and thrombocytopenia
* Rash - the rash characteristically begins on the face and spreads to the trunk and extremities. It will usually resolve within three days in the same order in which it appeared (face first and then body)

During pregnancy, maternal viremia may occur 5 to 7 days after exposure with spread of the virus throughout the body as well as transplacental infection of the fetus.

Vertical transmission (transplacental) from mother to fetus can only occur during viremia of a primary infection. The effect of maternal infection on the fetus depends on the gestational age:
* <8 weeks: up to 85% of fetuses will be infected and all will have clinical manifestations of the congenital rubella syndrome.
* 8≤ but <12 weeks: 50-80% of fetuses will be infected and of those 65-85% will be clinically affected.
* 13-16 weeks: 30% of the fetuses will be infected and of those, 1/3 will have sensorineural deafness.
* 16-19 weeks: 10% of the fetuses will be infected. Clinical features are rare, yet deafness can be a possibility.
* >19 weeks: there is no apparent risk.

NOTE - reinfection is associated with far less risk of fetal infection of approximately 5%.

Abnormalities associated with congenital rubella syndrome include:
* Central nervous system dysfunction (10-25%, intellectual impairment, developmental delay, microcephaly)
* Eye abnormalities (10-25%, cataracts, retinopathy, glaucoma, strabismus, micropthalmos)
* Sensorineural deafness (60-75%)
* Cardiac abnormalities (10-20%, PDA, PA stenosis)
* Intrauterine growth restriction, short stature
* Inflammatory lesions of the brain, liver, lungs and bone marrow

All pregnant women, who have contact with rubella or clinical features consistent with rubella – like illness should be screened for the presence of rising antibody titre (IgG) and rubella specific IgM, regardless of previous vaccination history or infection. Interpretation of the results and corresponding action is as follows: See photo below

If maternal infection is confirmed, antenatal testing is recommended at least 6 weeks after known maternal infection and is best performed after the 20th week of gestation. Rubella PCR, rubella culture and fetal IgM can be performed following chorionic villus sampling (CVS) / amniocentesis or cordocentesis; however, due to the very high risk of fetal infection and consequent fetal anomalies, if maternal infection occurs in the first trimester, termination of pregnancy should be recommended.

NOTE - In all pregnant women, serologic testing for rubella (IgG only) should be offered and performed in the first antenatal visit. With titers ≥10 IU/ml, risk of reinfection is minimal. However, if the titers are ≤15, vaccination after delivery should be performed.

*https://www.sahealth.sa.gov.au/wps/wcm/connect/d81 * https://www.asid.net.au/documents/item/368

180
Q

Which one of the following statements is incorrect regarding the use of combined hormone replacemet therapy (HRT) with estrogen and progesterone in a 55-year-old postmenopausal woman?

A. If used longer than 5 years, it significantly increases the risk of breast cancer.
B. It is not recommended for treating postmenopausal women presenting with osteoporosis, who are older than 60 to 65 years and where the management of osteoporosis is the only concern.
C. It should be avoided inpatient with history of deep vein thrombosis.
D. Combination with alendronate is usually discouraged.
E. In women who are more than three years post menopausal, raloxifene is effective in reducing vertebral fracture risk.

A

A. If used longer than 5 years, it significantly increases the risk of breast cancer

Studies suggest that combined HRT is associated with a slight increase in the incidence of breast cancer if used more than five years. However, the risk is not significant unless there is personal history of breast cancer. The benefits of HRT definitely outweigh the slightly increased risk of breast cancer.

OPTION B : Vasomotor symptoms, such as hot flushes are the only indications for commencement of hormone replacement therapy (HRT). Although HRT decreases mineral bone loss, osteoporosis is not an indication to start HRT. HRT prevents osteoporosis, but do not treat it.

OPTION C : Estrogen should be avoided in women at risk of venous thromboembolism.

OPTION D : Combination therapy with estrogen and alendronate should generally be avoided unless there is specialist guidance.

OPTION E : Raloxifene is an estrogen receptor modulator. It has an agonist effect on bone, but not the endometrium. Raloxifene is effective in reducing vertebral fracture risk but not the non-vertebral fractures. Raloxifene is not the first-line treatment option for osteoporosis even in the presence of personal history of breast cancer.

*http://www.ncbi.nlm.nih.gov/pmc/articles/PMC254436 * https://www.nhmrc.gov.au/_files_nhmrc/publications

181
Q

Zara is a recent 28-year-old immigrant from Sudan, who is being assessed for several medical issues including fatigue, insomnia, forgetfulness, and irritability. Her older sister accompanies her and says they have recently lost two family members due to the conditions in their county and believes Zahra could be depressed. She is vegetarian and back at home her diet mostly consisted of corn and wheat. On examination, she has desquamated plaques on an erythematous skin base, especially on the back of her hands, around her eyes, and on her neck. Which one of the following could be the most likely diagnosis?

A. Vitamin B12 deficiency.
B. Niacin deficiency.
C. Vitamin D deficiency.
D. Zinc deficiency.
E. Vitamin E deficiency.

A

B. Niacin deficiency

Zara’s background (an immigrant from Sudan), her diet, and the constellation of symptoms are highly suggestive of pellagra. Pellagra, a systemic disease, results from niacin (nicotinic acid or vitamin B3) deficiency.

Primary pellagra due to a deficient diet is common, particularly in developing countries where corn is a major food source or following prolonged disasters including famine or war. Although corn is rich in niacin, it is in a bound form unable to be accessed unless treated with alkaline solutions, and the proteins in corn are low in tryptophan. Pellagra remains endemic in India, China, and some African countries. In developed countries, pellagra is rare because many foods, such as bread and cereal, are fortified with niacin. However, it may still result from chronic alcohol abuse, anorexia nervosa, or self-imposed diets. Alcoholism is the most common cause of pellagra in developed countries.

Secondary pellagra develops despite an adequate diet due to:
* Malabsorption — inflammatory bowel disease, chronic diarrhea, Whipple disease, and gastrointestinal surgery including gastric bypass surgery
* Drug-induced e.g., azathioprine, chloramphenicol, 5-fluorouracil (5FU), isoniazid, ethionamide, carbamazepine, leucine.
* Metabolic steal — carcinoid syndrome steals tryptophan from the niacin pathway to make serotonin.

A form of pellagra is seen in Hartnup disease (MIM 234500), an autosomal recessive inherited disorder of the gene resulting in impaired absorption of tryptophan from the bowel and excessive excretion by the kidneys.

Pellagra presents with the mnemonic 4D’s (1) photosensitive dermatitis, (2) diarrhea, (3) dementia, and (4) death. This is the only photosensitivity syndrome where death is included as a cardinal clinical feature. Of note, not every patient, especially infants and children, with pellagra has the whole tetrad.

Early symptoms of pellagra include lassitude, weakness, loss of appetite, mild digestive disturbances, and psychiatric or emotional distress (e.g., anxiety, irritability, depression).

Characteristic and pathognomonic skin changes in pellagra are also known as pellagroderma, and often follow the following clinical order:

  1. Early findings:
    Acute pellagra resembles sunburn in the beginning: the skin is red with large blebs or blisters that may exfoliate and leave large areas of denuded epithelium resembling sunburns. Over time and with subsiding lesions, dusky, brown-red skin discoloration may happen.
  2. Late findings:
    In the second stage, dermatosis becomes hard, rough, cracked, blackish, and brittle. The skin may look like that of a goose (goose skin). Patients have thickened skin that is dry, scaly, and hyperkeratotic with a parchmentlike appearance and a yellowish-brown hue. The skin is darkly pigmented.

Pellagra can affect any part of the body surface, but it more frequently appears in certain areas. The usual sites are the dorsal surfaces of the hands, face, neck, arms, and feet. The backs of the hands are the most common sites for the lesions (in 77-97% of cases), with accentuation of the radial border of the dorsal aspect.

In a nutshell, photosensitive skin changes in pellagra can be summarized as:
* Bilateral and symmetrical with a well-demarcated sharp edge
* Initially resembles sunburn – acute onset, red, swollen, painful burning
* Occasional vesicles and/or bullae
* Persistent redness, scaling, pigmentation
* Becomes hyperpigmented, thickened, dry, and rough
* Casal necklace — hyperpigmented band or collar around the neck
* Pellagrous glove and boot — hyperpigmented plaques and fissuring of the hands and feet

See photos below.

Other mucocutaneous manifestations of pellagra may present as anogenital and mucosal lesions, glossitis, stomatitis, cheilitis, and sebaceous gland changes.

Gastrointestinal manifestations:
* Early symptoms — loss of appetite, vomiting, abdominal pain
* Diarrhea — develops in 50% of cases, watery, mucoid, or bloody, persists longer than 4 weeks

Neuropsychiatric manifestations:
* Late manifestation
* Initial apathy and weakness
* Progresses to headache, confusion, irritability, anxiety, tremor, depression
* Dementia and encephalopathy

NOTE - Pellagra is usually treated with oral supplementation of niacin.

  • DermNet - Pellagra
  • Medscape - Dermatologic Manifestations of Pellagra
182
Q

An 18-year-old girl was vaccinated against rubella as a part of antenatal work up and was advised to avoid pregnancy for at least 3 months. She was found to be pregnant 4 weeks after the rubella vaccination and is concerned about the outcome. Which one of the following would be the correct management?

A. Termination of the pregnancy.
B. Reassurance.
C. Re-vaccination against rubella.
D. Check serum IgG level.
E. Check serum IgM level.

A

B. Reassurance

It is recommended that women who receive rubella vaccine avoid pregnancy for 3 months post-vaccination, despite the fact that no study has proven the increased risk of congenital rubella syndrome if the patient becomes pregnant during this period.

If a patient becomes pregnant within 3 months of receiving the vaccine, reassurance is all needed and pregnancy should be continued.

Termination of pregnancy is considered if there is serological evidence of rubella infection during first trimester of pregnancy.

*https://www.sahealth.sa.gov.au/wps/wcm/connect/d81 * https://www.mja.com.au/journal/2002/176/5/1-infect

183
Q

A 23-year-old woman presents to the Emergency Department with lower abdominal pain and bleeding per vagina after a 6-week history of amenorrhea. Which one of the following sets of symptoms, if present, is highly suggestive of a tubal ectopic pregnancy?

A. Board-like abdominal rigidity with both guarding and rebound tenderness.
B. Little guarding but marked rebound tenderness in the suprapubic region.
C. Rapid pulse and upper abdominal rebound tenderness.
D. Tenderness in the pouch of Douglas and tender adnexal mass.
E. Profound shock, tachycardia and hypotension.

A

B. Little guarding but marked rebound tenderness in the suprapubic region

Marked rebound tenderness in the suprapubic region with only little or absent guarding is the most common exam finding in patients with EP. The rebound tenderness is caused by the presence of blood in the peritoneal cavity.

Board-like rigidity is evident when chemicals or purulent material irritate the peritoneal cavity. Therefore, board-like rigidity is not an expected finding in EP.

NOTE - The pain caused by a tubal ectopic pregnancy in often felt in the lower abdomen and suprapubic region rather than the affected side.

Patients with EP have cervical motion tenderness rather than tenderness of the pouch of Douglas. Pelvic inflammatory disease (PID) is another differential diagnosis for cervical motion tenderness.

In patients with EP, adnexal tenderness, or pelvic tenderness in general, is much more common than pelvic mass on exam. A mass, if present, is more likely to have been caused by blood collection from an ectopic pregnancy rather than the ectopic pregnancy itself.

Shock and its presentations (e.g., tachycardia, hypotension) are present when the ectopic pregnancy has ruptured and caused internal hemorrhage.

  • AMC Handbook of Multiple-choice Questions – page 529
184
Q

A 32-year-old woman comes to your office to discuss about her recent cervical screening test result with you. The result reports low-grade squamous intraepithelial lesion (LSIL). Her last testing wilth the old pap smear 2 years ago was reported normal. Which one of the following would be the most appropriate action to take?

A. Repeat the test in one year.
B. Refer for colposcopy.
C. Refer to an oncologist.
D. Repeat the test smear in 6 months.
E. Repeat the test in 2 years.

A

A. Repeat the test in one year

The following table includes guidelines for management of different pap smear results:

Pap smear report AND respective Investigation and management :
See table below.

  1. Negative test – inflammatory cells - Repeat the test in 5 years
  2. Unsatisfactory test - Repeat the smear in 6-12 weeks – this time period allows regeneration of the cells

Low-grade squamous intraepithelial lesion (LSIL)
1. Possible LSIL And Definite LSIL - Repeat the test in 12 months High-grade squamous intraepithelial lesion (HSIL)
2. Possible HSIL or Definite HSIL - Referral to specialist for colposcopy
3. Glandular abnormalities including adenocarcinoma in situ - Referral to gynecologist
4. Invasive squamous cell carcinoma or adenocarcinoma - Referral to appropriate specialist gynecologist or unit
5. Inconclusive – raising possibility of high-grade disease - Referral for colposcopy and possible biopsy

According to this guideline, as this woman had a normal pap smear 2 years ago, the next step would be repeating the smear in 12 months (one year).

  • National Cervical Screening Program
185
Q

On a routine cervical screening, a 34-year-old woman is diagnosed with high-grade squamous intraepithelial lesion (HSIL) of the cervix. She was referred to a gynecologist, who treated her successfully. Now this patient has been referred back to you. Which one of the following would be the next best step regarding surveillance?

A. Colposcopy and cervical cytology in 4 to 6months.
B. Start 2-yearly colposcopy after 2 years.
C. Annual human papillomavirus typing.
D. No further testing is required.
E. Cervical screening every 2 years.

A

A. Colposcopy and cervical cytology in 4 to 6months

Once a woman has been treated for high grade squamous intraepithelial lesion(HSIL), she should undergo monitoring by:
* Colposcopy and cervical cytology in 4 to 6 months after treatment.
* Cervical cytology and human papilloma virus typing at 12 months after treatment and annually until tested negative for both tests on 2 consecutive occasions. She then can be safely returned to the standard 5-yearly screening intervals for cervical cancer.

  • National Cervical Screening Program
186
Q

A 43-year-old woman presents to your GP clinic with a cervical screening test result reporting invasive squamous cell carcinoma of the cervix. Which one of the following would be the next best step in management?

A. Repeat the test in 6weeks.
B. Repeat the test in12months.
C. Colposcopy in your clinic.
D. Refer to a gynecological oncologist.
E. Reassure the patient and perform speculum examination.

A

D. Refer to a gynecological oncologist

According to the current guidelines, as shown in the following table, with a cervical screening test result significant for invasive carcinoma, the next best step would always be urgent referral to a gynecological oncologist.

TOPIC REVIEW

Action plans based on the cervical screening result are summarized in the following table: See photo below

  1. Negative test – inflammatory cells - Repeat the test in 5 years
  2. Unsatisfactory test - Repeat the smear in 6-12 weeks – this time period allows regeneration of the cells

Low-grade squamous intraepithelial lesion (LSIL)
1. Possible LSIL And Definite LSIL - Repeat the test in 12 months High-grade squamous intraepithelial lesion (HSIL)
2. Possible HSIL or Definite HSIL - Referral to specialist for colposcopy
3. Glandular abnormalities including adenocarcinoma in situ - Referral to gynecologist
4. Invasive squamous cell carcinoma or adenocarcinoma - Referral to appropriate specialist gynecologist or unit
5. Inconclusive – raising possibility of high-grade disease - Referral for colposcopy and possible biopsy

  • National Cervical Screening Program
187
Q

A 35-year-old woman had a cervical screening at your clinic one year ago, which was reported as low-grade squamous intraepithelial lesion (LSIL). You repeat the test after 12 months reporting the same result. Which one of the following would be the next best step in management?

A. Refer for colposcopy.
B. Repeat the cervical screening in 12 months again.
C. Perform cervical screening every 3 months.
D. Advise the patient for safe sex practices to reduce the risk of cervical cancer.
E. No action is required.

A

A. Refer for colposcopy

If a woman is older than 30 year old and cervical screening is positive for low-grade squamous intraepithelial lesion (LSIL), the test should be repeated in 12 months, provided that she has had a normal smear within the past 2-3 years. After 12 months, if the repeated cervical screening still shows LSIL, the patient should be referred for colposcopy as the most appropriate next step in management.

  • Cervical Screening Essentials
  • Murtagh’s General Practice – McGraw Hill – 7th Edition
188
Q

A 24-year-old pregnant woman presents in the first trimester of her pregnancy. Her pap smear report is significant for high-grade cervical squamous intraepithelial lesion (HSIL). Which one of the following would be the next best step in management?

A. Monitor her symptoms until after delivery.
B. Cone biopsy with endocervical curettage.
C. Hysterectomy.
D. Colposcopy.
E. Abortion induction before 9 weeks.

A

D. Colposcopy

All women (pregnant or non-pregnant) with high-grade cervical squamous intraepithelial neoplasia on cervical screening test should have colposcopy of the cervix and biopsy of any abnormal areas observed during the procedure.

If no lesion is identified at colposcopy, it is advisable to request a review of all the cytological slides. If the diagnosis of a high-grade abnormality is confirmed, a second opinion from another colposcopist with wide experience in the colposcopic evaluation of pregnant women is recommended. Re-evaluation in 20-24 weeks for cytology and colposcopy should be performed.

  • Cancer Council - Cervical cancer screening: Screening in pregnancy
189
Q

An 18-year-old girl comes to see you at a medical centre. She just has become sexually active with her new boyfriend. She is concerned about her risk of cervical cancer and is keen to follow your advice. Which one of the following would be the most appropriate advice in this regard?

A. She does not need cervical screening until she gets pregnant.
B. She should start 5-yearly HPV cervical screening from the age of 25 years and continue until the age of 70-74.
C. She is not at risk of contracting cervical cancer.
D. She should have 2 cervical screening tests at 5-year intervals and stop if both are normal.
E. She should not have sex with her boyfriend before marriage.

A

B. She should start 5-yearly HPV cervical screening from the age of 25 years and continue until the age of 70-74

Guidelines for national cervical cancer screening program has been changed since December 2017. It is postulated that implementation of new guidelines will result in a 30% reduction in the mortality rate from cervical cancer. The following table outlines the differences between the new and old guidelines: See table below

  • Women who have never engaged in sexual relationship do not need pap smears
  • Lesbian women require pap testing similar to heterosexual women even if they never had a male sexual partner

Since this girl has just started sexual relationship she should start 5-yearly HPV testing after the age of 25 years and continue unitl the age of 70-74 years.

  • National Cervical Screening Program
190
Q

A 29-year-old woman comes to your clinic for advice regarding cervical screening. She has never had sexual activity with men or women. Regarding cervical screening, which one of the following would be the best advice for her?

A. Annual cervical screening.
B. As long as sexually inactive, no cervical screening is required for her.
C. 2-yearly cervical screening.
D. She should start sexual activity as it is safe now.
E. She should see a psychiatrist, as she might have underlying mental illness.

A

B. As long as sexually inactive, no cervical screening is required for her

Current guidelines recommend 5-yearly HPV testing of all vaccinated and unvaccinated women from the age of 25 years or 2 years after the first sexual relationship, whichever is later, to 70-74 years. This has replaced the previous recommendations of performing 2-yearly pap smears for women from the age of 18 years or 2 years after the first sexual relationship.

However, women, who never had sexual relationship (neither with men, nor women), like this woman, do not need cervical screening. Once this woman starts sexual relationship, cervical screening would be indicated in 2 years and repeated every 5 years until the age of 70-74.

Cervical cancer is so strongly associated with sexual relationship, which can be classified as a sexually transmitted disease (STD); therefore, not having sexual relationship usually means no cervical cancer.

Sexual relationship without precautionary measures increases the risk of cervical cancer at any age. The risk of cervical cancer rises after the first unprotected sexual relationship, regardless of the age at which it is started.

Advising her to see a psychiatrist is not appropriate because it is not uncommon for women to not have experienced any sexual relationship for many years through their puberty. On the other hand, there is no history of mental illness to necessitate psychiatric evaluation.

  • Cervical Screening Essentials
191
Q

A 30-year-old woman presents to the emergency department at 37 weeks’ gestation after sudden onset of severe abdominal pain, vaginal bleeding (approximately 1200 cc) , and cessation of contractions after 18 hours of active pushing at home. The pregnancy has been uneventful until the event. On examination, she is conscious and pale, with a blood pressure of 70/45 mmHg and pulse rate of 115bpm. The abdomen is irregularly distended. Shifting dullness and fluid thrill are present. Fetal heart sounds are not audible. Which one of the following is the most likely diagnosis?

A. Placenta previa.
B. Placental abruption.
C. Uterine rupture.
D. Cervical laceration.
E. Disseminated intravascular coagulation.

A

C. Uterine rupture

The scenario is typical for uterine rupture. The condition presents with sudden abdominal pain, cessation of uterine contraction, the urge to push, and vaginal bleeding. On examination, the fetal heart rate is decreased or lost. Signs of fluid collection are often present. These signs include fluid thrill and shifting dullness caused by entrance of the blood into the peritoneal cavity.

Other manifestations of uterine rupture include:
* Loss of the station of the fetal presenting part
* Vaginal bleeding that might not be proportionate to the hemodynamic status
* Maternal tachycardia and hypotension ranging from subtle to severe (shock)
* Uterine tenderness
* Change in uterine shape and contour
* Easily palpable fetal parts
* No fetal presentation on vaginal examination
* Hematuria if the rupture extends to the bladder

The most common site of spontaneous uterine rupture is the anterior lower transverse segment.

OPTION A : Placenta previa causes painless vaginal bleeding, so cannot be the diagnosis here.

OPTION B : Placental abruption presents with vaginal blood loss and often a tender and tense uterusl. Contrary to uterine rupture, in placental abruption uterine contractions continue and do not stop.

OPTION D : Cervical laceration can be a possibility, but if it was the source of bleeding, more amount of vaginal blood loss was expected in this patient with hemodynamic instability. Furthermore, deformed uterus, abdominal distention and cessation of the contractions are inconsistent with cervical laceration as the cause of the bleeding.

OPTION E : Disseminated intravascular coagulation (DIC) is a condition resulting from excessive generation of thrombin and fibrin in the circulating blood. It leads to increased platelets aggregation and consumption of coagulation factors with consequent bleeding at one site and thromboembolism at another. The most common obstetrical causes of DIC are placental abruption and retained products of conception in the uterine cavity. DIC does not fit this clinical scenario.

*http://www.sahealth.sa.gov.au/wps/wcm/connect/0f2c * http://www.uptodate.com/contents/rupture-of-the-un

192
Q

48-year-old woman with past history of hysterectomy comes to discuss about hormone replacement therapy (HRT) for her post-menopausal symptoms, while she is also concerned about the associated risk of breast cancer. Which one of the following is the best advice for her?

A. Postmenopausal women with BMI <25 have a lower relative risk of developing breast cancer associated with estrogen-only HRT than those women with higher BMIs.
B. There is consistent evidence indicating that tibolone maybe associated with an increased risk of breast cancer.
C. Use of estrogen-alone HRT for up to seven years appears to have no effect on risk of breast cancer.
D. She does not need hormone replacement therapy.
E. She should use hormone replacement therapy only for few weeks.

A

C. Use of estrogen-alone HRT for up to seven years appears to have no effect on risk of breast cancer

The best option for HRT in women who do not have a uterus due to previous hysterectomy is estrogen alone. These patients should be advised that this regimen seems not to increase the risk of developing breast cancer if used for only up to seven years. Beyond this time, however, the risk increases proportionate to the duration of HRT.

Post-menopausal women with a BMI less than 25 have a higher relative risk of contracting breast cancer associated with estrogen-only hormone replacement therapy, than women with higher BMI. The reason can be attributed to less metabolization of estrogen in the adipose tissue.

Tibolone is used for hormone replacement therapy and for treatment of endometriosis. There is very inconsistent evidence indicating that tibolone may be associated with an increased risk of breast cancer.

While this woman has menopausal symptoms, advising against HRT is not appropriate. She can use HRT for a longer duration compared to women who still have uterus and are at risk of developing endometrial cancer.

193
Q

A 3-year-old Sudanese boy is being assessed as a part of a health check for refugees. He has dark skin, and Further inquiry reveals that he has a history of multiple fractures at different times, which the parents attribute to his clumsiness. When asked why they feel he is clumsy, they answer that he started walking very late and prefers to sit down rather than walk or play with other children. Based on multiple risk factors in the history, you order a serum vitamin D level among other tests which is positive for vitamin D deficiency. Which of the following is the most appropriate treatment option for him at this stage?

A. Calcitonin.
B. Calcitriol.
C. Calcium supplementation.
D. Cholecalciferol.
E. More sun exposure.

A

D. Cholecalciferol

There is increasing recognition that many Australians and people from specific groups within the community suffer from vitamin D deficiency.

Vitamin D deficiency can be classified as:
* mild (25–50 nmol/L)
* moderate (12.5–25.0 nmol/L)
* severe (<12.5 nmol/L) with levels of 50–100 nmol/L thought of as being vitamin D insuffici><12.5 nmol/L)

With levels of 50–100 nmol/L, vitamin D insufficiency rather than deficiency is used.

Vitamin D is important for muscle and bone health. Vitamin D deficiency is associated with:
* Cortical bone loss
* Increased bone turnover
* Increased parathyroid hormone levels
* Predisposing to osteoporosis
* Vitamin D supplementation increases bone density in established deficiency.
* Vitamin D deficiency has associations with:
* Cardiovascular disease
* Insulin resistance and β-cell dysfunction
* The development of autoimmune diseases (including type 1 diabetes, rheumatoid arthritis, and multiple sclerosis)
* Colon, breast, and prostate cancers

The following groups are at significant risk of vitamin D deficiency:
* The institutionalized or housebound individuals
* Those with sun-avoidant behavior
* Refugees
* Those with celiac disease (or other malabsorptive conditions)
* Those from areas of high vitamin D deficiency prevalence
* Those in whom osteoporosis therapy is considered
* Those with chronic idiopathic musculoskeletal pain * Pregnant women (particularly if dark-skinned or veiled)

Vitamin D deficiency is often a silent disease. Prolonged vitamin D deficiency results in Rickets in children whose growth plates have not fused. These children are often found to have started walking late or prefer to sit down for prolonged periods. In adults, vitamin D deficiency results in osteomalacia, which presents as a poorly mineralized skeletal matrix. Adults in these cases can experience chronic muscle aches and pains.

Current guidelines recommend screening only those individuals who are at high risk for vitamin D deficiency, including patients with osteoporosis or a malabsorption syndrome, as well as black and Hispanic individuals, obese persons (BMI >30 kg/m2), and those with several other medical conditions.

The daily maintenance dose of vitamin D varies by age, but most children and adults generally require 600-2000 IU of vitamin D daily. For vitamin D-deficient children and adults, higher doses of vitamin D given either daily or weekly are recommended, followed by an increase in the daily dose of vitamin D.

Current guidelines endorse supplementation with cholecalciferol (25- hydroxyvitamin D). also known as D3, or ergocalciferol (plant bases vitamin D). Cholecalciferol is the best treatment option for this child at this stage.

For maintenance, a diet rich in vitamin D should also be recommended.

Sun exposure is the most important source of vitamin D production in the human body. Safe sun exposure is recommended for maintaining healthy levels of vitamin D; however, with vitamin D deficiency or insufficiency, supplementing vitamin D is required first to push vitamin D levels within normal ranges. Moreover, dark-skinned people may take less benefit from sun exposure (option E). In fact, their skin color could have contributed to such deficiency in the first place.

Vitamin is required for calcium absorption and metabolism. Reduced amounts of vitamin D result in impaired calcium absorption and this child may have calcium issues as well. However, calcium supplementation (option C) does not treat vitamin D deficiency. Moreover, vitamin D must reach a safe level before calcium supplementation is considered.

Calcitonin (option A) is used to treat hypercalcemia due to Paget’s disease of the bone, or hypercalcemia due to other causes. It also may be used to prevent bone loss in postmenopausal women with osteoporosis, but it has no role in the treatment of vitamin D deficiency.

Calcitriol (1,25-dihydroxyvitamin D3) (option B) is the most active form of vitamin D and generally not suitable for the treatment of vitamin D deficiency as it has a narrow therapeutic window resulting in an increased risk of hypercalcemia or hypercalciuria. This is especially true in nursing home residents who often have severe vitamin D deficiencies. Calcitriol has a role in the treatment of vitamin D deficiency in renal failure where there is an inability to convert 25-hydroxyvitamin D (vitamin D3) to 1,25-dihydroxy vitamin D. Serum calcium concentrations and renal function must be monitored closely under these circumstances.

  • Medscape - Vitamin D Deficiency and Related Disorders
  • RCH - Vitamin D deficiency
  • Australian Prescriber - Vitamin D deficiency in Adults
194
Q

A 37-year-old woman presents to your clinic for an antenatal checkup. She has past obstetric history of breech presentation, premature rupture of membranes and twin pregnancy. She also had postpartum depression during her last two pregnancies as well as anemia and gestational diabetes in her last pregnancy.Which one of the following would not increase the risk for cord prolapse during delivery?

A. Breech presentation.
B. Multiple gestation.
C. Premature rupture of membrane.
D. Anemia.
E. Gestational diabetes.

A

D. Anemia

Cord prolapse should be suspected where there is an abnormal fetal heart rate pattern (bradycardia, variable decelerations etc), particularly if such changes commence soon after the ruputre of the membranes, spontaneously or with amniotomy.

Anemia is not a predisposing factor to cord prolapse.

  • Royal College of Obstetricians and Gynaecologists – Green-top Guideline – No.50
  • UpToDate - Umbilical cord prolapse
195
Q

A 21-year-old woman, G1P0 comes to your clinic at 12 weeks pregnancy. She is complaining of mild vaginal bleeding for the past 12 hours, associated with bouts of mild cramping lower abdominal pain. On vaginal examination, the cervical os is closed and there is mild discharge containing blood clots. Ultrasonography confirms the presence of a live fetus with normal heart rate. Which one of the following is the most likely diagnosis?

A. Threatened abortion.
B. Inevitable abortion.
C. Incomplete abortion.
D. Missed abortion.
E. Complete abortion.

A

A. Threatened abortion

Uterine bleeding in the presence of a closed cervix and sonographic visualization of an intrauterine pregnancy with detectable fetal cardiac activity is diagnostic of threatened abortion.

The term ‘threatened’ is used because abortion does not always follow uterine bleeding in early pregnancy, even after repeated episodes or large amounts of bleeding.

In fact, in 90 to 96% of cases who present with vaginal bleeding in the presence of a closed os and a detectable fetal heart rate, the pregnancy continues. The more advance the gestational age, the less likely the condition will end in miscarriage.

OPTION B : In inevitable abortion, there is a dilated cervix as well as progressive uterine bleeding and painful uterine contractions. The gestational tissue often can be felt or seen through the cervical os and passage of this tissue typically occurs within a short time.

OPTION C : In incomplete abortion, the membranes may have ruptured, and the conception products may have partly passed, but significant amounts of placental tissue are still left in the uterus. This is the most common presentation of an abortion in late first and early second trimester. On examination, the cervical os is open, gestational tissue may be observed in the cervix, and the uterine size is smaller than expected for gestational age. The uterine is not well contracted. The amount of bleeding varies but can be severe enough to cause hypovolemic shock. Painful contractions are often present. Ultrasound reveals tissue in the uterus.

OPTION D : A missed abortion refers to in utero death of the embryo or fetus prior to 20 weeks gestation. Women may notice that symptoms associated with early pregnancy (nausea, breast tenderness, etc.) have abated and they do not ‘feel pregnant’ anymore. Vaginal bleeding may occur, and the cervix usually remains closed. Ultrasound reveals an intrauterine gestational sac with or without an embryonic/fetal pole, but no embryonic/fetal cardiac activity.

OPTION E : In complete abortion, a miscarriage occurs before week 12 and the entire contents of the uterus is expelled. If this has occurred, the uterus is small on physical examination and well contracted with an open or closed cervix. There is scant vaginal bleeding, and only mild cramping. Ultrasound will reveal an empty uterus and no extra-uterine pregnancy.

  • UpToDate - Pregnancy loss (miscarriage): Risk factors, etiology, clinical manifestations, and diagnostic evaluation
196
Q

A 35-year-old woman presents to your GP clinic with complaints of irritability, anxiety, breast tenderness and headaches about 5 days before the onset of menses every months for the past 6 months. These symptoms are relieved with the onset of menses. She has to take a week off from work because of these symptoms. She is non-alcoholic and does not use any regular medication. Which one of the following is the most likely diagnosis?

A. Premenstrual dysphoric disorder.
B. Premenstrual syndrome.
C. Normal menstrual physiology.
D. Generalized anxiety disorder.
E. Depression.

A

B. Premenstrual syndrome

This woman fufils diagnostic criteria for premenstrual syndrome (PMS) as the most likely diagnosis.

Diagnostic criteria for premenstrual syndrome include affective and somatic symptoms starting within one week before menstruation and resolving within 4 days after menstrual flow starts. The symptoms should be recurrent for at least 3 menstrual cycles and must be absent in the preovulatory phase of the menstrual cycle. The specific symptoms are less important than their temporal relationship to the menstrual cycle.

Affective symptoms include, but not limited to:
* Depression
* Anger outbursts
* Irritability
* Social withdrawal
* Anxiety
* Tearfulness
* Helplessness/hopelessness
* The patient suffers from identifiable dysfunction in social or economic performance

Somatic symptoms include, but not limited to:
* Breast tenderness
* Abdominal bloating and pain
* Headache
* Muscle and joint pain
* Eedema
* Weight gain
* Heart pounding
* Confusion, dizziness

Symptoms are usually relieved within 4 days of onset of menses.

It is important that these symptoms cannot be attributed to any medical or psychological disorder, medications, drug, or alcohol.

OPTION A : Premenstrual dysphoric disorder is severe form of premenstrual syndrome and is characterised by severe feeling of sadness, emotional labiality with frequent tearfulness, loss of interest in daily activities, decreased concentration, fatigue, insomnia and feeling of being overwhelmed or out of control. Symptoms are more related to affections rather than being somatic. In other words, affective impairment is the dominating feature.

The Diagnostic and Statistical Manual of Mental Disorders, Fifth Edition (DSM-5), established 4 research criteria (A through D) for the diagnosis of PMDD.

Criterion A - in most menstrual cycles during the past year, at least 5 of the following 11 symptoms (including at least 1 of the first 4 listed) were present:
1. Markedly depressed mood, feelings of hopelessness, or self-deprecating thoughts
2. Marked anxiety, tension, feelings of being “keyed up” or “on edge”
3. Marked affective lability (e.g. feeling suddenly sad or tearful or experiencing increased sensitivity to rejection) 4. Persistent and marked anger or irritability or increased interpersonal conflicts
5. Decreased interest in usual activities (e.g. work, school, friends, and hobbies)

197
Q

A 27-year-old female, who is 37 weeks pregnant, presents to the Emergency Department with sudden onset of severe headache. She feels nauseous and dizzy. Her mother noted slurred speech and repetition of words after the headache started. She is clinically stable and has not had any obstetric complication so far. She has no history of migraine. Which one of the following would be the most appropriate next step in management?

A. Aspirin.
B. CT scan of the brain.
C. MRI of the brain.
D. Lumbar puncture.
E. Observation in the ward.

A

B. CT scan of the brain

The clinical picture is suggestive of subarachnoid hemorrhage (SAH), and non-contrast CT scan of the brain would be the next best step in management.

Pregnancy is a recognized risk factor for aneurysmal SAH. SAH is a very rare complication of pregnancy and may be confused with eclampsia, due to similar presentation with high blood pressure, seizures and altered levels of consciousness. Once the condition is suspected, the diagnosis is confirmed with CT scan (preferred), MR, or cerebral angiography. Although CT scanning exposes the fetus to radiation, the benefits significantly outweighs the risks. Appropriate covering of the uterus may reduce the risks to the fetus.

OPTION A : Aspirin should be avoided unless full assessment excludes the possibility of intracranial hemorrhage.

OPTION C : CT scan is more sensitive than MRI in detection of blood and is the first-line investigation where intracranial hemorrhage is suspected.

OPTION D : If CT scan of the brain is inconclusive, lumbar puncture comes next. It should be performed within 12 hours after the onset of the symptoms.

OPTION E : Subarachnoid hemorrhage is a real emergency demanding immediate workup; therefore, just observing the patient in the ward would be inappropriate.

  • Aneurysmal Subarachnoid Haemorrhage in Pregnancy: A Case Series
  • UpToDate - Cerebrovascular disorders complicating pregnancy
198
Q

In which one of the following conditions, it might be advised that breastfeeding be stopped?

A. Engorged breasts.
B. Inverted nipples.
C. Cracked nipples.
D. Breast abscess.
E. Mastitis.

A

D. Breast abscess

It is recommended that breastfeeding be continued from both breasts in cases of breast abscess. However, as the treatment of the abscess would be incisional drainage, the location of the incision may prevent the baby from being directly breastfed. In case of breast abscess it is recommended that breastfeeding be discontinued until the abscess resolves. Milk should be expressed manually and be given to the baby.

OPTION A : Engorged breast occurs with excessive milk production or decreased output due to inappropriate breastfeeding technique or the baby not taking enough milk. It results in accumulation of milk in breasts and engorgement. Breast feeding should be continued. Proper breastfeeding on baby’s demand is the key management of this condition.

OPTION B : Inverted nipple occurs when one or both nipples invert into the breast instead of pointing outwards. Breast feeding should be continued in mothers with inverted nipples.

OPTION C : Cracked nipples are usually due to the baby clamping the mouth around the nipple instead of applying the jaws behind them. The best approach is to rest the nipple for 1 to 2 days. The milk of the affected breast should be expressed manually and given to the baby with bottle. After 1 to 2 days of rest, breastfeeding should be resumed with short feeds and proper technique.

OPTION E : Mastitis is cellulitis of the interlobular connective tissue of the breast. It is mainly associated with cracked nipples or poor milk drainage (engorgement) in a lactating mother. The infecting organism is Staphylococcus aureus. Breast feeding from the affected side can be continued as infection is confined to the interstitial breast tissue.

  • RACGP - Lactational mastitis and breast abscess - Diagnosis and management in general practice
  • Australian Breastfeeding Association - Breast Abscess
199
Q

A 32-year old woman comes to your clinic for advice. She is 12-week pregnant, and has already been on folic acid for the past 24 weeks. Which one of the following should be started for her?

A. Iodine for a few weeks.
B. Vitamin B12 injections.
C. Vitamin A supplements.
D. Vitamin C.
E. Iodine throughout the pregnancy.

A

E. Iodine throughout the pregnancy

Clinical guidelines for antenatal care by National Health and Medical Research Council of Australia recommend folic acid 0.5 mg daily 12 weeks before conception and during first trimester. The dose of folic acid is increased to 5mg daily in the following situations:

  • History of neural tube defects in previous pregnancies
  • The pregnant woman is diabetic
  • The pregnant woman is on anticonvulsant medications

The mother is also recommended to take 150 microgram of iodine on a daily basis throughout the pregnancy for prevention of iodine deficiency in the fetus.

She has taken folic acid for the first trimester; therefore it can safely be stopped.

While there is evidence to support routine supplementation with folic acid and iodine in pregnancy, other vitamin and mineral supplements are not of benefit unless there is an identified deficiency. Vitamin A, B and C supplements are not routinely used in pregnancy.

  • http://www.health.gov.au/internet/main/publishing.
200
Q

A 32-year-old woman presents to your clinic for advice. She has been married for the past 2 years and has not been able to conceive. Blood tests show the following results:

Luteinizing hormone (LH): 1 U/L (3-16)
Follicle stimulating hormone (FSH): 0.5 U/L (2 – 8) Thyroid stimulating hormone (TSH): 0.3 mU/L (0.5 -5) Prolactin: 8750 U/L (<600 U/L)

Which one of the following is most likely to result in feritility in this woman?

A. Clomiphene citrate.
B. In vitro fertilisation.
C. Metformin.
D. Bromocriptine.
E. Surgical resection of the pituitary tumour.

A

D. Bromocriptine

The laboratory picture suggests hyperprolactinemia probably due to a prolactinoma. High levels of circulating prolactin, through a negative feedback, lead to co-inhibition of THS, FHS, LH and probably ACTH secretion. Once the prolactinoma is treated successfully with dopamine agonists such as bromocriptine, the condition is controlled. Prolactin-secreting adenomas of the pituitary gland (prolactinomas) can lead to hyperprolactinemia-induced infertility.

Bromocriptine is initially used for induction of tumor shrinkage. As the tumor shrinks, the prolactin levels become more normal and fertility resumes. Once the prolactinoma is biochemically diagnosed, MRI scan of the head should be considered to confirm and localize the adenoma before commencement of the therapy.

  • Therapeutic Guidelines – Endocrinology; available from http://tg.org.au
201
Q

A 34-year-old pregnant woman presents at 32 weeks pregnancy for an antenatal visit. On pelvic examination, breech presentation is suspected. Ultrasound shows that fetal hips are flexed, and the knees extended. Which one of the following is this presentation called?

A. Complete breech.
B. Frank breech.
C. Footling breech.
D. Kneeling breech.
E. Transverse lie.

A

B. Frank breech

Types of breech presentation include:
1. Frank breech: The fetal hips are flexed, and the knees extended.
2. Complete breech: The fetus seems to be sitting with hips and knees flexed.
3. Footling breech: One or both legs are completely extended and present before the buttocks.
4. Kneeling breech: The baby is in a kneeling position, with one or both legs extended at the hips and flexed at the knees.

Fetal position is transverse if the fetal long axis is oblique or perpendicular rather than parallel to the maternal long axis.

With the hips flexed and knees extended, this fetus has Frank breech presentation. This type is the most common breech presentation.

*https://www.sahealth.sa.gov.au/wps/wcm/connect/227

202
Q

A 23-year old Asian woman comes to your clinic at 36 weeks gestation. She was diagnosed with breech at 32 weeks. She is not in labor and manual examination of the uterus is suggestive of breech position. Which one of the following would be the next best step in management?

A. Induction of labor.
B. Cesarean delivery.
C. Steroids.
D. Pelvic ultrasound.
E. Admission to the hospital and arrange for blood transfusion.

A

D. Pelvic ultrasound

When a breech presentation suspected by manual exam, ultrasonography is needed as the next best step in management for confirmation, as well as evaluation of maternal pelvis, the fetal size and viability. This should be considered in every future visit. Breech presentation has the chance to spontaneously turn into cephalic 36 to 37 weeks. Once the breech position persists beyond this time, the chance of spontaneous version reduces to 25%. It is recommended that all women with breech presentation be offered an external cephalic version provided that there is no contraindication and cesarean delivery is not indicated for other reasons..

*http://www.kemh.health.wa.gov.au/development/manua * https://www.sahealth.sa.gov.au/wps/wcm/connect/227

203
Q

Which one of the following is not a contraindication to vaginal delivery in breech presentation at term?

A. Placenta previa.
B. Footling breech presentation.
C. Estimated fetal weight>3800g.
D. Clinically inadequate pelvis.
E. Anemia.

A

E. Anemia

Vaginal breech delivery may be offered provided that there are no contraindications to vaginal delivery.

Contraindications to vaginal breech delivery include:
* Contraindication to vaginal birth (e.g. placenta previa, contracted pelvis, cord presentation)
* Prior cesarean deliveries
* Fetal anomalies that may cause dystocia
* Estimated fetal weight <2500 g and >3800 g.
* Gestational age <36 weeks
* Hyperextension of the fetal head (i.e. an extension angle of greater than 90 degrees)
* Any presentation other than frank or complete breech presentation (incomplete breech)
* Anticipated mechanical difficulty

Anemia is not a contraindication to vaginal delivery for breech presentation.

*http://www.kemh.health.wa.gov.au/development/manua

204
Q

A 32-year-old woman presents to your practice at 28 weeks gestation with recurrent vulval warts previously treated with cryotherapy. Which one of the following options would be the most appropriate management option?
A. Electrocautery.
B. Surgical excision.
C. Cervical screening.
D. Podophyllum.
E. Cryotherapy.

A

E. Cryotherapy.

Genital warts can be managed either medically or surgically:
Medical management
1. Podophyllotoxin - 0.15% cream or 0.5% paint topically applied by the patient to each wart, twice daily for 3 days followed by a 4-day break.
This should be repeated for 4 to 6 cycles until the warts disappear.

  1. Imiquimod - 5% cream topically applied by the patient to each wart, 3 times per week at bedtime (wash off after 6 to 10 hours) until warts are cleared (usually 12-24 weeks)
  2. Podophyllum - 25% solution in compound benzoin tincture topically applied by the clinician to each wart, washed off after 6 hours (repeat once weekly until the warts disappear)

NOTE - Podophyllum and podophyllotoxin should not be used in pregnancy. Although imiquimod is category B1 in pregnancy, its use is not recommended during the pregnancy due to lack of evidence on its safety.

Surgical management
Overall, surgical management of genital warts is associated with fewer recurrences compared to medical treatment. The surgical options include: 1. Cryosurgery (cryotherapy) - it is the method of choice for simple warts, and is performed in the office with ot without anesthesia and/or analgesia once in a week until the warts disappear. This method is safe in pregnancy and the preferred option.
2. Surgical excision - surgical excision of the wart is the method with the least recurrence rate. The procedure is painful and needs anesthesia and/or analgesia. Simple surgical excision with a scalpel, scissors, or curette is used to remove warts, especially the large genital ones. This method is reserved for refractory or extensive warts. The procedure is usually performed in an outpatient surgical suite.
3. Electrosurgical modalities (diathermy or electrocautery) - these methods use high-frequency current to cut and coagulate warts. Electrodesiccation with a bipolar needle is most effective with external genital warts. LEEP is primarily used to treat cervical squamous intraepithelial lesions (SILs) after confirmation with a cervical biopsy but may also be used to remove large external genital warts. Electrosurgical methods usually require only topical anesthesia and may be employed in an outpatient setting if the appropriate equipment is available.
4. Large unresponsive lesions around the rectum or vulva can be treated with scissor excision of the bulk of the mass followed by electrocautery of the remaining tissue down to the skin surface. Removal of a very large mass of warts is a painful procedure and is best performed with the patient under either general or spinal anesthesia.
5. Laser therapy – carbon dioxide laser vaporization is typically used for treatment of refractory disease or extensive warts of the anogenital mucosal warts and is particularly useful in the treatment of periurethral and vaginal warts. It is the treatment of choice for pregnant women with extensive mucosal lesions.

Since this woman is pregnant, podophyllum and podophyllotoxin should not be used due to the risk of teratogenicity. Imiquimod (Aldara®) is not recommended (however, it can be used after informed consent). Cryotherapy, surgical excision, laser ablation and electrocautery can all be used safely during pregnancy. If the lesions are readily available and are few in numbers, cryotherapy is the method of choice.

References * SA Health - Wart Treatment Algorithm * AAFP - Management of Genital Warts

205
Q

A 25-year-old pregnant woman comes to your clinic with vaginal bleeding at 14 weeks’ gestation. Which one of the following is the main reason for performing ultrasonography?

A. Excluding ectopic pregnancy.
B. Excluding neural tube defects.
C. Determining the cause of bleeding.
D. Determining if the fetus is alive.
E. Excluding congenital heart defects.

A

C. Determining the cause of bleeding

Transvaginal ultrasound is the cornerstone in evaluation of vaginal bleeding in the second trimester. The main reason for ultrasonography is to find an underlying cause for the bleeding. Reasons to perform an ultrasound include:

  1. To determine whether the placenta is covering the cervical os (placenta previa).
  2. Whether there is evidence of decidual hemorrhage causing placental separation (abruption of placenta)
  3. Whether the cervix shows signs suggestive of cervical insufficiency (short length, dilated internal os, funneling of the fetal membranes).
  4. Whether there is a cervical polyp

The common causes of vaginal bleeding in second and third trimesters of pregnancy are:

-Bloody show associated with cervical insufficiency or labor
-Placenta previa
-Placental abruption
-Uterine rupture
-Vasa previa

*http://www.uptodate.com/contents/overview-of-the-e

206
Q

Which one of the following is the most common cause of postpartum hemorrhage requiring hysterectomy?

A. Genital lacerations.
B. Uterine atony.
C. Uterine inversion.
D. Placenta accreta.
E. Retained products of conception.

A

D. Placenta accreta

The most common reasons for hysterectomy as the inevitable treatment of postpartum hemorrhage are placental abnormalities such as placenta previa and placenta accreta.

Placental villi normally invade only the superficial layers of endometrial deciduas basalis. When the invasion is too deep into the uterine wall, the condition is termed as placenta accreta, increta or percreta depending on the depth of invasion:

Placenta accreta – the villi invade the deeper layers of the endometrial deciduus basalis, but not the myometrium. This is the most common type of decidual invasion, accounting for approximately 75% of the cases.

Placenta increta – the villi invade the myometrium, but do not reach the uterine serosa or the bladder. This type accounts for 15% of cases.

Placenta percreta – the villi invade into the uterine serosa or the bladder. This happens in 5% of cases.

Prior uterine surgery is the main risk factor for placenta accreta. The best management is elective cesarean hysterectomy.

Post-partum hemorrhage can also be caused by following conditions:

-Genital lacerations
-Uterine atony
-Retained products of conception
-Uterine inversion

In most of above cases, hysterectomy would not be required and remains the last resort in really desperate situations.

NOTE - Uterine atony is the most common cause of post-partum hemorrhage and is often manageable medically.

207
Q

Which one of the following is not elevated in the third trimester of pregnancy?

A. Serum prolactin.
B. Serum alkaline phosphatase.
C. Serum iron binding capacity.
D. Serum free T4.
E. Serum cortisol.

A

D. Serum free T4

Normally, estrogen stimulates and progesterone inhibits prolactin secretion; despite this action and counteraction, there is a slight raise in prolactin level throughout pregnancy.

Serum alkaline phosphatase is increased in pregnancy due to placental ALP.

Physiologically, the concentration of thyroid stimulating hormone (TSH) normally decreases during the first trimester of pregnancy when there is maximal cross-stimulation of the TSH receptor by hCG. The TSH concentration then returns to its pre- pregnancy level in the second trimester and then rises slightly in the third trimester. However, most of changes still occur within the normal non-pregnant range. Serum free T3 and T4 concentrations remain essentially unchanged during pregnancy but total concentrations, which include both free and protein-bound fractions, are significantly elevated due to increased circulating binding globulins..

Iron binding capacity reflects the protein used for iron transport, transferrin. Transferrin is a globulin in the beta band on electrophoresis. Both transferrin and iron binding capacity are elevated in pregnancy to counteract decreased plasma iron during pregnancy.

Cortisol levels can be elevated up to three times normal when compared to non-pregnant levels.

*http://www.australianprescriber.com/magazine/28/4/
*http://www.ncbi.nlm.nih.gov/pmc/articles/PMC398211

208
Q

Which one of the following is incorrect regarding hypothyroidism in pregnancy?

A. Thyroxin requirement increases by 25-30% during pregnancy.
B. Children born to women whose hypothyroidism was inadequately treated in pregnancy are at increased risk of neuropsychological impairment.
C. Thyroxine dose should be increased by 30% at the beginning of pregnancy.
D. TSH should be monitored every 4 to 6 weeks in the first half of the pregnancy, and thyroxin dose adjusted if necessary.
E. Thyroxin requirement does not increase in pregnancy and maintenance dose must be continued.

A

E. Thyroxin requirement does not increase in pregnancy and maintenance dose must be continued

During pregnancy, the size of the thyroid gland increases 15%, mostly due to increased vasculature. Estrogen causes the liver to produce more thyroid binding globulin (TBG). This results in increased total T3 and T4 levels; however, free T3 and T4 remain unchanged. Increased glomerular infiltration rate (GFR) leads to more renal excretion of iodine.

Pre-existing maternal hypothyroidism can lead to neuropsychological deficits of the baby; therefore, the thyroxin requirement should be precisely adjusted during pregnancy. Pregnancy is associated with an increase in demand of thyroxin by 25 to 30% requiring a 30% increase is thyroxin dose as soon as the pregnancy is diagnosed. This is best achieved by taking 2 additional doses of thyroxin per week on confirmation of pregnancy. (In most cases increasing from 7 to 9 doses per week)

TSH and free T4 should be rechecked after 4 weeks and monitored every 4-6 weeks in the first half of pregnancy and at least once between 26 and 32 weeks. The values should be maintained within the trimester specific range. After delivery, the thyroxin dose can be reduced to pre-pregnancy dose. Breastfeeding mothers do not need extra doses of thyroxin.

NOTE - subclinical hypothyroidism is associated with adverse pregnancy outcomes, particularly miscarriage but not impaired cognitive function.

  • RACGP - AFP - Thyroid disease in the perinatal period
209
Q

Which one of the following statements is correct regarding the management of deep venous thrombosis in pregnancy?

A. Warfarin therapy is contraindicated only in the first trimester of pregnancy.
B. Warfarin therapy is contraindicated throughout pregnancy but safe during breastfeeding.
C. Warfarin therapy is contraindicated throughout pregnancy and postpartum period.
D. Both warfarin and heparin are safe to use for treatment of deep vein thrombosis in pregnancy.
E. Low-molecular weight heparin is contraindicated in the first trimester of pregnancy.

A

B. Warfarin therapy is contraindicated throughout pregnancy but safe during breastfeeding

Pregnancy is a hypercoagulable state with increased risk of deep venous thrombosis (DVT) and pulmonary embolism (PE). PE is a significant cause of maternal death in Australia.

In pregnant women, venous thromboembolism should be treated with heparin because warfarin is contraindicated. Warfarin crosses the placenta and should be avoided throughout pregnancy, especially during the first and third trimesters.

Warfarin at 6-12 weeks’ gestation results in fetal warfarin syndrome characterized by the following features:

-A characteristic nasal hypoplasia
-Short fingers with hypoplastic nails
-Calcified epiphyses, namely chondrodysplasia punctuta (evident by stippling of epiphyses on X-ray)
-Intellectual disability
-Low birth weight

Recent studies show the risk of fetal warfarin syndrome in babies of women who require warfarin throughout pregnancy is around 5%. The risk is dose dependent.

Later exposure (after 12 weeks) is associated with central nervous system anomalies including microcephaly, hydrocephalus, agenesis of corpus callosum, Dandy-Walker malformation (complete absence cerebellar vermis and enlarged fourth ventricle), and mental retardation, as well as eye anomalies such as optic atrophy, microphthalmia, and Peter anomaly (anterior segment dysgenesis). Blindness can be seen in newborns exposed to warfarin in all three trimesters. Perinatal intracranial hemorrhage and other major bleeding is another complication in neonates exposed to warfarin.

Warfarin is not secreted into the breast milk, and is safe to use in post-partum period.

  • MJA - Consensus guidelines for warfarin therapy
210
Q

A 32-year-old female presented in a regional Australian hospital with acute onset shortness of breath, tachycardia, and sweating after landing from a long-distance flight from California. She is 34 weeks pregnant. On examination, she has a blood pressure of 110/87 mmHg, pulse rate of 110 bpm, and respiratory rate of 26 breaths per minute. Heart and lung auscultation is clear. There is no leg swelling, warmth o tenderness. Which one of the following is the best investigation in this situation?

A. CT pulmonary angiogram (CTPA).
B. Electrocardiography.
C. Bilateral lower exteremity Doppler ultrasound.
D. D-dimer.
E. Ventilation-perfusion scan.

A

E. Ventilation-perfusion scan

In developed countries, pulmonary embolism (PE) remains a significant cause of mortality in pregnant women. Despite advances in diagnostic modalities, diagnosis of PE in this patient group is challenging due to the following reasons:

  1. There are two patients at risk rather than one
  2. Overdiagnosis results in unnecessary and dangerous treatment, jeopardizing both the mother and fetus
  3. Choosing a modality for diagnosis of PE in pregnant women is more complicated than in non-pregnant patients.

There is about the equal risk of venous thromboembolism in all three trimesters of pregnancy. DVT is three times more common than PE. In 85% of the cases, pregnancy-associated DVT occurs on the left side of the body, probably due to compression of the left iliac vein by the pregnant uterus.

Isolated pelvic DVT is much more common in pregnancy or in early postpartum period than in non-pregnant patients. All three elements of Virchow’s triad (hypercoagulability, vascular damage, and venous stasis) are present in pregnancy and early postpartum period.

Neither clinical judgment nor clinical decision rules (e.g., Well’s score) that are commonly used in non-pregnant patients have proven ineffective for PE during pregnancy. For example, D-dimer levels rise gradually during pregnancy and then drop in immediate postpartum period but do not return to normal until 4-6 weeks postpartum.

A normal D-dimer level appears to have a high negative predictive value in patients with a low clinical suspicion for VTE, but false-positive levels are very common.

The decision to use imaging modalities associated with radiation exposure in pregnant patients is difficult due to concern of teratogenicity. Radiation exposure more than 1 Gy at any stage of the pregnancy may induce congenital malformations. Currently, the most commonly used diagnostic tests for PE are ventilation/perfusion scintigraphy (V/Q scan) and computed tomographic pulmonary angiography (CTPA):

  1. V/Q scan
    The fetal radiation dose of VQ/ scan is 0.00028-0.00051 Gy. The negative predictive value of a normal V/Q scan in pregnant patients is excellent. For a V/Q scan to be of predictive value, a normal chest X-ray is mandatory. With an underlying lung pathology, CTPA would be the modality of choice.
  2. CTPA
    The fetal radiation dose of CTPA is 0.000003-0.000131 Gy. Although the radiation dose to fetus is lower in CTPA compared to V/Q scanning, CTPA produces higher levels of radiation exposure to women’s breasts.

Breast cancer risk in 20-year-old women after a CTPA is estimated to be 1 event in 143 exposures, with a lifetime attributable risk of 0.7%. The risk for malignancy is inversely related to age at the time of exposure. Therefore, it is recommended that women use breast shields when undergoing CTPA to decrease radiation dose from 0.05-0.08 Gy to 0.02-0.06 Gy.

A major benefit of CTPA over other imaging modalities is that it often provides alternative diagnoses. Thus far, no studies have addressed the accuracy or outcome of CTPA in pregnant patients. However, technical limitations might produce poor vessel opacification in pregnant patients that simulates PE and leads to false-positive results. This appears to be more common when apparent subsegmental PEs are identified. Hence, it is usually recommended that subsegmental PEs identified on CTPA be confirmed with further testing (e.g., VQ scan or lower extremity ultrasonography) before making a final diagnosis.

Which test to choose?

V/Q scan is the option of choice for pregnant women provided there is no underlying chest X-ray abnormality to make interpretation difficult. The pregnant woman should be informed that there is a slight risk of increased childhood malignancies for her baby. But the risk of radiation to the mother is virtually negligible.

OPTION A : CTPA is the second best option if V/Q scan is not an option. The fetal radiation is almost zero, but there is a slight risk of breast cancer for the mother. The risk, however, can be reduced by shield protection of the breasts during the procedure.

OPTION B: Electrocardiography is among initial tests for exclusion of other causes of chest pain but it cannot diagnose PE.

OPTION C: Doppler ultrasound of both legs is the next best step for diagnosing suspected PE if there are signs suggestive of DVT. If a clot is detected, no further diagnostic test is required and the patient should be started on anticoagulation therapy. If no clot is found the routine pathway will be followed. This woman has no leg symptoms; therefore, Doppler ultrasound is unlikely to have diagnostic benefit for her.

OPTION D : Negative predictive value of a normal D-dimer level for exclusion of VTE including PE is high, especially when there is a low pretest probability; however, this test is not of much use in pregnant women because D-dimer levels progressively increase throughout pregnancy. After delivery, D-dimer levels drop but not to the baseline for 4-6 weeks.

  • UpToDate - Deep vein thrombosis and pulmonary embolism in pregnancy: Treatment * Medscape - PE in Pregnancy: A Complicated Diagnosis
  • Diagnostic Imaging Pathways - PE in pregnancy
  • Medscape - Pulmonary Embolism in Pregnancy - Workup
211
Q

A 35-year-old woman develops deep vein thrombosis (DVT) of her left lower leg at 20 weeks pregnancy. Which one of the following statements is true regarding DVT in pregnancy?

A. Decreased levels of protein C and S is a precipitating factor.
B. DVT of right leg is more common than left leg.
C. Decreased levels of factor V and Vll is a precipitating factor.
D. Deep vein thrombosis is a rare condition in pregnancy.
E. Warfarin is the treatment of choice.

A

A. Decreased levels of protein C and S is a precipitating factor

Pregnancy is considered a hypercoagulable state caused by a decrease in anticoagulation activity due to a drop in proteins C and S, and an increase in activity of pro-coagulants, factor V and Vll.

Such physiological changes make DVT a common complication in pregnancy with an incidence rate of 1 in 2200 of pregnancies in Australia.

In pregnant women, deep vein thrombosis occurs much more commonly in the left leg than in the right leg (85%) due to compression of the left iliac vein by the pregnant uterus.

Anticoagulant therapy with low-molecular weight heparin (LMWH) or unfractionated heparin is the treatment of choice in pregnant women with DVT.

Warfarin therapy is contraindicated throughout pregnancy due to association with fetal warfarin syndrome and increased risk of fetal intracranial hemorrhage in the second and third trimesters.

  • UpToDate - Deep vein thrombosis in pregnancy: Epidemiology, pathogenesis, and diagnosis * Medscape - PE in Pregnancy: A Complicated Diagnosis
212
Q

A 30-year-old woman developed deep vein thrombosis at 30 weeks pregnancy and was started on the therapeutic dose of low molecular weight heparin (LMWH). Now and at 34 weeks gestation, she is in your office for a follow-up visit. Her expected date of delivery is due in 4 weeks. Which one of the following would be the best advice for her today?

A. The LMWH should be replaced with unfractionated heparin.
B. Continue the LMWH until delivery.
C. Switch to warfarin and check INR in 2days.
D. Add warfarin to the LMWH.
E. Cease anticoagulation.

A

A. The LMWH should be replaced with unfractionated heparin

A patient with deep vein thrombosis during pregnancy requires 3-6 months of treatment with therapeutic dose of LMWH or subcutaneous unfractionated heparin (UFH). The duration of therapy depends on the site of the thrombosis.

LMWH may be associated with an increased risk of epidural hematoma in women receiving epidural anesthesiae in labor; therefore, it is recommended that it be switched to subcutaneous UFH at 36 weeks or sooner if delivery is expected earlier to make regional anesthesia at labor (if indicated) safe.

Subcutaneous UFH can be discontinued 24-36 hours prior to anticipated delivery or induction. If there is high risk of thromboembolism in the absence of anticoagulation, the patient can be started on intravenous UFH because it has a shorter half-life. Intravenous UFH can be discontinued 4-6 hours before delivery.

Prior to neuraxial anesthesia, an ATTP should be checked.

Anticoagulation can be restarted with UFH or LMWH 4 to 6 hours after vaginal delivery or 6-12 hours after cesarean delivery. If a neuraxial catheter was used, anticoagulation should not be restarted for at least 12 hours after it is removed.

  • Medscape - Thromboembolism in Pregnancy
  • UpToDate - Deep vein thrombosis and pulmonary embolism in pregnancy: Treatment
213
Q

Warfarin is contraindicated during pregnancy. Which of the following complications develop if warfarin is used in second trimester of pregnancy?

A. Fetal chondrodysplasia punctata.
B. Hypercalcemia.
C. Facial anomalies.
D. Maternal cerebral bleeding.
E. Fetal optic atrophy.

A

A. Fetal chondrodysplasia punctata

Warfarin crosses the placenta and should be avoided throughout pregnancy, especially during the first and third trimesters.

Warfarin at 6-12 weeks’ gestation results in fetal warfarin syndrome characterized by the following features:

-A characteristic nasal hypoplasia
-Short fingers with hypoplastic nails
-Calcified epiphyses, namely chondrodysplasia punctuta (evident by stippling of epiphyses on X-ray)
-Intellectual disability
-Low birth weight

Recent studies show the risk of fetal warfarin syndrome in babies of women who require warfarin throughout pregnancy is around 5%. The risk is dose dependent.

Later exposure (after 12 weeks) is associated with central nervous system anomalies including microcephaly, hydrocephalus, agenesis of corpus callosum, Dandy-Walker malformation (complete absence cerebellar vermis and enlarged fourth ventricle), and mental retardation, as well as eye anomalies such as optic atrophy, microphthalmia, and Peter anomaly (anterior segment dysgenesis). Blindness can be seen in newborns exposed to warfarin in all three trimesters. Perinatal intracranial hemorrhage and other major bleeding is another complication in neonates exposed to warfarin.

NOTE - There may be a place for mid-trimester warfarin in pregnant women with prosthetic heart valves, but this choice should be made only after full discussion of the implications with the patient. Unfractionated heparin and low molecular weight heparin are alternatives that do not cross the placenta.

  • SA Health - Cardiac disease in pregnancy
  • Australian Prescriber - Managing warfarin therapy in the community
214
Q

A 31-year-old woman develops deep vein thrombosis (DVT) one week after delivery. She is started on enoxaparin. You are planning to start her on warfarin on discharge, but the mother is reluctant to take warfarin because she wants to breastfeed her baby and she thinks that warfarin may cause problems to her child. Which of the following is correct about warfarin and breastfeeding?

A. She cannot breastfeed her baby as long as on any type of anticoagulation as this increases the risk of hemorrhage in the baby.
B. She can breastfeed her baby while she is on warfarin.
C. Warfarin is contraindicated in breastfeeding mothers.
D. She should take enoxaparin injections for six months before warfarin can be started.
E. She can go home without anticoagulation as breastfeeding of her baby is a priority.

A

B. She can breastfeed her baby while she is on warfarin

Low molecular weight heparin (enoxaparin) is the medication of choice for antenatal thromboprophylaxis.

After delivery, warfarin can be used safely as the anticoagulation method. Both warfarin and heparin are safe to use in breastfeeding mothers. These do not increase the risk of bleeding in the baby because only a very small amount is secreted in breast milk.

  • QLD Health - Guidelines for warfarin management in the community
215
Q

Of the following options, which one is NOT associated with an increased risk of preeclampsia?

A. Nulliparity.
B. History of preeclampsia in a first-degree relative.
C. Chronic rena ldisease.
D. Age between 18 and 40 years.
E. Prolonged intervals between pregnancies

A

D. Age between 18 and 40 years

Preeclampsia refers to the onset of hypertension and proteinuria after 20 weeks of gestation in a previously-normotensive woman.

Risk factors for pre-eclampsia include:

-Preeclampsia in a previous pregnancy
Family history of preeclampsia
-Poor outcome in a prior pregnancy (placental abruption, IUGR, fetal death in utero)
-Interdelivery interval greater than 10 years
-Nulliparity (8 times more risk)
-Pre-existing medical conditions or chronic hypertension
-Diabetes (pre-existing or gestational)
-Renal disease
-Thrombophilias e.g. protein C and S deficiency, antithrombin III deficiency, or -Factor V Leiden mutation
-Antiphospholipid syndrome
-Systemic lupus erythematosus
-Maternal age greater than or equal to 40 years
-Body Mass Index (BMI) greater than 30 kg/m2
-Multiple pregnancy
-Raised blood pressure at booking
-Gestational trophoblastic disease
-Fetal triploidy

Age between 18 and 40 years is associated with decreased risk of preeclampsia, not an increased risk.

NOTE - Previously, age 16 years or younger was considered a risk factor for preeclampsia; however, recent studies failed to establish a meaningful relationship.

  • Risk factors for pre-eclampsia at antenatal booking: systematic review of controlled studies
216
Q

A 20-year-old Asian woman with background history of primary pulmonary hypertension attends your clinic. She is planning to become pregnant in the next few months. She enjoys good health at the moment. Which one of the following would be the most appropriate advice?

A. Pulmonary hypertension becomes better during pregnancy.
B. If she becomes pregnant, termination of pregnancy is not required.
C. Pregnancy is contraindicated for her.
D. Maternal mortality in this setting is low.
E. Sudden death is a rare complication.

A

C. Pregnancy is contraindicated for her

Primary pulmonary hypertension is a contraindication to pregnancy and the patient should be advised on the risks of increased maternal mortality in the presence pulmonary hypertension.

With unwanted pregnancy, this woman should be encouraged to terminate the pregnancy, as pulmonary hypertension becomes worse in pregnancy and can result in the mother’s death. Sudden death secondary to hypotension is a common and feared complication of pulmonary hypertension during pregnancy.

  • SA Health - Cardiac disease in pregnancy
217
Q

Which one of the following is the most important single warning sign of diminishing blood volume within the first four hours postpartum?

A. Decrease in blood pressure.
B. Tachycardia.
C. Tachypnea.
D. Sweating.
E. Vomiting.

A

B. Tachycardia

For early recognition of postpartum hemorrhage, routine observation of all postpartum women for blood loss, fundal tone, blood pressure and pulse is carried out during the first four hours after delivery.

The single most important sign of diminished blood volume and mild shock is tachycardia, which often precedes a fall in blood pressure. Weakness, sweating and tachypnea may accompany. Vomiting is not of any significance for early recognition of shock.

*http://www.ranzcog.edu.au/college-statements-guide
*http://www.elsevieradvantage.com/samplechapters/97

218
Q

A 28-year-old woman presents for antenatal visit at 39 weeks gestation. On examination, the fetus is found to be in transverse lie. Which one of the following would be the most appropriate next step in management?

A. Immediate ultrasonography
B. Vaginal examination.
C. External cephalic version.
D. Lower segment cesarean section.
E. Ask her to come back in two weeks.

A

A. Immediate ultrasonography

Shoulder presentations, unstable lie, transverse lie and oblique lie may be detected in late pregnancy.

Contributing factors include:
-High parity
-Pendulous abdomen
-Placenta previa
-Polyhydramnios
-Pelvic inlet contracture and/or fetal macrosomia
-Uterine abnormalities (e.g. bicornuate uterus or uterine fibroids)
-Fetal anomaly (e.g. tumors of the neck or sacrum, hydrocephaly, abdominal distension) -Distended maternal urinary bladder
-Poorly formed lower segment
-Wrong dates i.e. more premature than appears
-Undiagnosed twins

The diagnosis of transverse lie is made clinically by the presence of a broad asymmetric uterus with a firm ballotable round head in one iliac fossa and a softer mass (buttock) in the other.

An ultrasonography is required as the next step to is if there is:
-Placenta previa
-Twins
-Fetal anomaly
-Pelvic tumor
-Polyhydramnios

When placenta previa has been excluded, a gentle cephalic version may be tried; in many cases, however, the fetus returns to its previous presentation. If the lie remains unstable, the expectant mother should be advised to report to hospital immediately once the labor starts. Those with unfavorbale social conditions may be admitted to the hospital for expectant management.

In some cases after the cephalic version, the head is held over the pelvic brim and with the head in place amniotomy is performed to let the head be pressed into the pelvis as the amniotic fluid is released. Cesarean section is another option.

If these presentations are found once the labor has been established, cesarean section is the preferred method.

*https://www.sahealth.sa.gov.au/wps/wcm/connect/249
*http://www.gpnotebook.co.uk/simplepage.cfm?ID=1697

219
Q

An 18-week pregnant woman is brought to the Emergency Department with complaint of sudden-onset severe frontal headache. On examination, she has a blood pressure of 80/60 mmHg, pulse of 110bpm, respiratory rate of 17 breaths per minute and temperature of 37.3°C. The rest of the examination is unremarkable. There is no neck stiffness, abdominal pain or tenderness, or uterine contractions. Which one of the following is the investigation more likely to establish the diagnosis?

A. Lumbar puncture.
B. CT scan of the head.
C. Ultrasonography.
D. MRI of the head.
E. Full blood exam (FBE) and liver function tests (LFT).

A

D. MRI of the head

With the clinical findings of frontal headache and hypotension in a pregnant woman, pituitary apoplexy should be considered as one of the most important differential diagnoses. Pituitary apoplexy is defined as sudden hemorrhage into the pituitary gland. Hemorrhage often occurs into a pituitary adenoma.

The most dramatic presentation of apoplexy is the sudden onset of excruciating headache, diplopia due to pressure on the oculomotor nerve, and hypopituitarism.

All pituitary hormonal deficiencies can occur, but a fall in ACTH and, consequently, cortisol is most serious because it can cause life-threatening hypotension.

In a series of 35 patients with pituitary apoplexy seen at one center, 97% had headache, 71% had visual field impairment, and 66% had decreased visual acuity. Only a minority had clinical manifestations of hormonal excess or deficiency, but there was biochemical evidence of gonadotropin deficiency in 79%, ACTH deficiency in 76%, and TSH deficiency in 50%.

CT scan and MRI can be used for detection of intra-pituitary hemorrhage; however, MRI is more sensitive for evaluation of the pituitary gland.

Hypopituitarism and the diplopia may improve after surgical decompression of the pituitary. Both problems may also improve spontaneously as the blood is absorbed over a course of weeks to months after the hemorrhage.
For hypotension, high-dose corticosteroids are the mainstay of therapy.

OPTION A and B : With headache and hypotension, subarachnoid hemorrhage can be another differential diagnoses, but headache of SAH tends to be much more severe (the worst headache in life) and often associated with neck stiffness. On the other hand, skull cannot accommodate enough blood to justify the hypotension and the tachycardia in this patient. CT scan of the head (non-contrast) and LP (if CT was inconclusive) are used if SAH is suspected.

OPTION C : Ultrasonography is of no use in assessment of this patient with a possible intracranial pathology.

OPTION E: Preeclampsia is defined as persistent hypertension and proteinuria after 20 weeks’ gestation. FBE, LFT, and urinalysis should be considered for excluding preeclampsia. This woman with hypotension is less likely to have preeclampsia.

  • RANZCOG - Pituitary apoplexy in pregnancy: a case report
  • Pituitary apoplexy in pregnancy: A case series and literature review
220
Q

A 31-year-old woman presents to your GP clinic for planning her mode of delivery at 38 weeks’ gestation. She insists that her mode of delivery be a caesarean section because she strongly believes that this is best for both her and her baby. She has had an uneventful pregnancy so far, and has no indications for cesarea. You fully explain about the risks of cesarean delivery and its complications to her and the baby, but she insists. Which one of the following statements is correct regarding this situation?

A. Comply with her wish because she is right about the safety of cesarean section.
B. She should take a second opinion from an obstetrician.
C. Refuse to refer her for cesarean delivery because it is not indicated.
D. Refer her to an obstetrician for cesarean delivery.
E. Ask her to bring her husband for more discussion on the issue.

A

B. She should take a second opinion from an obstetrician

The scenario is a case of cesarean delivery on maternal request (CDMR) that is defined as elective cesarean delivery for singleton pregnancy on maternal request at term in the absence of any medical or obstetrical indication.

In such cases and as a GP, if the patient is still adamant to undergo cesarean delivery, referral to an obstetrician for further counselling is the next best step in management.

If after full discussion the patient maintains a request for delivery by cesarean section, the obstetrician may consider either of the following:

-Agree to perform the cesarean section, providing the patient is able to demonstrate an understanding the risks and benefits.

-Decline to perform the cesarean section in circumstances where: (1) the obstetrician believes there are significant health concerns to the mother or baby if cesarean section is performed or (2) the patient appears to not have an understanding sufficient to enable informed consent to the procedure.

  • The Royal Australian and New Zealand College of Obstetricians and Gynaecologists - Caesarean Delivery on Maternal Request (CDMR) C-Obs 39
221
Q

A 32-year-old woman presents with placenta previa. You offer cesarean section as the most appropriate mode of delivery. She wants to know about the risks of cesarean delivery. Which one of the following is correct regarding cesarean section?

A. Decreased risk of maternal mortality.
B. Decreased need for repeated cesarean sections.
C. No damage to adjacent viscera.
D. Increased risk of adhesions.
E. Decreased risk of infections.

A

D. Increased risk of adhesions

Complications of caesarean section delivery may include:

  1. Maternal complications
    -Increased risk of maternal mortality
    -Increased risk of thromboembolism
    -Hemorrhage
    -Infection
    -Incidental surgical injuries
    -Extended hospitalization
    -More postpartum pain
    -Poor birth experience
    -Re-admission to the hospital
    -Adhesion formation
    -Infertility/subfertility
    -Uterine rupture in subsequent pregnancies -Abnormal placentation in subsequent pregnancies
  2. Neonatal complications
    -Increased risk of neonatal death
    -Increased neonatal respiratory problems
    -Asthma
    -Iatrogenic prematurity
    -Trauma especially lacerations with cuts (particularly in emergency cesarean sections) -Failure to breastfeed

Cesarean section is associated with increased risk of infections as a complication.

*http://www.medscape.org/viewarticle/512946_4

222
Q

A 34-year-old Aboriginal woman is found to be IgM positive, but IgG negative after exposure to a child with rubella during the first trimester of pregnancy. Rubella infection is confirmed with repeated serology testing showing not only a positive IgM, but also rising titers of IgG. Which one of the following would be the most appropriate next step in management?

A. Termination of the pregnancy.
B. Give natural human immunoglobulin (NHIG).
C. Check rubella serology again and get the test reported from another laboratory.
D. No action is required.
E. Give MMR vaccine.

A

A. Termination of the pregnancy

The positive IgM for rubella indicates active rubella infection. In such cases, repeating the test is indicated. If IgM still positive, diagnosis of rubella is confirmed. No further testing is required (option C).

If rubella infection occurs in the first trimester of pregnancy, the risk of fetal infection and damage is high (85% in the first 2 months of pregnancy, and 50 -80% in the third month); therefore, termination of pregnancy is usually recommended.

OPTION B : NHIG has been used in trials for prophylaxis and prevention of congenital rubella syndrome after contact to a case with rubella infection. The results have been discouranging; therefor, it is not advised for prophylaxis. This woman, on the other hand, has establsihed infection and NHIG wil not be of benefit as primary prevention.

OPTION E : Rubella-containing vaccines are contraindicated throughout pregnancy and shoud be avoided. Women who remain susceptible to rubella should receive MMR vaccine postpartum.MMR vaccine is a live attenuate vaccine and is contraindicated during pregnancy.

223
Q

A 52-year-old lady presents to your GP clinic for breast cancer screening. She has been on combined hormone replacement therapy (HRT) for the past year. Mammogram is the only available method of screening for breast cancer and because of HRT she is expected to have dense breast tissue. Which one of the following is the best recommendation to this patient regarding HRT and breast cancer screening?

A. Stop HRT six months before mammography.
B. Stop HRT 1 one year before mammography.
C. Continue HRT and perform mammography as recommended for other women.
D. Change combined HRT to estrogen-only hormone replacement therapy.
E. Decrease the dose of combined hormone replacement therapy.

A

C. Continue HRT and perform mammography as recommended for other women

About 10% of women on combined HRT will have extremely dense breasts; nonetheless, presently, there is not enough evidence to recommend that patients on combined hormone replacement therapy should stop HRT for any length of time before their mammography in an attempt to reduce breast density. Therefore, this patient should continue HRT and have mammography based on current recommendations for other women.

Women making decisions as to whether to commence combined HRT should be aware of the fact that HRT may lead to inaccurate mammography results. Furthermore, they should be informed about the increased risk of breast cancer if HRT extends beyond the recommended period (3 to 5 years).

224
Q

A 34-year-old multigravida, whose fetus is known to be in breech position, presents to the hospital for vaginal delivery. After spontaneous rupture of the membranes, you notice bradycardia and variable deceleration on fetal heart rate monitoring. Vaginal examination confirms cord prolapse; however the cord is still pulsating. Which one of the following would be the most appropriate next step in management?

A. Push the presenting part away, push the cord up as far as possible, and keep it back with digital pressure.
B. Continue the vaginal delivery.
C. Give oxytocin to enhance contractions.
D. Prophylactic antibiotics.
E. Fetal scalp blood sampling.

A

A. Push the presenting part away, push the cord up as far as possible, and keep it back with digital pressure

Cord prolapse presents with abnormalities in fetal heart rate patterns (bradycardia, variable decelerations, etc.). If these changes begin after the rupture of the membranes, either spontaneously or with amniotomy, cord prolapse should be strongly suspected.

Umbilical cord prolapse is abnormal position of the cord in front of the fetal presenting part, resulting in cord compression during labor and fetal hypoxemia.

Once the diagnosis of the cord prolapse is confirmed on pelvic examination and the cord is still pulsating (indicating the fetus is still alive), the presenting part should be pushed up with the examiner’s finger, followed by pushing the cord up as far as possible and maintaining the digital pressure.

The mother should be placed in knee-chest position and with the examiners hand in place and rushed to the operating room for emergency cesarean delivery.

With the child being unstable and severely hypoxemic (reflected by variable decelerations on heart rate monitoring) there is no place for vaginal delivery and emergency cesarean section should be performed.

225
Q

Which one of the following is the most common cause of cesarean section delivery in Australia?

A. Failed progress of labor.
B. Previous cesarean section.
C. Cephalopelvic disproportion.
D. Cord prolapse.
E. Fetal distress.

A

B. Previous cesarean section

The most common cause of performing a cesarean section in Australia is a history of previous cesarean section.

Other causes of performing cesarean section include:
1.Failed progress of the labor 2. Cephalopelvic disproportion
3. Cord prolapse
4. Fetal distress
5. Fetal malpresentation especially breech
6. Placenta previa
7. Failed induction of labor

226
Q

A 26-year-old woman with history of chronic immune thrombocytopenic purpura (ITP) presents to your clinic and seeks advice regarding pregnancy. She has a platelet count of 70000/mm3. Which one of the following is the correct statement in counselling her?

A. She should have splenectomy before pregnancy.
B. She should avoid pregnancy in the next 2 years.
C. She can become pregnant.
D. If she becomes pregnant, the mode of delivery should be cesarean section.
E. She cannot become pregnant while the platelet count is below 70000/mm3.

A

C. She can become pregnant

While uncommon, immune thrombocytopenia (ITP) is an important cause of thrombocytopenia in pregnant women either as a pre-existing condition or occurring at any time during pregnancy.

ITP developed in pregnancy is important to be distinguished from ‘incidental’ or gestational thrombocytopenia, which, is responsible for up to 80% of cases of thrombocytopenia in pregnant women, or from more serious conditions such as HELLP syndrome.

Gestational thrombocytopenia usually causes an approximately 10% decrease in platelet counts and is usually characterized by counts that are above 70,000/mm3. When platelets are lower than that, it is mostly ITP.

A diagnosis of ITP is usually made in 1-4% of cases of thrombocytopenia. ITP, either pre-exiting or developed during pregnancy, does not prevent a woman from becoming pregnant or safely delivering a healthy baby. The best advice for this woman is that she can become pregnant and ITP does not preclude vaginal delivery; however, the pregnancy is categorized as high risk and she will need extra care and probably treatment, especially around the delivery.

In pregnant women with ITP, treatment is usually not necessary as long as the platelet count is above 30,000, unless there is bleeding or easy bruising. For pregnant women with symptomatic platelet counts of above 30,000, or women with platelet count of below 30,000 regardless of symptoms, treatment either with prednisolone or intravenous immunoglobulin (IVIG) is required due to significant concern of bleeding, especially uterine bleeding.

It is important to maintain a platelet count that allows the pregnant woman with the condition to go through delivery. This safe count is at least 50,000. For regional anesethesia, a count between 70,000 and 100,000 is often demanded.

227
Q

The fetus of a 27-year-old woman is found to have breech presentation at 32 weeks gestation. She has gestational diabetes and assessment reveals macrosomia of the fetus. The fetus is otherwise normal. Which one of the following would be the best advice regarding the mode of delivery?

A. Induction of labor now.
B. Elective cesarean delivery.
C. Urgent cesarean delivery now.
D. Trial of vaginal delivery and then cesarean if needed.
E. Elective vaginal delivery.

A

B. Elective cesarean delivery

Contraindications to vaginal delivery in breech presentation are as follows:
-Cord presentation
-Any presentation other than frank or complete breech with a flexed or neutral head attitude
-Clinically inadequate maternal pelvis
-Fetal anomaly incompatible with vaginal delivery
-The fetal weight less than 2500 gr or over 3800 gr

Because of macrosomia of the baby, vaginal delivery is contraindicated and elective cesarean delivery would be the best advice.

NOTE - if ultrasound is not available for estimation of fetal weight or macrosomia, cesarean section is recommended.

The following are recommendations for vaginal delivery of a baby with breech presentation:

-Continuous electronic fetal heart monitoring is preferable in the first stage and mandatory in second stage of the labor.
-When membrane rupture, immediate vaginal examination is recommended to rule out prolapsed cord.
-In the absence of adequate progress in labor, cesarean section is advised.
-Induction of labor is not recommended for breech presentation.
-Oxytocin augmentation is acceptable in the presence of uterine dystocia.
-A passive second stage without active pushing may last up to 90 minutes, allowing the breech to descend adequately into the pelvis.
Once active phase starts, cesarean section is recommended if delivery is not imminent after 60 minutes.
-The active second stage of labor should take place in or near an operating room with equipment and personnel available to perform a timely cesarean section.
-A health care professional skilled in neonatal resuscitation should be present at the time of delivery.

228
Q

You are a resident medical officer in a high-risk pregnancy clinic. One of your patients with past obstetrical history of placenta previa and cesarean section is found to have breech presentation at 36 weeks gestation. Which one of the following is a risk factor for increased chance of term breech presentation?

A. Nulliparity.
B. Prior cesarean section.
C. Fetal anomalies.
D. Polyhydramnios.
E. All of the above.

A

E. All of the above

Contributing factors to breech presentation include the following:

  1. Maternal factors:
    -Nulliparity
    -Previous breech birth
    -Uterine (anatomical) anomaly
    -Placental abnormalities (previa, cornual)
    -Oligohydramnios
    -Polyhydramnios
    -Multiple pregnancy
    -Grand multiparity
  2. Fetal factors:
    -Extended fetal legs
    -Short umbilical cord
    -Early gestation
    -Fetal abnormality
    -Poor fetal growth

All the options are contributing factors to breech presentation.

229
Q

A 32-year-old woman is diagnosed with deep vein thrombosis (DVT) of the iliofemoral vein at 18 weeks gestation. Which one of the following is the next best step in management?

A. Prophylactic dose of low molecular weight heparin for 6 months.
B. Therapeutic dose of low molecular weight heparin for 3months.
C. Therapeutic dose of low molecular weight heparin for 6months.
D. Termination of the pregnancy.
E. Start her on warfarin and continue for 6 months with strict INR monitoring.

A

C. Therapeutic dose of low molecular weight heparin for 6months

As a general rule, any pregnant patient with DVT should undergo anticoagulation with therapeutic dose of low-molecular weight heparin (LMWH) for 6 months if the DVT is in veins above the knee and 3 months if the DVT has occurred below the knee. The treatment is then followed by prophylactic dose of LMWH until delivery. Anticoagulation is required to be continued for another 6 weeks postpartum. After delivery, warfarin can be used.

NOTE - LMWH should be changed with unfractionated heparin four weeks before the due date, as LMWH is more likely to be associated with spinal hematoma if the pregnant woman undergoes neuraxial anesthesia.

Termination of pregnancy is not required as this condition is manageable. Warfarin is contraindicated in pregnancy due to risk of warfarin embryopathy.

230
Q

Which one of the following is an indication for cervical cerclage at 14 weeks gestation?

A. Two or more consecutive previous second-trimester pregnancy losses.
B. Short cervix before 24 weeks gestation with no previous preterm birth.
C. One preterm birth before 34 weeks gestation.
D. Carcinoma of the cervix.
E. Vaginal polyp.

A

A. Two or more consecutive previous second-trimester pregnancy losses

Decision as to whether to perform a cervical cerclage is made based upon the following conditions:

  1. History-indicated: consider cervical cerclage at 12-14 weeks gestation for women with a history of:
    -
    Two or more second trimester
    pregnancy losses in the presence of cervical shortening
    -Each loss earlier than the previous pregnancy
  2. Ultrasound-indicated: perform cervical cerclage at 14-26 weeks if:
    -There is progressive cervical shortening on serial ultrasounds and the external os is closed
  3. Rescue-indicated: consider cerclage for women with a combination of:
    -Cervix dilated > 2 cm with no perceived uterine contractions
    -Premature cervical effacement > 50 %
    -Presence of pelvic pressure
    -Heavy mucoid vaginal discharge or bulging membranes through the cervical os

Short cervix before 24 weeks gestation without any history of preterm birth or miscarriages is not an indication for performing cerclage. Similarly carcinoma of the cervix and vaginal polyps are not indications to performing cerclage.

Cerclage removal should take place at 36-37 weeks, after fetal lung maturity has achieved, but before the usual onset of spontaneous labor that could result in avulsion of the sutures.

231
Q

A 23-year-old Aboriginal woman at 10 weeks gestation presents with 2-week history of nausea and vomiting. She feels dizzy. She has not seen any doctor during this period. On examination, she is dehydrated and her blood pressure is 95/60 mmHg with a drop in systolic blood pressure by more than 20% when she stands. She is unable to tolerate oral intake. Laboratory results show ketonuria. Which one of the following would be the next best step in management?

A. Start intravenous fluids and keep her nil by mouth.
B. Give metoclopramide and intravenous normal saline.
C. Give metoclopramide and intravenous dextrose.
D. Give ondansetron and intravenous fluids.
E. Give steroids and admit in intensive care unit.

A

B. Give metoclopramide and intravenous normal saline

The clinical picture is indicative of hyperemesis gravidarum with resultant hypovolemia and pre-shock state. This patient needs urgent fluid resuscitation and intravenous antiemetic medications. The first-line fluid for resuscitation is normal saline (0.9% NaCl) and the anti-emetic drug of choice in this situation is metoclopramide which is safe for use in pregnancy (category A).

Dextrose should be avoided because once the sugar in the solution is consumed a hypotonic fluid is left that leads to more hyponatraemic states and carries the risk of encephalopathy due to edema.

Ondansetron is the second-line antiemetic (category B1) and considered in the following situations:
-More refractory vomiting
-No response to metoclopramide
-Recurrent hospital admissions due to hyperemesis gravidarum

Steroids such as prednisolone are third-line medications used in resistant cases of hyperemesis gravidarum only after consultation with an expert in the field.

232
Q

Which of the following abnormalities is not seen in a pregnant woman with hyperemesis gravidarum?

A. Ketosis.
B. Hypothyroidism.
C. Hyponatremia.
D. Hypokalemia.
E. Hypochloremic alkalosis.

A

B. Hypothyroidism

Hyperemesis gravidarum affects many women, particularly in the first trimester of pregnancy. It presents with severe nausea and vomiting and may lead to following electrolyte and metabolic derangements:

Hyponatremia – caused by vomiting and GI loss
Hypokalemia – caused by vomiting and GI loss
Hypochloremic alkalosis – caused by vomiting and GI loss
Ketosis – resulting from decreased oral intake, starvation and dehydration Abnormal liver enzymes (ALT>AST)
Increased serum amylase and lipase

Beta hCG is a glycoprotein with structural similarities to the beta subunit of thyroid stimulating hormone (TSH). Significantly increased amounts of beta hCG, particularly around weeks 10 to 12 of pregnancy can cause subclinical hyperthyroidism and on rare occasions, overt hyperthyroidism. This condition is usually self-limiting, but occasionally anti-thyroid medications are required.

Hypothyroidism is not a feature of hyperemesis gravidarum.

233
Q

A 48-year-old woman presents complaining of hot flushes, insomnia and mood swings. She has been on sertraline for mood swings and also takes over-the-counter vitamins. Which one of the following is incorrect regarding menopause?

A. Decreased estrogen level.
B. Increased serum LH level.
C. Decreased serum FSH.
D. Increased LDL level.
E. Decreased bone density.

A

C. Decreased serum FSH

Menopause is characterized by irregular menstrual cycles and marked hormonal fluctuations, often accompanied by hot flushes, sleep disturbances, mood symptoms, and vaginal dryness.

Eventual depletion of oocytes and ovarian follicles in the ovaries result in drop in estrogen level and elevation of serum FSH and LH levels. Decrease in serum estrogen results in the characteristic features of menopause. Long term effects of low estrogen include osteoporosis and vaginal atrophy. After menopause there is a rise in LDL level.

Serum FSH is elevated (not decreased) in perimenopausal period and causes hot flushes.

234
Q

Which one of the following is the most common symptoms of hyperemesis gravidarum?

A. Nausea.
B. Vomiting.
C. Fever.
D. Headache.
E. Dizziness.

A

A. Nausea

The most common symptom of hyperemesis gravidarum is nausea. It occurs in 80% to 85% of the cases. Vomiting is present in about 50% of the cases.

The peak severity for hyperemesis is around 12 weeks (when beta hCG is at its peak). The symptoms resolve in most of women by 20 weeks, but 10% will continue to have the problem throughout pregnancy.

Hyperemesis gravidarum is a severe form of nausea and vomiting occuring in 0.3%1 to 1.5% of pregnancies.
Excessive vomiting may lead to the following:
-Dehydration
-Ketosis
-Electrolyte imbalance
-Vitamin deficiency in pregnancy (very rare)

The term ‘hyperemesis’, however, should be used only where one or more of the following exist:
-Persistent symptoms that have led to attendance at the hospital, and the need for intravenous (IV) therapy
-Weight loss of > 4 kg (or >5%) since conception associated with persistent vomiting/anorexia
-Lack of response to usual antiemetic and other medications

235
Q

A 26-year-old lady at 18 weeks gestation presents with clear vaginal discharge. She has the history of preterm labor at 24 weeks in her last pregnancy. On examination, you notice clear fluid coming out of the vagina. Which one of the following is the best predictor of preterm labour?

A. Cervical length of 15mm.
B. Cervical length of 40mm.
C. Elevated serum beta hCG.
D. Fibronectin test.
E. Abdominal pain.

A

A. Cervical length of 15mm

Measurement of cervical length by transvaginal ultrasound is the best way to predict preterm delivery between 14 and 24 weeks gestation. The predictive value is even higher between 20 and 24 weeks of pregnancy.

At 20 weeks, the cervical length is expected to be 40mm. This length is decreased to 34mm at 34 weeks gestation. A cervical length of 15 mm is definitely short for this gestational age and is associated with increased risk of pregnancy loss.

Cervical length screening is required at least twice between 14-24 weeks gestation in the following situations to predict preterm labor:
-Prior preterm birth less than 34 weeks gestation
-Previous cervical cone biopsy
-Women with suspected cervical incompetency
-Multiple pregnancies

Fetal fibronectin (fFN) level is another test that could be performed to predict the chance of preterm labor.
Candidates for testing should meet the following criteria:
1. Intact fetal membranes
2. Cervical dilation less than 3 cm
3. Gestational age of between 22+ 0d and 34+6d weeks

The absence of fetal fibronectin (fFN) in the cervical secretions is a very useful negative predictor of imminent birth (negative predictive value for birth within 7 days 97-98%). By negative prediction its meant that with a negative test the preterm labor is unlikely, but a positive test does not mean that preterm labor will occur.

NOTE - fFN is not useful for prediction of preterm labor if performed earlier than 22 weeks.

A cervical length, on the other hand, is a good positive predictor, meaning that with a cervical length of more than 30 mm the preterm labor is unlikely. In cases of threatened preterm labor, a threshold of 30 mm has been consistently reported to exclude preterm labor, but there is no threshold of cervical length that establishes the diagnosis. In women with contractions and cervical length less than 30 mm, fetal fibronectin may help predict the patient’s risk of preterm delivery within the next seven days.

OPTION B : A cervical length of 40mm is quite normal and against preterm labor as a diagnosis.

OPTION D: fFN test is not an appropriate option here because the gestational age is less than 22 weeks and secondly, clear vaginal discharge might be due to rupture of membranes and leaking amniotic fluid. This excludes the patient as a potential candidate.

OPTION C : Elevated beta hCG is not helpful in predicting miscarriage in second trimester of pregnancy.

OPTION E : Premenstrual-like cramping lower abdominal pain is one of the features seen in cervical insufficiency, but it is very nonspecific with no predictive value for miscarriage.

*http://www.kemh.health.wa.gov.au/development/manua
*http://www.uptodate.com/contents/fetal-fibronectin

236
Q

What is the diagnosis for this child?

A 7-year-old boy is brought to your clinic with a 2-week history of paroxysms of cough and vomiting at the end of coughing episodes during school holidays. Cough is dry and is worse at night. His mother is worried whether he should go to school, which is going to start in 2 days. Which one of the following would be the most appropriate approach regarding this situation?

A. No exclusion is required.
B. Exclude from school.
C. Exclude from the school and start antibiotics.
D. The child can go to school after antibiotics are started.
E. The decision depends on the child’s immunization status.

A

A. No exclusion is required.

The clinical picture is consistent with whooping cough (pertussis) caused by Bordetella pertussis. The incubation period is of 7 to 14 days (up to 20 days), and the disease then appears with the following three stages:
1. Catarrhal – Cough and coryza for one week
2. Paroxysmal – more pronounced cough in spells or paroxysms
3. Convalescence

WHEN IS ANTIBIOTICS INDICATED?
Antibiotics for the case with pertussis (index case) are indicated in the following situations:

  1. The patient is diagnosed in catarrhal or early paroxysmal phase (may reduce severity)
  2. Cough for less than 14 days (may reduce spread; reduces school exclusion period)
  3. The patient is admitted to the hospital
  4. There are complications (e.g., pneumonia, cyanosis, apnea)
Exclusion of the case and close contacts are based on the following recommendations from National Health and Medical Research Council (NHMRC) and Australian Health Department:

Exclusion of the index case: 

* exclude **until 5 days** after starting appropriate antibiotic treatment, OR 
* for **21 days** from the onset of any cough (catarrhal phase), OR
* 14 days after the onset of paroxysmal cough if the date is known.

Exclusion of close contacts: unimmunized (< 3 doses) household and close childcare contacts less than 7 years of age must be excluded from school or child care for 14 days from the last exposure to infection OR until they have taken 5 days of effective antibiotics for prophylaxis.

Since school time is after two weeks (14 days) of the date the cough paroxysm has been started, the case is not infectious and no exclusion from school will be require Antibiotics would have been indicated and could have reduced exclusion period if the presentation had been within 14 days of cough. With 14 days of paroxysmal cough (that would be 21 days of cough of catarrhal phase) this child is at the end of the infectious period (21 days).

Immunization status guides exclusion of the close contacts from school or childcare settings, not that of the index case.

237
Q

A 35-year-old woman presents with regular uterine contractions at 34 weeks gestation. Sterile speculum examination is performed showing cervical dilation of 2 cm and intact membranes. Which one of the following investigations is the best negative predictor of imminent birth within the next 7 days?

A. Transvaginal ultrasound.
B. Mid-stream urine sample.
C. The absence of fetal fibronectin in cervical secretions.
D. The presence of fetal fibronectinin cervical secretions.
E. Irregular uterine contractions.

A

C. The absence of fetal fibronectin in cervical secretions

Preterm labor is defined as regular uterine contractions after 20 weeks but before 37 weeks gestation, associated with cervical shortening and effacement.

Risk factors for preterm labour include:
-Infections
-Antepartum hemorrhage
-Uterine distension
-Spontaneous rupture of membranes

Fetal fibronectin (fFN) is thought to be a “trophoblast glue” that promotes cellular adhesion at uterine-placental and decidual-fetal membrane interfaces. It is released into cervicovaginal secretions when the extracellular matrix of the chorionic/decidual interface is disrupted; this is the rationale for measurement of fFN as a predictor of PTD.

Absence of fFN in cervical secretions is the most useful negative predictor of imminent birth within the next 7 days. This test is performed at bedside using a commercial test kit according to instructions which are provided by the kit. It is very expensive but useful.

Candidates for fibronectin test are pregnant women who meet the following criteria:
1.Intact fetal membranes
2.Cervical dilation less than 3 cm
3.Gestational age of between 22 +0d – 34+6d weeks

At 34 weeks gestation, with a cervical dilation of 2 cm and intact membrane fibronectin test is the best negative predictor, meaning that with a negative test, preterm labor within the next 7 days would be unlikely. In the presence of fFN (positive test), one cannot tell with certainty that preterm labor will follow.

Other steps to take are obtaining vaginal swabs and midstream urine for evaluation of vaginal infection and colonization with group B streptococcus. Ultrasonography is performed to assess cervical length, placental position and presentation before the fFN is performed. A sterile speculum examination should be considered to exclude rupture of membranes.

*http://www.kemh.health.wa.gov.au/development/manua
*http://www.uptodate.com/contents/fetal-fibronectin

238
Q

A 28-year-old pregnant woman is involved in a car accident at 26 weeks gestation, while wearing seatbelt. On examination, there are visible bruises on the abdomen. She is otherwise normal. Fetal heart sounds are audible and within normal parameters and CTG is reassuring. Which one of the following is the next best step in management?

A. Reassure and discharge home.
B. Perform 24-hour cardiotocography (CTG) monitoring.
C. Admit her and observe for 24hours.
D. Perform a continuous 30-minute CTG and discharge home if reassuring.
E. Observe for six hours and discharge home.

A

C. Admit her and observe for 24hours

Trauma is a major contributor to maternal mortality, and is the leading cause of pregnancy-associated maternal deaths.

Maternal trauma can compromise the fetus as a result of maternal hypotension or hypoxemia, placental abruption, uterine rupture, or fetal trauma.

With bruises on the abdomen (seatbelt marks) this woman has positioned the seat belt incorrectly over the uterus and there is a good chance that the uterus and its contents, including the fetus, has been affected by the impact. The correct position of the seat belt is when the lap belt is placed on the hip below the uterus and the sash between breasts and above the uterus.

Apart from routine trauma workup necessary for non-pregnant patients, a minimum 24-hour period of monitoring is recommended for all pregnant women who have sustained trauma if any of the following is present:
-Regular uterine contractions
-Vaginal bleeding
-A non-reassuring fetal heart rate tracing
-Abdominal/uterine pain
-Significant trauma to the abdomen

With bruises over the abdomen this patient should be considered as having significant abdominal trauma and observation for a minimum of 24 hours, should be considered. These patients should not be discharged until the clinician reasonably makes sure that they do not have an abruption or preterm labor.

*http://www.kemh.health.wa.gov.au/development/manua
*http://www.health.qld.gov.au/qcg/documents/g-traum
*https://www.sahealth.sa.gov.au/wps/wcm/connect/abd

Correct (green) vs. incorrect position of seat belt for pregnant women
239
Q

A 20-year-old woman presents to the emergency department with premature uterine contractions. He is 28 weeks pregnant and has the history of untreated mitral valve stenosis. After necessary evaluation and absence of contraindications, tocolysis is planned. Which one of the following would be the drug of choice for tocolysis?

A. Nifedipine.
B. Salbutamol.
C. Betablockers.
D. Magnesium sulphate.
E. Oxytocin antagonists.

A

E. Oxytocin antagonists

First-line medication for tocolysis is the calcium channel blocker nifedipine.
Beta agonists such as terbutaline and salbutamol are second-line choice.
As this patient has untreated mitral valve stenosis, nifedipine and beta agonists are contraindicated.

Nifedipine, (option A) by decreasing peripheral vascular resistance leads to a decrease in an already diminished cardiac output. Beta agonists do the same by inducing tachycardia.

Oxytocin is a hormone which increases uterine contractions. Oxytocin can be counteracted by atosiban. The counteraction is achieved through competitive binding of atosiban to oxytocin and vasopressin receptors, and by this, suppressing uterine contractions. It has about the same efficacy of nifedipine. This drug is the option of choice when other tocolytic agents such as calcium channel blockers or beta agonists are contraindicated.

Contraindications to oxytocin antagonists are generally the same for tocolysis and include:
-Gestational age below 24 or over 33 completed weeks
-Premature rupture of the membranes at >30 weeks of gestation
-Intrauterine growth retardation
-Abnormal fetal heart rate
-Antepartum uterine haemorrhage requiring immediate delivery
-Eclampsia and severe pre-eclampsia requiring delivery
-Intrauterine fetal death
-Suspected intrauterine infection
-Placenta praevia or abruptio placenta
-Any other conditions of the mother, or fetus, where continuation of pregnancy is hazardous
-Any known hypersensitivity to the active substance or any of the ingredients

Athough magnesium sulfate acts as a calcium antagonist, and inhibits the muscle contractions of the uterus, studies have shown that magnesium sulfate (previously used as a tocolytic agent) is unlikely to stop preterm labor. But if it is given to women in preterm labor, who are less than 32 weeks pregnant, it may help reduce the risk of cerebral palsy in preterm babies.

Beta blockers are not used to prevent uterine contractions.

*http://www.ncbi.nlm.nih.gov/pmc/articles/PMC297173 *http://www.kemh.health.wa.gov.au/development/manua
*http://www0.health.nsw.gov.au/policies/pd/2011/pdf
*http://www.rcpi.ie/content/docs/0000

240
Q

A fully breast-feed baby of a mother on a healthy, balanced diet could have the deficiency of which one of the following vitamins?

A. Vitamin A.
B. Vitamin B complex.
C. Vitamin C.
D. Vitamin D
E. Vitamin K

A

E. Vitamin K

Breast milk lacks or has insufficient vitamin K, but as this vitamin is given intramuscularly at birth, there is no more need for supplementation. Without supplementaion at birth, the baby may suffer vitamin K deficiency and develop severe complications such as intracranial hemorrhage. Vitamin K deficiency is more common among premature babies and those whose mothers have been on antiepileptic drugs.

241
Q

A 34-year-old woman from country-side of Victoria presents to the hospital at 37 weeks gestation after she noticed a sudden gush of clear fluid per vagina. Uterine contractions are absent. Speculum examination shows pooling of liquor in the posterior fornix. After 12 hours, she develops fever, tachycardia and chills. Apart from giving antibiotics, which one of the following would be the next best step in management?

A. Perform vaginal examination.
B. Continue the pregnancy until natural delivery.
C. Induce labour now.
D. Give corticosteroids.
E. Discharge from hospital after 14 days of antibiotics.

A

C. Induce labour now

This patient presented with premature rupture of membranes. Premature rupture of membranes (PROM) refers to membrane rupture before the onset of uterine contractions.

The classic clinical presentation of premature rupture of membranes is a sudden gush of clear or pale yellow fluid from the vagina. This patient also has developed signs of infection (chorioamnionitis) indicated by fever, tachycardia and chills. Once chorioamnionitis develops the induction of labour (IOL) should be considered as soon as possible.

Vaginal examination should be avoided in PROM unless delivery is imminent. Continuation of the pregnancy carries the significant risk of maternal and fetal jeopardy.

Corticosteroids are given up to 34 weeks gestation. At 37 weeks gestation maternal systemic corticosteroids will have no effect on fetal lung maturity. In the presence of chorioamnionitis where immediate induction of labour is indicated, corticosteroids would be of no use even if the gestational age was under 34 weeks.

242
Q

A 28 weeks pregnant woman comes to your clinic with presenting complaint of a gush of clear vaginal fluid. On speculum examination, premature preterm rupture of membranes (PPROM) is confirmed. The cervical os is closed. In addition to transfer to a tertiary care center, which one of the following would be the most appropriate management?

A. Systemic corticoseroids.
B. Nifedipine.
C. Salbutamol.
D. Cardiotocography.
E. Pelvic ultrasound.

A

A. Systemic corticoseroids

Premature preterm rupture of membranes (PPROM) is defined as spontaneous rupture of the membranes before the onset of labour prior to 37 weeks gestation.

The etiology is multifactorial. Risk factors for PPROM include:
-Intra-amniotic infections – the most common cause
-Placental abruption
-Invasive uterine procedures e.g. amniocentesis, cordocentesis, chorionic villus sampling, cervical cerclage

Typically, women with PPROM present with a large gush or steady trickle of clear vaginal fluid. The clinical signs of PPROM may become less accurate after one hour.

The interval between PPROM and the onset of labor is influenced by many factors including gestational age. Women with PPROM have a 50% chance of going into labor within 24 to 48 hours. The chance increases to70 to 90% chance within 7 days. If PPROM occurs between 24 and 28 weeks gestation the latency period before birth is generally longer than if occurring closer to term.

Perinatal complications of POM include:
-Respiratory distress syndrome -Infections
-Fetal intraventricular hemorrhage -Pulmonary hypoplasia
-Skeletal deformities
-Cord prolapse
-Malpresentation

PPROM is diagnosed by sterile speculum exam and the following criteria:
i) Positive pooling – there is pooling of fluid in the posterior fornix
ii) Positive nitrazine test – the pH paper will turn blue with the fluid
iii) Positive fern test – the fluid shows a ferning pattern when the fluid is allowed to air-dry and seen under light microscope

Management
In the absence of chorioamnionitis the management depends on the gestational age:

  1. Before 23 weeks
    The labor may be induced or the patient sent home for bed rest for signs of spontaneous delivery to start.
  2. Between 23 and 34 + 0/7 weeks
    The patient should be transferred to a tertiary hospital and be admitted - while being transferred to the tertiary hospital, it is important to administer systemic corticosteroids for fetal lung maturity.
    -Bed rest
    -Cervical and vaginal swabs for microscopy and culture
    -Prophylactic antibiotics for prevention of chorioamnionitis

Tips for management of PROM

1-Digital vaginal examination should be avoided unless the woman is in active labor or birth is imminent.

2-Between 23 and 23+6/7 weeks gestation the decision for corticosteroids administration is made after consultation with obstetric/pediatric medical team and the parents. Corticosteroids are not effective in promoting long maturation before 23 weeks.

3-A single course of antenatal corticosteroids should be administered to women with PPROM without signs of infection between 23 and 34 weeks gestation. In the presence of chorioamnionitis prompt delivery is indicated and there is not enough time for corticosteroids to be effective on lung maturity.

4-If gestation is less than 34 weeks and in the absence of infection or complications and in circumstances when a course of corticosteroids has not been completed, tocolysis may be considered for threatened premature labor.

5-Broad spectrum antibiotic administration is recommended following PPROM to prevent infection and prolongation of the pregnancy in the short term.

6-It is the obstetric consultant’s decision as to when to deliver a preterm baby. If expectant management continues >34weeks, women should be advised of the increased risk for chorioamnionitis and the decreased risk of respiratory problems in the neonate.

NOTE - chorioamnionitis is an absolute indication to terminate the pregnancy regardless of the gestational age

At 28 weeks gestation, this patient should be immediately transferred to a tertiary hospital. She should also receive systemic steroids to promote fetal lung maturation in case preterm delivery ensues.

*https://www.ranzcog.edu.au/Statements-Guidelines/Obstetrics/Preterm-Prelabour-Rupture-of-Membranes

243
Q

Which one of the following is not a contraindication to tocolysis?

A. A fetus with congenital malformations inconsistent with life.
B. Suspected fetal compromise determined by cardiotocography.
C. Fetal death in utero.
D. Placental abruption.
E. Maternal hypothyroidism.

A

E. Maternal hypothyroidism

The following are the contraindications to tocolysis in preterm labor:

-Gestation > 34 weeks or <24 weeks
-Labor is too advanced – advanced cervical dilation (>4cm)
-Chorioamnionitis
-In utero fetal death
-Abnormal CTG suggesting non-reassuring fetal status
-Lethal fetal anomalies
-Intrauterine fetal demise
-Suspected fetal compromise
-Significant antepartum hemorrhage, such as placental abruption/ active vaginal bleeding
-Suspected intrauterine infections (e.g. chorioamnionitis)
-Maternal hypotension
-Hemorrhage with hemodynamic instability
-Pregnancy-induced hypertension/ eclampsia/ pre-eclampsia
-Placenta previa
-Plancental insufficiency
-Intrauterine growth retardation
-Severe preeclampsia
-Maternal allergy to specific tocolytic agents, or where tocolytics are contraindicated due to specific co-morbidities (e.g. beta agonists should not be given in case of cardiac disease)

Known or suspected fetal pulmonary maturity is not an absolute contraindication to tocolysis, as there are nonpulmonary morbidities associated with preterm birth. These fetuses could potentially benefit from prolongation of pregnancy and the nonpulmonary benefits of glucocorticoid therapy.

Inhibition of preterm labor is less likely to be successful when cervical dilation is greater than 3 cm. Tocolysis can still be considered in these cases, especially when the goal is to administer antenatal corticosteroids or safely transport the mother to a tertiary care center.

Maternal hypothyroidism is usually treated with thyroxine and is not a contraindication to labor suppression.

*http://www.health.qld.gov.au/qcg/documents/ed-ptl.
*http://www.uptodate.com/contents/inhibition-of-acu

244
Q

Jane, a 65-year-old patient of yours, has presented to you with complaint of uterine prolapse. Which one of the following ligaments has the most important role in pathophysiology of uterine prolapse?

A. Broad ligament.
B. Round ligament.
C. Uterosacral ligament.
D. Ovarian ligament.
E. Levator ani muscle.

A

C. Uterosacral ligament

The uterus is normally held in place by surrounding and attached muscles and ligaments. Prolapse happens when the ligaments supporting the uterus become so incompetent that uterus cannot stay in place and slips down from its normal anatomic position. These ligaments are:
1. Round ligament
2. Uterosacral ligament
3. Broad ligament
4. Ovarian ligament

Of these, incompetency of the uterosacral ligament has the most important role in pathophysiology of uterine prolapse.

245
Q

A 36-year-old woman with asthma and a 5-year history of hypertension is keen to become pregnant. She has never been pregnant before and has stopped using contraception methods recently. Her only currennt medication is ramipril 10 mg daily. Her blood pressure is 130/85. Which one of the following is the most appropriate initial management of her hypertension?

A. Cease ramipril and start her on methyldopa.
B. Cease ramipril and start her on atenolol.
C. Cease ramipril and start her on irbesartan.
D. Cease the ramipril and start her on hydralazine.
E. Continue ramipril.

A

A. Cease ramipril and start her on methyldopa

Some commonly prescribed anti-hypertensive drugs are contraindicated, or are better avoided before conception and during pregnancy. These drugs include:

-Angiotensin converting enzyme (ACE) inhibitors e.g. ramipril, captopril
-Angiotensin receptor blockers (ARBs) e.g. losartan, irbesartan
-Diuretics
-Most beta blockers

Methyldopa is the first-line antihypertensive agent during pregnancy for controlling mild to moderate hypertension during pregnancy.

OPTION A : Beta blockers can cause fetal bradycardia and with long-term use result in growth restriction. Labetolol is an exception and can be used in hypertensive emergencies.

OPTION C and E : Angiotensin receptor blockers and ACE inhibitors both are teratogenic in first trimester. In the second and third trimesters they can result in fetal renal dysfunction, oligohydramnios and skull hypoplasia.

OPTION D : Hydralazine can be safely used for treatment of hypertensive emergencies during pregnancy.

Otherwise, methyldopa remains the choice.

Other anithypertensice medications:
Diuretics - Diuretics can cause fetal electrolyte disturbances and reduction in maternal blood volume. This can consequently lead to restricted fetal growth.

Calcium channel blockers (CCBs) - with the exception of nifedipine, CCBs should be avoided in pregnancy due to risk of maternal hypotension and fetal hypoxia. Nifedipine is commonly used as first-line tocolytic agent, but is not used as an antihypertensive medication during pregnancy.

NOTE - Hydralazine and labetalol are safe to use during pregnancy. However, these two drugs are used for treatment of hypertensive emergencies. For routine management of hypertension in pregnant women or in those who are planning pregnancy, methyldopa is the first-line option.

  • Therapeutic Guidelines – Cardiovascular: available from http://tg.org.au
246
Q

A 26-year-old woman presents for her first antenatal visit at 10th week of her pregnancy. On history, her mother has type II diabetes mellitus. Which one of the following would be the recommended screening test for gestational diabetes mellitus (GDM) for her?

A. 2-hour 75gr OGTT between 24 and 28weeks.
B. 2-hour 75gr OGTT now.
C. Fasting blood glucose (FBS) now.
D. Fasting blood glucose (FBS) between 24 and 28weeks.
E. 3-hour 100 gr OGTT between 24 and 28 weeks.

A

B. 2-hour 75gr OGTT now

All women should be assessed for risk of developing gestational diabetes mellitus in the first antenatal visit. The risk index is calculated according to the following table: see photo below.

Risk factors Score
i) BMI (pre-pregnancy or first antenatal visit)
BMI: 25-35 kg/m2 (1)
BMI>35 kg/m2 (2)
ii) Asian, Indian subcontinent, Aboriginal, Torres Strait Islander, Pacific Islander, Maori, Middle Eastern, non-white African (1)
iii) Previous GDM (2)
iv) Previous elevated blood glucose (2)
v) Maternal age≥40 years (2)
vi) Family history DM: (first degree relative with diabetes) or sister with GDM (2)
vii) Previous macrosomia: (birth weight > 4500 g or > 90th percentile) (2)
8) Previous perinatal loss (2)
9) Polycystic ovarian syndrome (2)
10) Medications (corticosteroids, antipsychotics) (2)

Every woman with a total score of 2 or greater is high risk and should have a 2-hour 75-gr oral glucose tolerance test (OGTT) now. If normal, another 2-hour 75-gr OGTT should be performed between 24-28 weeks of pregnancy.

With an abnormal 2-hour OGTT test, the patient should be advised to repeat the second 2-hour 75 gr OGTT after 10 hours of fasting while on her regular medications (if any). Pregnant women whose risk score is less than 2 should have their first 2-hour 75gr OGTT at 24-28 weeks gestation.

With a positive family history for diabetes of her mother this patient has a total score of 2; therefore she should undergo 2-hour 75 gr OGTT now.

  • RACGP - AJGP - Gestational diabetes mellitus: A pragmatic approach to diagnosis and management
  • ADIPS Consensus Guidelines for the Testing and Diagnosis of Gestational Diabetes Mellitus in Australia
247
Q

A 26-year-old woman with menstrual cycles ranging from 4 to 6 weeks, presents with vaginal bleeding following amenorrhea of 6 weeks duration, which is heavier than ever and not like her previous periods. A urine pregnancy test is positive. On vaginal examination, the cervical os is closed. An endovaginal ultrasound scan is performed that reveals an endometrial thickness of 12 mm and an empty uterus. Adenexa are clear and there is no fluid in the pouch of Douglas. There is a corpus luteum cyst in the left ovary. Which one of the following would be the most likely diagnosis?

A. Ectopic pregnancy.
B. False positive pregnancy test result.
C. Complete abortion.
D. Incomplete abortion.
E. Complicated corpus luteam cyst.

A

C. Complete abortion

A positive pregnancy test, while the uterus is empty on ultrasound exam can be due to either of the following:
-A false positive pregnancy test
-Complete abortion
-Ectopic pregnancy
-Early normal pregnancy

With an endometrial thickness of 12 mm on ultrasoound, false positive pregnancy test result is less likely. If the woman is not pregnant and she is experiencing just a normal menstruation at the 6th week (as she often does according to the history) the endometrial thickness would be expected to be 2-4 mm.

Several studies have been conducted to see whether measuring endometrial thickness would be helpful for telling ectopic pregnancy, early normal pregnancy, and complete abortion apart, but no meaningful results are yet obtained. In fact, the endometrial thickness could be similar in all these conditions; therefore, the only condition this parameter excludes is a false negative result. Considering the fact that a corpus luteum cyst is found on ultrasound, false positive result would become even less likely.

After ovulation, the remnant of the follicle,turns into a temporary endocrine structure that produces progesterone (pro=in favor of + gesterone= pregnancy) - an essential hormone for egg implantation and maintaining the pregnancy until the placenta fully takes over this function by 10-12 weeks gestation.

Corpus luteal cyst may internally or externally bleed and cause pain and peritoneal irritation (resembling acute abdomen). Ovarian torsion is another very serious complication but the clinical picture would be quite different.

If pregnancy does not take place, the copus luteum involutes and disappears; otherwise, it grows in size and persists. If the patient was not pregnant, the corpus luteum cyst would be expected to have resolved by the time she starts her menstruation. Since it has persisted, it is more likely that pregnancy has occurred.
With clear adenexa on ultrasound, ectopic pregnancy is less likely but not excluded, and should be thoroughly investigated. However, the question asks about the ‘most likely diagnosis’ not the ‘most appropriate management’.

The prevalence of ectopic pregnancies in terms of the location is as follow:
i) Distal tubal (ampullary) – 80%
ii) Isthmic – 12%
iii) Fimbrial – 5%
iv) Cornual/interstitial – 2%
v) Abdominal – 1.4%
vi) Ovarian – 0.2%
vii) Cervical – 0.2%

With clear adenexa almost 97% of EPs are excluded; however, since ultrasound is highly operator dependent, ectopic pregnancy cannot be cleared with certainty. It shoudl be noted that in uncomplicated ectopic pregnancies there is no fluid collection in the pouch of Douglas; therefore, an empty pouch does not exclude ectopic pregnancy.
The closed OS, excludes incomplete abortion because OS will not completely close unless all the products of pregnancy are expelled and uterine contractions have stopped.

Since the US findings do not favor EP, of the options, complete abortion remains the most likely diagnosis. This does not mean that EP is ruled out whatsoever. In fact what was seen as corpus luteum cyst could be an ovarian EP that has been misinterpreted but comparing to complete abortion that would far less likely.

TOPIC REVIEW

Common causes of falsely positive pregnancy test include:
-Dirty urine collecting cup (detergent residue for example is known to cause false-positive results)
-Faulty test kit (for example, the kit is damaged, past its use-by date or has been exposed to heat or moisture)
-Blood in the urine (e.g. from cystitis)
-Protein in the urine (e.g. from kidney damage)
-Certain medications including anti-convulsants, some fertility drugs, diuretics and tranquillizers
-Recent birth or miscarriage -a woman’s blood and urine may still contain detectable levels of hCG for a few weeks afterwards
-An ovarian tumor or some other type of hCG-producing growth

  • Medscape - Ectopic Pregnancy
  • Radiopaedia - Endometrial thickness
248
Q

A 32-year-old woman presents to the Emergency Department at 38 weeks gestation because she has not felt the fetal movements since yesterday. Investigations confirm fetal demise. Delivery is induced and she gives birth to a dead fetus. Which one of the following is least likely to reveal the cause the fetal death?

A. Autopsy of the fetus.
B. Indirect Coomb’s test of the mother.
C. Chromosomal analysis of the mother.
D. Kleihauer-Betki test.
E. Fetal X-ray.

A

C. Chromosomal analysis of the mother

It is essential that the cause of fetal demise (still birth) is investigated and determined. The purposes of such investigations are to provide an explanation for the death of their baby to the parents and family, who need to know what went wrong to relieve suffering, enable appropriate counselling about recurrent risk, and guide the management of future pregnancies. Providing an explanation to the incidence may be helpful in going through the grieving process and decreasing the feelings of guilt.

For all still births without an obvious cause, the following Investigations must be undertaken with the parents’ consent. Consideration should be given to omitting screening tests when the cause of death is absolutely clear.

At diagnosis of a fetal death (prior to delivery):
-Ultrasound scan to detect possible fetal abnormalities and to assess amniotic fluid volume
-Amniocentesis (where available and warranted) for cytogenetic and infection investigations
-A low vaginal and peri-anal swab, to culture for anaerobic and aerobic organisms
-Full blood examination - maternal blood must be collected for:
Serology for cytomegalovirus, toxoplasmosis, and parvovirus B19
Rubella and syphilis if not already undertaken in the pregnancy
-Blood group determination and antibody screen if not already undertaken in this pregnancy -Kleihauer –Betke test
-Renal function tests including uric acid
-Liver function tests
-Bile acids
-HbA1c
-Anticardiolipin antibodies
-Lupus Anticoagulant
-Activated protein C (APC)

Following birth:
-External examination of the baby (by a perinatal pathologist, neonatologist or a pediatrician if possible)
-Clinical photographs
-Surface swabs (ear and throat) for microbiological cultures
-Babygram (X-ray of the entire body in one film) (or ultrasound (where post-mortem autopsy is refused)
-Post-mortem examination
-Blood samples from the cord or cardiac puncture for investigations of infection
-Blood samples for chromosomal analysis
-Detailed macroscopic examination of the placenta and cord
-Placental microbiological cultures
-Placental and amnion biopsy for chromosomal analysis
-Placental histopathology

Further investigation for thrombophilia must be undertaken 8-12 weeks after the birth where:
a) Fetal death is associated with:
Fetal growth restriction
Preeclampsia
Maternal thrombosis and/or maternal family history of thrombosis

b) The stillbirth remains unexplained following the standard investigations

c) Tests for thrombophilia were positive at the time of the intrauterine fetal death (IUFD) as follows:
-Anticardiolipin antibodies and Lupus anticoagulant repeated if positive at the time of the intrauterine fetal death or initial testing if not previously undertaken
-APC resistance if it was not undertaken at birth
-Factor V Leiden mutation if APC resistance was positive at birth
-Fasting Homocysteine and if there is a positive test for MTHFR gene mutation
-Protein C and S deficiency
-Prothrombin gene mutation 20210A

Of the given options, chromosomal analysis of the mother is not recommended, because this test does not add any further explanation to the cause of fetal demise.

  • Prenatal Society of Australia and New Zealand - Investigations of stillbirths
  • Queensland Clinical Guidelines - Stillbirth care
249
Q

A 16-year-old girl is brought to your GP clinic because her parents are concerned about her development. She has not started her menses yet, and according to the parents she is the shortest of all her classmates at school. On examination, the breasts have not started budding, but pubic and axillary hair growth is present. On examination, no abdominal mass, tenderness or rebound tenderness is noted. The remainder of the exam is unremarkable. The patient refuses a vaginal exam. Which one of the following can be the most likely diagnosis?

A. Ovarian dysgenesis.
B. Mullerian agenesis.
C. Imperforated hymen.
D. A pituitary tumor.
E. Absent vagina.

A

A. Ovarian dysgenesis

Menses, the final piece of puberty, is expected by the age of 16 years. Failure to menstruate beyond this age is termed primary amenorrhea. Primary amenorrhea can be a feature of all the given options; however, short stature makes ovarian dysgenesis (e.g., Turner syndrome) the most likely diagnosis.

Female puberty consists of three components:

  1. Adrenarche – growth of pubic and axillary hair as a response to increased production of androgens by zona reticularis layer of the adrenal gland
  2. Thelarche – breast development (the first sign of puberty in girls is breast budding)
  3. Menarche – commencement of menses (the final event in puberty)

In patients with ovarian dysgenesis, adrenarche often occurs around the expected time like a normal female, but thelarche and menarche are almost always delayed to various extents, depending on the underlying cause, because of decreased levels of estrogen, the adequate levels of which is crucial to thelarche and menarche. Estrogen deficiency usually leads to short stature as a characteristic finding.

OPTION B, C, and D : In mullerian agenesis , imperforated hymen and absent vagina , sexual development occurs normally because ovaries have full function but there is no menstruation because of menstrual flow obstruction (e.g., imperforated hymen, absent vagina), or absent or abnormal menstruation (e.g., absent vagina).

Many infiltrative disease and tumors of the hypothalamus and pituitary (option E) can result in diminished GnRH release or gonadotropin destruction and consequently amenorrhea. Examples are craniopharyngioma, germinoma and Langerhans cell histiocytosis. Such diseases, however, are often associated with other manifestations such as visual field defects and headaches, none of which present in history.

  • UpToDate - Evaluation and management of primary amenorrhea * Medscape – Disorders of Sex Development
250
Q

A 28-year old woman visits your office for her first prenatal check-up. She has smokes one pack of cigarettes per day for the past five years. Which one of the following is incorrect regarding disadvantages of smoking during pregnancy?

A. Small for gestational age.
B. Increased risk of sudden infant death syndrome.
C. Increased risk of developing small teeth with faulty enamel.
D. Higher perinatal mortality.
E. Developmental delay of the baby at least in early years.

A

C. Increased risk of developing small teeth with faulty enamel

Effects of tobacco smoking (active or passive) during pregnancy on fetal, childhood and adult life include:

-Preterm labor
-Low birth weight
-Intrauterine growth restriction
-Birth defects – limb reduction, club foot, oral clefts, eye defects, gastrointestinal effects and many other insignificant anomalies
-Sudden infant death syndrome (SIDS)
-Developmental delay (at least at early stages of development)
-Type II diabetes
-Obesity
-Hypertension
-Decreased HDL
-Nicotine dependence
-Asthma, lower respiratory tract infections
-Impaired academic performance and cognitive abilities
-Conduct disorders, ADHD, antisocial behaviour

Small teeth and faulty enamel are features seen in fetal alcohol syndrome (FAS), and not smoking.

*http://www.racgp.org.au/afp/2014/januaryfebruary/s
*http://www.uptodate.com/contents/cigarette-smoking

251
Q

An Aboriginal woman is in your practice for antenatal care. She is a smoker and has the history of hypertension. She does not seem to be motivated to quit smoking. You want to consult her on harms of smoking to her child, which one of the following is not an effect smoking on the fetus during pregnancy if she continues to smoke?

A. Stillbirth.
B. Premature birth.
C. Placental abruption.
D. Hydrops fetalis.
E. Spontaneous abortion.

A

D. Hydrops fetalis

Effects of tobacco smoking (active or passive) during pregnancy include:

  1. Obstetric complications:
    -Spontaneous abortion and miscarriage
    -Placental abruption (double the risk for smokers who consume more than 20 cigarettes a day)
    -Placenta previa
    -Preterm birth (<37 weeks)
    -Pre-eclampsia
    -Premature rupture of membranes
    -Still birth
    -Ectopic pregnancy
  2. Fetal complications:
    -Low birth weight
    -Intrauterine growth restriction
    -Birth defects – limb reduction, club foot, oral clefts, eye defects, gastrointestinal defects and many other less insignificant anomalies
  3. Childhood and adult life complications:
    -Sudden infant death syndrome (SIDS)
    -Developmental delay (at least at early stages of development)
    -Type II diabetes
    -Obesity
    -Hypertension
    -Decreased HDL
    -Nicotine dependence
    -Asthma, lower respiratory tract infections
    -Impaired academic performance and cognitive abilities
    -Conduct disorders, ADHD, antisocial behavior

Hydrops fetalis is a serious fetal condition, defined as abnormal accumulation of fluid in two or more fetal compartments, including ascites, pleural effusion, pericardial effusion, and skin edema. Hydrops fetalis can be caused by the following conditions:

-Hemolytic disease of the newborn
-Severe anemia
-Chromosomal abnormalities -Congenital heart disease

*Smoking is not a cause of hydrops fetalis.

  • RACGP - AFP - Management of smoking in pregnant women
  • UpToDate - Cigarette and tobacco products in pregnancy: Impact on pregnancy and the neonate
252
Q

A 33-year-old woman presents at 18th week of her pregnancy complaining of fishy-smelling, thin, white homogenous and offensive vaginal discharge. A sample of the discharge is positive for clue cells under light microscopy. Which one of the following statements is true regarding this condition?

A. Metronidazole is contraindicated in this patient.
B. There is are lapse rate of over 50% within 6months.
C. The condition is a sexually transmitted disease (STD).
D. Her sexual partner needs to be treated as well.
E. Reassurance is the only action required.

A

B. There is are lapse rate of over 50% within 6months

The clinical case is a typical description of bacterial vaginosis (BV). This condition is caused by overgrowth of predominantly anaerobic organisms such as Gardnerella vaginalis. Although bacterial vaginosis is exclusively seen in sexually active women (or sometimes after menopause), it is not a sexually transmitted disease.

BV is suspected in the presence of ‘fishy-smelling’ thin white vaginal discharge.
The criteria (Amsel’s criteria) for diagnosis of BV includes:
-Clue cells on microscopy
-Vaginal pH> 4.5
-Positive whiff test: addition of potassium hydroxide to the discharge results in a fishy odor

Since the cause of BV is unclear, current treatment is directed toward alleviation of symptoms and restoration of normal flora rather than eradication of a specific etiologic agent.
Internationally recommended therapies include:
i) First-line: 7 days of oral metronidazole (400 mg twice daily) or,
ii) Second-line: Vaginal clindamycin (1 g at night)

These antibiotics are associated with cure rates of 70–90%, but there is a recurrence in more than 50% of patients within 6 months of treatment. Single dose therapies such as 2g metronidazole have been shown to be less effective than 7-day course of metronidazole.

Australian national guidelines recommend either clindamycin 300 mg orally 12 hourly for 7 days (category A) or metronidazole 400 mg orally 12 hourly for 7 days (category B2). Oral therapy is currently recommended over vaginal therapy, as there is some concern that topical therapy may not be effective against BV organisms in the endometrial cavity; however, this has not been supported by recent clinical trials.

NOTE - clindamycin (300 mg orally, 12 hourly for 7 days) is first-line medication for treatment of BV during pregnancy, but metronidazole (400 mg orally, 12 hourly for 7 days) can also be used safely as the second-line therapy.

Bacterial vaginosis is only seen in women, and men will never have it; therefore, treating the male partner is not necessary.

Symptomatic bacterial vaginosis in non-pregnant women and even asymptomatic BV in pregnant women need treatment, and reassuring the patient is not a correct option. BV in pregnancy may lead to obstetric complications and pelvic infections.

  • RACGP - Bacterial vaginosis
253
Q

A 27-year-old woman presents to the emergency department of the hospital after giving birth to a healthy male baby four days ago with complaint of fever. During the vaginal delivery, she sustained small vaginal laceration. The lesion was small and did not require sutures for repair. She is breastfeeding her baby. On examination, no uterine tenderness is noted. The rest of the exam is unremarkable. Which one of the following can be the most likely cause of her fever?

A. Infection of the unrepaired vaginal laceration.
B. Endometritis.
C. Urinary tract infection( UTI).
D. Breast engorgement.
E. Atelectasis.

A

A. Infection of the unrepaired vaginal laceration

The time of fever (4th day postpartum) and absence of uterine tenderness on exam makes infection of the vaginal laceration the most likely cause of this presentation.

OPTION B : With endometritis, exquisite uterine tenderness was expected. However, sometimes malodorous lochia is the only finding in addition to fever. Furthermore, endometritis would be expected earlier on days 2-3 postpartum (not a strict rule).

OPTION C : There are no urinary symptoms suggesting UTI as the likely cause of the presentation. Moreover, UTI is often expected on days one or two postpartum.

OPTION D : It is too soon for breast engorgement to develop (7-21 days). Besides, there are no findings suggesting breast engorgement as the likely cause of the fever.

OPTION E : Atelectasis is expected during the first 24 hours postpartum.

TOPIC REVIEW

  1. Postpartum fever
    Postpartum fever is a common complication, with a frequency of approximately 5-7% of births and the majority of occurrences happening more than two days after birth. Postpartum fever is defined as a temperature of 38.7 C or greater for the first 24 hours, or greater than 38.0°C of any two of the first 10 days postpartum.

Local spread of colonized bacteria is the most common etiology for postpartum infection following vaginal delivery.

The following conditions are associated with increased risk of postpartum infections and fever:
-History of cesarean delivery
-Premature rupture of membranes
-Frequent cervical examination (sterile gloves should be used in examinations. Other than a history of cesarean delivery, this risk factor is most important in postpartum infections.)
-Internal fetal monitoring
-Preexisting pelvic infection including bacterial vaginosis
-Diabetes
-Malnutrition
-Obesity

Significant causes of postpartum fever, in order of temporal occurrence after delivery, are summarised in the following table:

**Cause of postpartum fever - Expected time of onset (days postpartum)
Atelectasis 0 (within the first 24 hours)
Urinary tract infections / pyelonephritis (1-2days)
Endometritis (2-3days)
Wound infection (4-5days)
Septic thrombophlebitis (5-6days)
Mastitis (7-21days)

Atelectasis (and resultant respiratory tract infections)

Clinical picture include tachypnea, rales, crackles, rhonchi, and consolidation. General anesthesia is the most common risk factor, followed by cigarette smoking. No chest X-ray is required.
Management includes respiratrory toliet and postural drainage.

  1. Urinary tract infection (UTI)
    Patients with pyelonephritis or UTIs may have costovertebral angle tenderness, suprapubic tenderness, and an elevated temperature.

Bacteria most frequently found in UTIs are normal bowel flora, including E.coli, Klebsiella, Proteus, and Enterobacter species. Any form of invasive manipulation of the urethra (e.g. Foley catheterization) increases the likelihood of a UTI. UIT is more commonly associated with repeated catheterization and vaginal exam in prolonged labor.

  1. Endometritis
    Endometritis is the most common infection during the postpartum period and is characterized by lower abdominal tenderness on one or both sides of the abdomen, adnexal and parametrial tenderness elicited with bimanual examination, and temperature elevation (most commonly >38.3°C).

Some women have foul-smelling lochia without other evidence of infection. Some infections, most notably caused by group A beta-hemolytic streptococci, are frequently associated with scanty, odourless lochia.

The mode of delivery is the single most important factor in the development of endometritis. The risk increases dramatically after cesarean delivery. However, there is some evidence that hospital readmission for management of postpartum endometritis occurs more often in those who delivered vaginally.

Other risk factors include prolonged rupture of membranes, prolonged use of internal fetal monitoring, anemia, and lower socioeconomic status.

Perioperative antibiotics have greatly decreased the incidence of endometritis.

In most cases of endometritis, the bacteria responsible are those that normally reside in the bowel, vagina, perineum, and cervix.

The uterine cavity is usually sterile until the rupture of the amniotic sac. As a consequence of labor, delivery, and associated manipulations, anaerobic and aerobic bacteria can contaminate the uterus.

Management includes provision of antibiotic coverage against polymicrobial infection. The recommended regimen is ampicillin/amoxicillin + gentamicin + metronidazole (all intravenously)

  1. Wound infections
    Patients with wound infections, or episiotomy infections, or traumatic vaginal lacerations have erythema, edema, tenderness out of proportion to expected postpartum pain, and discharge from the wound or episiotomy site.

Drainage from wound site should be differentiated from normal postpartum lochia and foul-smelling lochia suggestive of endometritis.

Most often, the causative organisms associated with perineal cellulitis and episiotomy site infections are Staphylococcus or Streptococcus species and gram-negative organisms, as in endometritis. Vaginal secretions contain as many as 10 billion organisms per gram of fluid. Yet, infections develop in only 1% of patients who had vaginal tears or who underwent episiotomies.

Those who underwent cesarean delivery have a higher readmission rate for wound infection and complications than those who delivered vaginally.
Wound toilet, abscess drainage and broad spectrum antibiotics are mainstay of treatment.

  1. Septic pelvic thrombophlebitis
    Numerous factors cause pregnant and postpartum women to be more susceptible to thrombosis. Pregnancy is known to induce a hypercoagulable state secondary to increased levels of clotting factors. Also, venous stasis occurs in the pelvic veins during pregnancy.

Patients with septic pelvic thrombophlebitis, although rare, may have palpable pelvic veins. These patients also have tachycardia that is out of proportion to the fever. Septic pelvic thrombosis and thrombophlebitis are rather rare cause of postpartum fever.

Broad-spectrum antibiotics should be administered. Initial choice of antibiotics should cover gram-positive, gram-negative, and anaerobic organisms. Ampicillin and gentamicin with metronidazole or clindamycin is a common regimen.

Anticoagulation either with unfractionated heparin (UFH) or low molecular weight heparin (LMWH) may be used. There is no universal guideline or recommendation for anticoagulation therapy in septic pelvic thrombosis.

  1. Mastitis
    Patients with mastitis have very tender, engorged, erythematous breasts. Infection is frequently unilateral.

The most common organism reported in mastitis is Staphylococcus aureus.
The organism usually comes from the breastfed neonate/infant’s mouth or throat.

Mastitis should be treated with antibiotics to prevent abscess formation. Oral di/flucloxacillin, 6-hourly, provides good coverage against the most common causative organisms. For those with hypersensitivity to penicillin (immediate hypersensitivity excluded) cephalexin is an alternative. For those with immediate hypersensitivity (anaphylaxis) clindamycin is used.

In patients with evidence of cellulitis and more pronounced systemic symptoms, intravenous di/flucloxacillin is the preferred. For patients with penicillin hypersensitivity (immediate hypersensitivity excluded) intravenous cefazolin/cephalothin is an alternative. For those with immediate hypersensitivity, intravenous clindamycin can be used.

*http://emedicine.medscape.com/article/796892-overv
*http://www.medscape.com/viewarticle/804263

254
Q

A 37-year-old woman presents to your practice with complaint of malodorous grey vaginal discharge. A wet mount prep under light microscopy reveals bacteria attached to vaginal epithelial cells. Which one the following organisms could be the most likely pathogen?

A. Chlamydia.
B. Trichomonas.
C. Candida albicans.
D. Gardnerella vaginalis.
E. Mycoplasma hominis.

A

D. Gardnerella vaginalis

The malodorous greyish discharge and more importantly the bacteria attached to vaginal epithelial cells are suggestive of bacterial vaginosis (BV) as the most likely diagnosis.

BV represents a complex change in the vaginal flora characterized by a reduction in concentration of normally dominant hydrogen-peroxidase producing lactobacilli and an increase in concentration of other organisms, especially anaerobic gram negative rods, such as:

-Gardnerella vaginalis
-Prevotella species
-Porphyromonas species
-Bacteroids
-Peptostreptococcus species
-Mycoplasma hominis
-Ureaplasma urealyticum
-Mobiluncus species
-Fusobacterium species
-Atopobium vaginae

Increasing evidence suggests that Gardnerella vaginalis plays the most important role in pathogenesis of BV.

Most of women with BV are asymptomatic. The symptoms, if present, include a grayish white, thin, and homogenous discharge with the unpleasant ‘fishy smell’ that may be more noticeable after sexual intercourse (when the semen has made the vagina more alkaline) and during the menses.

BV alone does not cause dysuria, dyspareunia, pruritus, burning, or vaginal inflammation. The presence of these symptoms suggests mixed vaginitis.

Although gram stain is the gold standard for diagnosis of BV, the diagnosis is usually based on Amstel criteria, which are simple and useful in an office practice where microscopy is available. The first 3 findings are sometimes also present in Trichomoniasis:
1. Homogeneous, thin, grayish-white discharge that smoothly coats the vaginal walls
2. Vaginal pH >4.5
3. Positive whiff-amine test, defined as the presence of a fishy odor when a drop of 10% potassium hydroxide (KOH) is added to a sample of vaginal discharge
4. Clue cells on saline wet mount - clue cells are vaginal epithelial cells studded with adherent coccobacilli that are most evident at the edge of the cell. For a positive result, at least 20% of the epithelial cells on wet mount should be clue cells. The presence of clue cells diagnosed by an experienced microscopist is the single most reliable predictor of BV.

255
Q

A 38-year-old primigravida woman is admitted to the Labor Unit at 39 weeks gestation due to the onset of regular uterine contractions. Her cervix is 8 cm dilated and 100% effaced, with the fetus’ vertex at +1 station. The fetal heart rate is 150 bpm. As the labor progresses, the fetal heart rate falls to 80 bpm without any changes in the mother’s general condition. Which one of the following would be the next best step in management?

A. Fetal scalp blood sampling.
B. Cardiotocography.
C. Vaginal examination to exclude any cord prolapse.
D. Emergency delivery by cesarean section.
E. Percutaneous umbilical sampling.

A

B. Cardiotocography

The case describes fetal bradycardia detected on fetal heart auscultation. Bradycardia of <100bpm for more than five minutes or <80 bpm for more than three minutes is always considered abnormal.

Severe bradycardia is caused by:
-Prolonged cord compression Cord prolapse
-Epidural and spinal anesthesia
-Maternal seizures
-Rapid fetal descent

In clinical situations where the fetal heart rate pattern is considered abnormal, immediate management should include:

-Identification of any reversible cause of the abnormality and initiation of appropriate action (i.e., maternal repositioning, correction of maternal hypotension, rehydration with intravenous fluid, cessation of oxytocin, tocolysis for excessive uterine activity, and initiation or maintenance of continuous CTG.

-Consideration of further fetal evaluation or delivery if a significant abnormality persists.

In this scenario where the baby is 1+ station and abnormal fetal heart rate is detected on auscultation the next step would be performing a confirmatory cardiotocography (CTG). If CTG confirms the condition despite initial measures mentioned above, prompt action should be taken. If CTG indicates fetal compromise, measures such as fetal scalp blood sampling , or emergency delivery by caesarean section can be undertaken.

With the mother having normal general condition, cord compression or prolapse should come on the top of the differential diagnoses list, but since the cervix is 8 cm dilated, 100% effaced and the fetal head is already engaged, cord prolapse would be unlikely; therefore, repeating vaginal exam is not as important as confirmatory CTG; however, if the scenario indicated cord prolapse as a possibility, a vaginal exam should have been performed to exclude cord compression or prolapse.

In case a blood sample of the fetus is required, scalp blood sampling is the preferred method. Percutaneous blood sampling is not routinely performed.

NOTE - In cases of severe prolonged bradycardia, if the cause cannot be identified and corrected, immediate delivery is recommended.

  • RANZCOG Intrapartum Fetal Surveillance Clinical Guideline
  • WA Health Department – Obstetrics and Gynaecology Guidelines
256
Q

A 28-year-old woman is admitted to the Maternity Unit in early labor. She has had an uncomplicated pregnancy to date and is otherwise healthy. She asks you about fetal monitoring during labor and says she has heard that cardiotocography (CTG) is helpful for assessment of the baby’s wellbeing and preventing fetal problems. In counselling regarding to the use of CTG as a predictor of fetal outcome and satisfactory labor in comparison to intermittent auscultation and whether CTG monitoring will reduce the risk of neonatal developmental abnormalities, which one of the following would be the most appropriate advice?

A. There is no evidence to support admission CTG.
B. CTG is a predictor of previous fetal oxygenation.
C. Compared to intermittent auscultation, CTG is superior in monitoring the baby’s wellbeing.
D. CTG is a good predictor of the fetal outcome.
E. CTG will reduce the risk of neonatal developmental abnormalities.

A

A. There is no evidence to support admission CTG

Continuous fetal heart rate monitoring with CTG is mandatory in all high-risk pregnancies but has been shown to have limited value in low-risk pregnancies. Intermittent fetal heart auscultation has the same efficacy in such pregnancies.

In fact, CTG in low-risk pregnancies, by recognizing abnormalities that are of little if any significance, leads to unnecessary obstetrical interventions such as instrumental delivery or cesarean section.

CTG has not reduced the incidence of neonatal developmental abnormalities such as cerebral palsy. CTG is a predictor of the current fetal condition, not the previous, unless there are severe abnormalities as soon as the CTG is applied on admission.

In conclusion, there is no evidence to support routine admission CTG.

  • NCBI - PubMed - Cardiotocography versus intermittent auscultation of fetal heart on admission to labour ward for assessment of fetal wellbeing.
257
Q

Christina, 38 years old, presents for discussion about her risk of developing ovarian cancer. Her detailed history shows that she is the mother of 4 children, all breastfed. She used oral contraceptive pills for 5 years after the birth of her last child. Two year ago, she underwent hysterectomy to ‘get rid of OCPs’. Apart from a BMI of 31, she is otherwise healthy on examination. Which one of her conditions is considered a risk factor for developing ovarian cancer?

A. History of oral contraceptive pills.
B. Hysterectomy.
C. Her body mass index.
D. Breastfeeding.
E. Multiparity.

A

C. Her body mass index

The following are associated with INCREASED risk of ovarian cancer:

  1. Family history of ovarian cancer - Family history of ovarian cancer is a strong risk factor, present in approximately 10-15% of women who develop the disease. The risk of ovarian cancer is increased when the family history suggests a sporadic case, but is substantially greater when there is a hereditary cancer syndrome.
  2. Familial ovarian cancer syndromes - Clues to the presence of a hereditary cancer syndrome include the presentation of ovarian cancer in a first- or second-degree family member (both maternal and paternal lineage) at an early age (under 50 years) or the occurrence of ovarian or related cancers in multiple members (i.e. 2 to 4 generations). Familial ovarian cancer syndromes are uncommon, accounting for 5-10% of ovarian cancer cases. These hereditary syndromes include:
    a) Lynch II syndrome (cancers of the colon, breast, endometrium, and ovary with hereditary nonpolyposis colorectal cancer or HNPCC)
    b) Breast-ovarian cancer syndrome (usually associated with a BRCA1 or BRCA2 mutation) – the most common hereditary ovarian cancer syndrome.
  3. Age - The incidence of ovarian cancer increases with age.
  4. Infertility or Nulliparity
  5. Early menarche or late menopause - these factors are associated with greater numbers of ovulation that are associated with increased risk of ovarian cancer
  6. Endometriosis
  7. Premenopausal or postmenopausal hormone replacement therapy (HRT)
  8. Obesity – The mechanism through which obesity increases the chance of contracting ovarian cancer is increased oestrogen production by adipose tissue.

The risk of ovarian cancer is DECREASED in women with history of:
-Pregnancy –the more the number of pregnancies the less the chance of ovarian cancer
-Use of the oral contraceptive pill
-Breastfeeding
-Tubal ligation or hysterectomy

Of the given options, the BMI of this woman (obese) is a risk factor for developing ovarian cancer.

Oral contraceptive pills, multi-parity, breastfeeding and hysterectomy are protective factors against ovarian cancer.

*http://emedicine.medscape.com/article/255771-overv *http://www.uptodate.com/contents/screening-for-ova

258
Q

A young woman with history of valvular heart disease comes to your clinic for advice regarding pregnancy. She is planning to become pregnant. Which one of the following, if present, can lead to most significant complications during pregnancy?

A. Mitral valve regurgitation.
B. Mitral valve stenosis.
C. Aortic valve regurgitation.
D. Aortic valve stenosis.
E. Tricuspid valve regurgitation.

A

B. Mitral valve stenosis

Of the given options, mitral valve stenosis is the cardiac condition associated with the most significant complications during pregnancy.

The pregnancy-induced increase in blood volume, cardiac output, and tachycardia can increase the trans-mitral pressure gradient and cause pulmonary edema in women with mitral valve stenosis.

Pregnancy associated with long-standing mitral valve stenosis may result in pulmonary hypertension. On the other hand, pregnant women with mitral stenosis are at increased risk for the development of atrial fibrillation and other tachyarrhythmias.

Mitral valve stenosis, grade III or IV heart failure, and atrial fibrillation are the most significant cardiac conditions with high risk for complications.

Mitral valve regurgitation (option A), aortic valve regurgitation (option C), aortic valve stenosis (option D) and tricuspid valve regurgitation (option E) are often well tolerated during pregnancy, as the pregnancy-induced decrease in systemic vascular resistance reduces the risk of cardiac failure with these conditions.

  • KEMH - Clinical guidelines
  • Medscape - Cardiovascular Disease and Pregnancy
259
Q

Which one the following is unlikely to require RhoGAM (anti-D immunoglobulin) administration in an Rh negative woman, who has not actively formed their own Anti-D?

A. Threatened abortion before 12weeks pregnancy.
B. Spontaneous abortion.
C. External cephalic version.
D. Significant abdominal blunt trauma.
E. Termination of pregnancy.

A

A. Threatened abortion before 12weeks pregnancy

There is insufficient required in cases of
evidence to suggest that a threatened abortion before 12 weeks gestation requires Anti-D antibody unless the bleeding is significant. Royal college of Obstetricians and Gynaecologists (UK) recommends that RhoGAM is not threatened abortion in the first 12 weeks if there is no significant bleeding. This recommendation is endorsed and practiced by Royal Australian and New Zealand College of Obstetricians and Gynaecologists.

TOPIC REVIEW

An Rh-negative mother has no antibodies against Rh (D) antigen. If she is pregnant and the fetus is Rh-positive any mixing up the fetus’ blood to the mother’s will trigger an immune response by lymphocytes and will lead to production of anti-Rh antibodies in the mother’s blood (anti-D antibody). RhoGAM® is anti-D IgG passive antibody that will eliminate the D-antigen from the mother’s blood, before the mother’s immune system start to sensitize. It is administered intramuscularly (IM).

RhoGAM is available is two forms:
1. CLS-250 IU – one dose contains 50μg of anti-D antibody
2. CLS-265 IU – one dose contains 125μg of anti-D antibody

Each 300μg of anti-D antibody neutralizes 15 mL of fetal packed red cells (30 mL of whole blood)

RhoGAM should be administered within the first 72 hours after the precipitating even, however, with much less success rate it can be given up to 9-10 days.

INDICATIONS

  1. First trimester indications (up to and including 12 weeks)
    -Spontaneous abortion (complete, incomplete, or missed abortion) probably excluding threatened abortion
    -Induced abortion (medically or by D&C)
    -Ectopic pregnancy
    -Chorionic villous sampling (this procedure is performed at 10-12 weeks)
    -Molar pregnancy – chorionic villi may contain D-antigen

MANAGEMENT - in the first 12 weeks the maximum amount of fetal blood that can mix with mother’s is 2.5 mL of RBC (5 mL whole blood). A single dose of CLS 250-IU is sufficient for neutralization of circulating fetal D-antigen. Multifetal pregnancies need extra doses. In a singleton pregnancy Kleihauer-Betki test is not indicated because the amount of fetomaternal blood mix is not significant and a single dose CLS – 250 will be enough.

  1. Second and third trimester indications
    -Spontaneous or induced abortions of all kind after 12 weeks
    -Amniocentesis (this procedure is performed > 15 weeks)
    -Cordocentesis (this procedure is performed >20 weeks)
    -Fetal blood sampling
    -Fetal death
    -External cephalic version of breech presentation (successful or unsuccessful)
    -Blunt abdominal trauma in pregnancy considered sufficient to cause fetomaternal haemorrhage
    -Antepartum haemorrhage (revealed or concealed) in the second or third trimester (e.g. placental abruption, placenta praevia, etc)

MANAGEMENT - in the second or third trimester CLS 625 (containing 125μg anti D-antibody) should be used. A Kleihauer-Betki test is indicated in the second or third trimester events to assess the need for extradoses of CLS 625 IU. Ideally the sample for the test should be collected within 15 minutes of the precipitating event, but if not possible it can be performed up to 72 hours.

*RhoGAM will remain in maternal circulation for up to 6 weeks.

**RhoGAM should not be given to women in which anti-D antibody has already formed.

Routine administration of RhoGAM - in the absence of a precipitating event, RhoGAM (CLS – 625 IU) is routinely given to all Rh-negative pregnant women at 28 weeks and 34 weeks of pregnancy.

Within the first 72 hours postpartum all R-negative women whose baby is Rh-positive should receive a dose of CLS-625 IU and undergo quantification of fetomaternal blood mix using Kleihauer-Betki test to evaluate whether extra doses of CLS-625 IU are indicated.

*http://www.australianprescriber.com/magazine/23/2/
*https://www.nhmrc.gov.au/_files_nhmrc/publications
* Royal Australian and New Zealand College of Obstetricians and Gynaecologists – College Statement C-Obs 6 (2015)

260
Q

A 38-year-old Rh-negative woman was given Anti-D antibody (RhoGAM®) at 28 and 34 weeks of pregnancy three years ago during her first pregnancy, and extra postpartum doses after the baby was found to be Rh-positive. Six month ago, she underwent an emergency orthopedic surgery and received blood transfusion. She has had no miscarriages since then. She is planning to become pregnant again. In the first antenatal visit, anti-D antibody level is found to be positive. Which one of the following is most likely to be the cause of sensitization to D antigen in this woman?

A. Occult fetomaternal bleeding.
B. Blood transfusion.
C. Previous pregnancy.
D. RhoGAM injection.
E. Orthopedic surgery.

A

B. Blood transfusion

An Rh-negative woman with an Rh-positive fetus is likely to become sensitized against fetal D antigen in the following situations:

-Delivery
-Spontaneous abortion (complete, incomplete, missed or threatened abortion)
-Induced abortion (medically or by D&C)
-Ectopic pregnancy
-Chorionic villous sampling, amniocentesis, cordocentesis, or fetal blood sampling
-Fetal death
-External cephalic version of breech presentation (successful or unsuccessful)
-Blunt abdominal trauma in pregnancy considered sufficient to cause fetomaternal hemorrhage
-Antepartum hemorrhage (overt or concealed) in the second or third trimester (e.g. placental abruption, placenta previa, etc) -Molar pregnancy

Timely administration of adequate anti D immunoglobulin, removes fetal D-antigen from maternal circulation before sensitization occurs in the mother.

This patient has received RhoGAM during her previous pregnancies and has had no abortion or any obstetrical procedures to pose the risk of fetomaternal blood mixing. Therefore, the only likely cause of exposure to Rh antigen can be transfusion of mismatched blood during the orthopedic surgery.

*http://www.australianprescriber.com/magazine/23/2/
*https://www.nhmrc.gov.au/_files_nhmrc/publications

261
Q

A 34-year-old woman presents with history irritability around her periods. She also mentions breast tenderness at these times. She is not sexually active and is not planning to start any relationship with a male partner in the future. Which one of the following would be the next best step in management?

A. Hysterectomy.
B. Selective serotonin reuptake inhibitors (SSRIs).
C. Oral contraceptive pills.
D. Topical anesthetics.
E. Evening primrose oil.

A

E. Evening primrose oil

Most women of reproductive age experience one or more mild emotional or physical symptoms for 1 to 2 days before the onset of menses. These symptoms, such as breast soreness and bloating, are mild, do not cause distress or functional impairment, and are not considered to represent premenstrual syndrome (PMS).

In contrast, clinically significant premenstrual syndrome (PMS) is defined by the American College of Obstetricians and Gynecologists as at least one symptom associated with ‘economic or social dysfunction’ that occurs during the 5 days before the onset of menses and is present in at least 3 consecutive menstrual cycles. Symptoms may be affective (e.g. anger outbursts, depression) or physical (e.g. breast pain and bloating).

Common symptoms of PMS include:

  1. Psychological and behavioral

-Mood swings and depression
-Tiredness, fatigue or lethargy Irritability
-Difficulty concentrating
-Sleep disturbances
-Aggression
-Tearfulness or feeling low
-Tension or unease
-Clumsiness/poor coordination
-Altered libido
-Food cravings/increased appetite
-Loss of self-control

  1. Physical

-Bloated abdomen and fullness
-Breast pain/tenderness
-Headaches
-Puffiness of face, abdomen or fingers -Constipation or diarrhea
-General aches and pains, especially backache
-Weight gain
-Acne or other skin rashes
-Muscle or joint stiffness
-Abdominal pain/cramp
-Exacerbation of epilepsy, migraine, asthma, rhinitis or urticaria.

This patient has irritability and cyclic mastalgia as her two main symptoms. Irritability is the most common psychological symptom in PMS. Of physical symptoms, abdominal bloating is the most common one (90%) followed by headache (>50%) and mastalgia/breast tenderness (>50%)

Treatment of PMS starts with non-drug strategies (e.g. CBT, relaxation training) and lifestyle changes (e.g. regular exercise). Vitamin E, B6, calcium and magnesium have shown some efficacy and can be used in conjunction with CBT, regular exercise, relaxation therapy, etc. This treatment should be continued for at least 2-3 cycles.

Evening primrose oil (EPO) has been shown to have no more effect on PMS symptoms in general, but it can be effective on cyclic mastalgia either in isolation or as a symptom of PMS. For cyclic mastalgia addition of EPO is an appropriate initial step in management. EPO contains an essential fatty acid claimed to be lacking in the diet, and replacement allows for the production of prostaglandin E, which counteracts the effects of estrogen and prolactin on breasts.

If this patient was considering contraception, OCPs (option C) could be an option for her. Some evidence suggest that OCP preparations, especially those with shortened pill-free intervals can reduce physical symptoms of PMS and (including mastalgia).

Topical anesthetics (option D) are not typically used for management of mastalgia; however, topical analgesics may benefit some patients.

SSRIs (option B) are used in treatment of women with premenstrual dysphoric disorder (PDD) where symptoms are negatively impacting the patient’s economics and/or social functioning. Even so, a trial of more conservative management with the emphasis on lifestyle modification should be considered first.

Cyclical mastalgia (or PMS/PDD) in general are hypothesized to be caused by hormonal imbalance. Hysterectomy (option A) does not affects a woman’s hormones, therefore not effective in treatment such conditions.

  • Royal College of Obstetrics and Gynaecology – Green top Guideline No.48 * RACGP - Management of benign breast conditions
  • UpToDate - Breast pain
262
Q

A 32-year-old office secretary presents with symptoms of premenstrual syndrome associated with severe mastalgia, unresponsive to conservative measures, evening prim rose oil and analgesic. Which one of the following is the most appropriate treatment?

A. Mefenamic acid.
B. Spironolactone.
C. Danazol.
D. Bromocriptine.
E. Vitamin E.

A

C. Danazol

Although never first-line, danazol is the most effective treatment for cyclic mastalgia either in isolation or as a part of premenstrual syndrome (PMS) . The regimen includes 200mg danazol daily from the onset of the symptoms to the onset of menstruation.

OPTION D : GnRH antagonists and bromocriptine are also very effective but not appropriate for use in severe cyclic mastalgia because their significant adverse effect profiles.

OPTION A : NSAIDs such as mefenamic acid are more effective for management of dysmenorrhea than PMS.

OPTION B : Spironolactone, with controversial effects, has been considered for treatment of choice for PMS if fluid retention is the dominating feature. It should be used from 3 days before the anticipated time for development of symptoms to day 1 of menstruation.

OPTION E : Evidence favoring use of vitamin E for treatment of cyclic mastalgia is very weak.

TOPIC REVIEW

Approach to cyclical mastalgia often starts with non-pharmacological measures, including:
i) Well-fitting brassiere - this approach is widely advocated. For patients with pendulous breasts a support bra with steel underwire is more effective. In addition, use of a ‘sports bra’ during exercise has been shown to reduce pain related to breast movements.

ii) Compress - Some women obtain relief from application of warm compresses or ice packs or gentle massage.

iii)Evening primrose oil (EPO) - this natural remedy has shown to have some effects on cyclic mastalgia (but not on PMS in general)

Medical therapy as the next step if conservative measures fail:

  1. First line: paracetamol or NSAIDs or both can be used to relieve breast pain. Topical NSAIDs may also be useful
  2. Second line: danazol is the only medication approved by FDA for the treatment of mastalgia with the greatest efficacy; however, it causes significant androgenic effects, which generally limits its use and result in discontinuation of treatment.
  3. Third line: Tamoxifen can be used in patients with more severe mastalgia failing to respond to other measures; however, this medication is associated with side effects including menopause-like symptoms such as hot flushes, vaginal dryness, joint pain, and leg cramps. Tamoxifen also increases the risk of blood clots, strokes, uterine cancer, and cataracts. Tamoxifen is infrequently used for this indication.

NOTE - Bromocriptine and GNRH antagonists have been studied, but are not advocated for use in patients with severe mastalgia because of significant adverse effects.

263
Q

A 27-year-old woman presents to your practice with complaint of premenstrual abdominal bloating, irritability and uneasiness for the past 6 months. These symptoms begin 3-4-days before her anticipated periods and ease off within 1 to 2 days of her menstrual flow. Recently, she had to take few days off work because of the intensity of her symptoms. Which one of the following is the most appropriate next step in management?

A. Relaxation therapy.
B. Sertraline 50mg daily in the second half of anticipated luteal phase.
C. Evening primrose oil.
D. NSAIDs.
E. Low dose OCP with shortened pill free intervals.

A

A. Relaxation therapy

Most women of reproductive age experience one or more mild emotional or physical symptoms for 1 to 2 days before the onset of their menses. These symptoms, such as breast soreness and bloating, are mild, do not cause distress or functional impairment, and are not considered to represent premenstrual syndrome (PMS).

In contrast, clinically significant premenstrual syndrome (PMS) is defined by the American College of Obstetricians and Gynecologists as at least one symptom associated with ‘economic or social dysfunction’ that occurs during the 5 days before the onset of menses and is present in at least 3 consecutive menstrual cycles. Symptoms may be affective (e.g. irritability, anger outbursts, depression) or physical (e.g. abdominal bloating, headache, breast pain, etc).

The characteristic features in the history, as well as apparent interference of symptoms with her job (economic dysfunction) make PMS the most likely diagnosis with high index of certainty.

Management of PMS (and PMDD) always starts with non-drug measures including the following:
-Cognitive behaviour therapy (CBT)
-Relaxation therapy (Yoga included)
-Regular exercise
-Healthy lifestyle and diet - cessation of alcohol/smoking, high-fiber diet, low-glycemic index carbohydrates and cutting down on caffeine intake. Caffeine has not shown to be related to PMS but generally, intake reduction is advised.
-Vitamin E, B6, calcium and magnesium

NOTE - evening primrose oil has no more effect than placebo on PMS, but studies show that it may be helpful in treatment of mastalgia as a main symptom.

Low-dose SSRIs (fluoxetine, sertraline, paroxetine and citalopram) are used if the above measures have been tried to no avail for at least 2-3 cycles. SSRIs are particularly effective if the mood and affective symptoms dominate.

SSRIs have shown greatest effectiveness, as well as most acceptable adverse effect profile among other treatment options.

Combined new-generation pill, such as Yasmin®, Cilest® (cyclically or continuously) can be tried particularly if physical symptoms are pronounced and bothersome. Their effect on PMS, however, is still controversial.

Estradiol patches, danazol and GnRH antagonists with add-back hormone replacement therapy, bilateral oophorectomy and total hysterectomy with adding back HRT and testosterone are other treatment options considered in a step-wise manner.
Of the given options, only relaxation therapy falls in the category of measures considered for initial management of PMS (and PMDD).

TOPIC REVIEW

A step-wise approach to management of PMS and PMDD

  1. First line
    Treatment Starts with non-drug strategies (e.g. CBT, relaxation training) and lifestyle changes (e.g. regular exercise). Vitamin E, B6, calcium and magnesium have shown some efficacy and can be used in conjunction with CBT, regular exercise and healthy diet. This treatment should be continued for at least 2-3 cycles.
    If proved ineffective consider the next steps:
  2. Second line
    a) Continuous or in luteal phase (day 15–28) low-dose SSRIs – especially if psychological symptoms are of main concern.
    b) Combined new-generation pill, such as Yasmin®, Cilest® (cyclically or continuously) – these contraceptive are not as effective of SSRs but may be considered the preferred option if the physical symptoms are bothersome. They do not relieve mood and psychological symptoms.
  3. Third line
    i) Estradiol patches (100 micrograms) PLUS either oral progestogen such as duphaston or Mirena®
    ii) Higher-doses of SSRIs continuously or in luteal phase may be tried with care as evidence suggest that there are more adverse effects, with added benefit compared to lower doses
  4. Fourth line
    GnRH antagonist or danazol , as well as adding back hormone replacement therapy (either continuous or cyclical) to counteract effects of induced estrogen deficiency.
  5. Fifth line
    Total abdominal hysterectomy and bilateral oophorectomy PLUS add-back HRT (including testosterone)

NOTE - Treatment of PMS and PMDD always starts with non-pharmacological measures for at least 2-3 menstrual cycles.

264
Q

A 28-year-old woman presents to the Maternity Unit in labor. Her last pregnancy ended in a cesarean section due to breech presentation. After delivery of her baby, spontaneous complete expulsion of the placenta is followed by 1000 ml of blood loss, after which the woman entered a state of shock. Which one of the following is the most likely cause of her bleeding?

A. Uterine atony.
B. Cervical laceration.
C. Uterine inversion.
D. Uterine rupture.
E. Retained products of pregnancy.

A

D. Uterine rupture

Uterine atony, cervical lacerations, uterine inversion and retained products of pregnancy can all cause postpartum hemorrhage.

Normally after delivery, 600-800 ml of blood is lost per vagina. The plasma volume is increased by 50% during pregnancy. Assuming that a regular adult has 5000 ml of blood, during the pregnancy this volume increases to approximately 7500 ml. A blood loss of 1000 ml amounts to approximately 13% of total blood volume. Loss of this amount is very unlikely to result in shock; therefore, a concealed source of bleeding should exist. Considering the fact that shock has occurred with only 1000 ml of blood loss, either uterine rupture or uterine inversion should be suspected; however, given the history of cesarean section and the rarity of uterine inversion, uterine rupture would be the most likely diagnosis.

In uterine rupture (option A) , there are often signs of hemorrhagic shock unjustifiable with the amount of per vaginal hemorrhage, because the blood spillage occurs into the abdominal cavity. With uterine atony and hemorrhage to shock , more blood is expected per vagina, as is in cervical or vaginal lacerations (option B).

OPTION C : Uterine inversion is a potentially lethal condition that occurs when the fundus of the uterus turns inside out. Uterine inversion can be incomplete or complete. It is a rare condition. Diagnosis of uterine inversion is usually based on clinical signs and symptoms. When there is complete inversion, the diagnosis is most easily made by palpating the inverted fundus at the cervical os or vaginal introitus. In incomplete inversion, palpating the fundal wall in the lower uterine segment and cervix might be required for diagnosis.

OPTION E : With a complete placenta, retained products of pregnancy as the underlying cause of uterine atony and postpartum bleeding is not likely.

265
Q

A 20-year-old woman presents to your clinic because she has never had a period so far. On examination, the genitalia appear completely normal. Breasts are immature, but pubic and axillary hair are present. Pelvic ultrasound shows normal uterus and fallopian tubes, but ovaries contain small amount of connective tissue and no follicles. Which one of the following at is the most likely cause of her amenorrhea?

A. A hypothalamic lesion.
B. Mullerian agenesis.
C. Turner syndrome.
D. Hyperprolactinemia.
E. A pituitary lesion.

A

C. Turner syndrome

The clinical features described in the scenario are consistent with diagnosis of Turner syndrome. Turner syndrome is characterized by the Karyotype of 45-XO.

Clinical features of Turner syndrome include:
-Short stature
-Webbed neck
-Puffy hands and feet
-Coarctation of the aorta
-Cardiac abnormalities
-High-arched palate
-Absent secondary sexual characteristics when puberty is expected

In Turner syndrome, the ovaries consist of small amounts of connective tissue and no or just a few atrophic follicles (streak gonads). The external genitalia, vagina, uterus and Fallopian tubes are well-developed at birth and anatomically normal, but fail to function when estrogen-induced maturity should take place.

In older adolescents and adults, presenting symptoms usually involve issues of puberty and fertility as well as short stature. Adrenarche, the beginning of pubic hair growth, occurs as predicted, but is not an indication that puberty will progress normally. Breast development is absent when ovarian failure occurs before puberty, but pubic hair growth is normal. Up to 13% of girls with Turner syndrome have spontaneous breast development or menses; some of these are XO/XX mosaics, with normal gonadotropin responses to luteinizing hormone–releasing hormone (LHRH). Turner syndrome should be suspected in individuals with primary or secondary amenorrhea and in adult women with unexplained infertility, particularly when such individuals also have short stature.

Approximately 30% of girls with Turner syndrome have some spontaneous pubertal development

266
Q

A 32-year-old female presented for antenatal follow-up. She is 14 weeks pregnant. On examination, the fundal height is 18 cm. which one of the following would be the next best step in management of this patient?

A. Check serum beta hCG level.
B. Perform an ultrasound scan.
C. Reassure that this is normal.
D. Refer to a gynecologist.
E. Review in four weeks.

A

B. Perform an ultrasound scan

The fundus is expected to be palpable above the pubis symphysis at 12 weeks gestation. At 20 weeks it is expected to be felt at the level of umbilicus. Between 12 and 20 weeks the uterus stands in between. After 20 weeks the fundal height in centimeters is equivalent to the number of weeks of pregnancy.

A fundal height of 18cm is definitely large for a 14-week pregnant uterus. The most common cause of such discrepancy is dating errors; hence, the next best step would be performing an ultrasound scan for more accurate estimation of the gestational age. Furthermore, ultrasonography can give additional information about the conditions that might have led to a large-for-date uterus such as polyhydramnious, multiple gestation, etc, if the case is not simply a dating error.

The most common causes of a large-for-gestational-age are:
1. Dating errors - the most common
2. Twin pregnancy
3. Gestational diabetes
4. Polyhydramnios
5. Gestational trophoblastic disease (molar pregnancy)

267
Q

Which one of the following is a protective factor against developing uterine fibroids?

A. Obesity.
B. Early menarche.
C. Smoking.
D. First-degree relatives with uterine fibroid. E. Nulliparity.

A

C. Smoking

Uterine leiomyoma (fibroid) is a benign uterine tumor commonly seen in females of reproductive age. Fibroid growth is highly dependent on estrogen.

The following are associated with INCREASED risk of uterine leiomyomas:

-Black race - the condition is 2- to 3-fold greater in black women than in white women -Family history of uterine leiomyomas
-Early menarche
-Prenatal exposure to diethylstilbestrol
-Obesity - most studies show a relationship between fibroids and increasing body mass index
-Significant consumption of beef and other red meats or ham
-Consumption of alcohol, especially beer (controversial)
-Frequent uterine infections (controversial)
-Hypertension (controversial)

The following are associated with DECREASED risk for uterine leiomyomas:

-Smoking - by decreasing estrogen levels
-Green vegetables and fruit (especially citrus fruit)
-Parity (having one or more pregnancies extending beyond 20 weeks) - it has been hypothesized that the postpartum remodeling of the uterus may have the effect of clearing smaller fibroids
-Long-acting progestogen

NOTE - Combined oral contraceptive pills (COCPs) do not seem to increase the risk of developing uterine leiomyomas.

Of the given options, smoking is associated with decreased risk of uterine leiomyomas.

268
Q

Which one of the following is a risk factor for endometrial cancer?

A. Combined hormone replacement therapy. B. Age more than 30years.
C. Early menarche.
D. Early menopause before the age of 45.
E. Multiparity.

A

C. Early menarche

Risk factors for endometrial hyperplasia are essentially the same for endometrial cancer. The following table summarizes the risk factors for endometrial hyperplasia/cancer and their relative risk: see table below.

Risk factor (Relative Risk, %)

-Unopposed estrogen therapy (2-10%)
-Increasing age (1.4%) (50-70 years)
-Late menopause (>55 years) (2%)
-Nulliparity (2%)
-Chronic anovulation (e.g. in PCOS) (3%)
-Obesity (2-4%)
-Diabetes mellitus (2%)
-Tamoxifen therapy (2%)
-Lynch syndrome (hereditary nonpolyposis colorectal cancer) 22-50% (lifetime risk)
-BRCA1/BRCA2 gene mutation (20%)
-Early menarche (Not specified)
-Estrogen secreting tumor (Not specified)
-Family history of endometrial, ovarian, breast, or colon cancer (Not specified)

Of the options, only early menarche is associated with increased risk of endometrial hyperplasia and endometrial cancer.

OPTION A : Combined HRT does not seem to increase the risk of endometrial hyperplasia/cancer. Estrogen-alone HRT, however, is associated with an increased risk.

OPTION B : While age over 50 appears to be a risk factor, age over 30 is not a risk factor for endometrial cancer.

OPTION D : Early menopause, by shortening the time of exposure of endometrium to estrogen, decreases the risk of endometrial hyperplasia and cancer.

OPTION E : Multiparity is associated with decreased risk of endometrial hyperplasia.

269
Q

A 55-year-old woman presents to your clinic complaining of vaginal dryness and irritation, which has resulted in painful sexual intercourse. Speculum examination confirms the presence of atrophic vaginitis. No other abnormality is noted. She has the history of breast cancer and is on tamoxifen. Which one of the following would be the most appropriate management?

A. Non-hormonal vaginal lubricants.
B. Vaginal estrogen cream.
C. Oral estrogen replacement therapy.
D. Combined oral contraceptive pills.
E. Progesterone-only pills.

A

B. Vaginal estrogen cream

The primary indication for treatment of vaginal atrophy is the presence of bothersome genitourinary symptoms related to estrogen deficiency that cause distress in women with low estrogen such as menopausal women.

As the hypoestrogen state continues, symptoms of vaginal atrophy progress and often worsen; therefore, the condition demands treatment once it is encountered. The first-line therapy for post-menopausal atrophic vaginitis is topical vaginal estrogen creams.

With tamoxifen in the history, the patient is very likely to have had an estrogen receptor-positive tumor. Studies show that topical hormone therapy in not associated with an increased risk in patients with estrogen-dependent cancers such as breast cancer; therefore, estrogen cream (in low-doses) can be safely used for treatment of atrophic vaginitis.

NOTE - All hormonal contraception methods and systemic hormone replacement therapies are contraindicated in the presence of a history of breast cancer.

  • Local estrogen therapy and risk of breast cancer recurrence among hormone-treated patients
270
Q

Which one of the following is not associated with increased risk of cervical cancer?

A. Sexual activity in the past.
B. Current sexual activity.
C. Age older than 25years at first sexual intercourse.
D. Multiple sexual partners.
E. Human papilloma virus infection.

A

C. Age older than 25years at first sexual intercourse

Risk factors for carcinoma of cervix include:

-Persistent infection with high-risk HPV types (the most significant risk factor – without HPV infection, there is no risk of cervical cancer even in the presence of multiple other factors)
-Smoking
-Lack of regular cervical screening tests
-Age – cervical cancer risk increases after the age of 35 years
-Prolonged use of OCPs (more than 5 years) –Immunosuppression
-Previous screening abnormalities or previous cervical cancer
-Multiparity (5 or more)
-Exposure to diethylstilbestrol (DES)

Age over 25 at first sexual intercourse decreases the risk of cervical cancer, as women at this age group are more likely to take precautionary measures in sexual relationships.

271
Q

A 27-year-old woman presents to your clinic complaining of a painful swelling down below. She is unable to sit and feels very uncomfortable. On examination, there is vulval redness and tenderness. Which one of the following you should inspect to confirm the diagnosis?

A. Posterior end of the labia minora.
B. Posterior end of the labia majora.
C. Anterior end of the labia minora.
D. Anterior end of the labia majora.
E. Anterior and posterior ends of the labia majora.

A

A. Posterior end of the labia minora

Labia minora (singular: labium minus) are two flaps of skin on either side of the vaginal opening situated between the labia majora. They extend from clitoris obliquely downwards, laterally and backwards.

On the front, each lip divides into two portions surrounding around the clitoris. The upper part of each lip passes above the clitoris to meet the upper part of the other lip, forming a fold that overhangs the clitoris called clitoral hood. The posterior ends of labia minora are joined across the midline by a fold of skin called frenulum of labia minora or fourchette. The part of each lip that is near clitoris is called anterior end of the labium minus (the nomenclature is based on a woman in a standing position where the clitoris is in the front and fourchette in the back). When a woman is in lithotomy position and the vulva observed anteriorly, the posterior end is in the bottom.

Bartholin glands are two pea-sized glands, each being in the posterior (bottom) part of the labium minus. Observed in the lithotomy view, the glands are approximately at 4 o’clock and 8 o’clock positions. The gland secretions, through a 2- to 2.5-cm duct, drains into the vestibule. Cysts and abscesses of these glands leads to gland enlargement and tenderness (in case of abscess) felt in the posterior end of the labium minus.

First picture : Schematic anatomy of Bartholin glands and the ducts and their position in the labia minora

Second picture: An extensive Bartholin abscess in the left labium minus. The swelling has extended from the posterior end of the labium minus to involve almost the entire labium minus

272
Q

Which one of the following is highly suggestive of polycystic ovarian syndrome (PCOS)?

A. Elevated FSH.
B. Elevated LH.
C. Elevated testosterone levels.
D. Decreased free testosterone.
E. Hyperprolactinemia.

A

C. Elevated testosterone levels

Polycystic ovarian syndrome (PCOS) is a common condition associated with the following:
1. Clinical or biochemical hyperandrogenism. Clinical features suggestive of PCOS include hirsutism, acne, deepened voice, etc 2. Menstrual dysfunction, irregularity or lack of ovulation.
3. Polycystic ovaries are present on the ultrasound.

Only the first two mentioned features suffice to establish the diagnosis of PCOS, because cystic structures on ultrasound are often not seen, particularly in women who have been on treatment.

The following hormonal changes are seen in PCOS:
-Serum FSH (follicle stimulating hormone) is either normal or low
Elevation of LH (luteinizing hormone)
-The LH/FSH ration is a way to differentiate several hormonal abnormalities associated with different conditions. This is best measured on day 3 of the menstrual cycles.
-A normal LH/FSH ratio in premenopausal women is about 1:1, while in PCOS this ration may be 2:1 or 3:1. It should be noted that approximately 30% of patients with established diagnosis of PCOS will have a LH/FSH ration of 1:1 Serum estrogen level is either decreased or normal. Overall, estrogen level is very nonspecific.
-Serum free testosterone is the first-line investigation in women suspected of having PCOS. Hyperandrogenism is one of the essential criteria to establish the diagnosis of PCOS.
-Prolactin level is usually normal or mildly elevated.

An elevated LH or FHS level can be seen in other conditions such as premature ovarian failure, menopause, etc; therefore, not highly suggestive PCOS.

*http://emedicine.medscape.com/article/256806-worku
*http://www.ncbi.nlm.nih.gov/pmc/articles/PMC106906

273
Q

Jessica, 51 years old, presents to your practice with complaints of hot flushes, irritability, and disturbed sleep for the past two months which have been progressively worsening. She had been otherwise in good health prior to that. she has her last period 6 months ago after a 12-month history of irregular, widely spaced periods. Her recent symptoms have negatively affected her family and social lives. Which one of the following options would you recommend for treatment?

A. Fluoxetine.
B. Continuous daily estrogen therapy, with medroxyprogesteroneacetate (MPA) given daily for 12days each month.
C. Continuous estrogen and MPA.
D. Daily progesterone therapy with MPA.
E. Continuous estrogen therapy alone.

A

B. Continuous daily estrogen therapy, with medroxyprogesteroneacetate (MPA) given daily for 12days each month

Jessica has classic symptoms of menopause. Bothersome vasomotor symptoms of menopause i.e., hot flushes are the main indication for hormone replacement therapy in perimenopausal women.

There are two recommended regimens for combined HRT therapy:

  1. Cyclical HRT with daily estrogen PLUS MPA given only for 12 days each month – this method is best used within the first 1-2 years of cessation of periods because compared to continuous method, occurrence of unpredictable breakthrough bleeding - an adverse effect of progesterone - will be reduced. Such bleedings can be troublesome for patient and my result in noncompliance with the therapy.
  2. Continuous therapy with daily estrogen and MPA – this method is based on taking both estrogen and MPA on regular daily basis. Unpredictable breakthrough bleeding as an effect of progesterone makes this method appropriate 1-2 years after cessation of menses.

Since the menses has been ceased for only the past 6 months, cyclical HRT with continuous daily estrogen and MPA for 12 days each month will the most appropriate management option for her.

OPTION A : Selective serotonine reuptake inhibitors (SSRIs) are used for treatment of hot flushes of menopause if HRT is contraindicated. Paroxetine is approved for such purpose.

OPTION C : Continuous daily estrogen and progesterone is appropriate for women after 1 to 2 years of menopause.

OPTION D : Progesterone-only method is not an effective HRT because estrogen is the main component counteracting menopausal symptoms.

OPTION E : Estrogen-only method is the best and most effective method of HRT, but it can be used if the patient has no uterus. With a uterus in place, estrogen therapy alone increases the risk of endometrial hyperplasia and cancer.

*http://www.ncbi.nlm.nih.gov/pmc/articles/PMC254436
*https://www.nhmrc.gov.au/_files_nhmrc/publications
* AMC Handbook of Multiple Choice Questions – page 530

274
Q

Which one of the following is increased during pregnancy?

A. Serum alkaline phosphatase.
B. Serum albumin.
C. Serum calcium.
D. Parathyroid hormone.
E. TSH in the first trimester of pregnancy.

A

A. Serum alkaline phosphatase

Normal pregnancy is associated with profound changes in almost every organ system to accommodate the demands of the fetoplacental unit.

Serum albumin, transaminases (AST and ALT) and total bilirubin are lowered compared with the non-pregnant state due to expansion of extracellular volume. The only exception is serum alkaline phosphatase (ALP) which is elevated due to ALP of placental origin.

During pregnancy, the serum total calcium, phosphate and magnesium tend to be low due to the expanded intravascular space. Concentrations of calcium are also affected by the reduced albumin concentration.

The concentration of serum parathyroid hormone tends to be 50% lower in pregnancy, despite the fact that serum concentration of calcium is decreased.

The reasons for hypocalcemia during pregnancy are (1) plasma expansion that through dilutional effect causes decreased plasma calcium and albumin concentration and (2) increased urinary excretion of calcium due to increased GFR

Thyroid stimulating hormone (TSH) normally decreases during the first trimester of pregnancy, during which time there is maximal cross-stimulation of the TSH receptor by human chorionic gonadotropin hormone (HCG).

275
Q

A 24-year-old woman comes for preconception evaluation and consultation. She has the past history of tonic-clonic epilepsy, but has been seizure-free for the past 2 years while on phenytoin. She is currently on oral contraceptive pills. An EEG is obtained that excludes any epileptiform activity. On examination, there is no focal neurological finding. Which one of the following would be the most appropriate advice?

A. Cease phenytoin and change to carbamazepine.
B. Discontinue phenytoin.
C. Advise that she should not become pregnant.
D. Continue phenytoin, but add high dose folic acid according to current recommendations.
E. Check serum phenytoin level and decrease the dose if appropriate.

A

B. Discontinue phenytoin

Antiepileptic therapy may be ceased after the patient has been seizure-free for at least 2 years, particularly if there is no epileptiform activity on EEG. Medication may be withdrawn earlier in women planning a pregnancy; however, the epileptic
disorder needs to be reviewed beforehand.

The decision to withdraw the drug and the time it should be considered depends on the patient’s specific epileptic syndrome and previous history. Factors associated with a poor prognosis associated with withdrawal include:

-A symptomatic epilepsy
-Neurological abnormalities on examination
-History of difficult-to-control seizures (such as patients with juvenile myoclonic epilepsy) –Epileptiform changes on the EEG
-Abnormalities on MRI
-A history of seizure recurrence after previous attempts to withdraw medication

Antiepileptic drugs are potential teratogens and assumed not safe to the fetus. Therefore, withdrawal of antiepileptics should always be considered provided that all the **following are considered(( and strictly followed:

  1. If the patient has been seizure-free for 2 years or more
  2. There is no epileptic activity on EEG
  3. There is no abnormal focal neurological findings on physical exam and imaging studies
  4. Withdrawal from antiepileptic drugs is managed or guided by a specialist.

The dose is gradually decreased over several months and even more slowly for barbiturates. Ideally, complete cessation should take place at least 6 months before conception. If seizures recur, the previously effective dose is resumed.

NOTE - In juvenile myoclonic epilepsy it is best not to withdraw the mediation, or at least to do so after the patient has been seizure free for many years.

Since this woman has been stable for two years and has a normal EEG, gradual withdrawal would be the next best option in management.

276
Q

A 28-year-old lady comes to your clinic at 10 weeks pregnancy. She is keen to know how she can minimize the chance of toxoplasma infection. Which one of the following pieces of advice is not helpful in preventing toxoplasma infection?

A. Wash fruits and vegetables before consumption.
B. Wash hands and kitchen surfaces thoroughly after contact with raw meat.
C. Daily shower.
D. Avoid untreated water while travelling.
E. Avoid changing cat litter boxes.

A

C. Daily shower

Risk of transmission of Toxoplasma can be minimized by washing fruits and vegetables before consumption, washing hands and kitchen surfaces after contact with raw meat and avoiding changing cat litter boxes.

She should also avoid using untreated water while travelling especially in developing countries.

These measures should be adopted throughout the pregnancy.

Daily shower is not recommended for prevention of Toxoplasma.

*http://www.cdc.gov/parasites/toxoplasmosis/prevent